MCAT- Chemical and Physical Foundations of Biological Systems

Réussis tes devoirs et examens dès maintenant avec Quizwiz!

What would be the volume of a 1 L sample of helium if its pressure is changed from 12 atm to 4 atm under isothermal conditions?

3L P1V1=P2V2 12atm(1L)=4atm(V2)

If the Ka of an acid is 1.8 x 10^-5, then what is its pKa?

4.82 (actual 4.72) pKa=-log(1.8 x 10^05) = 5-log 1.8 = 5-0.18

What are the wavelengths of visible light? What are the order of the colors of the visible spectrum?

400-700 nm ROYGBV in order of DECREASING wavelength. Red has longest, violet has shortest.

How many sigma and pi bonds are present in H3C-HC=O

6 sigma and 1 pi

which of the following molecules contains the oxygen atom with the most negative formal charge? A. H2O B. CO3^-2 C. O3 D. CH2O

B

What is the difference between Keq, Kp, and Kc?

Keq is the equilibrium constant Kc is the equilibrium constant and the subscript c indicates that it is in terms of concentration. Kp is the equilibrium constant and the subscript p indicates that it is in terms of pressure.

what is damping or attenuating (waves)?

a decrease in amplitude of a wave caused by an applied or nonconservative force

What is radioactive decay?

a naturally occurring spontaneous decay of certain nuclei accompanied by the emission of specific particles

what is the third law of thermodynamics?

absolute zero cannot be reached "the entropy of a perfectly organized crystal at absolute zero is zero"

What are nodes and antinodes of a wave?

as the waves move in opposite directions, they interfere to produce a new wave pattern characterized by alternating opints of maximum displacement (amplitude) and points of no displacement. The points in a standing wave with no fluctuation in displacement are called nodes. The points with maximum fluctuation are called antinodes.

what is not included in the mass calculations of an atom and why?

electrons bc they tiny

what is the effective nuclear charge (Zeff)?

electrostatic attraction between the valence shell electrons and the nucleus -a measure of the net positive charge experienced by the outermost electrons

what do oxidizing agents do in redox reactions?

it is the element of compound that accepts an electron

Modern atomic therapy postulates that any electron in an atom can be completely described in 4 quantum numbers. What are they and what is a good way to think of them?

n, l, m(sub l), and m (sub s ) think of them as becoming more specific as one goes from n to m (sub s). Like an address: one lives in a particular state (n), in a city (l), on a street (m (sub l)), at a house number (m (sub s)).

The interquartile range can be used to determine what? Define this term.

outliers. Any value that falls more than 1.5 interquartile ranges below the first quartile or above the third quartile is considered an outlier.

What is the coordination number?

the number of atoms that surround and are bonding to a central atom (relevant when determining molecular geometry)

how can you tell which harmonic a wave of an open pipe is on?

the number of nodes present will tell you which harmonic it is.

What is saponificaiton?

the process by which fats are hydrolyzed under basic conditions to produce soap.

what is the spectroscopic notation?

the shorthand representation of the principal and azimuthal quantum numbers. l=0 (s) l=1 (p) l=2 (d) l-3 (f) Thus, an electron in shell n=4 and sub shell l=2 is in the 4d sub shell

What is translational motion?

when forces cause an object to move without any rotation

A 2000kJ experimental car can accelerate from 0 to 30 m/s in 6s. What is the avg power of the engine needed to achieve this acceleration?

150kW W=delta Kinetic energy = 1/2 m (vf^2-vi^2) = .5 (2000kg) (900-0) = 900kJ P=W/t = 900kJ/t = 150kW

A holding take at sea level (atm pressure 1.01 x 10^5 Pa) containing water has a pressure of 2 x 10^5 Pa at 10 m depth. What is the pressure in an equivalent holding take containing mercury (specific gravity = 13) at 1 m depth in Denver, CO (atm pressure 8.4 x 10^4 Pa)? Formula for hydrostatic pressure in a fluid Ptot=P0 + pgh

2 x 10^5Pa Ptot= (8.4 x 10^4) + (13,000kg/m^3 x 10m/s^2 x 1m)

An electron returns from an excited state to its ground state, emitting a photon at wavelength 500nm. What would be the magnitude of the energy change if one mole of these photons were emitted?

2.39 x 10^5 J E=(hc/wavelength) x avagardos #

What are the standard conditions for measuring the enthalpy, entropy, and Gibbs free energy changes of a reaction?

25C (298K), 1 atm pressure, 1M [ ]

Consider the name 2,3-diethylpentane. The correct IUPAC name would be?

3-ethyl-4-methylhexane

A weight lifter lifts a 275 kg barbell from the ground to a height of 2.4m. How much work has he done in lifting the barbell, and how much work is required to hold the weight at that height?

6468J and 0J, respectively. W=fd = 2750N (2.4m) = 6468J W=fd= 2750N (0m)=0J

As the confidence level increases, a confidence interval: A. becomes wider B. becomes thinner C. shifts to higher values D. shifts to lower values

A

Which of the following lists the correct common names for ethanal, methanol, and ethanol, respectively? A. Acetaldehyde, formaldehyde, ethyl alcohol B. Ethyl alcohol, propionaldehyde, isopropyl alcohol C. Ethyl alcohol, formaldehyde, acetaldehyde D. isopropyl alcohol, ethyl alcohol, formaldehyde

A

How is it that ships float?

Any object will float if its average density is less than that of water. The steel hull of the ship would sink by itself, but all the air submerged beneath the water level, between the ship's lower decks, lowers the ship's avg density to be less than that of water.

What are Arrhenius acids and bases?

Arrhenius acids dissociate to form an excess of H+ in solution. Arrhenius bases will dissociate to form an excess of OH- in solution. These are easily identified: Acids contain an H at the beginning of their formula (HCl, HNO3), bases contain OH at the end of their formula (NaOH, Fe(OH)3)

What is the median of the following data set? 7, 17, 53, 23, 4, 2, 4 A. 4 B. 7 C. 15.7 D. 23

B

Sn1 reactions show first order kinetics because: The rate limiting step is the first step to occur in the reaction The rate limiting step involves only one molecule There is only one rate limiting step The reaction involves only one molecule

B.

What is the highest energy orbital of elements with valence electrons in the n=3 shell? A. s-orbital B. p-orbital C. d-orbital D. f-orbital

C

Water flows from a pipe of diameter 0.15m into one of diameter 0.2m. If the speed in the 0.15m pope is 8m/s, what is the speed in the 0.2m pipe? A. 3m/s B. 3.7m/s C. 4.5m/s D. 6m/s

C VaAa=VbAb

Which of the following can be used to reduce a ketone to a secondary alcohol? A. CrO3 B. KMnO4 C. LiAlH4 D. Ag2O

C Hydrides like LiAlH4 and NaBH2 are reducing agents; as such, they will reduce aldehydes and ketones to alcohols. The other reagents listed are oxidizing agents, which will not act on a ketone

Chlorate

ClO3-

Perchlorate

ClO4-

Chromate

CrO4 2-

Copper(I)

Cu+

A massless spring initially compressed by a displacement of 2 cm is now compressed by 4 cm. How has the potential energy of this system changed? A. The potential energy has not changed B. The potential energy has doubled C. The potential energy has inc by 2 J C. The potential energy has quadrupled

D

Compared to single bonds, triple bonds are: A. weaker B. longer C. made up of fewer sigma bonds D. more rigid

D

Which of the following compounds possesses at least one sigma bond? A. CH4 B. C2H2 C. C2H4 D. All of the above contain at least one sigma bond

D

Imines naturally tautomerize to form: A. Oximes B. Hydrazones C. Semicarbazones D. Enamines

D During tautomerization, the double bond between the carbon and nitrogen in an immune is moved to lie between two carbons. This results in an enamine- a compound with a double bond and an amine.

For a three second period of time, a 50 kg wooden crate slides across the concrete floor at exactly 5 m/s. The coefficient of kinetic friction between the wooden crate and the concrete floor is 0.2. What is the net force of this crate during this three second period? A. 500N B. 100N C. 50N D. 0N

D Since there is a constant velocity, there is no net acceleration and no net force.

Where are electrons found? What is an electrons charge? What is their mass in relation to a proton?

In the space surrounding the nucleus (orbitals). -1. Mass is approx 1/2000 of a proton

What are the peaks of O-H, N-H, and C=O, in infrared spectroscopy?

O-H (broad around 3300 cm-1) N-H (sharp around 3300 cm-1) C=O (sharp around 1750 cm-1)

Oxygen molecules travel at an average speed of approximately 500m/s at a given temp. Calculate the avg speed of hydrogen molecules at the same temperature.

Oxygen's molar mass is 32g/mol. Hydrogen's molar mass is 2g/mol. Plugging into Graham's law we get: see pg 281 chemistry (r1/r2)=sqrrt(M1/M2) (500m/s)(sqrrt32/2)=2000m/s

what is the ground state of an atom?

State of lowest energy, in which all electrons are in the lowest possible orbitals

What are confidence intervals?

a range of values so defined that there is a specified probability that the value of a parameter lies within it.

what's the difference between a dilute and a concentrated solution?

a solution in which the proportion of solute to solvent is SMALL is dilute. If the proportion is LARGE it is a concentrated solution.

How is a magnetic field created? What is the SI unit?

any moving charge, whether a single electron traveling through space or a current through a conductive material, creates a magnetic field. SI Unit: Tesla

what is a natural (resonant) frequency?

any solid object, when hit, struck, rubbed, or disturbed in any way will begin to vibrate.

what are isotopes?

atoms that share an atomic number but have different mass numbers. they differ in their number of neutrons

Consider a stationary negative source charge and a positive test charge that can be moved. Because these two charges are unlike, they will exert _______ forces between them. Therefore, the closer they are to each other, the _______ stable they will be. Consider two positive charges. As like charges, the will exert _____ forces, and the potential energy of the system will be ______.Therefore, the closer they are to each other, the ______ stable they will be.

attractive more repulsive positive less

Explain hyperopia and myopia using bifocal lenses

bifocal lenses are corrective lenses that have two distinct regions-one that causes convergence of light to correct for farsightedness (hyperopia) and a second that causes divergence of light to correct for nearsightedness (myopia) in the same lens

Explain blinding in an experiment. Explain single blind vs double blind.

blinding means they subjects and/or investigators do not have info about which group the subject is in. in single blind experiments, only the patient or the assessor (the person who makes measurements on the patient or performs subjective evaluation) is blinded. In double-blind experiments, the investigator, subject, and assessor all do not know the subject's group.

What are bond enthalpies/bond dissociation energies?

bond dissociation energy is the average energy that is required to break a particular type of bond between atoms in the gas phase-remember, bond dissociation is an endothermic process

In general, how does bond strength trend in the periodic table? What about acidity? The more electronegative an atom, the ___________ the acidity. What happens when these trends oppose each other?

bond strength decreases down a periodic table acidity, therefore, increases the higher the acidity low bond strength takes precedence

What is a coordinate covalent bond?

both of the shared electrons originated on the same atom - generally, this means that a lone pair of one atom attacked another atom with an unhybridized p-orbital to form a bond

What are capacitors? Give an example.

can hold charge at a particular voltage ex: defibrillators - while a defibrillator is charging, a high-pitched electronic tone sounds as electrons build up on the capacitor. When the defibrillator is fully charged, that charge can be released in one surge of power (after the operator yells CLEAR!). ex: the clouds and the ground during a lightning storm also act as a capacitor, with the charge building up between them eventually discharging as a bolt of lightening. The MCAT focuses on a particular type of capacitor called a parallel plate capacitor

Electrons closer to the nucleus are at ______ energy levels, while those further out (in ________ _______ _______) have _____ energy.

closer are at lower energy levels. those further out (in higher electron shells) have higher energy

What is gamma decay?

emission of high energy photons (no charge, just lower the energy of the emitting nucleus) called gamma-rays. The high-energy state of the parent nucleus may be represented by an asterisk

what is convection?

heat transfer in a fluid in which hot fluid rises and cold fluid sinks, setting up a cycle

How are amplitude and intensity related?

intensity is proportional to the square of the amplitude. Therefore doubling the amplitude produces a sound wave that has 4x the intensity.

What is the base SI unit for force?

kg . m /s^2 (the newton is a derived unit, with these base units making it up)

What are the differences between lenses and mirrors?

lenses refract light while mirrors reflect light. When working with lenses, there are two surfaces that affect the light path. For example, a person wearing glasses sees light that travels from an object through the air into the glass lens (first surface). Then the light travels through the glass until it reaches the other side, where again it travels out of the glass and into the air (second surface). The light is refracted twice as it passes from air to lens and from lens to air.

what is volatile

low melting point, sublimable solids or vaporizable liquids

What is the mode?

most frequently occurring score

The atomic number (Z) of an element is equal to the number of ______________ in one of its atoms.

protons

In a neutral atom, number of _______ = number of ______

protons = electrons

What factors affect reaction rate?

reaction [ ]s temperature medium catalysts

what is the difference between relative and absolute configuration of a chiral molecule?

relative configuration- its configuration in relation to another chiral molecule (often through chemical interconversion) *we can use the relative configuration to determine whether molecules are enantiomers, diastereomers, or the same molecule absolute configuration- describes the exact spatial arrangement of these atoms or groups, independent of other molecules

What is hydrolysis?

salt ions react with water to give back the acid or base, opposite of neutralization

explain the concept of steric hindrance

steric hindrance- the prevention of reactions at a particular location within a molecules due to the size of substituent groups.

What is terminal velocity?

the constant velocity of a falling object when the force of air resistance is equal in magnitude and opposite in direction to the force of gravity

What are triaclyglycerols?

the storage form of fats inthe body, esters of long-chain carboxylic acids (fatty acids) and glycerol.

Two atomic orbitals may combine to form: I. a bonding molecular orbital II. an anitbonding molecular orbital III. Hybridized orbitals

I, II, and III

The molecule shown in FIGURE 14 ORGANIC CHEMISTRY is: I. synthesizable from a gamma-hydroxycarboxylic acid II. a lactone III. a form of an ester

I, II, and III The molecule shown, gamma-nonalactone, is a cyclic ester, also called a lactone. This molecule could arise from intromolecular attack in a gamma-hydroxycarboxylic acid.

Within one principal energy level, which sub shell has the least energy? A. s B. p C. d D. f

A

A low-pressure weather system can decrease the atmospheric pressure from 1 to 0.99atm. By what percent will this decrease the force on a rectangular window from the outside. Assume the window is 6x3m and the glass is 3cm thick. A. 1% B. 10% C. 1/3% D. 30%

A P=f/A If pressure dec by 1% and area does not change, the force will be dec by 1%

What is the correct electron configuration for Zn2+?

1s^2 2s^2 2p^6 3s^2 3p^6 4s^0 3d^10

what is temperature?

The measure of the average kinetic energy of the molecules.

What is power?

the rate at which energy is transferred from one system to another

Hypochlorite

ClO-

Chlorite

ClO2-

Dichromate

Cr2O7 2-

What is Newton's third law?

Every action has an equal and opposite reaction

hydrogen sulfate (bisulfate)

HSO4-

What are conservative forces? Examples? What about nonconservative forces? Examples?

They are path independent forces that do not dissipate the mechanical energy of a system. * if only conservative forces are acting on an object, the total mechanical energy is conserved. * examples gravity and electrostatic forces are conservative delta E= delta U + delta K = 0 -nonconservative forces are like frictional forces, viscous drag, and air resistance dissipate mechanical energy as thermal or chemical energy delta E= delta U + delta K = W nonconservative

Permanganate

MnO4-

Nitride

N3-

Oxide

O2-

Sulfite

SO3 2-

How many total electrons are in a 133Cs cation? A. 54 B. 55 C. 78 D. 132

A

Explain chromatic aberration.

a dispersive effect within a spherical lens. Depending on the thickness and curvature of the lens, there may be significant splitting of white light, which results in a rain bow halo around images. This phenomenon is corrected for in visual lenses like eyeglasses and car windows with special coatings that have different dispersive qualities than the lens itself

Which of the following elements has the highest electronegativity? A. Mg B. Cl C. Zn D. I

B

what is a voltmeter and a petentiometer?

a voltmeter is used to measure the emf of a cell. A potentiometer is a kind of voltmeter that draws no current and fives a more accurate reading of the difference in potential between 2 electrodes.

can a dielectric material ever decrease the capacitance?

no! thus, k(dielectric constant) can never be less than 1

what is a mole?

the number of "things" (atoms, ions, molecules) equal to Avagadros number-6.02 x 10^23

Carbon and silicon are the basis of biological life and synthetic computing, respectively. While these elements share many chemical properties, which of the following best describes a difference between the two elements. A. Carbon has a smaller atomic radius than silicon B. Silicon has a smaller atomic radius than carbon C. Carbon has fewer valence electrons than silicon D. Silicon has fewer valence electrons than Carbon

A

A NAOH solution has an OH- concentration of 1 x 10^-5 M. What is the pH of the solution? A. 2 B. 5 C. 9 D. 12

C poH=-log[OH-]

Dihydrogen Phosphate

H(2)PO(4)-

What is the half equivalence point?

It is when exactly one half of the acid has been neutralized by the base; in other words, [HA] = [A-]

amino acids are _______ molecules. Why?

amphoteric (can act as both acids and bases). b/c they have an acidic carboxyl group and a basic amino group

what is the difference between an isovolumetric (isochoric) process and an isobaric process?

an isovolumetric (isochoric) process is when the volume stays constant as the pressure changes, so no work is done an isobaric process is when the pressure remains constant

Rank the carboxylic acid derivatives from most reactive to least reactive

anhydrides > carboxylic acids and esters > amides

What is the difference between converging and diverging lenses? Who uses each?

converging lens is thicker at the center while diverging lenses are thinner at the center. converging lenses (reading glasses) are needed by people who are "farsighted" and diverging lenses (standard glasses) are needed by people who are "nearsighted."

Is it easy or difficult to deprotonate the alpha hydrogen of an aldehyde or ketone? Why?

easy. through induction, oxygen pulls some of the electron density our of the C-H bonds, weakening them.

Using E=-Rh/n^2. How would you say the angular momentum of an electron changes?

in discrete amounts with respect to the principal quantum # (b/c n is the only thing that is not a constant)

Using L=nh/2pi. How would you say the angular momentum of an electron changes?

in discrete amounts with respect to the principal quantum # (b/c n is the only thing that is not a constant)

what is the Zeff trend across a period what about the atomic radius?

increases left to right across a period atomic radius as a result decreases from left to right (essentially the opposite of other periodic trends)

what are the threshold of hearing and the threshold of pain?

threshold of hearing- the softest sound that the avg human ear can hear I=1 E-12 W/m^2 threshold of pain- intensity that causes perforation of the eardrum. 1 E4 W/m^2

What is laminar flow?

opposite of turbulent flow. characterized by smooth, parallel layers of fluid often modeled as layers of fluid that flow parallel to each other

what is timbre

quality of sound

By convention, the direction of the electric field vector is given as what? What are field lines?

the direction that a positive test charge would move in the presence of the source charge. field lines are imaginary lines that represent how a positive test charge would move in the presence of the source charge.

The conversion of ubiquinone to ubiquinol requires what type of reaction? A. Condensation B. Oxidation C. Reduction D. Hydrolysis

C two ketones are reduced to two hydroxyl groups

what is radiation?

The transfer of energy by electromagnetic waves

The atomic mass of an atom is nearly equal to what?

it's mass number (protons+neutrons)

Borate

BO3 3-

Which of the following is not a Bronsted-Lowry base? FIGURE 7 GENERAL CHEMISTRY A. B. C. D.

D A Bronsted-Lowry base is defined as a proton acceptor.

Thiocyanate

SCN-

What is a decomposition reaction?

When something is broken down into smaller molecules (AB→A+B)

What is nonmalificence?

the obligation to avoid treatments or interventions in which the potential for harm outweights the potential for benefit

what are conjugate acids and bases?

a conjugate acid is the acid formed when a base gains a proton, and a conjugate base is the base formed when an acid loses a proton.

What are the three types of observational studies in medicine?

cohort studies, cross-sectional studies, and case-control studies

at room temp, the majority of atoms in a sample are in what state?

in the ground state

What are cyclic esters called? How are they named?

lactones and are named by replacing -oic acid with lactone

what is a rechargeable cell (rechargeable battery)?

one that can function as both a galvanic and electrolytic cell

Bohr came to describe the structure of the hydrogen atom as a nucleus with one proton forming a dense core, around which a single electron revolved in a defined pathway (_____), at a discrete value

orbit

What makes up the mass number (A) of an atom

the total number of protons and neutrons in a nucleus.

The following system obeys second-order kinetics. 2NO2-->NO3 + NO. (slow) NO3 + CO --> NO2 + CO2. (fast) What is the rate law for this reaction? A. rate=k[NO2][CO] B. rate=k[NO2]^2 [CO] C. rate=k[NO2][NO3] D. rate=k[NO2]^2

D. To answer this question, recall that the slow step of a reaction is the rate determining step. The rate is always related to the [ ]s of the reactants in the rate-determining step (not the overall reaction), so NO2 is the only compound that should be included in the correct answer. The [ ] of NO2 is squared in the rate law because the question stem tells us that the system obeys second-order kinetics.

Ferric

Fe3+

What are the properties of the chalcogens? what group are they?

Group VIA or 6. nonmetals and metalloids. each have 6 electrons in their valence electron shell and due to their proximity to the metalloids, have small atomic radii and large ionic radii oxygen in this group

what are the properties of the halogens? what group are they?

Group VIIA or 7. highly reactive nonmetals with 7 valence electrons. (desperate to complete their octets) very reactive towards the alkali and alkaline earth metals the halogens are so reactive that they are not naturally found in their elemental state but rather as ions (halides) or diatomic molecules

hydrogen carbonate (bicarbonate)

HCO3-

Sulfate

SO4 2-

Reverse osmosis is a process that allows fresh water to be obtained by using pressure to force an impure water source through a semi-permeable membrane that only allows water molecules to pass. What is the minimum pressure that would be required to purify seawater at 25C that has a total osmolarity of 1000mOsm/L? A. 23.4atm B. 24.5atm C. 24000atm D. 24500atm

B Osmotic pressure is given by the formula, osmotic pressure=iMRT. osmotic pressure=(1000E-3mOsm/L)(0.0812L atm/mol K)(298K) =1 x 0.08 x 300 = 24 atm (actual value 24.2 atm)

A new study of a weight loss drug uses a radio advertisement to generate study participation. What type of error is most likely to result? A. Hawthorne effect B. Selection bias C. Confounding D. Detection bias

B Requiring subjects to volunteer for a study and to seek the study out will introduce selection bias. The people who end up volunteering listen to the radio, which the general population may not, and are interested int he topic and willing to volunteer. Most studies suffer from selection bias and it is the most common impediment to generalizability.

Which of the following bases is the weakest? A. KOH B. NH3 C. CH3NH2 D. Ca(OH)2

B Soluble hydroxides of GroupIA and IIA metals are strong bases, eliminating A and D. B and C are both weak base; however, methylamine contains an alkyl group, which is electron-donating. This increases the electron density on the nitrogen in methylamine, making it a stronger (Lewis) base. Therefore, ammonia is the weakest base.

Ultrasound machines calculate distance based upon: A. intensity of the reflected sound B. travel time of the reflected sound C. angle of incidence of the sound D. the detected frequency of the sound

B While intensity, A, could be used to measure distance, time of travel is an easier indication and most commonly used by ultrasound machines. Apparent frequency, D, is only used in Doppler ultrasound, and not calculate distance. Angle of incidence, C, can be used to position various structures on the screen of an ultrasound, but is not used to calculate distance

Nylon, a polyamide, is produced from hexanediamine and a substance X. This substate X is most probably a(n) A. Amine B. Carboxylic acid C. Ketone D. Alcohol

B an amide is formed from an amine and a carboxyl group or its acyl derivatives. In this question, an amine is already given; the compound to be identified must be an acyl compound. The only acyl compound among the choices is a carboxylic acid

In order to convert phenols into hydroxyquinones, how many steps of oxidation or reduction are required? A. 1 oxidation step B. 2 oxidation steps C. 1 reduction step D. 2 reduction steps

B in order to convert phenols into hydroxyquinones, they must first be converted to quinones through an oxidation step; a second oxidation step is required to further oxidize quinones to hydroxyquinones

A 30 kg girl sits on a seesaw at a distance of 2 m from the fulcrum. Where must her father sit to balance the seesaw if he has a mass of 90 kg? A. 67 cm from the girl B. 67 cm from the fulcrum C. 133 cm from the girl D. 267 cm from the fulcrum

B.

What is the pH of a solution with [HClO4] = 10M?

Because perchloric acid is a stron acid, it will fully dissociate in solution. Therefore, [H+] = 10M (note that the contribution from the autoionization of water is negligible). pH=-log[H+] = -log10M=-1 This question points out that the pH scale does not end at 0 and 14. There can be negative pH values and pH values greater than 14 but this implies a very high [ ] of a strong acid or base

What are Bronsted-Lowry acids and bases?

Bronsted-Lowry acids are molecules or ions that are proton donors. Bronsted-Lowry bases have molecules or ions that are proton acceptors.

A figure shows 2 compartments side by side in a thick metal container. Compartment A is full of a hot gas, while compartment B is full of a cold gas. What is the primary mode of heat transfer in this system? A. Radiation B. Convection C. Conduction D. Enthalpy

C

If the rate law for a reaction is rate= k[A]^0 [B]^2 [C]^1 what is the overall order for the reaction A. 0 B. 2 C. 3 D. 4

C

The four C-H bonds of CH4 point toward the vertices of a tetrahedron. This indicates that the hybridization of the carbon atom in methane is A. sp B. sp2 C. sp3 D. sp3d

C

What is the final temp of a 3 kg wrought iron fireplace tool that is left in front of an electric heater, absorbing heat energy at a rate of 100 W for 10 minutes? Assume the pendant is initially at 20 C and that the specific heat of wrought iron is 500 J/kg . K A. 40C B. 50C C. 60C D. 70C

C E= P(t)=100W (10min) (60s/1min)=6E4J q=mCdeltaT 6E4 J=3kg (500 J/kg . K) (deltaT)

maximum # of e-s in a subshell=?

4l+2

Which will fill first, the 5d subshell or the 6s subshell?

6s. use the n+1 rule (the lower the sum of the values of the 1st and 2nd quantum numbers, n and l, the lower the energy of the sub shell. The sub shell with the lowest energy will fill first 5d- n=5, l=2 so n+l=7 6s- n=6, l=0, so n+l=6 (lower energy, fills first)

What are STP conditions (MCAT) and standard state conditions?

STP 273K and 1atm for gas law standard state 25C (298K)

What is an acid equivalent and a base equivalent?

an acid equivalent is equal to one mole of H+ (or more properly, H3O+) ions; a base equivalent is equal to one mole of OH- ions.

What is the difference between chemical and physical properties?

physical properties are characteristics of

What are polyvalent titrations?

they have multiple equivalence points.

In aqueous solution, pyrophosphate will likely: A. form insoluble complexes B. be stable and inert C. degrade into inorganic phosphate D. decrease the polarity of the solvent

C Pyrophosphate is unstable in aqueous solution and will degrade to form two equivalents of inorganic phosphate. The solvent is water, which should retain its polarity regardless of the presence of solutes. Pyrophosphate and inorganic phosphate are small, charged molecules which are relatively soluble.

Which of the following relationships is incorrect? A. Isinθ x cosθI < IsinθI + IcosθI B. sinθ / cosθ = tanθ C. tan 90 is undefined D. sinθ = sin (90-θ)

D sinθ is not equal to sin (90-θ), although sinθ=cos (90-θ).

- In the Bohr model, the group of H emission lines corresponding to transitions from energy levels n>/=2 to n=1 is known as the ______ series. - The group corresponding to transitions from energy levels n>/=3 to n=2 is the ______ series. - The group corresponding to transitions from energy levels n>/=4 to n=3 is the ______ series.

- Lyman - Balmer - Pachen

how can one use formal charges to assess the stability of resonance structures?

- a Lewis structure with small or no formal charges is preferred over a Lewis structure with large formal charges - A lewis structure with less separation between opposite charges is preferred over one with a large separation of opposite charges - A Lewis structure in which negative formal charges are placed on more electronegative atoms is more stable than one in which the negative formal charges are placed on less electronegative atoms

What is electric potential?

-the ratio of the magnitude of a charge's change in electric potential energy to the magnitude of the charge itself V=U/q

What is the density of water?

1 g/cm3 or 1000kg/m3qa

Explain the SN2 reaction

1 step (concerted reaction). Bimolecular the nucleophile attacks the compound as the same time as the LG leaves The nucleophile actively displaces the LG in a backside attack. For this to happen, the nucleophile must be strong and the substrate cannot be sterically hindered. Therefore, the less substituted the carbon, the more reactive it is in SN2 reactions (opposite trend from SN1) The single step involves 2 reacting species, the substrate (often an alkyl halide, tosylate, or mesylate) and the nucleophile. Therefore, the [ ]s of both have a role in determining the rate SN2 reactions have an inversion of stereochemistry

A 2L sample of 100C and 20 atm contains 5 moles of a gas. If an additional 25 moles of gas at the same pressure and temp are added, what is the final volume of the gas?

12L n1/V1=n2/V2 5 mol/2L=5+25 mol/V2

What volume would 12 g of helium occupy at 27C and a pressure of 380mmHg?

148L math on pg 266 of chem R=8.21E-2 L(atm)/mol(k) P=380mmHg[1atm/760mmHg]=0.5atm T=27C +273=300K n=12gHe[1molHe/4g]=3 mol He PV=nRT (0.5atm)(V)=(3 mol He)(0.0821 L(atm)/mol(K))(300K) V=148L

What is a double-displacement reaction?

AB + CD --> AC + BD

what is the dipole moment?

Any time charge is not evenly distributed within a bond. p=qd p=dipole moment q= charge d=distance between charges. shown by an arrow pointing from the pos to neg charge

Antimony is used in some anti parasitic medication specifically those targeting Leishmania Donovani What type of element is it? Metal Metalloid Halogen Nonmetal

B

Water is a rare substance in that the solid is less dense than the liquid at the freezing point, resulting in a solid form that floats on top of the liquid. Which of the following best explains this phenomenon? A. The bent structure of the water molecule results in a molecular dipole that maximizes the close molecular packing structure in the solid state. B. The bent structure of the water molecule and ratio of covalently-bonded hydrogens to lone pairs of electrons on the oxygen atom maximizes the hydrogen bonding that occurs in the solid phase, producing a hexagonal structure with large empty spaces. C. The degree of ionization in the solid state is less than in the liquid phase. D. The London dispersion forces of water significantly decrease in the solid phase as compared with the liquid phase.

B is correct. For most substances, the liquid becomes more dense as the average kinetic energy (temperature) decreases, and the solid is more dense than the liquid due to close-packing solid-state structures, resulting in the formation of the solid at the bottom of the liquid. However, as stated in the question, solid water (ice) is significantly less dense than the liquid form at 0°C (the melting/freezing point). Remember, the water molecule is bent (shown below), with a bond angle of approximately 104.5°. This, combined with the degree of hydrogen bonding that can occur between water molecules, yields a solid crystalline structure with relatively large amounts of empty space. As a result, solid water is less dense than its liquid form. A: While water does have a bent structure and a molecular dipole, "close" packing of the molecules in the solid state would imply that the solid phase of water is more dense than the liquid. The question asks to explain the opposite phenomenon. C: Water does auto-ionize in its liquid, but presumably not in its solid, phase. However, this activity has no clear effect on the density of the solid compared to the liquid. D: Water does experience London dispersion forces, but these are the weakest of the intermolecular forces and are unlikely to have a significant effect in this case. Content Foundations: Solid Phase of Water Within the scope of the MCAT, there are three fundamental phases of matter: solid, liquid, and gas. The phase of a given substance is closely related to intermolecular forces—that is, the forces that exist between molecules. Stronger intermolecular forces tend to be associated with solids, weaker intermolecular forces with gases, and liquids are in between. For water (H2O), in its liquid and solid phases, molecules are held together by relatively strong intermolecular forces: hydrogen bonds. A solid is a structure with a rigid, tightly-packed organization of atoms, such as sodium chloride (table salt) and most metals, like iron. Solids are characterized by a fixed volume, meaning that they do not expand and are not compressible to any significant degree, as well as a fixed shape. Solids also do not flow, although their particles do vibrate in place. Like solids in general, the solid phase of water (ice) exhibits these properties. The solid phase of water predominates under low temperatures, and we might expect it to also predominate under high pressures, since we can imagine liquid molecules "packing" more tightly together when compressed. Interestingly, while this is true of many substances, it is not the case for water! The solid "ice" phase is actually less dense than the liquid phase, meaning that ice floats on top of water. (Just remember: icebergs float!) For the MCAT, this concept may appear in the form of a phase diagram. In such a diagram, a line separates the regions that correspond to the solid and liquid phases. For most substances, since the solid phase is denser than the liquid, this line has a positive slope. This indicates that it is possible to convert the liquid substance into solid by increasing pressure. Since the opposite is true for water, however, the line between these regions has a negative - or downward-tilting - slope. This hallmark is one easy way to distinguish the phase diagram of water from that of most other compounds.

Explain a box and whisker plot.

Bounded by Q1 and Q3; Q2 (the median) is the line in the middle of the box (This is what the grade distributions on canvas were)

Acetate

CH3COO-

Substances A and B have the same freezing and boiling points. If solid samples of both substances are heated in the exact same way, substance A boils before substance B. Which of the following would NOT explain this phenomenon? A Substance B has a higher specific heat B. substance B has a higher heat of vaporization C. substance B has a higher heat of fusion D. substance B has a higher internal energy

D (physics ch 3)

What is electric potential energy?

Energy a charge has due to its location in an electric field it is the work necessary to move a test charge from infinity to a point in space in an electric field surrounding a source charge

Which of the following are considered terminal functional groups? I. Aldehydes II. Ketones III. carboxylic acids

I and II

Which of the following are Lewis bases? I. Ag+ II. H2O III. NH3

II and III only

What is the correct compound rank by ascending BP? I. Acetone II. KCl III. Kr IV. Isopropyl alcohol

III < I < IV < II

What is fission?

a process by which a large nucleus splits into smaller nuclei. Spontaneous fission rarely occurs. However, through the absorption of a low-energy neutron, fission can be induced in certain nuclei.

What is Kirchhoff's Junction Rule?

at any point or junction in a circuit, the sum of currents directed into that point equals the sum of currents directed away from that point. I(into junction)=I(leaving junction)

What is electron capture?

certain unstable radionuclides are capable of capturing an inner electron that combines with a proton to form a neutron, while releasing a neutrino. The atomic number is now one less than the original but the mass number remains the same. Electron capture is a rare process that is perhaps best thought of as the reverse of B- decay

what is the spin quantum number?

m (sub s). +1/2 or -1/2. whenever 2 electrons are in the same orbital, they must have opposite spins. They are paired. Electrons in different orbitals with the same m (sub s ) values have parallel spins.

explain diastereomers

non-mirror-image configurational isomers. Diasteroemers occur when a molecule has two or more stereogenic centers and it differs at some, but not all, of these centers For any molecule with n chiral centers, there are 2^n possible stereoisomers diastereomers have different chemical properties. However, they might behave similarly in particular reactions because they have the same functional groups -diastereomers will rotate plane-polarized light however, knowing the specific rotation of one diastereogmer gives no indication of the specific rotation of another diastereomer

What is the zeroth law of thermodynamics?

objects are in thermal equilibrium when they are at the same temperature if a=b and b=c then a=c.

What is fusion?

occurs when small nuclei combine to form a larger nucleus.

What are oxidation-reduction (redox) reactions?

reactions in which the oxidation states of the reactants change. Oxidation state-an indicator of the hypothetical charge that an atom would have if all bonds were completely ionic

What are mixed-order reactions?

reactions where rate orders vary over the course of the reaction (Broken-order)

What are the two types of images created by a mirror? How can you tell the difference?

real or virtual. An image is real if the light actually converges at the position of the image. An image is virtual if the light only appears to be coming from the position of the image but does not actually converge there. One of the distinguishing features of real images is the ability of the image to be projected onto a screen.

What is the atomic radius of an element?

refers to the size of a neutral element. equal to 1/2 fo the distance between the centers of two atoms of an element that are briefly in contact with each other. - the distance between two centers of circles in contact is akin to a diameter, making this radiation calculation simple

what is the reduction potential of a species?

the tendency of a species to gain electrons and to be reduced. Each species has its own intrinsic reduction potential; the more positive the potential, the greater the tendency to be reduced

Explain half life

the time it takes for half of the sample to decay. In each subsequent half life, one half of the remaining sample decays so that the remaining amount asymptotically approaches zero.

What is the atomic weight? What is the utility of the atomic weight?

the weighted avg of the different isotopes (it is the # on the periodic table) the utility is that it represents both the mass of the avg atom of that element, in amu, and the mass of one mole of the element in grams - Example:Chlorine-35 is about 3 times more abundant than chlorine-37 and therefore the atomic weight of chlorine is closer to 35 than 37

What is a combination reaction?

two or more reactants form one product A + B --> C

What is a bimodal distribution?

when 2 scores both occur with the greatest frequency. A distribution containing two peaks with a valley in between.

What are morally relevant differences?

those differences between individuals that are considered an appropriate reason to treat them differently. Ex: age.

Valence electrons are much more likely to do what?Why?

to become involved in bonds with other atoms because they experience the least electrostatic pull from their own nucleus

how can you tell which harmonic a wave of a closed pipe is on?

make sure to actually count the number of quarter-wavelengths contained in the pipe to determine the harmonic

What is glycogen phosphorylase?

functions by cleaving glucose from the nonreducing end of a glycogen rbanch and phosphorylating it, thereby producing glucose-1-phosphate, which plays an important role in metabolism

What is the difference between resistors connected into a circuit in series vs in parallel?

in series, all current must pass sequentially through each resistor connected in a linear arrangement in parallel, the current will divide to pass through resistors separately.

what is periodic law

the physical and chemical properties of the elements are periodic functions of their atomic numbers

What describes the max number of electrons that can fill a sub shell?

4l +2

If Kc >> 1: A. The equilibrium mixture will favor products over reactants B. The equilibrium mixture will favor reactants over products. C. The equilibrium concentrations of reactants and products are equal. D. The reaction is essentially irreversible

A The larger the value of Keq (whether Kc or Kp), the larger the ratio of products to reactants. Therefore, if Kc>>1, there are significantly larger [ ]s of products than reactants at equilibrium

Ionization energy contributes to an atom's chemical reactivity. Which of the following shows an accurate ranking of ionization energies from lowest to highest? A. first ionization energy of Be < second ionization of Be < first ionization energy of Li B. first ionization energy of Be < first ionization energy of Li < second ionization energy of Li C. first ionization energy of Li < first ionization energy of Be < second ionization energy of Be D. first ionization energy of Li < second ionization energy of Be < first ionization energy of Be

C

What is Newton's second law?

F=ma

The cyclic forms of monosaccharides are: I. hemiacetals II. hemiketals III. acetals

I and II Monosaccharides can exist as hemiacetals or hemiketals, depending on whether they are aldoses or ketoses. When a monosaccharide is in its cyclic form, the anomeric carbon is attached to the oxygen in the ring and a hydroxyl group. Hence, it is only a hemiacetal or hemiketal beacuse an acetal or ketal would require the -OH group to be converted to another -OR group

What is beneficence?

Promoting the well being of individuals and the public (do good) Ex- doing dental screenings

what is conduction?

The direct transfer of heat from one substance to another substance that it is touching.

If the temperature of 2L of gas at constant pressure is changed from 290K to 580K, what would be its final volume?

V1/T1=V2/V2 2L/290K=V2/580K =4L

What is the electron configuration of osmium (Z=76)

[Xe]6s^2 4f^14 5d^6

What are electrochemical cells?

contained systems in which oxidation-reduction reactions occur

Generally speaking, the valence electrons determine the ______ of an atom.

reactivity

If the reduction potential is -0.34V, what is the oxidation potential?

+0.34 V

what is chemoselectivity

-preferential reaction of one functional group in the presence of other functional groups

Aldehydes are generally more reactive than equivalent ketone to nucleophiles. This is likely due to difference in: Steric hindrance Leaving group ability Resonance stabilization Electron-withdrawing character

A

The half-life of carbon-14 is approximately 5730 years, while the half-life of carbon-12 is essentially infinite. If the ratio of carbon-14 to carbon-12 in a certain sample is 25% less than the normal ratio in nature, how old is the sample? A. Less than 5730 years B. Approximately 5730 years C. Significantly greater than 5730 years but less than 11460 years. D. Approximately 11460 years

A Because the half life of carbon-12 is essentially infinite, a 25% decrease in the ratio of carbon-14 to carbon-12 means the same as a 25% decrease in the amount of carbon-14. If less than half of the carbon-14 has deteriorated, then less than one half-life has elapsed. Therefore, the sample is less than 5730 years old. Be careful with the wording here-the question states that the ratio is 25% less than the ratio in nature, not 25% of the ratio in nature, which would correspond to D.

IN an IR spectrum, how does extended conjugation of double bonds affect the absorbance band of carbonyl (C=O) stretches compared with normal absorption? A. The absorbance band will occur at a lower wavenumber B. The absorbance band will occur at a higher wavenumber C. The absorbance band will occur at the same wavenumber D. The absorbance band will disappear.

A Carbonyl groups in conjugation with double bonds tend to absorb at lower wavenumbers because the delocalization of pi electrons causes the C=O bond to lose double-bond character, shifting the stretching frequency closer to C-O stretches. Remember that higher-order bonds tend to have higher absorption frequencies, so loss of double-bond character should decrease the absorption frequency of the group.

Two enantiomers will: A. Have identical IR spectra because they have the same functional groups B. Have identical IR spectra because they have the same specific rotation C. Have different IR spectra because they are structurally different D. Have different IR spectra because they have different specific rotations

A Enantiomers will have identical IR spectra because they have the same functional groups and will therefore have the exact same absorption frequencies. Enantiomers have opposite specific rotations, but specific rotation actually has no effect on the IR spectrum.

Consider the cooling of an ideal gas in a closed system. The process is illustrated in the pressure-volume graph shown in FIGURE 2 (general chemistry). Based on this info, the process may be: A. adiabatic B. isobaric C. isothermal D. isochoric

A Given that the gas was cooled, it did not maintain constant temp, eliminating C. Isobaric and isovolumetric processes appear as horizontal and vertical lines in pressure-volume graphs, respectively, eliminating B and D. Adiabatic processes appear hyperbolic on P-V graphs.

Which reactant could be combined with butanol to form butyl acetate? A. (CH3CO)2O and catalytic acid B. (CH3CH2CO)2O and catalytic acid C. Ch3Ch2CONH2 and catalytic acid D. CH3CONH2 and catalytic acid

A In order to prepare butyl acetate from butanol, we need to perform a nucleophilic acyl substitution reaction. If the product is an ester, we need to start with a reactant that is more reactive than the ester itself, or the reaction will not proceed. Anhydrides are more reactive than esters, but amides are less reactive, eliminating C and D. Reaction with propanoic anhydride, as in B, would result in butyl propanoate.

IR spectroscopy is most useful for distinguishing: A. double and triple bonds B. C-H bonds C. chirality of molecules D. relative percentage or enantiomers in mixtures

A Infrared spectroscopy is most useful for distinguishing between different functional groups. Almost all organic compounds have C-H bonds so except for fingerprinting a compound, these absorptions are not useful. Little info about the optical properties of a compound can be obtained by IR spectroscopy

Consider the following data: Hg2+ + 2e- --> Hg Ered= +0.85V Cu+ + e- --> Cu Ered= +0.52V Zn2+ + 2e- --> Zn Ered= -0.76V Al3+ + 3e- --> Al Ered= -1.66V The anode of a certain galvanic cell is composed of copper. Which of the metals from the data table can be used at the cathode, assuming equal concentrations of the two electrolytic solutions? A. Hg B. Cu C. Zn D. Al

A Oxidation occurs at the anode, and reduction occurs at the cathode. Because Cu is the anode, it must be oxidized. The reduction potential of the cathode cannot be less than that of the anode for a galvanic cell. Therefore, mercury, A, must be the cathode. In a [ ] cell, the same material is used as both the cathode and the anode; however, this question assumes equal [ ]s. If both electrolyte solutions have the same [ ], there will be no oxidation reduction reaction and, therefore, no anode or cathode, this eliminates B.

If propanamide were treated with water, what products would be observed? A. Propanamide B. Propanoic acid C. Equal concentrations of propanamide and propanoic acid D. Propyl propanoate

A Propanamide is an amine; as such, it is the least reactive of the carboxylic acid derivatives. Without strong acid or base, propanamide will not be able to undergo nucleophilic acyl substitution and no reaction would occur.

What is the minimum value of 2 cosθ- 1? A. -3 B. -2 C. -1 D. 0

A The minimum value of the cosine function is -1 (cos 180 = -1). Therefore, the minimum value of 2 cosθ -1 is 2 x (-1) -1 = -3

Which of the following would be the most appropriate setup for estimating the value 3.6 x 4.85 for questions in which answer choices differ by a small margin? A. 3.5 x 5 B. 3.5 x 4.5 C. 4 x 4 D. 4 x 5

A When estimating the product of two numbers, it is best to round one up while rounding the other down.

successfully converting 3-phenylpropanol to 3-phenylpropanoic acid by Jones oxidation requires the oxidizing agent, the solvent, and: A. dilute sulfuric acid B. dilute sodium hydroxide C. anhydrous conditions D. high temperature

A Acidic conditions, provided by dilute sulfuric acid, are required to complete the Jones oxidation. This reaction is carried out in aqueous conditions, eliminating C. While heat may speed up the reaction, high temps are not required for this reaction, eliminating D.

The gas eluent in gas chromatography and the liquid eluent in paper chromatography are examples of which component of these systems? A. Stationary phase B. Mobile phase C. Column D. Fraction

B Each of these is the mobile phase, in which the solutes are dissolved and move. The stationary phase in gas chromatography is usually a crushed metal or polymer; the stationary phase in paper chromatography is paper.

The atomic weight of hydrogen is 1.008 amu. what is the percent composition of hydrogen by isotope assuming that hydrogens only isotopes are 1H and 2D A. 92% H , 8% D B. 99.2% H , 0.8% D C. 99.92% H , 0.08% D D. 99.992% H ,0.008% D

B H + D = 1 (precent H + percent D = 100%) 1 H + 2 D = 1.008 (atomic weight calc) (1-D) + 2D = 1.008 or D=0.008 0.008 is 0.8%, so there is 0.8% D

What is the final product of the following reaction? CH3(CH2)4CH2OH-----> react with CrO3, H2SO4 and acetone A. CH3(CH2)4CHO B. CH3(CH2)4COOH C. CH3(CH2)4CH3 D. HOOC(CH2)4COOH

B Jones reagent (chromium trioxide in aqueous sulfuric acid) is an oxidizing agent. As such, it oxidizes primary alcohols directly to carboxylic acids. This reagent is too strong an oxidant to give an aldehyde so (A) is incorrect; remember that pyridinium chlorochromate (PCC) is a common oxidizing agent used to convert alcohols to aldehydes without progressing to a carboxylic acid. (D), a dicarboxylic acid, cannot form because there is no functional group on the other end of the molecule for the reagent to attack, and it cannot attack an inert alkane. (C) represents reduction, not oxidation.

The p character of the bonds formed by the carbon atom in HCN is: A. 25% B. 50% C. 67% D. 75%

B The carbon bond in hydrogen cyanide is triple bonded, and b/c triple bonds require two unhybridized p-orbitals, the C must be sp-hybridized; sp-hybridized orbitals have 50% s character and 50% p character

Pi bonds are formed by which of the following orbitals? A. two s-orbitals B. two p-orbitals C. one s- and one p-orbital D. two sp2 hybridized orbitals

B pi bonds are formed by the parallel overlap of unhybridized p orbitals.

Order the following compounds by increasing boiling point: butane, butanol, butanone A. butanol < butane < butanone B. butane < butanone < butanol C. butanone < butane < butanol D. butane < butanol < butanone

B Assuming the length of the carbon chain remains the same, the alkane consistently has the lowest boiling point. The boiling point of the ketone is elevated by the dipole in the carbonyl. The boiling point of the alcohol is elevated further by hydrogen bonding

Although the octet rule dictates much of molecular structure, some atoms can violate the octet rule by being surrounded by more than eight electrons. Which of the following is the best explanation for why some atoms can exceed the octet.? A. atoms that exceed the octet already have eight electrons in their outermost electron shell B. Atoms that exceed the octet only do so when bonding with transition metals C. Atoms that exceed the octet can do so because they have d-orbitals in which extra electrons can reside D. Some atoms can exceed the octet because they are highly electronegative

C

If the methyl groups of butane are 120 degrees apart as seen in a Newman projection this molecule is in its: A. highest-energy gauche form B. lowest-energy staggered form C. middle-energy eclipsed form D. highest-energy eclipsed form

C

Which of the following correctly lists methanol, isobutyl alcohol, and propanol by decreasing boiling point? A. methanol > isobutyl alcohol > propanol B. Isobutyl alcohol > methanol > propanol C. Isobutyl alcohol > propanol > methanol D. methanol > propanol > isobutyl alcohol

C All else being equal, BPs increase with increasing size of the alkyl chain because of increased van Der Waals attractions. Isobutyl alcohol has the largest alkyl chain and will thus have the highest boiling point; methanol has the smallest chain and will thus have the lowest BP

Which of the following is considered a tautomer of the imine functional group? A. cyanohydrin B. hydrazine C. enamine D. semicarbazone

C All of the answer choices are nitrogen-containing functional groups, but only enamines are tautomers of imines. Imines contain a double bond between a C and a N; enamines contain a double bond between two carbons as well as an amine.

The acid-catalyzed conversion of propyl ethanoate to benzyl ethanoate is likely: A. reduction B. hydrolysis C. transesterification D. oxidation

C As far as we cal tell, we are converting one ester to another in this reactino. The fact that this reaction is acid catalyzed should confirm the suspicion that this is a trans-esterification reaction

Cuprous

Cu+

Fluoride

F-

Hydride

H-

NADH is a coenzyme that releases high-energy electrons into the electron transport chain. It is known as nicotinamide adenine dinucleotide or diphosphopyridine nucleotide. What functional groups exist in this molecule? I. Phosphate II. Amide III. Anhydride

I and II only

Which phases of solvent and solute can form a solution? I. Solid solvent, gaseous solute II. Solid solvent, solid solute III. Gaseous solvent, gaseous solute

I, II, and III.

Which of the following statements is true of movement on a plane with friction? I. Acceleration is a function of applied force only II. More force is needed to accelerate a stationary object than an identical moving object III. The force of friction is independent of the mass of objects

II only

What is a single displacement reaction?

a reaction in which one element takes the place of another element in a compound Ex: Cu (s) + AgNO3 (aq) --> Ag (s) + CuNO3 (aq)

What is a combustion reaction?

a special type of reaction that involves a fuel (usually a hydrocarbon) and an oxidant (usually oxygen). In its most common form, these reactants form the two products of CO2 and H2O. Ex: CH4 + 2O2 --> CO2 + 2H2O

What is a titration?

a technique where a solution of known concentration is used to determine the concentration of an unknown solution

what is a zwitterion?

a zwitterion is a species with both a positive and a negative charge

When Lewis acids and bases interact, they form ______ bonds

coordinate covalent bonds

Electrolytes are what?

solutes that enable solutions to carry currents

What are aqueous solutions?

solutions in which water is the solvent

what is respect for persons ?

the need for honesty between the subject and the researcher and generally-but not always-prohibits deception. Also includes informed consent.

what is the ion product (IP)

the reaction quotient, Q.

When capacitors are connected in parallel, what happens to the total capacitance?

the resultant capacitanc eis equal to the sum of the individual capacitances. Therefore, Cp increases as more capacitors are added.

What is rotational motion?

when forces are applied against an object in such a way as to cause the object to rotate around a fixed pivot point (fulcrum) Application of force at some distance from the fulcrum generates torque (the moment of force). The distance between the applied force and the fulcrum is termed the lever arm

what is thermal equilibrium?

when heat transfer stops because both substances are the same temperature (balanced)

What is rectilinear propagation?

when light travels through a homogeneous medium, it travels in a straight line

What is work?

work is not actually a form of energy itself, but a process by which energy is transferred from one system to another - it is one of only 2 ways energy can be transferred. the other is heat.

REFER TO FIGURE 16 ORGANIC CHEMISTRY 1. What would be the effect on the Rf values if thin-layer chromatography (TLC) were run with hexane rather than ether as the eluent? A. No effect B. Increase tenfold C. Double D. Decrease 2. If these compounds were separated by column chromatography with ether on silica gel, which would elute first? A. Benzyl alcohol B. Benzyl acetate C. p-Nitrophenol D> 1-Naphthalenemethanol

1. D Hexane is less polar than ether and, therefore, is less likely to displace polar compounds adsorbed to the silica gel. This would decrease the distance these polar compounds would travel, decreasing Rf values. 2. D In column chromatography, as in TLC, the less polar compounds travel most rapidly. This means that 1-naphthalenemethanol, with the highest Rf value, would travel most rapidly and would be the first to elute from the column

Which of the following outliers would most likely be the easiest to correct? A. A typographical error in data transfer B. A measurement error in instrument calibration C. A heavily skewed distribution D. A correctly measured anomalous result

A

Which of the following will convert a cyclic acetal to a carbonyl and a dialcohol? A. aqueous acid B. LiAlH4 C. CrO3 D. Acetone

A

How might succinic anhydride, shown in FIGURE 11 ORGANIC CHEMISTRY, be formed from succinic acid (butanedioic acid) A. Catalytic acid B. Catalytic base C. Heat D. Oxidation

C Anhydrides, particularly cyclic anhydrides, will form spontaneously from dicarboxylic acids when heated

Which of the following does NOT show optical activity? A. (R)-2-butanol B. (S)-2-butanol C. a solution containing 1M (R)-2-butanol and 2 M (S)-2-butanol D. A solution containing 2M (R)-2-butanol and 2M (S)-2-butanol

D

Which of the following species is represented by the electron configuration 1s^2 2s^2 2p^6 3s^2 3p^6 4s^1 3d^5 I Cr II Mn+ III Fe2+ A. I only B. I and II C. II and III D. all

D

Why do halogens often form ionic bonds with alkaline earth metals? A. The alkaline earth metals have much higher electron affinities than the halogens B. By sharing electrons equally, the alkaline earth metals and halogens both form full octets C. Within the same row, the halogens have smaller atomic radii than the alkaline earth metals D. The halogens have much higher electron affinities than the alkaline earth metals.

D

If the half life of a certain isotope is 4 years, what fraction of a sample of that isotope will remain after 12 years?

If 4 years is one half life, then 12 years is 3 half lives. During the first half life-the first 4 years- half of the sample will decay. During the second half-life (years 4 to 8), half of the remaining half will decay, leaving one-fourth of the original. During the third and final half-life (years 8 to 12), half of the remaining fourth will decay, leaving one-eighth of the original sample. Thus, the fraction remaining after 3 half lives is (1/2)^3=1/8.

What is a chiral object?

an object is considered chiral if its mirror image cannot be superimposed on the original object, this implies that the molecule lacks an internal plane of symmetry (ex: hands) chiral objects like forks can be superimposed

Gravitational potential energy depends on what?

an object's position with respect to some level identified as a datum (ground or the zero potential energy position). This zero potential energy level is usually chosen for convenience

Iron (III)

Fe3+

Nitrite

NO2-

How is an object in uniform circular motion kept moving in a circle instead of along a tangent?

by a centripetal force, which points radially inward. centripetal force generates centripetal acceleration. It is this centripetal acceleration that keeps an object in its circular pathway.

What is cellulose and what is its function in the body?

cellulose is the main structural component of plants. A homopolysaccharide, cellulose is a chain of B-D-glucose molecules linked by B-1,4 glycosidic bonds, with hydrogen bonds holding the actual polymer chains together for support. Human's cant digest cellulose b/c we lack the cellulase enzyme responsible for hydrolyzing cellulose to glucose monomers. Therefore, cellulose found in fruits and veggies serves as a great source of fiber in our diet, drawing water into the gut.

cellulose is a polymer of ______ linked what? Starches are polymers of _____ linked what?

cellulose- 1,4 linked B-D-glucose starch- 1,4 linked a-D-glucose

What are the two types of spherical mirrors? What are the differences? What are the two parts of spherical mirrors.

concave and convex If you look from the inside of a sphere to its surface, you would see a concave surface. Concave is like looking into a cave. For a concave surface, the center of curvature and the radius of curvature are located in front of the mirror. If you look from the outside of a sphere, you see a convex surface. For a convex surface, the center of curvature and radius of curvature are located behind the mirror. Concave mirrors are called converging mirrors and convex mirrors are called diverging mirrors because they cause parallel incident light rays to converge and diverge after they reflect, respectively. The word spherical implies that the mirror can be considered a spherical cap or dome taken from a much larger spherically shaped mirror. Spherical mirrors have an associated center of curvature (C) and a radius of curvature (r). The center of curvature is a point on the optical axis located at a distance equal to the radius of curvature from the vertex of the mirror; in other words, the center of curvature would be the center of the spherically shaped mirror if it were a complete sphere.

Esterification is a __________ reaction with what as a side product

condensation reaction with water as a side product

3-methylpentane and hexane are related in that they are.. A. enantiomers D. diastereomers C. meso compounds D. the same molecule

constitutional isomers

What is a common method for supplying energy for nonspontaneous reactions?

coupling them with spontaneous ones.

What is a chair flip in cyclohexane?

cyclohexane can undergo a chair flip in which one chair form is converted to the other. In this process, the cyclohexane molecule briefly passes through a fourth conformation called the half-chair conformation. After the chair flip, all axial groups become equatorial and all equatorial groups become axial. All dashes remain dashes and all wedges remain wedges.

materials consisting of atoms that have only paired electrons will be slightly repelled by a magnetic field and are said to be ______.

diamagnetic

ionic bonds form between atoms that have significantly different __________. The atom(metals or nonmetals) that loses the electrons becomes a(n)_________. The atom(metals or nonmetals) that gains the electrons becomes a(n)_______.

different electronegativities meTals-loses- caTion- posiTive Nonmetals- gains-aNion- Negative

What charge do protons and electrons have?

e=1.6 x 10^-19 C protons are positively charged (q=+e) electrons are negatively charged (q=-e)

what is polyvalence

each mole of the acid or base liberates more than one acid or base equivalent

What is the Lorentz force?

eesum of the electrostatic and magnetic forces acting on a body

When a spring is stretched or compressed from its equilibrium length, the spring has what?

elastic potential energy

Explain endothermic vs exothermic processes.

endothermic processes- system absorbs heat exothermic processes- system releases heat

What is a filtration? Explain a gravity filtration and a vacuum filtration?

filtration isolates a solid from a liquid. In the chem lab, one pours a liquid-solid mixture onto a paper filter that allows only the solvent to pass through, much like a coffee filter. At the end of filtration, one is left with the solid, called the residue, and the flask full of liquid that passed through the filter, known as the filtrate. Filtration can be modified depending on whether the substance of interest is the solid or is dissolved i nthe filtrate. Gravity filtration, in which the solvent's own weight pulls it through the filter, is more commonly used when the product of interest is in the filtrate. Hot solvent is generally used to keep the product dissolved in liquid. Vacuum filtration, in which the solvent is forced through the filter by a vacuum connected to the flask, is more often used when the solid is the desired product.

What is the difference between first ionization energy and second ionization energy? Which will be larger?

first is the energy necessary to remove the first electron, second is the energy necessary to remove the second electron, and so on. second will be larger

What is the pH of a solution made from 1L of 0.05M acetic acid (CH3COOH, Ka=1.8E-5- mixed with 500mL of 1M acetate (CH3COO-)?

for math see pg 374 of general chemistry Nf,ch3coo- = (1N)(0.5L)/(1.5L)=0.33N pH=pKa + log[A-]/[HA] = -log[1.8E-5] + log 0.33N/0.033N pH = approx 4.82 + 1 = 5.82 (actual=5.74)

what is the standard enthalpy of formation of a compound? what about the standard enthalpy if of a reaction?

formation: the enthalpy required to produce one mole of a compound from its elements in their standard states reaction: the enthalpy change accompanying a reaction being carried out under standard conditions. This can be calculated by taking the difference between the sum of the standard heats of formation for the products and the sum of the standard heats of formation of the reactants

explain metals

found on the left side and in the middle of the periodic table. include the active metals, the transition metals, and the lanthanide and actinide series of elements - metals are lustrous (shiny) solids, except for mercury, which is a liquid under standard conditions. - They generally have high mending points and densities. - metals can be deformed without breaking. the ability of metal to be hammered into shapes is called malleability, and its ability to be pulled or drawn into wires is called ductility. At the atomic level, a metal is defined by a low effective nuclear change, low electronegativity, large atomic radius, small ionic radius, low ionization energy, low electron affinity. (metals easily give up electrons) -b/c the valence electrons of all metals are only loosely held to their atoms, they are free to move, which makes metals good conductors of heat and electricity

explain nonmetals

found predominantly on the upper right side of the periodic table. - generally brittle in the solid state and show little or no metallic luster - high ionization energies, electron affinities, and electronegativities, as well as small atomic radii and large ionic radii -poor conductors of heat and electricity - inability to easily give up electrons

what is equipoise?

genuine uncertainty about which arm of a clinial trial will provide the most benefits or harms to patients, research subjects, or future patients in studies comparing two potential treatment options, one cannot approach the research with the knowledge that one treatment is superior to the other.

What is glycogen and what is its function in the body?

glycogen is a carbohydrate storage unit in animals. It is similar to starch, except that it has more a-1,g glycosidic bonds, which makes it a highly branched compound. This branching optimizes the energy efficiency of glycogen and makes it more soluble in solution, thereby allowing more glycose to be stored in the body. Also, its branching pattern allows enzymes that cleave glucose from glycogen, such as glycogen phosphorylase to work on many sites within the molecule simultaneously.

What are the physical properties of aldehydes and ketones?

governed by the presence of the carbonyl group. The dipole of the carbonyl is stronger than the dipole of an alcohol because the double-bonded oxygen is more electron-withdrawing than the single bond to oxygen in the hydroxyl group. In solution, the dipole moments associated with these polar carbonyl groups increase intermolecular attractions, causing an elevation in boiling point relative to their parent alkanes. However, even though aldehyde and ketones have dipoles more polar than those of alcohols, the elevation in BP is less than that in alcohols because no H bonding is present. In reactions, aldehydes and ketones both act as electrophiles, making good targets for nucleophiles. This is due to the electron-withdrawing properties of the carbonyl oxygen, which leaves a partial positive charge on the carbon. Generally, aldehydes are more reactive toward nucleophiles than ketones because they have less steric hindrance and fewer electron-donating alkyl groups.

what are the properties of the transitional metals? what group are they?

groups IB to VIIIB or 3 to 12. metals, low electron affinities, low ionization energies, low electronegativities. hard metals, high melting and boiling points. malleable. good conductors due to loosely held electrons that progressively fill the d-orbitals in their valence shells. many of the transition metals can have different possible charged forms (oxidation states) b/c they are capable of losing different #s of electrons from the s- and d- orbitals in their valence shells. These different oxidation states often correspond to different colors.

how can you tell whether a solution is unsaturated, saturated, or supersaturated?

if a salt's IP < salt's Ksp then the solution is not yet at equilibrium (unsaturated) IP > Ksp (Supersaturated)- solution is beyond equilibrium IP=Ksp (saturated)- solution is at equilibrium

When is a group of outcomes said to be exhaustive?

if there are no other possible outcomes

What are streamlines?

imaginary lines that show the path of a fluid The velocity vector of a fluid particle will always be tangential to the streamline at any point. Streamlines never cross each other.

What is the difference between independent and dependent events>

independent events have no effect on one another. dependent events have an impact on one another

What are configurational isomers?

isomers that can only be interchanged by breaking and reforming bonds two categories are enantiomers and diastereomers

what is the magnetic quantum number?

m (sub l ). specifies the particular orbital within a sub shell where an electron is most likely to be found. Each orbital can hold a max of 2 electrons. The possible values of m (sub l ) are the integers between -1 and +1, inc 0. Ex: s subshell, l=1, m (sub l)=1 (1 orbital) p subshell, l=1, m (sub l )=-1, 0, +1 (3 orbitals) d sub shell, l=2, m (sub l)= -2, -1, 0, 1, 2 (5 orbitals) the orbitals in the s sub shell are spherical p sub shell are dumbbell shaped

How can you determine the magnification of an image? What do the signs mean?

magnification (m) is a dimensionless value that is the ratio of the image distance to the object distance m=i/o The magnification also gives the ratio of the size of the image to the size of the object. The orientation of the image (upright or inverted) can be determined: a negative magnification signifies an inverted image, while a positive value signifies an upright image. If ImI < 1, the image is smaller than the object (reduced); if ImI > 1, the image is larger than the object (enlarged); and if ImI = 1, the image is the same size as the object.

What is the principal quantum number? What is the maximum number of electrons within a shell? The larger the value of the principal quantum number, the larger the _____ and _____ is.

n. (the same number used in Bohr's model). Can be any positive integer (any # greater than 0). The larger the value of n, the higher the energy level and radius of the electron's shell. 2n^2

explain ionic radius trend across a periodic table

nonmetals closer to the metalloid line possess a larger ionic radius than their counterparts closer to Group VIIIA. for metals, the trend is opposite. Metals close to the metalloid line have very small ionic radii compared to ones closer to Group 1A

what are the 5 categories of amino acids

nonpolar nonaromatic, aromatic, polar, negatively charged (acidic), positively charged (basic)

Are the colors that we see absorbed or not absorbed?

not absorbed.

what is a state function? give examples. what about a process function? give examples.

state function- a thermodynamic quantity whose value depends only on the state at the moment ex: pressure, density, temp, volume, enthalpy, internal energy, Gibbs free energy, and entropy process function-describe the path taken to get from one state to another Ex: work and heat

what is the specific heat of a substance?

the amount of heat energy required to raise one gram of a substance by one degree Celsius or one unit Kelvin

what is the standard heat of combustion (deltaHcomb)?

the enthalpy change associated with the combustion of a fuel

How do reaction [ ]s affect the rate of a reaction?

the greater the [ ]s of the reactants, the greater # of effective collisions per unit time, which leads to an inc in the frequency factor (A), per the Arrhenius equation. Therefore, the reaction rate will inc for all but zero-order reaction. For reactions occurring in the gaseous state, the partial pressures of the gas reactants serve as measure of [ ]

what is a photon and how does its energy relate to frequency?

the light beam consists of an integral number of light quanta called photons. Energy of each photon (E)=hf energy increases with increasing frequency

what is the solubility of a substance?

the maximum amount of that substance that can be dissolved in a particular solvent at a given temperature

what is entropy?

the measure of the spontaneous dispersal of energy at a specific temperature: how much energy is spread out or how widely spread out energy becomes in a process.

for all electrochemical cells, the movement of electrons is from what to what and the movement off the current (I) runs from what to what

the movement of electrons is from anode to cathode and the current runs from cathode to anode.

The Roman numeral above each group represents what? Explain A vs B elements.

the number of valence electron elements in that group have in they natural state. A elements (representative elements) include groups IA through VIIIA. The elements in these groups have their valence electrons in the orbitals of either s or p sub shells B elements (non representative elements) include both transition elements, which have valence electrons in the s and d sub shells, and the lanthanide and actinide series, which have valence electrons in the s and f sub shells.

what is electrostatics?

the study of stationary charges and the forces that are created by and which act upon these charges

What is absolute (hydrostatic) pressure?

the total pressure that is exerted on an object that is submerged in a fluid

What are conformational isomers?

these are not true isomers. they are different spatial representations of the same molecule due to spinning around a single bond. eclipsed, staggered and gauche are examples of these. Easiest to see when the molecules are depicted in a Newman projection

What are electromagnetic waves?

transverse waves because the oscillating electric and magnetic field vectors are perpendicular to the direction of propagation.

what are indicators?

weak organic acids or bases that have different colors in their protonated and deportonated states. This small structural change-the binding or release of a proton-leads to a change in the absorption spectrum of the molecule, which we perceive as a color change. Indicators are generally vibrant and can be used in low [ ]s without significantly alterating the equivalence point. The indicator must always be a weaker acid or base than the acid ro base being titrated; otherwise, the indicator would be titrated first. The point at which the indicator changes to its final color is not the equivalence point but rather the endpoint.

when is an atom in an excited state?

when at least one electron has moved to a sub shell of higher than normal energy

What is saponification and how does it happen?

when long-chain carboxylic acids react with sodium or potassium hydroxide, a salt is formed. This process, called saponification, occurs by mixing fatty acids with lye (sodium or potassium hydroxide), resulting in the formation of a salt that we know as soap.

what is a saturated solution? What happens if more solute is added?

when the maximum amount of solute has been added and the dissolved solute is in equilibrium with its undissolved state. If more solute is added it will not dissolve, instead it will remain in solid form, precipitating to the bottom of the container.

How would you use the Doppler equation if you are driving down the street and hear an ambulance approaching from behind

you are the detector and the ambulance is the sound source. At this time, you are driving AWAY from the ambulance, so you use the lower sign (-). The driver of the ambulance is driving TOWARD you, so use the upper sign (-) f'=f(v-VsubD)/(v-Vsubs)

what is the % composition by mass of a salt water oslution if 100g of the solution contains 20g of NaCl?

(mass of solute/mass of solution)x 100% (20g/100g)x 100% = 20% NaCl solution

A scientist takes a 0.5 g/ml solution of an unknown use dextrorotatory organic molecule and places it in a test tube with a diameter of 1cm. He observes that the plane of polarized light is rotated 12 degrees under these conditions. What is the specific rotation of this molecule?

+240 degrees alpha=Aobs/ (c)(l) = +12/(0.5g/ml)(o.1dm)

what are the characteristic physical properties of ionic compounds? - in the solid state, the ionic constituents of the compound form what? what is this thing?

- very high melting and boiling points- b/c of the strength of the electrostatic force between the - - - ionic constituents of the compound - dissolve readily in water or other polar solvents good conductors of electricity - form a crystalline lattice consisting of repeating positive and negative ions

The combustion of octane is a common reaction in engines: 1) 2C8H18 (l) + 25O2 (g) --> 16CO2 (g) + 18H2O (g) delta H1 = -250kJ/mole 2) 18H2O (l) -->18H2O (g) delta H2 = 88kJ/mol Net Reaction: 3) 2C8H18 (l) + 25O2 (g) --> 16CO2 (g) + 18H2O (1) What is the delta H?

-338kJ/mole

What are the three causes of outliers?

1. A true statistical anomaly 2. A measurement error 3. A distribution that is not approximated by the normal distribution

If 4E-4 moles of gas are dissolved in 2L of solution under an ambient pressure of 2 atm, what will be the molar [ ] of the gas under 10 atm?

10^-3 M Start by determining the initial [ ] of the gas in solution. [A]1=4E-4mol/2L=2E-4M Next, utilize the direct relationship between solubility and pressure according to Henry's law. [A]1/P1=[A]2/P2 2E-4M/2atm=[A]2/10atm

Determine the number of protons, neutrons, and electrons in a nickel-58 atom and in a nickel-60 +2 cation.

58Ni has an atomic number of 28 and a mass number of 58. Therefore, it will have 28 protons, 28 electrons, and 58-28 (30) neutrons. 60Ni2+ has the same number of protons as the neutral 58Ni atom. However it has a + charge because it has lost 2 electrons. It will have 26 electrons. The mass number is 2 units higher than the 58Ni atom, due to 2 extra neutrons. So it has 32 neutrons.

3 moles of N2O4 is placed in a 0.5L container and allowed to reach equilibrium according to the following reaction: N2O4(g)--> 2NO2(g) What is the equilibrium [ ] of NO2, given Keq for the reaction is 6E-6?

6E-3M Keq=[NO2]^2/[N2O4] starting [ ] of N2O4: (3M/0.5L)=([N2O4]/1L) [N2O4]=6M Using x to represent the amount of N2O4 that reacts, 2x to represent the amount of NO2 that is produced.... 6E-6=[2x]^2/[6-x] which is roughly equal to [2x]^2/6 x=3E-3M The value of x represents the amount of N2O4 that reacts. the final answer is the amount of NO2 produced, which is twice the amount of N2O4 that reacts, or 2x.

A student making a coffee cup calorimeter fails to use a second coffee cup and inadequately seals the lid. What was her initial goal, and what was the result of this mistake? A. She was trying to create an isolated system but created an open system instead B. She was trying to create an isolated system but created a closed system instead C. She was trying to create a closed system but created an open system instead D. She was trying to create a closed system but created an isolated system instead

A

For a cell with the following half reactions: Anode: So2 + 2H2O --> So4^2- + 4H+ + 2e- Cathode: Pd2+ + 2e- --> Pd How would decreasing the pH of the solution inside the cell affect the emf? A. The emf would decrease B. The emf would remain the same C. The emf would increase D. The emf would become zero

A A change in pH has a direct correlation to the hydrogen ion (H+) [ ]. Decreasing the pH increases the H+ [ ], which means that the [ ] of products has increased in the oxidation of sulfur dioxide. This means it would be harder to liberate electrons, thus decreasing the emf. Once could also view this decrease in oxidation potential as an increase in reductino potential. If Ered,anode increases, the Ecell must decrease according to Ecell=Ered,cathode-Ered,anode

Which of the following statements best describes the effect of lowering the temperature of the following reaction: A = B <--> C + D deltaH=-1.12 kJ/mol A. [C] and [D] would increase B. [A] and [B] would increase C. delta H would increase D. delta H would decrease

A A negative deltaH value indicates an exothermic reaction, meaning that the forward reaction produces heat. Visualize this as follows: A + B <--> C + D + heat This means that removing heat by decreasing the temp is similar to removing any other product of the reaction. To compensate for this loss, the reaction will shift to the right, causing an inc in the [ ]s of C and D and a dec in the [ ]s of A and B

Which of the following hybridizations does the Be atom in BeH2 assume? A. sp B. sp2 C. sp3 D. sp3d

A Beryllium has only 2 electrons in its valence shell. When it bonds to 2 hydrogens, it requires two hybridized orbitals, meaning that its hybridization must be sp. Note that the presence of only single bonds does not mean that the hybridization must be sp3; this is a useful assumption for C, but does not apply to beryllium b/c of its smaller number of valence electrons.

An object is placed at the center of curvature of a concave mirror. Which of the following is true about the image? A. It is real and inverted B. It is virtual and inverted C. It is real and upright D. It is virtual and upright

A One could solve this question with a ray diagram, but be wary about using these on Test Day. it is easy to make small mistakes. So solve the question using the sign convention. If the object is at the center of curvature, its distance is 2f. We can plug into the optics equation: 1/f=1/o+1/i --> 1/i=1/f-1/o=1/f=1/2f=1/2f i=2f=r because i is positive, the image is real. For single mirrors or lenses, all real images are inverted.

In which of the following compounds is the percent composition of carbon by mass closest to 62%? A. Acetone B. Ethanol C. Propane D. Methanol

A Acetone C3H6O (3 x 12g/ mol C) / [(3 x 12 g/ mol C) + (6 x 1 g/ mol H) + (16 g/ mol O)] = 36g/molC / 58 g/ mol total = 66.7 approx

What is the gram equivalent weight of H2SO4 with respect to protons?

A It's helpful to know the molar mass of one mole of H2SO4. (2 x 1 g/mol H) + (1 x 32.1 g/mol S) + (4 x 16 g/ mol O) = 98.1 g/ mol H2S04 Gram equivalent weight is the mass that would release one mole of protons. Because sulfuric acid has two hydrogens per molecule, the gram equivalent weight is 98.1/2= 49.1

What is a neutralization reaction?

A double displacement reaction between an acid and a base in an aqueous solution. acid + base --> salt + water

what is a buffer solution?

A system that minimises pH changes when small amounts of an acid or a base are added. a mixture of a weak acid and its salt (which is composed of its conjugate base and a cation) or a mixture of a weak base and its salt (which is composed of its conjugate acid and an anion).

A consumer is comparing two new cars. Car A exerts 250 horsepower, while Car B exerts 300 horsepower. The consumer is most concerned about the peak velocity that the car can reach. If nonconservative forces can be ignored, which of the following statements is true? (1 horsepower = 745.7W) A. Car A and Car B both have unlimited velocities B. Car A will reach its peak velocity more quickly than Car B C. Car A will dissipate less energy to the surroundings than Car B D. Car A will have a lower peak velocity than Car B

A. Horsepower is a unit of power. Power is a rate of energy expenditure over time. Given the unlimited time, both care are capable of unlimited increases in kinetic energy, meaning that they have unlimited maximum velocities.

The concentrations of all reactants in a zero-order reaction are increased two-fold. What is the new rate of the reaction? A. It is unchanged B. It is decreased by a factor of 2 C. It is increased by a factor of 2 D. It cannot be determined from the information given

A. By definition, zero-order reactions are unaffected by the [ ]s of any reactants in the reaction.

What functional groups act as acids? Bases?

Acids: alcohols, aldehydes, ketones (at the alpha carbon), carboxylic acids, most carboxylic acid derivatives Bases: amides and amines

Assign oxidation numbers to the atoms in the following reaction to determine the oxidizing and reducing agents. SnCl2 + PbCl4 --> SnCl4 + PbCl2

All of these species are neutral, so the oxidation numbers of each compound must add up to zero. In SnCl2, tin must have an oxidation number of +2 because there are two chlorines present and each chlorine has an oxidation number of -1. Similarly, the oxidation number of Sn in SnCl4 is +4; the oxidation number of Pb is +4 in PbCl4 and +2 in PbCl2. The oxidation number of Sn goes from +2 to +4; it loses electrons and thus is oxidized, making it the reducing agent. Because the oxidation number of Pb has decreased from +4 to +2, it gains electrons and is reduced, making it the oxidizing agent. The sum of the charges on both sides of the reaction is equal to zero, so charge has been conserved. Keep in mind that oxidation state also plays a role in nomenclature; the reactants in this reaction would be called tin(II) chloride and lead(IV) chloride.

In Michaelis-Menten enzyme kinetics, what is the velocity of the reaction when the substrate concentration is equal to Km? A. 1/4 Vmax B. 1/2 Vmax C. Vmax D. 2Vmax

B

Mechanical advantage and efficiency are both radios. Which of the following is true regarding the quantities used in these ratios? A. Mechanical advantage compares values of work; efficiency compares values of power. B. Mechanical advantage compares values of forces; efficiency compares values of work. C. Mechanical advantage compares values of power; efficiency compares values of energy D. Mechanical advantage compares values of work; efficiency compares values of forces.

B

Molecular orbitals can contain a maximum of: A. one electron B. two electrons C. four electrons D. 2n^2 electrons, where n is the principal quantum number of the combining atomic orbitals

B

What is the character of the bond in carbon monoxide? A. Ionic B. Polar covalent C. non polar covalent D. coordinate covalent

B

Similarities in chemical behavior can be observed between elements within the same family in the periodic table. For instance, sodium and potassium react with oxygen to produce a metal oxide. Which of the following gives the balanced reaction for the oxidation of an alkali metal (M)? A. 2M(s) + O2(g) → 2MO(s) B. 4M(s) + O2(g) → 2M2O(s) C. 3M(s) + O2(g) → M3O2(s) D. M(s) + O2(g) → MO2(s)

B Sodium and potassium are congeners found in the first column of the periodic table. They are included in the alkali metal family and exhibit similar chemical reactivity. In the presence of oxygen, alkali metals are oxidized. To form a complete octet, alkali metals lose one valence electron and take on their preferred oxidation state of +1. Oxygen, on the other hand, completes its octet by gaining two electrons to form an anion with a charge of -2. Thus, in the reaction above, two metal (M) atoms must each lose one electron in order for each oxygen atom to gain two electrons. Of the reactions above, B is the only choice where the metal (M) has a charge of +1 and oxygen had a charge of -2 with a 2:1 coefficient ratio of metal to oxygen. Thus, B is the correct answer for the oxidation of alkali metals.

Why does a sound wave travel through air at a speed of roughly 767mph and through water at a speed of roughly 3300mph? A. Water is less dense than air. B. Water has a greater bulk modulus than air. C. The wave changes phase as it changes medium. D. Sound is a transverse wave.

B The speed of sound is proportional to the square root of the bulk modulus and inversely proportional to the square root of the density of the medium. Although the density of water is much greater than air, the bulk modulus of water is also greater than air. Answer choice b) is correct. A. Water is less dense than air. Incorrect, Water has greater density than air. B. Water has a greater bulk modulus than air. Correct. C. The wave changes phase as it changes medium. Incorrect, Although beyond the scope of the MCAT, the phase of a wave refers to a horizontal offset of the waveform that affects neither amplitude nor frequency. A phase change would not affect wave speed. D. Sound is a transverse wave. Incorrect. Sound is a longitudinal wave, and the type of wave does not affect the speed of propagation.

A 95% confidence interval will fall within what distance from the mean? A. +/-σ B. +/-2σ C. +/-3σ D. +/-4σ

B Approx 95% of values fall within two standard deviations of the mean for a normal distribution. A confidence interval is constructed using the same values. Approximately 68% of the values are within one standard deviation, and 99% are within three standard deviations, eliminating the other answer choices.

What is the gram equivalent weight of phosphoric acid? A. 24.5 g B. 32.7 g C. 49 g D. 98 g

B Gram equivalent weight is the weight (in grams) that releases 1 acid or base equivalent from a compound. Because H3PO4 contains 3 protons, we find the gram equivalent weight by dividing the mass of one mole of the species by 3. The molar mass of phorphoric acid is 98g/mol, so the gram equivalent weightis 32.7g.

The following titration curve in FIGURE 18 PHYSICS is an example of: A. a sigmoidal relationship on a log-log graph B. a sigmoidal relationship on a linear graph C. A logarithmic relationship on a semilog graph D. A logarithmic relationship on a log-log graph

B Sigmoidal curves are S-shaped. Because the axes have the same scale throughout, this is a linear graph. Note that even though the y-axis represents logarithmic changes in H+ [ ] (pH=-log[H+]), the actual unit that is used is pH points, which increase linearly in this graph.

The reaction below is an example of: FIGURE 3 (organic chemistry)? A. esterification B. tautomerization C. elimination D. dehydration

B Tautomerization is the interconversion of two isomers in which a hydrogen and a double bond are moved. The keto and enol tautomers of aldehydes and ketones are common examples of tautomers seen on Test Day. Note that the equilibrium lies to the left because the keto form is more stable.

At physiological pH, which two forms of phosphoric acid have the highest concentrations? A. H3PO4 and H2PO4- B. H2PO4- and HPO4^2- C. HPO4^2- and PO4^3- D/ PO4^3- and H3PO4

B The pKa of phosphoric acid is close to physiological pH therfore [H2PO4-]=[HPO4^2-] at this pH

The reaction of formic acid with sodium borohydride will yield what final product? A. An aldehyde B. A carboxylic acid C. A ketone D. An alcohol

B The reaction of formic acid, which is a simple carboxylic acid, with sodium borohydride, which is a mild reducing agent, will result in no reaction, and therefore will result in maintenance of the carboxylic acid. Sodium borohydride is too mild to reduce carboxylic acids, and therefore cannot produce the primary alcohols that lithium aluminum hydride, a strong reducing agent, would.

A gas at a temp of 27C has a volume of 60mL. What temp change is needed to increase this gas to a volume of 90mL A. A reduction of 150C B. An increase of 150C C. A reduction of 13.5C D. An increase of 13.5C

B Use Charles' law. First, we must convert the temp to K by adding 273 to get 300K as the initial temp. Think of this as a proportionality: If the volume is multiplied by 3/2, the final temp will also have to be multiplied by 3/2. Thus the final temp is 450K, which represents a 150K increase (which is equivalent to an inc of 150C)

Wavenumber is directly proportional to: A. wavelength B. frequency C. percent transmittance D. absorbance

B Wavenumber (1/wavelength) is directly proportional to frequency (c/wavelength).

Treatment of (S)-2-bromobutane with sodium hydroxide results in the production of a compound with an (R) configuration. This reaction has most likely taken place through: Sn1 Sn2 Sn1 & Sn2 mechanism in sequence Sn1 & Sn2 mechanism simultaneously

B inversion of stereochemistry = Sn2

What is the approximate pH of a solution with a pKa of 3.6, [HA] = 100mM, and [A-] = 0.1M? (Note: pH = pKa + log[A-]/[HA]) A. 1.6 B. 3.6 C. 5.6 D. 7.6

B pH=pKa + log [A-]/[HA] = 3.6 + log(1) = 3.6

When a hydrogen atom electron falls to the ground state from the n=2 state, 10.2eV of energy is emitted. What is the wavelength of this radiation? (Note: 1eV=1.6E-19J, and h=6.626E-34Js) A. 5.76E-9m B.1.22E-7m C. 3.45E-7m D. 2.5E15m

B solutions on page 340-341 physics 10.2eV (1.6E-19J/eV) = 1.6E-18J Next, to determine the wavelength of the radiation, we can combine the formulas E=hf and c=fλ: E=hf=hc/λ --> λ=hc/E =(6.626E-34Js)(3E8m/s)/(1.6E-18J) =(4)(3)(10^-26)/(10^-18)=12E-8=1.2E-7m

The resistance of two conductors of equal cross-sectional area and equal lengths are compared, and are found to be in the ration 1:2. The resistivities of the materials from which they are constructed must therefore be in what ratio? A. 1:1 B. 1:2 C. 2:1 D. 4:1

B the resistance of a resistor is given by the formula R=pL/A Thus, there is a direct proportionality between resistance and resistivity. Because the other variables are equal between the two resistors, we can determine that if R1:R2 is a 1:2 ratio, then p1:p2 is also a 1:2 ratio

Which of the following best describes the purpose of a catalyst? A. Catalysts are used up in the reaction, increasing reaction efficiency. B. Catalysts increase the rate of the reaction by lowering the activation energy. C. Catalysts alter the thermodynamics of the reaction to facilitate the formation of products or reactants D. Catalysts stabilize the transition state by bring it to a higher energy

B. By definition, a catalyst inc the rate of a reaction by lowering the activation energy, making it easier for both the forward and reverse reactions to overcome this energy barrier. Catalysts are neither used up in the reaction, nor do they alter the equilibrium of a reaction. Catalysts stabilize the transition state by lowering its energy, not raising it.

Element X is radioactive and decays via alpha decay with a half-life of four days. If 12.5% of an original sample of element X remains after n days, what is the value of n? A. 4 B. 8 C. 12 D. 16

C Bet.cause the half-life of element X is four days, 50% of an original sample remains after four days, 25% remains after 8 days, and 12.5% remains after 12 days. Therefore, n=12 days. Another approach is to set (1/2)^x =0.125, where x is the number of half-lives that have elapsed. Solving for x gives x=3. This, 3 half lives have elapsed, and because each half-life is four days, we know that n=12 days.

Which of the following phase changes is associated with the largest decrease in entropy? A. Fusion B. Solidification C. Deposition D. Sublimation

C Gases have the highest entropy, and solids have the lowest. Therefore, a phase change from a gas to a solid-deposition-would have the largest decrease in entropy of any phase change process

Which of the following molecules could be classified as a soap? A. CH3(CH2)19COOH B. CH3COOH C. CH3(CH2)19COO-Na+ D. CH3COO-Na+

C Soap is a salt of a carboxylate anion with a long hydrocarbon tail. (A) and (B) are not salts of anionic compounds. (D) is sodium acetate, which is a salt but does not contain the long hydrocarbon tail needed to be considered a soap.

During gravity filtration, a student forgets to heat the solution before running it through the filter. After capturing the filtrate, the student analyzes the sample via IR spectroscopy and finds none of the desired product in the filtrate. What likely occurred to the student's product? A. The product degraded because of a prolonged filtration time B. The product evaporated with collection of the filtrate C. The product precipitated and is present in the residue D. The product was dissolved in the solvent.

C Warm or hot solvent is generally used in gravity filtration to keep the desired product soluble. This allows the product to remain in the filtrate, which can b=then be collected. In this case, the student likely used solvent that was too cold, and the product crystallized out. The product should be present in the residue.

Each of the following reaction types occurs during the Gabriel synthesis EXCEPT: A. decarboxylation B. nucleophilic substitution C. dehydration D. hydrolysis

C gabriel synthesis includes two nucleophilic substitution steps followed by hydrolysis and decarboxylation. Dehydration is not included

At standard temp and pressure, a chemical process is at equilibrium. What is the free energy of reaction (deltaG) for this process? A. deltaG > 0 B. deltaG < 0 C. deltaG = 0 D. Not enough info

C standard temp and pressure indicate 0C and 1 atm. Gibbs free energy is temp dependent, but if a reaction is at equilibrium, deltaG=0

Cyanide

CN-

How many valence electrons are present in elements in the third period? A. 2 B. 3 C. The # dec as the atomic # inc D. the # inc as the atomic # inc

D

Solution pK1 pK2 pK3 1 2.10 3.86 9.82 2 2.10 4.07 9.47 3 2.32 9.76 Not applicable 4 2.18 9.04 12.48 Which solution contains an amino acid that would be most likely to stabilize an anionic substrate in an enzyme pocket at physiological pH? A. Solution 1 B. Solution 2 C. Solution 3 D. Solution 4

D

Which sample would be the most appropriate participants for a study on hormone replacement therapy for postmenopausal symptoms? A. Prepubescent girls B. Premenopausal adult women C. Pregnant women D. Postmenopausal women

D

The standard reduction potentials for Zn and Cu are given below: Zn2+(aq) + 2e- → Zn(s) E°= -0.76 V Cu+(aq) + e- → Cu(s) E°= 0.52 V What is the standard state cell potential for the following reaction? Zn(s) + 2Cu+(aq) → Zn2+(aq) + 2Cu(s) A. -1.8V B. -1.28V C. 0.28V D. 1.28V

D Given the standard reduction potentials: Zn2+(aq) + 2e- → Zn(s) E°= -0.76 V Cu+(aq) + e- → Cu(s) E°= 0.52 V The reduction potentials are used as is or multiplied by -1 if the reaction direction is reversed. Stoichiometry does not matter, the reduction potentials are summed for the standard cell potential. Zn(s) + 2Cu+(aq) → Zn2+(aq) + 2Cu(s) 0.76V + 0.52V=1.28V -1.8V, incorrect, This uses Zinc in the wrong direction (it should be +0.76V), and copper is also in the wrong direction and multiplied twice. There should be no multiplying for stoichiometric coefficients. -1.28V, incorrect, This uses Zinc in the wrong direction and copper in the wrong direction. 0.28V, incorrect, This uses Zinc in the right direction and Cu in the wrong direction. 1.28V, correct.

Which of the following equations is incorrect? A. A^3 x B^3 = (AB)^3 B. A^5 / A^7 = A^-2 C. (A^0.5)^4 + A^2 = 2A^2 D. (A^3)^2 = A^9

D (A^3)^2 = A^6

Which of the following can alter the emf of an electrochemical cell? A. The mass of the electrodes B. The length of the wire connecting the half-cells C. the overall size of the battery D. The temp of the solutions in the half-cells

D E(not) cell is dependent upon the change in free energy of the system through the equation RT ln Keq= nFE(not)cell. the temp, T, appears in this equation; this a change in temp will impact the E(not)cell

All of the following are true of epimers EXCEPT: A. they differ in configuration about only one carbon B. they usually have slightly different chemical and physical properties C. they are diastereomers (with the exception of glyceraledhyde) D. they have equal but opposite optical activities

D Epimers are monosaccharide diastereomers that differ in their configuration about only one carbon. As with all diastereomers, epimers have different chemical and physical properties, and their optical activities have no relation to each other. Enantiomers have equal but opposite optical activities. Therefore, D is the only statement that does not apply to epimers.

A source of light (f= 6E14 Hz) passes through three plane polarizers. The first two polarizers are in the same direction, while the third is rotated 90 degrees with respect to the second polarizer. What is the frequency of the light that comes out of the third polarizer? A. 3E14 Hz B. 6E14 Hz C. 9E14 Hz D. Light will not pass through the third polarizer

D Plane polarized light is light in which the electric fields of all the waves are oriented in the same direction. Light passing through the first two polarizers will only contain rays with their electric field vectors in the same direction. When it reaches the third polarizer, however, the light will not be able to pass through because all the light rays will be oriented in the direction dictated by the first and second polarizers

Given a solution of insulin (molecular weight=5.8kD) and titin (molecular weight=3816kD), which chromatographic technique wold be the most effective for separating out usable molecules of titin? A. Thin-layer chromatography B. Ion-exchange chromatography C. Affinity chromatography D. Size-exclusion chromatography

D Size-exclusion would remove the smaller insulin molecules into the fraction retained in the column and allow the titin to be eluted. C could also be used but comes with a risk of rendering the titin unusable; the eluent run through an affinity chromatography column often binds to the target molecules.

Consider the following reaction: C6H12O6 (s) + 6 O2 (g) → 6 CO2 (g) + 6 H2O (l) ΔH = - 2808 kJ / mol Under what conditions will this reaction proceed spontaneously? A. Low temperature, low pressure B. Low temperature, any pressure C. High temperature, any pressure D. Any temperature, any pressure

D Spontaneity is determined by a negative ΔG value in Gibbs free energy equation: ΔG = ΔH - TΔS A reaction with a negative ΔH and a positive ΔS will thus be spontaneous under any conditions. The reaction given has a negative ΔH. It also involves converting 1 mole of solid and 6 moles of gas into 6 moles of liquid and 6 moles of gas. That phase change means entropy is increasing (solid to liquid). Thus, with a negative ΔH and a positive ΔS this reaction is spontaneous under any conditions and (D) is the right answer. A, B: Reactions with a negative ΔH and a negative ΔS are spontaneous at low temperatures (e.g. the freezing of water from liquid to solid). C: Reactions with a positive ΔH and a positive ΔS are spontaneous at high temperatures (e.g. the boiling of liquid water into steam)

Which of the following measures of distribution is most useful for determining probabilities? A. Range B. Average distance from mean C. Interquartile range D. Standard deviation

D Standard deviation is the most common measure of distribution. Is is the most closely linked to the mean of a distribution and can be used to calculate p-values, which are probabilities.

Carboxylic acids have higher boiling points than their corresponding alcohols primarily because: A. molecular weight is increased by the additional carboxyl group B. the pH of the compound is lower C. acid salts are soluble in water D. hydrogen bonding is much stronger than in alcohols

D The BP of compounds depend on the strength of the attractive forces between molecules. In both alcohols and carboxylic acids, the major form of intermolecular attraction is hydrogen bonding; however, hydrogen bonding is much stronger in carboxylic acids as compared to alcohols because carboxylic acids are more polar and the carbonyl also contributes to hydrogen bonding in addition to the hydroxyl group. The stronger hydrogen bonds elevate the boiling points of carboxylic acids compared to alcohols. Boiling points also depend on molecular weight, (A), but in this case, the difference in molecular weight is insignificant compared to the effect of hydrogen bonding. (B) and (C) are both true but do not explain the difference in boiling points.

Which of the following statements is inconsistent with the Bohr model of the atom? A. Energy levels of the electron are stable and discrete B. An electron emits or absorbs radiation only when making a transition from one energy level to another C. To jump from a lower energy to a higher energy orbit, an electron must absorb a photon of precisely the right frequency such that the photon's energy equals the energy difference between the two orbits. D. To jump from a higher energy to a lower energy orbit, an electron absorbs a photon of a frequency such that the photon's energy is exactly the energy difference between the two orbits.

D The Bohr model is based on a set of postulates originally put forward to discuss the behavior of electrons in hydrogen. In summary, these postulates state that the energy levels of the electron are stable and discrete, and they correspond to specific orbits, eliminating A. They also state that an electron emits or absorbs radiation only when making a transition from one energy level to another, eliminating B. Specifically, when an electron jumps from a lower-energy orbit to a higher-energy one, it must absorb a photon of light of precisely the right frequency such that the photon's energy equals the energy difference between the two orbits, eliminating C. When falling from a higher-energy orbit to a lower-energy one, an electron emits a photon of light with a frequency that corresponds to the energy difference between the two orbits. This is the opposite of D, which makes it the right answer.

The hybridizations of the carbon and nitrogen atoms in CN- are A. sp3 and sp3 B. sp3 and sp C. sp and sp3 D. sp and sp

D The carbon and nitrogen atoms are connected by a triple bond. A triple-bonded atom is sp hybridized; one s-orbital hybridizes with one p-orbital to form two sp-hybridized orbitals.

If the value of E(not)cell is known, what other data is needed to calculate deltaG? A. Equilibrium constant B. Reaction quotient C. Temperature of the system D. Half-reactions of the cells

D This answer comes directly from the equation relating Gibbs free energy and Ecell. deltaG(not) =-nGE(not)cell.

A 2 L sealed vessel contains 2 moles of nitrogen gas at 0oC. All of the following will decrease the pressure inside the vessel EXCEPT: A. nitrogen leaking out of the tank B. decreasing the temperature of the gas C. expanding the volume to 3 L D. replacing 1 mole of nitrogen gas for 1 mole of oxygen gas

D This question is testing your understanding of gas behavior according to the ideal gas law. PV = nRT When nitrogen leaks out of the tank, the moles of nitrogen, n, decreases. Since pressure is proportional to moles, decreasing the moles of gas will decrease the pressure making A incorrect. Since temperature is also proportional to pressure, a decrease in temperature will lower the pressure making B false. Pressure and volume are inversely related, which means that increasing the volume will result in a decrease in pressure. Therefore, C is not the correct answer, which leaves D as the correct answer. Exchanging 1 mole of nitrogen gas for 1 mole of oxygen changes the partial pressures within the system but has no effect on total pressure.

During the assigning of oxidation numbers, which of the following elements would most likely be determined last? A. Ar B. F C. Sr D. Ir

D When assigning oxidation numbers, one starts with elements of known oxidation state first, and determines the oxidation state of the other elements by deduction. As a noble gas, argon, A, will always have an oxidation state of 0. As a Group VIIA element, fluorine, B, will have an oxidation state of 0 (by itself) or -1 (in a compound). As a Group IIA element, strontium, C, will have an oxidation state of 0 (by itself) of +2(in a compound). Like most transition metals, iridium, D, can have various oxidation states, ranging from -3 to +8. Therefore, one would have to determine the oxidation states of other atoms in an iridium-containing compound to determine iridium's oxidation number.

Which of the following would be the most reactive toward nucleophiles? A. Propyl ethanoate B. Propanoic acid C. Propanamide D. Propanoic anhydride

D With the same R groups, steric influence is the same in each case, so we can therefore rely solely on electronic effects. When this is all that is taken into account, reactivity toward nucleophiles is highest for anhydrides, followed by esters and carboxylic acids, then amides.

What is the product of the reaction between benzaldehyde and an excess of ethanol (CH3CH2OH) in the presence of anhydrous HCl? See Figure 1 Organic Chemistry A. B. C. D.

D because an excess of ethanol is present, the product of the reaction will be an acetal. The benzaldehyde will first be converted to.a hemiacetal, shown in C, but will then proceed to completion as an acetal

The following equillibrium exists when AgBr(Ksp=5.35E-13) is in solution: AgBr(s) <--> Ag+(aq) + Br-(aq) What is the solubility of AgBr in a solution of 0.0010M NaBr? A. 5.35E-13g/L B. 1.04E-12g/L C. 5.35E-10g/L D. 1.04E-7g/L

D solution on pg 341 general chemistry The solubility of AgBr can be determined using the Ksp value given in the equation. Some amount of AgBr will dissolve; this is the molar solubility x for these conditions. When AgBr dissociates, there will be x amount of silver(I) formed and x amount of bromide which is added to the 0.0010M Br- already present from NaBr. AgBr<--> Ag+ + Br- Ksp=[Ag+][Br-] 5.35E-13 = (x)(0.0010M-x) Given that the Ksp of 5.4E-13, x will be negligible compared to 0.0010M. Thus, the math can be simplified to: 5.35E-13 = x (0.0010) x=5.35E-13/10E-3 = 5.35E-10 Therefore, x, the molar solubility is 5.35E-10, which looks like C. However, the units of the answer choices are grams per liter, not molarity, and the result must be multiplied by the molar mass (187.8g/mol): 5.35E-10mol/L (187.8g/mol) = 5E-10 x 200 =1000E-10 = 1E-7

What is the product of FIGURE 2 (organic chemistry)? A. B. C. D.

D. One mole of aldehyde reacts with one mole of alcohol via a nucleophilic addition reaction to form a product called a hemiacetal. In a hemiacetal, an -OH group, an -OR group, a hydrogen atom, and an -R group are attached to the same carbon atom

A 40kg block is resting at a height of 5 m off the ground. If the block is released and falls to the ground, which of the following is closest to its total mechanical energy at a height of 2m, assuming negligible air resistance? A. 0 J B. 400 J C. 800 J D. 2000 J

D. assuming negligible air resistance, conservation of energy states that the total mechanical energy of the block is constant as it falls. At the starting height of 5m, the block only has PE=mgh=40kg (10m/s^2)(5m)=2000J B/c the KE at this point is 0J, the total mechanical energy is 2000J at any point during the block's descent

Which of the following best describes ionic compounds? A. Ionic compounds are formed from molecules containing two or more atoms. B. Ionic compounds are formed of charged particles and are measured by molecular weight. C. Ionic compounds are formed of charged particles that share electrons equally. D. Ionic compounds are three-dimensional arrays of charged particles.

D. Ionic compounds are composed of atoms held together by ionic bonds. Ionic bonds associate charged particles with large differences in electronegativity. Rather than forming molecules or being measured by molecular weight, ionic compounds form large arrays of ions in crystalline solids and are measured with formula weights. In ionic bonds, electrons aren't really shard but are rather donated from the less electronegative atom to the more electronegative atom

Which of the following experimental methods should NEVER affect the rate of a reaction? A. Placing an exothermic reaction in an ice bath B. Increasing the pressure of a reactant in a closed container C. Putting the reactants into an aqueous solution D. Removing the product of an irreversible reaction

D. Temp directly affects the rate constant. Changing the partial pressure of a gas will affect the number of effective collisions per tie. Solvents affect the rate of reactions depending on how the reactants interact with the solvent.

An electron is accelerated over a distance d by an electric potential V. the electric potential applied to this electron is then increased by a factor of 4 and the electron is accelerated over the same distance d. The speed of the electron at the end of the second trial will be larger than at the end of the first trial by a factor of: A. 16 B. 8 C. 4 D. 2

D. The electric potential (V) is equal to the amount of work done (W) divided by the test charge (q). This means that the potential is directly proportional to the amount of energy gained by particle; therefore, the overall amount of energy increases by a factor of 4. Because energy is directly proportional to the square of the speed (according to 1/2mv^2), the speed must increase by a factor of 2.

In a certain equilibrium process, the activation energy of the forward reaction is greater than the activation energy of the reverse reaction. This reaction is .... A. endothermic B. exothermic C. spontaneous D. non-spontaneous

D. The terms endothermic and exothermic are associated with enthalpy. There is not enough info in this question to reliably determine the entropy though, which is needed to determine if the reaction is endothermic or exothermic. In drawing a reaction coordinate diagram, the reactants lie below the products on the y axis (free energy axis), because the activation energy of the forward reaction is greater than the reverse reaction. The net free energy change is positive, indicating an endergonic (nonspontaneous) reaction

What would increasing the concentration of reactants accomplish in a solution containing a saturated catalyst? A. It would increase the rate constant but not the reaction rate B. It would decrease the rate constant but increase the reaction rate C. It would increase the rate constant and increase the reaction rate D. The reaction rate would be unaffected

D. While increasing the [ ] of reactants can alter the reaction rate in first orh higher-order reactions, saturated solutions containing a catalyst have a maximum turnover rate and cannot increase the rate constant or the reaction rate any higher by adding more reactant molecules

Ferrous

Fe2+

Iron(II)

Fe2+

Rank the following in order of decreasing leaving group ability: H2O, HO-, Br-, H-

H2O > Br- > HO- > H- Good LGs are weak bases, which are the conjugates of strong acids. LGs must also be stable once they leave the molecule. H2O is by far the most stable LG. Br- is the conj base of HBr; HO- is the conj base of water. HBr is a much stronger acid than water, so Br- is a better LG than OH-. H- is very poor LG b/c it's extremely unstable in solugion

The IR spectrum of a fully protonated amino acid would likely contain which of the following peaks? I. A sharp peak at 1750 cm-1 II. A sharp peak at 3300 cm -1 III. A broad peak at 3300 cm -1

I and II Animoo acids in their fully protonated form contain all three of the peaks that should be memorized for test day (C-O, N-H, and O-H). While statements I and II correctly give the peaks for the C=O bond (sharp peak at 1750cm-1) and the N-H bond (sharp peak at 3300cm-1), the peak for the O-H bond is in the wrong place. In a carboxylic acid, the C=O bond withdraws electron density from the O-H bond, shifting the absorption frequency down to about 3000 cm-1.

Which of the following describe(s) pyridine chlorochromate (PCC)? I. An oxidant that can form aldehydes from primary alcohols II. An oxidant that can completely oxidize primary alcohols III. An oxidant that can completely oxidize secondary alcohols

I and III

Why is the C-N bond of an amide planar? I. It has partial double-bond character II. It is sp^3 hybridized III. It has some sp^2 character

I and III One resonance structure of a C-N bond in an amide has the double bond between the C and N, not between the C and O. Thus, the C-N bond of an amide has some sp2 character, and sp2 hybridized atoms exhibit planar geometry

Plane-polarized light will NOT be rotated by which of the following molecules: SEE ORGANIC CHEMISTRY FIGURE 6

I and III Plane-polarized light is not rotated if a molecule is meso (like I) or if it has no chiral centers (like III). To see how structure I is meso, remember that in a Newman projection, groups can be rotated freely, since free rotation is possible around a single bond. If the front carbon is spun 180º, each of the substituents lines up with the same group on the rear carbon. Thus, the molecule has a plane of symmetry and is meso. For structure III, note that the front central carbon is bound to two CHO groups, while the rear central carbon is attached to two hydrogen atoms. Thus, no carbon in the compound is bound to four different substituents, meaning that this compound has no chiral centers. As a result, we know that structures I and III will not rotate plane-polarized light.

The aldol condensation is an example of which reaction type(s)? I. Dehydration II. Cleavage III. Nucleophilic addition

I and III The aldol condensation is a dehydration reaction because a molecule of water is lost and a nucleophilic addition reaction.

alpha hydrogens of a ketone are acidic due to I. resonance stabilization II. the electron-withdrawing properties of the alkyl groups III. the electronegative carbonyl oxygen

I and III When alpha carbons are deprotonated, the negative charge is resonance stabilized in part by the electronegative carbonyl oxygen, which is electron-withdrawing. Alkyl groups are actually electron donating, which destabilizes carbanion intermediates.

Which of the following amino acids contain(s) sulfur? I. Cysteine II. Serine III. Methionine

I and III.

As an officer approaches a student who is studying with his radio playing loudly beside him, he experiences the Doppler effect. Which of the following statements remains true while the officer moves closer to the student? I. The apparent frequency of the music is increased II. the same apparent frequency would be produced if the officer were stationary and the student approached him at the same speed III. The apparent velocity of the wave is decreased

I only physics pg 268 Here, an observer is moving closer to a stationary source. The applicable version of the Doppler effect equation is f'=f(v+VsubD)/v where v is the speed of sound. Because the numerator is greater than the denominator, f' will be greater than f: therefore, statement I is true. The scenario described in statement II will produce a similar, but not identical, frequency for the officer: the frequency formula would be f'=f(v/v-VsubS). The apparent frequency will increase, but the increase will not be exactly the same as if the officer had been moving. Statement III is false because we already know the frequency increases for the officer-a decrease in velocity would be associated with a decrease in frequency.

Ideal gases I. have no volume II. have particles with no attractive forces between them III. have no mass

II only Ideal gases are said to have no attractive forces between molecules. While each particle within the gas is considered to have negligible volume, ideal gases as a whole certainly do have a measurable volume.

A resonance structure describes: I. The hybrid of all possible structures that contribute to electron distribution II. A potential arrangement of electrons in a molecules III. The single form that the molecules most often takes

II only Statement 1 has reversed the terminology for resonance structures. The electron density in a molecule is the weighted avg of all possible resonance structures, not the other way around

If the surface of electrode material in an electrochemical cell is tripled, what else is necessarily tripled? I. E(not)cell II. Current III. Keq

II only Potential, as measured by E(not)cell, is dependent only on the identity of the electrodes and not the amount present. Similarly, the equilibrium constant depends only on the identity of the electrolyte solutions and the temp. However, as the electrode material si increased, the surface area participating in oxidation-reduction reactions is increased and more electrons are released, making statement II correct.

The period for a certain wave is 34ms. If there is a Doppler shift that doubles the perceived frequency, which of the following must be true? I. The detector is moving toward the source at a velocity equal to the speed of sound II. The source is moving toward the detector at a velocity equal to half the speed of sound III. The perceived period is 17 ms IV. The perceived period is 68 ms.

III only Period is inversely related to frequency. because the perceived frequency is doubled, the perceived period must be halved, from 34 ms to 17 ms. While either condition I or II would cause a doubling of the perceived frequency, ,either condition must necessarily be true because the opposite could be true instead.

What is a ground ( in relation to electrostatics)?

It is a way of returning charge to the earth. Ex: The shock that occurs when your hand gets close enough to a metal doorknob after shuffling your feet across this carpet allows that excess charge to jump from your fingers to the knob, which acts as a ground.

A chemist wishes to prepare 300mL of a 1.1 M NaOH solution from a 5.5M NaOH stock solution. What volume of stock solution should be diluted with pure water to obtain the desired solution?

MiVi=MfVf Vi=(1.1M)(300mL)/(5.5M) = 300/5 = 60mL

Do plane mirrors (flat reflective surfaces) cause convergence or divergence of reflected light rays? Why?

Neither. Because the light does not converge at all, plane mirrors always create virtual images. In a plane mirror, the image appears to be the same distance behind the mirror as the object is in front of it. In other words, plane mirrors create the appearance of light rays originating behind the mirrored surface. Because the reflected light remains in front of the mirror but the image appears behind the mirror, the image is virtual. Plane mirrors can be conceptualized as spherical mirrors with an infinite radius of curvature.

A 3E-6F capacitor is connected to a 4V battery. If a piece of ceramic having dielectric constant k=2 is placed between the plates, find the new charge, capacitance, and voltage of the capacitor.

New voltage-held constant by a battery-still 4V. By introducing a dielectric with a dielectric constant of 2, the capacitance of the capacitor is multiplied by 2 (C'=kC). Hence the new capacitance is 6E-6F. now, the new charge on the capacitor can be determined Q'=C'V'=(6E-6F)(4V)=24E-6C

What is an adiabatic process? How does it appear on a pressure volume graph.

Occur when no heat is exchanged between the system and the environment; thus, the thermal energy of the system is constant throughout the process. When Q=0, the first law simplifies to deltaU=-W (the change in internal energy of the system is equal to work done on the system [the opposite of work done by the system]). Appears as a hyperbolic curve (like isothermal processes)

How do you calculate the probability of independent events occurring at the same time? How do you calculate the probability of two dependent events occurring at the same time When you see the word and, do what. When you see the word or, do what?

P(A B) = P (A and B) = P(A) x P(B) P (A U B) = P(A or B)= P (A) + P(B) - P(A and B) and- multiply probabilities or- add the probabilities and subtract the probability of both happening together

Phosphide

P3-

List all the state functions

Pressure (P) Density (p) Temperature (T) Volume (V) Enthalpy (H) Internal Energy (U) Gibbs Free Energy (G) Entropy (S) When I'm under PRESSURE and feeling DENSE, all I want to do is watch TV and get HUGS

Resistance in a wire depends on: I. cross-sectional areaII. the material comprising the wireIII. the length of the wire a) I only b) I and II only c) II and III only d) I, II and III

Resistance in a wire follows the equation R=ρL/A where ρ is a property of the wire material, L is the length of the wire, and A is the cross-sectional area of the wire. a) I only, incorrect. b) I and II, incorrect. c) II and III, incorrect. d) I, II and III, correct.

What is the percent composition of chromium in K2Cr2O2?

The molar mass of K2Cr2O2 is (2 x 39.1g/mol) + (2x52g/mol) + (7 x 16g/mol) = 294.2g/mol % Composition = [(2 x 52g/mol) / (294.2g/mol)] x 100% = 35.4%

What are colligative properties?

The physical properties of solutions that are dependent on the concentration of dissolved particles but not on the chemical identity of the dissolved particles. Ex: Vapor pressure depression, boiling point elevation, freezing point depression, and osmotic pressure.

What is the common ion effect?

The solubility of a salt is considerably reduced when it is dissolved in a solution that already contains one of its constituent ions as compared to its solubility in a pure solvent. This reduction in molar solubility is the common ion effect.

Explain deshielding

When dealing with H-NMR on the MCAT, think of a proton as being surrounded by a shielf of electrons. As we add electronegative atoms or have resonance structures that pull electrons away from the proton, we Deshield and move Downfield

What is energy density

a measure of a battery's ability to produce power as a function of its weight.

what is recrystallization?

a method for further purifying crystals in solution. in this process, we dissolve our product in a minimum amount of hot solvent and let it recrystallize as it cools. The solvent chosen for this process should be one in which the product is only soluble at high temps. Thus, when the solution cools, only the desired product will recrystallize out of solution, excluding the impurities.

what is a complex ion (coordination compound)? what is a ligand?

a molecule in which a cation is bonded to at least one electron pair donor (which could include the water molecule). The electron pair donor molecules are called ligands.

explain mess compounds

a molecule with chiral centers that has an internal plane of symmetry. b/c they have this plane of symmetry, they are not optically active

What is a correlation coefficient?

a number between -1 and +1 that represents the strength of the relationship. +1 indicates a strong positive relationship and -1 indicates a strong negative relationship.

What is energy

a system's ability to do work or cause change

What is a neutralization reaction?

acid + base -> salt + water

What is a double displacement reaction?

also called metathesis reactions. AB + CD --> AC + BD

What is an amphoteric species? What about amphiprotic?

amphoteric species is one that reacts like an acid is a base environment and like a base in an acidic environmtne. Amphiprotic species can either gain or lose a proton. t

What is an isothermal process? How does it appear on a pressure volume graph.

an isothermal process is a process that occurs at constant temperature constant temp implies that the total internal energy of the system (U) is constant throughout the process. This is because temp and internal energy are directly proportional. When U is constant, deltaU=0 and the first law simplifies to Q=W (The heat added to the system equals the work done by the system). An isothermal process appears as a hyperbolic curve on a pressure volume graph. Work is the area under the curve

Suppose a parent nucleus X alpha decays as follows: 238/92X-->A'/Z'Y + alpha What are the mass number and atomic number of the daughter isotope Y?

answer on pg 326 of physics To solve this question, we simply need to balance the atomic numbers and mass numbers: A/ZX--> (A-4)/(Z-2)Y +4/2 alpha 238/92U-->234/90Y +4/2 alpha Y msut be thorium (Th) because its atomic number is 90. Therefore, the daughter nucleus is 234/90Th

Which of the following has the largest dipole moment? A. HCN B. H2O C. CCl4 D. SO2

A

What are the three types of electrochemical cells?

galvanic cells (voltaic), electrolytic cells, and [ ] cells

A tractor pulls a log with a mass of 500kg along the ground for 100m. The rope (between the tractor ad the log) makes an angle of 30 degrees with the ground and is acted on by a tensile force of 5000N. How much work does the tractor perform in this scenario?

433kJ W=fdcostheta 5000N (100m) cos 30 = 433kJ

If the pressure of a sample of gas with a temperature of 300K changes from 2 atm to 5 atm during heating, what would be the final temp if volume is held constant?

750K 2atm/300K=5atm/T2

(+)-Glyceraldehyde and (-)-Glyceraldehyde refer to the (R) and (S) forms of 2,3-dihydroxypropanal, respectively. These molecules are considered A. enantiomers B. diastereomers C. meso compounds D. structural isomers

A

What is surface tension?

A measure of how difficult it is to stretch or break the surface of a liquid. Causes the liquid to form a thin but strong layer like a "skin" at the liquid's surface. Surface tension results from cohesion. Cibsuder the IMFs between the separate molecules of water. For those molecules below the surface, there are attractive IMFs coming from all sides; these forces balance out. However, on the surface, the molecules only have these strong attractive forces from the molecules below them, which pulls the surface of the liquid toward the center. This establishes tension in the place of the surface of the water; when there is an indentation on the surface, then the cohesion can lead to a net upward force.

How can the relative reactivities of different half cells be compared to predict the direction of electron flow?

A more positive Ered (standard reduction potential) means a greater relative tendency for reduction to occur, while a less positive Ered means a greater relative tendency for oxidation to ocur.

How many moles of electrons pass through a circuit containing a 100V battery and a 2 omega resistor over a period of 10 seconds? (Note F=9.65E4C/mol e^-) A. 5.18E-3 moles B. 500 moles C. 5.18E3 moles D. 5.2E6 moles

A pg 230 physics and math To determine the moles of charge that pass through the circuit over a period of 10s, we will have to calculate the amount of charge running through the circuit. Charge is simply current multiplied by time, and the current can be calculated using Ohm;s law: V=IR and I=Q/deltat --> Q=Vdeltat/R Q=(100V)(10s)/2 omega= 500C Then calculate the number of moles and charge that this represents by using the Faraday constant and approximating F as 10^5C/mol e^- 500C[mol e^-/10^5] = 5E-3 mol e^-

Explain blood flow through the body

As blood flows away from the heart, each vessel has a progressively higher resistance until the capillaries; however, the total resistance of the system decreases because the increased number of vessels are in parallel with each other. Like parallel resistors in circuits, the equivalent resistance is therefore lower for the capillaries in parallel than in the aorta. Return flow to the heart is facilitated by mechanical squeezing of the skeletal muscles, which increases pressure in the limbs and pushes blood to the heart, and the expansion of the heart, which decreases pressure in the heart and pulls blood in. Finally, the pressure gradients created int he thorax by inhalation and exhalation also motivate blood flow. Venous circulation holds approximately three times as much as blood in arterial circulation. Heart murmurs, which result from structural defects of the heart, are heard because of turbulent blood flow.

(R)-2-chloro-(S)-3-bromobutane and (S)-2-chloro-(S)-3-bromobutane are.... A. enantiomers B. diastereomers C. meso compounds D. the same molecule

B

A carbon atom participates in one double bond. As such this carbon contains orbitals with: A. hybridization between the s-orbital and one p-orbital B. hybridization between the s-orbital and two p-orbitals C. hybridization between the s-orbital and three p-orbitals D. unhybridized s character

B

Which of the following atoms only has paired electrons in its ground state? A. Sodium B. Iron C. Cobalt D. Helium

D

What is the difference between diffusion and effusion? What are the similarities?

Diffusion is when gases mix with one another. Effusion is when a gas moves through a small hole under pressure. Both will be slower for larger molecules. Both conditions use the same equation.

what do insulators do? what are they like on a molecular level? most ________ are insulators (metals or nonmetals)

Do not allow heat or electricity to pass through easily they will not easily distribute a charge over its surface and will not transfer that charge to another neutral object very well- esp to another insulator. the electrons of insulators tend to be closely linked with their nuclei most nonmetals are insulators

Is static charge buildup or static electricity more significant in drier or more humid air? Why?

Drier air because the lower humidity makes it easier for charge to become and remain separated.

A vessel contains 0.75 mol of nitrogen, 0.2 mol hydrogen, and 0.05 mol of fluorine at a total pressure of 2.5 atm. What is the partial pressure of each gas?

First calculate the mole fraction of each gas. XN2=0.75mol/1mol=0.75 XH2=0.2mol/1 mol=0.20 XF2=0.05mol/1mol=0.05 Then calculate the partial pressure P(subA)=X(subA)P(subT) PN2=0.75(2.5atm)=1.875atm PH2=0.2(2.5atm)=0.5atm PF2=0.05(2.5atm)=0.125atm

Calculate the [ ] of H3O+ in a 2.0M aqueous solution of acetic acid, CH3COOH (Note: Ka=1.8E-5)

First, write the equilibrium reaction CH3COOH(aq) + H2O(l) <--> H3O+(aq) + CH3COO-(aq) Next, write the expression for the acid dissociation constant: Ka=[H3O][CH3COO-]/[CH3COOH] = 1.8E-5 Then, recognize that acetic acid is a weak acid, so the [ ] of CH3COOH at equilibrium is equal to its initial [ ], 2M, minus the amount dissociated, x. Likewise, [H3O+]=[CH3COO-]=x because each molecule of CH3COOH dissociates into one H+ ion and one CH3COO- ion. Note that the contribution of H3O+ from water is negligible. Thus, the equation can be rewritten: Ka=[x][x]/[2.0M=x]=1.8E-5 Remember that the value of x is generally very small. Therefore, we can approximate that 2.0M-x=2.0M. Thisis further supported because acetic acid si a weak acid and only slightly dissociates in water. This simplifies the calculations: Ka=[x][x]/2M =1.8E-5 x^2=3.6E-5 = 36E-6 x=6E-3 x represents the [ ] of H3O+; therefore, [H3O+] = 6E-3M. Note: When needing to take the square root adjust the coefficient as needed to amke the power of 10 an even number. This way the square root only requires cutting the power of 10 in half.

as electrons move from a lower energy to higher energy level in the Bohr model, they do what 4 things?

Get AHED Absorb light Higher potential Excited Distant (from the nucleus)

Hemiacetals and Hemiketals usually keep reacting to form acetals and ketals. Why is it difficult to isolate hemiacetals and hemiketals. I. These molecules are unstable II. The hydroxyl group is rapidly protonated and lost as water under acidic conditions, leaving behind a reactive carbocation III. The molecules are extremely basic and react rapidly with one another.

I and II

An object is placed 6cm in front of a concave mirror that has a 10cm radius of curvature. Determine the image distance, the magnification, whether the image is real or virtual, and whether it is inverted or upright.

If you have trouble with the math, refer to pic on phone. Use the optics equation: 1/f=1/o+1/i=2/r 1/i=2/r=1/o 1/i=2/10cm=1/6cm=12-10/60=2/60 i=60/2=+30cm A positive value for i signifies that the image is in front of the mirror and is therefore real. For a single lens or mirror with o>0, a real image will always be inverted. After determining i, the magnification m can be calculated as: m=-i/o=-30cm/6cm=-5 The negative sign on the magnification confirms that the image is inverted, and the fact that ImI>1 indicates that the image is enlarged.

what's the definition of Lewis acids and bases?

Lewis acid- electron acceptor Lewis base- electron donor

explain alpha state vs Beta state in NMR.

NMR spectroscopy is based on the fact that certain atomic nuclei have magnetic moments that are oriented at random. When such nuclei are placed in a magnetic field, their magnetic moments tend to align either with or against the direction of this applied field. Nuclei with magnetic moments that are aligned with the field are said to be in the alpha state (lower energy). The nuclei can then be irradiated with radiofrequency pulses that match the energy gap between the two states, which will excite some lower-energy nuclei into the B-state (higher energy). The absorption of this radiation leads to excitation at different frequencies, depending on an atom's magnetic environment.

Nitrate

NO3-

Strong acid + strong base: HCl + NaOH ---> ?? Strong acid + weak base: HCl + NH3 -->?? Weak acid + strong base: HClO + NaOH -->?? Weak acid + weak base: HClO + NH3--??

Strong acid + strong base: HCl + NaOH ---> NaCl+H2O Strong acid + weak base: HCl + NH3 -->NH4Cl Weak acid + strong base: HClO + NaOH -->NaClO + H2O Weak acid + weak base: HClO + NH3--NH4ClO

What is alpha decay?

The loss of one alpha particle (He nucleus, which has mass number of 4 and atomic number of 2). The alpha particle is very massive compared to a beta particle and carries double the charge. Alpha particles interact with matter very easily; hence, they do not penetrate shielding (such as lead sheets) very extensively.

What is the threshold frequency and how does it relate to the photoelectric effect?

The minimum frequency of light that causes ejection of electrons is known as the threshold frequency, fsubT. The threshold frequency depends on the type of metal being exposed to the radiation. The photoelectric effect is, for all intents and purposes, an "all-or-nothing" response: if the frequency of the incident photon is less than the threshold frequency, then no electron will be ejected because the photons do not have sufficient energy to dislodge the electron from its atom. But if the frequency of the incident photon is greater than the threshold frequency, then an electron will be ejected, and maximum kinetic energy of the ejected electron will be equal to the difference between hf and hfsubT (also called the work function).

What is Hess' law?

The total enthalpy change of a reaction is always the same, no matter which route is taken. the enthalpy changes of reactions are additive.

Which of the following accurately depicts the field lines created by a proton that is moving toward the right on a page? FIGURE 4 physics. A. B. C. D.

You should know that the field lines for a positively charged particle will always point away from the particle in a radial pattern, regardless of the direction in which the particle is moving. This is because field lines point in the direction a positive test charge would move in that field (that is, the direction that a force would be exerted on a positive test charge in that field).

What is the maximum number of electrons in a single atomic energy level in terms of the principal quantum number, n?

2n^2

A certain substance has a specific heat of 1 J/mol . K and a melting point of 350K. if one mole of the substance is currently at a temp of 349K, how much energy must be added in order to melt it? A. more than 1 J B. Exactly 1J C. Less than 1J but more than 0 J D. Less than 0J

A

A reaction that utilizes oxygen and hydrocarbons as reactants and that produces carbon dioxide and water as products is best characterized as: A. single-displacement B. combustion C. metathesis D. decomposition

B

The common names for the aldehydes and carboxylic acids that contain only one carbon start with which prefix? A. para- B. Form- C. Meth- D. Acet-

B

If a certain metal has multiple oxidation states, its acidity as an oxide generally increases as the oxidation state increases. Therefore, which of the following tungstem compounds is likely to be the strongest acid? A. WO2 B. WO3 C. W2O3 D. W2O5

B Recall that oxygen has an oxidation state of -2. Therefore, in tungsten (IV) oxide, A, tungsten has an oxidation state of +4. In tungsten(VI) oxide, B, it has an oxidation state of +6. In tungsten (III) oxide, C, it is +3. In tungsten pentoxide, D, it is +5.

If the electric field at a distance r away from charge Q is 36N/C, what is the ratio of the electric fields at r, 2r, and 3r? A. 9:3:1 B. 36:9:4 C. 36:18:9 D. 36:18:12

B The first step in answering this question is to remember that the magnitude of the electric field is inversely proportional to the square of the distance: E=kQ/r^2 ---> E proportional to 1/r^2 Therefore, if the electric field at radius r, Esubr, is 36N/C then the electric field at radius 2r will be: Esub2r is proportional to [1/(2r)^2 ] = [1/4r^2]=Esubr/4 =36N/C/4=9N/C Similarly the electric field at radius 3r is equal to: Esub3r is proportional to [1/(3r^2)]=[1/9r^2]=Esub4/9 =36N/C/9 = 4N/C Therefore, the radio of Er:E2r:E3r is 36:9:4

How would the number 17,060 be written in scientific notation? A. 1706 x 10^1 B. 1.706 x 10^4 C. 1.7040 x 10^4 D. 0.17060 x 10^5

B because there is no decimal point, the last zero is not significant and should not be used in scientific notation. The significand in scientific notation should always be between one and twn

Metals are often used for making wires that conduct electricity. Which of the following properties of metals explains why? A. metals are malleable B. metals have low electronegativities C. metals have valence electrons that can move freely D. metals have high melting points

C

In order to separate a biological effector from solution, which chromatographic technique would be the most effective A. Thin-layer chromatography B. Ion-exchange chromatography C. Affinity chromatography D. Size-exclusion chromatography

C Affinity chromatography, using the target for the biological effector or a specific antibody, would work best in this case. It will specifically bind the protein of interest and keep it in the column.

which of the following is digestible by humans and is made up of only one type of monosaccharide? A. Lactose B. Sucrose C. Maltose D. Cellubiose

C While maltose and cellobiose both have the same glucose subunits, only maltose is digestible by humans because the B-glycosidic linkages in cellubiose cannot be cleaved in the human body

Copper(II)

Cu2+

Cupric

Cu2+

(2R,3S)-2,3-dihydroxybutanedioic acid and (2S,3R)-2,3-dihydroxybutanedioic acid are: I. meso compounds II. the same molecule III. .enantiomers

I and II

ammonium

NH4+

What is the electron configuration of Fe3+?

[Ar] 3d^5 Electrons are removed from the 4s subshell before the 3d sub shell because it has a higher principal quantum number.

Which of the following will convert cyclohexanol to cyclohexanone? I. Chromium trioxide II. Pyridinium chlorochromate III. Sodium dichromate

all 3.

For CH4, NH3, and H2O, explain the electronic and molecular geometries.

all have tetrahedral electronic geometry molecular geometry: CH4-tetrahedral NH3-pyramidal H2O- bent/angular

what is the difference between electronic and molecular geometry? what is the most important implication of electronic geometry?

electronic geometry describes the spatial arrangement of all pairs of electrons around the central atom, including both the bonding and the lone pairs. - determination of the ideal bond angle. molecular geometry-describes the spatial arrangement of only the bonding pairs of electrons

Charge is transmitted by a flow of _______ in a ___________. How does this charge move? what is the conventional direction of current?

flow of electrons in a conductor because electrons are negatively charged, they move from a point of lower electrical potential to a point of higher electrical potential by convention, the direction of current is the direction in which positive charge would flow (from higher potential to lower potential). Thus, the direction of current is opposite to the direction of actual electron flow.

What is Le Chatelier's principle?

if a stress is applied to a system in dynamic equilibrium, the system changes in a way that relieves the stress

How can you identify a nucleophile?

lone pairs, pi bonds, anions good bases tend to make good nucleophiles

relate acidity to pKa

more acidic molecules will have a smaller (or even negative) pKa. more basic molecules will have a larger pka

what is the difference between specific heat and heat capacities?

specific heat is the amount of energy required to raise the temperature of one gram of a substance by one degree Celsius heat capacity is the products mass times the specific heat. ex: it requires less heat to raise the temp of a glass of water the same amount as a swimming pool

What is translational equilibrium? (also known as what?)

the first condition of equilibrium- exists only when the vector sum of all the forces acting on an object is zero

What is conjugation?

the presence of alternating single and multiple bonds.

what are Doppler ultrasounds and how do they work?

they are used to determine the flow of blood within the body by detecting the frequency shift that is associated with movement toward or away from the receiver.

What does the second law of thermodynamics, also called the law of entropy, state?

that energy spontaneously disperses from being localized to becoming spread out if it is not hindered from doing so

what is transmittance

The amount of light that passes through the sample and reaches the detector (vs wavenumber)

Which of the following values corresponds to the probability of a type I error? A. alpha B. Beta C. power D. confidence

A Type I error is the probabilit of mistakenly rejecting the null hypothesis. We set the type I error level by selecting a significance level (alpha)

Which of the following has a formula weight between 74 and 75 grams per mole? A. KCl B. C4H10O C. MgCl2 D. BF3

A . B and D are covalent and thus are measured in molecular weights not formula weights. The formula weight of MGCl2 is way too high. KCl (39.1 amu + 35.5 amu)

what are the need to know solubility rules for aqueous solutions for the MCAT?

1. All salts containing ammonium (NH4+) and alkali metal (Group 1) cations are water-soluble. 2. All salts containing nitrate (NO3-) and acetate (CH3COO-) anions are water-soluble.

The ejection fraction is the proportion of the blood volume in the left ventricle expelled with each contraction of the heart. A patient is known to have an ejection fraction of 0.6, a cardiac output of 5L/min, and a heart rate of 80bpm. What is the volume of blood in this person's left ventricle just prior to contraction?

(5L/min)/(80bpm) = 1/16 = 0.0625L/beat The question also explains that only 60% of the blood volume is expelled from the left ventricle per heartbeat. 0.0625L/beat = 0.6 x ventricular volume/beat 0.0625L/0.7 = ventricular volume = 0.1 L

Using a given mass of KClO3, how would one calculate the mass of oxygen produced in the following reaction, assuming it goes to completion? 2 KClO3 --> 2 KCl + 3O2

(grams KClO3 consumed)(3 moles O) (molar mass O2) / (molar mass KClO3)(2 moles KClO3)

Element Q consists of 3 different isotopes:A, B, and C. Isotope A has an atomic mass of 40 amu and accounts for 60% of naturally occurring Q. Isotope B has an atomic mass of 44 amu and accounts for 25% of naturally occurring Q. Isotope C has an atomic mass of 41 amu and accounts for 15% of Q What is the atomic weight of Q?

0.60 (40 amu) + 0.25 (44 amu) + 0.15 (41 amu)= 41.15 amu

what is the specific heat of water

1 cal/g K

What 4 major factors are nucleophilicity determined by? Explain them.

1. Charge- nucleophilicity increases with increasing electron density (more negative charge) 2. Electronegativity- nucleophilicity decreases as electronegativity increases because these atoms are less likely to share electron density 3. Steric hindrance- bulkier molecules are less nucleophilic 4. Solvent- protic solvents can hinder nucleophilicity by protonating the nucleophile or through hydrogen bonding (in polar protic solvents, nucleophilicity increases down the periodic table. in polar aprotic solvents, it increases up the periodic table)

What are the 5 assumptions of the Kinetic Molecular Theory?

1. Gases are made up of particles with volumes that are negligible compared to the container volume 2. Gas atoms or molecules exhibit no intermolecular attractions or repulsions 3. Gas particles are in continuous, random motion, undergoing collisions with other particles and the container walls 4. Collisions between any two gas particles (or between particles and the container walls) are elastic, meaning that there is conservation of both momentum and kinetic energy. 5. The average kinetic energy of gas particles is proportional to the absolute temperature of the gas (in kelvins), and it is the same for all gases at a given temp, irrespective or chemical identity or atomic mass

In a sealed 1 L container, 1 mole of nitrogen gas reacts with 3 moles of hydrogen gas to from 0.05 miles of NH3, at equilibrium. Which of the following is closest to the Kc of the reaction? A. .0001 B. .001 C. .01 D. 1

A. Start with the balanced equation for the reaction of H2 and N2 to produce NH3: N2 + 3H2 <--> 2NH3 Kc=Keq=[NH3]^2/[N2][H2}^3 Because the volume is 1L, the amount of each gas in moles is equal to the value of the concentration of each gas in moles per liter (M). The relatively small amount of NH3 produced indicates that it will be possible to consider the amount of N2 and H2 that reacts to be negligible compared to their starting [ ]s. Keq=0.05^2/[1-x][3-x]^3 which is roughly equal to : 0.05^2/[1][3]^3=0.0001

An elevator is designed to carry a max weight 9800N and to move upward at a speed of 5m/s after an initial period of acceleration. What is the relationship between the max tension in the elevator cable and the max weight of the elevator while the elevator is accelerating upward? A. The tension is greater than 9800N B. The tension is less than 9800 N C. The tension equals 9800N

A. the forces on the elevator are the tension upward and the weight downward, so the net force on the elevator is the difference between the 2. For the elevator to accelerate upwards, the tension in the cable will have to be greater than the max weight so that there is a new force directed upwards.

What is the most important difference between Bohr's model and the modern quantum model?

Bohr postulated that electrons follow a clearly defined circular pathway or orbit at a fixed distance from the nucleus. Now we know that electrons move rapidly and are localized within regions of space around the nucleus called orbitals. Now we can just described the probability of finding an electron within a given region of space surrounding the nucleus.

What are Bronsted-Lowry acids and bases? what about amphoteric species?

Bronsted-Lowry acids are molecules or ions that are proton donors. Bronsted-Lowry bases have molecules or ions that are proton acceptors. amphoteric species have the ability to act as either Bronsted-Lowry acids or bases (Ex: water)

A 20 m steel rod at 10C is dangling from the edge of a building and is 2.5cm from the ground. if the rod is heated to 110C, will the rod touch the ground? (Note: alpha=1.1 x 10^-5 K^-1) A. Yes, because it expands by 3.2 cm B. Yes, because it expands by 2.6 cm C. No, because it expands by 2.2 cm D. No, because it expands by 1.8 cm

C deltaL=alpha (L)(delta T)

Consider the following reaction Zn(s) + CuSO4(aq) --> Cu(s) + ZnSO4(aq) Write the complete ionic equation and the net ionic equation. What is the difference?

Complete: Zn(s) + Cu2+(aq) + SO4^2-(aq) --> Cu(s) + Zn2+(aq) + So4^2-(aq) Net ionic equation: Zn(s) + Cu2+(aq) --> Cu(s) + Zn2+(aq) net ionic does not have the spectator ions (ions that are not taking part in the overall reaction but simply remaining in the solution unchanged).

The molar heat capacities (J/mole ŸK) for zinc, copper, silver, and gold is 25.2, 24.5, 24.9, and 25.6, respectively. If 1 mole of each substance is heated until the temperature increases by 10K, which metal required the addition of the most heat? A. Zinc B. Copper C. Silver D. Gold

D

Which of the following atoms or ions has the largest effective nuclear charge? A. Cl B. Cl- C. K D. K+

D

What is the first law of thermodynamics? What equation accounts for it?

Energy can be transferred and transformed, but it cannot be created or destroyed. E= U+K (total mechanical energy=potential energy + kinetic energy)

Give the following half reactions and Ered values, determine which species would be oxidized and which would be reduced in a galvanic cell. Ag+ + e- --> Ag(s) Ered=+0.80V Tl+ + e- -->Tl(s) Ered=-0.34V

Ered indicates the reduction potential, or the likelihood of a compound to be reduced via a given reaction. A positive Ered value indicates a spontaneous reduction, and a negative value indicates a non-spontaneous reduction. In a galvanic cell, Ag+ will be spontaneously reduced to Ag(s) and Tl(s) will be spontaneously oxidized to Tl+ because Ag+ has the more positive Ered and thus the more favorable reduction reaction. Therefore, the net ionic equtation would be: Ag+ + Tl(s) --> Tl+ + Ag(s)

What are electric fields? What effects do they have other charges?

Every electric charge sets up a surrounding electric field, just like every mass creates a gravitational field. Electric fields make their presence known by exerting forces on other charges that move into the space of the field. -Whether the force exerted is attractive or repulsive depends on whether the stationary test charge q (the charge placed in the electric field) and the stationary source charge Q (which actually creates the electric field) are opposite charges (attractive) or like charges (repulsive)

Determine the volume of a cylinder with a radius that is measured as 7.45 m and a height of 8.323 m. (Note: use 3.14159 as pi, and round the answer to the correct number of significant digits).

V=Abase x h (pir^2)() (3.14159)(7.45)^2 (8.323m) =1451.249 Because all of the factors are multiplied, the answer should have the same number of significant digits as the factor with the fewest number of significant digits. In this case, that is the radius, which has only two significant digits (remember that, in the case of measurements, the last digit is an estimate and is not considered significant). Therefore, the correct answer is 1500 or 1.5E3.

What is electronegativity?

a measure of the tendency of an atom to attract a bonding pair of electrons

delta H delta S Outcome + + ? + - ? - + ? - - ?

delta H delta S Outcome + + Spontaneous at high T + - Nonspontaneous at all T - + Spontaneous at all T - - Spontaneous at low T

What are the three classifications of materials? Explain each, give examples.

diamagnetic, paramagnetic, and ferromagnetic diamagnetic materials-made of atoms with no unpaired electrons and that have no net magnetic field. These materials are slightly repelled by a magnet and so can be called weakly antimagnetic. Ex: common materials that you wouldn't expect to get stuck in a magnet: wood, plastics, water, glass. skin, etc The atoms of both paramagnetic and ferromagnetic materials have unpaired electrons, so these atoms do have a net magnetic dipole moment, but the atoms in these materials are usually randomly oriented so that the material itself creates no net charge. Paramagnetic materials will become weakly magnetized in the presence of an external magnetic field, aligning the magnetic dipoles of the material with the external field. Upon removal of the external field, the thermal energy of the individual atoms will cause the individual magnetic dipoles to reorient randomly. Ex: aluminum, copper, gold Ferromagnetic materials, like paramagnetic materials, have unpaired electrons and permanent atomic magnetic dipoles that are normally oriented randomly so that the material has no net magnetic dipole. Unlike paramagnetic materials, ferromagnetic materials will become strongly magnetized when exposed to a magnetic field or under certain temps. Ex: iron, nickel, cobalt. -Bar magnets are ferromagnetic materials with a north and South Pole.

How can you tell which harmonic a wave of a string is in?

for strings attached at both ends, the number of antinodes present will tell you which harmonic it is

what is mass spectrometry

involves the ionizaiton and fragmentation of compounds; these fragments are then run through a magnetic field, which separates them by mass-to-charge ration. The total molecular weight can thus be determined, or the relative [ ]s of the different fragments can be calculated and compared against reference values to identify the compound.

What is the difference between ionic and covalent bonding?

ionic- one or more electrons from an atom with a low ionization energy, typically a metal, are transferred to an atom with a high electron affinity, typically a nonmetal. the resulting electrostatic attraction between opposite charges is what holds the ions together (Ex: NaCl) covalent- an electron pair is shared between the two atoms, typically non-metals, that have relatively similar values of electronegativity (the degree to which the pair of electrons is shared equally or unequally between the two atoms determines the degree of polarity in the covalent bond)

What is an isovolumetric process? How does it appear on a pressure volume graph?

isovolumetric (isochoric) processes experience no change in volume. Because the gas neither expands nor compresses, no work is performed in such a process. Thus, the first law simplifies to deltaU=Q (the change in internal energy is equal to the heat added to the system). An isovolumetric process is a vertical line on a P-V graph; the area under the curve, which represents the work don by the gas, is zero.

What is the weak nuclear force?

it contributes to the stability of the nucleus, but is about one-millionth as strong as the strong nuclear force.

What are the two categories of conductivity? Explain them.

metallic conductivity- solid metals and the molten forms of some salts electrolytic conductivity- seen in solutions.

Neutrons have what charge? Where are they located? What is their mass in relation to a proton?

neutral. in the nucleus. Mass is slightly larger than a proton

what are mutually exclusive outcomes

outcomes that cannot occur at the same time. one cannot flip both heads and tails in one throw, or be both 10 and 20 years old

Electrons that are farthest from the nucleus have the _____ interactions with the surrounding environment and the ________ interactions with the nucleus. What are these electrons called?

stronger interactions with the surrounding environment and weakest interactions with the nucleus. Valence electrons

What is mechanical advantage?

the ratio of magnitudes of the force exerted on an object by a simple machine to the force actually applied on the simple machine

What is reflection

the rebounding of incident light waves at the boundary of a medium

what are ideal solutions?

when the overall strength of the new interactions after solvation is approx equal to the overall strength of the original interactions. In this case, the overall enthalpy change for the dissolution is close to zero.

- For electrons to move from a lower to higher energy level, they must ______ energy in the form of. ____. - for electrons to move from a higher energy to a lower energy level, they must _____ energy in the form of _____.

- absorb energy in the form of light - emit energy in the form of light

- electrons can be excited to higher energy levels in the Bohr model by what? - Because the lifetime of an excited state is brief, the electrons will rapidly return to the ground state, resulting in what? - These energy transitions are quantized to certain values (do not form a continuum). Thus, the spectrum is composed at light at specific frequencies. It is sometimes called a ________, where each line on the emission spectrum corresponds to what? - Because each element can have its electrons excited to a different set of distinct energy levels, each possesses a unique _________, which can be used as a ______ for the element.

- heat or other energy forms. - resulting in the emission of discrete amounts of energy in the form of photons with a wavelength characteristic of the specific energy transition it undergoes - light spectrum-each line corresponds to a specific electron transition - atomic emission spectrum, can be used as a fingerprint for the element

Consider the table. Which of the following terms best describes these two electrons? n. l. ml. ms 2. 1. 1. +1/2 3. 1. -1. +1/2 A. Parallel B. Opposite C. Antiparallel D. Paired

A

Josh, who has a mass of 80kg, and Sarah (m=50kg) jump off a 20m tall building and land on a fire net. The net compresses, and they bounce back up at the same time. Which of the following is NOT true? A. Sarah will bounce higher than Josh B. For Josh, the change in speed from the start of the jump to contacting the net is 20m/s C. Josh will experience a greater force upon impact than Sarah D. The energy in this event is converted from potential to kinetic to elastic to kinetic

A

The value of 200^0.25 is closest to: A. 4 B. 14 C. 50 D. 800

A 200^0.25 = 200^(0.5 x 0.5) = sqrt sqrt 200 = 4sqrt200 The square root of 200 should be a bit larger than 14 (14^2=196); therefore, the fourth root of 200 should be a bit less than 4.

At low pressures and temperatures, water can go directly from ice to water vapor. This state change is known as: A. sublimation B. deposition C. vaporization D. gaseous melting

A A state change from solid to gas is sublimation. A. sublimation, correct. B. deposition, incorrect, Deposition is gas to solid. C. vaporization, incorrect, vaporization is liquid to gas. D. gaseous melting, incorrect, there is no such thing as gaseous melting.

Which of the following represents chloric acid? A. HClO3 B. ClO3- C. HClO2 D. HClO

A Acids ending in -ic are derivatives of anions ending in -ate, while acids ending in -ous are derivatives of anions ending in -ite. ClO3- is chlorate because it has more oxygen than the other commonly occurring ion, ClO2-, which is named chlorite. Therefore, HClO3 is chloric acid. HClO2 represents chlorous acid. HClO represents hypochlorous acid.

Which of the following compounds would be most reactive toward a nucleophile? A. Pentanal B. 3-pentanone C. Pentane D. 2-nonanone

A Aldehydes are generally more reactive than ketones because the additional alkyl group of a ketone is sterically hindering; this alkyl group is also electron-donating, destabilizing the carbanion intermediate. This eliminates B and D. The carbonyl carbon is highly electrophilic; alkanes lack any significant electrophilicity, eliminating C.

In a reaction between ammonia and glutaraldehyde, what is the major product? A. An imine B. A cyanohydrin C. A semicarbazone D. A hydrazone

A Ammonia, NH3, will react with an aldehyde like glutaraldehyde to form an immune. This is a condensation and a substitution reaction, as the C=O bond of the carbonyl will be replaced with a C=N bond

A study is performed on a new medication. Subjects in the experimental group are told about the potential side effects of the medication, while subjects in the placebo group are not. The subjects have no contact with each other and do not know in which group they are placed. The side effects end up being significantly more severe in the treatment group, when seen by the same assessor physician. This is most likely caused by which of the following? A. Physician unblinding only B. Patient unblinding only C. Both physician and patient unblinding D. Both physician and patient blinding

A Because the same physician sees both the control groups and the experimental groups, there is the potential for the physician to realize which of the groups is receiving which treatment-especially if the subject mentions expected side effects. In this study, patients were not told which group they were in, but if they were assigned to the medication group, they were told about its side effects. If the patients talked to each other they could experience patient unblinding, but we are told that they have no communication with each other.

Which compound will migrate the fastest during thin layer chromatography using silica plates? FIGURE 8 ORGANIC CHEMISTRY A B C D

A During thin layer chromatography on silica plates, polar molecules are attracted to the silica and migrate slower than non-polar compounds. Of the answer choices, only cyclohexane is non-polar. B. Incorrect, ketone group makes this molecule polar. C. Incorrect, the carboxylic acid group is polar. D. Incorrect, the alcohol is polar.

If cinnamaldehyde was treated with LiAlH4 what reaction would occur? A Reduction, resulting in a primary alcohol B. Oxidation, resulting in a carboxylic acid C. Acid-base, resulting in a diol D. no reaction would occur

A it's an aldehyde. LiAlH4 reduces aldehydes to primary alcohols

What is a wash?

Another way to take advantage of solubility properties is to perform the reverse of an extraction to remove unwanted impurities. In this case, a small amount of solute is used to extract and remove impurities, rather than the compound of interest. This process is called a wash.

Which of the following is NOT a state function? A. Internal energy B. Heat C. Temp D. Entropy

B

Which of the following is closest to the pH of a solution containing 5mM H2SO4? A. 1 B. 2 C. 3 D. 4

B First, convert the [ ] to 5E-3M. Next, because sulfuric acid is a strong acid, we can assume that, for the majority of sulfuric acid molecules (although not all), both protons will dissociate. The [ ] of hydrogen ions is therefore 2 x 5 x 10E-3, or 10E-2. The equation for pH is pH=-log[H+]. If [H+]=10^-2M, then pH=2.

Which of the following would make the strongest electrolytic solution? A. A non polar covalent compound with significant solubility B. An ionic compound composed of one cation with a +3 charge and three anions with a -1 charge C. A polar covalent compound with a small dissociation constant D. An ionic compound composed of two cations with +1 large and one anion with -2 charge

B The best electrolytes dissociate readily (have high dissociation constants) and are ionic compounds with large amounts of cations and anions.

Which of the following describes the image formed by an object placed in front of a convex lens at a distance smaller than the focal length? A. Virtual and inverted B. Virtual and upright C. Real and upright D. Real and inverted

B The image produced by a convex lens can either be real or virtual. It is real if the object is placed at a distance greater than the focal point, and virtual if the object is placed at a distance less than the focal point (between the focal point and the lens). Remember that for a single mirror or lens, an image that is real must be inverted and one that is virtual must be upright. In this question, the object is placed in front of the focal point, so the image must be virtual and therefore upright. We could also determine this from the optics equation. If f>0, then 1/f - 1/o si negative and i is therefore negative (virtual).

In a sample of hospital patients, the mean age is found to be significantly lower than the median. Which of the following best describes this distribution? A. Skewed right B. Skewed left C. Normal D. Bimodal

B The mean is to the left of the median, which implies that the tail of the distribution is on the left side; therefore, this distribution is skewed left.

One hundred grams of sucrose are dissolved in a cup of hot water at 80C. The cup of water contains 300mL water. What is the % composition by mass of sugar in the resulting solution? (Note: Sucrose=C12H22O11, density of water at 80C=0.975g/mL) A. 25% B. 25.5% C. 33.3% D. 34.2%

B pg 340 general chemistry mass%=(mass of solute/total mass of solution) x 100% = [100g sucrose/(300mL H2O)(0.975g/mL) + 100g sucrose] x 100% =[100/(300+100)]x 100% = 100/400 x 100% =25% approx keep in mind that in rounding wile calculating, the denominator was estimated to be larger than the actual value, thus giving an answer that is slightly lower than the actual value.

A graph of an exponential decay process is created. The y-axis is the natural logarithm of the ratio of the number of intact nuclei at a given time to the number of intact nuclei at time t=0. The x-axis is time. What does the slope of such a graph represent? A. gamma B. -gamma C. e^-gamma(t) D. n/nsub0

B pg 342 physics The expression n=nsubo^e-λt is equivalent to n/nsubo = e^-λ. Taking the natural logarithm of both sides, we get: ln(n/nsubo) =-λt From this expression, it is clear that plotting ln(n/nsubo) vs t will give a straight line with a slope of -λ

Which of the following best describes the number and character of the bonds in an ammonium cation? A. Three polar covalent bonds B. Four polar covalent bonds, of which none are coordinate covalent bonds C. Four polar covalent bonds, of which one is a coordinate covalent bond D. Four polar covalent bonds, of which two are coordinate covalent bonds

C

Which of the following is a conservative force? A. air resistance B. friction C. gravity D. convection

C

A 150 pound man must be given a drug that is dosed at 1.5 mg drug/kg body mass. Approximately how many milligrams of the drug should be administered per dose? (Note: 1 lb= 4.45N) A. 33mg C. 67mg C. 100mg D. 225mg

C 150lb x 4.45N/lb = 150 x 4.4 = 600+60 = 660 660N/(9.8m/s^2) = (660 kg.m/s^2)/(9.8m/s^2) = 67kg 67kg (1.5 mg drug/kg body mass) = approx 100 mg

The intramolecular reaction of 5-aminopentanoic acid through nucleophilic acyl substitution would result in a(n): A. anhydride B. lactone C. lactam D. carboxylic acid

C 5-aminopentanoic acid contains a carboxylic acid and an amine. If this molecule undergoes intramolecular nucleophilic acyl substitution, it will form a cyclic amide. These molecules are called lactams. Lactones are cyclic esters, not amides

A solution is prepared with an unknown [ ] of a theoretical compound with a Ka of exactly 1.0. What is the pH of this solution? A. Higher than 7 B. Exactly 7 C. Less than 7 D. There is not enough information to answer the question

C A higher Ka implies a stronger acid. Weak acids usually have a Ka that is several orders of magnitude below 1. The pKa of a compound is the pH at which there are equal [ ]s of acid and conjugate base; the pKa of this compond would be -log1=0. With such a low pKa, this compound must be an acid. Therefore, the pH of any [ ] of this compound must be below 7.

A certain carbon nucleus dissociates completely into alpha particles. How many particles are formed? A. 1 B. 2 C. 3 D. 4

C A typical carbon nucleus contains 6 protons and 6 neutrons. An alpha particle contains 2 protons and 2 neutrons. Therefore, one carbon nucleus can dissociated into 6/2=3 alpha particles.

If a light ray has a frequency of 5E14Hz, in which region of the electromagnetic spectrum is it located? A. X-Ray B. UV C. Visible D. Infrared

C It is unnecessary to memorize the entire electromagnetic spectrum for test day; however, it is important to know that the visible spectrum runs from 400-700nm. c=f(wavelength) wavelength=c/f= (3E8m/s)/(5E14Hz)=600nm

Which of the following explanations best describes the mechanism by which solute particles affect the melting point of ice? A. Melting point is elevated because the kinetic energy of the substance increases. B. Melting point is elevated because the kinetic energy of the substance decreases. C. Melting point is depressed because solute particles interfere with lattice formation. D. Melting point is depressed because solute particles enhance lattice formation.

C Melting point depresses upon solute addition, making A and B incorrect. Solute particles interfere with lattice formation, the highly organized state in which solid molecules align themselves. Colder-than-normal conditions are necessary to create the solid structure.

Which of the following data sets is sufficient to determine the linear speed through an area of a rigid pipe? A. The cross-sectional area in another segment of pipe and the cross sectional area in the region of interest. B. The Reynolds number, viscosity of the fluid, density, and diameter of the pipe. C. The radius of the pipe, pressure gradient, viscosity, and length of the pipe. D. The absolute pressure and density.

C Poiseuille's law, which can then be used to determine the linear speed by dividing the cross sectional area

One mole of compound consists of approx 72g carbon and 6g H. What is its empirical formula? A. C6H6 B. CHO C. CH D. C6H12

C The empirical formula is the smallest whole number radio of the elements making up a molecule. A is the molecular formula

Consider the following equation: 6Na(s) + 2NH3(aq) --> 2Na3N(s) + 3H2(g) Which species acts as an oxidizing agent? A. Na B. N in NH3 C. H in NH3 D. H2

C The oxidized agent is the species that is reduced in any given equation. In this problem, six hydrogen atoms with +1 oxidation states in NH3 are reduced to three neutral H2 molecules.

What is the oxidation state of chromium in the dichromate ion, Cr2O72-? A. -2 B. 0 C. +6 D. +12

C The rules for oxidation states are as follows: In a compound, oxidation states of all elements sum to the net charge on the ion or neutral compound. Free elements have an oxidation number of 0. The more electronegative element has a negative oxidation state, fluorine and oxygen being the most and second most electronegative. Group IA elements are +1, Group IIA elements are +2, Group VIIA elements are usually -1 unless bonded to a more electronegative element, oxygen usually -2, hydrogen usually +1, fluorine always -1, chlorine usually -1. In this case, the oxidation states of all elements sums to -2. 7 oxygens have an oxidation state of -2, summing to -14. The 2 chromiums must sum to +12, giving an oxidation state of +6 for each chromium. -2, incorrect, This is the overall oxidation state. 0, incorrect, This answer indicates that 7 oxygens have a total oxidation state of -2. +6, correct. +12, incorrect, This answer neglects the fact that there are 2 chromiums, therefore the +12 must be divided by 2.

Solid Fe(OH)3 is added to distilled water and allowed to reach equilibrium. At that time, [Fe3+]aq= 4 x 10-10 M. Calculate the Ksp of Fe(OH)3. A. 4 x 10-10 B. 4.8 x 10-19 C. 6.9 x 10-37 D. cannot be determined without knowing the [OH-] concentration

C The solubility product, Ksp, is given by the product of the equilibrium concentrations of the dissociated ions [A]a + [B]b in the form Ksp= [A]a[B]b. In this case, FeOH3 dissociates to [Fe3+]aq and 3[OH-]aq. Even though we are not given the concentration of OH-, we know that it must be three times the concentration of Fe3+, since one Fe(OH)3 molecule dissociates into one iron cation and three OH- anions. Therefore, the solubility product Ksp = [Fe3+]1[OH-]3 = [Fe3+]1{3[Fe3+]}3 The math works out to (4 x 10-10) x (27 x 64 x 10-30), and only the correct choice c) is close enough. A. 4 x 10-10, incorrect, This answer disregards the [OH-] concentration. B. 4.8 x 10-19, incorrect, This answer disregards the power of 3 on the [OH-] ion. C. 6.9 x 10-37, correct. D. cannot be determined without knowing the [OH-] concentration, incorrect, The [OH-] concentration is known by the multiplying the given [Fe3+] concentration by 3.

Consider the following fission reaction: FIGURE 16 PHYSICS The masses of the species involved are given in atomic mass units below each species, and 1 amu can create 932 MeV of energy. What is the energy liberated due to transformation of mass into energy during this reaction? A. 0.003 MeV B. 1.4 MeV C. 2.8 MeV D. 5.6 MeV

C This problem presents a reaction and asks for the energy liberated due to transformation of mass into energy. To convert mass into energy, we are told that 1 amu can be converted into 932MeV of energy. All we need to do now is calculate how much mass, in amu, is converted in the reaction. Because we are given the atomic mass for each of the elements in the reaction, this is simply a matter of balancing the equation: mass defect = 1.0087amu+10.0129amu-7.0160amu+4.0026amu =11.0216-11.0186 -0.0030amu Given both the small magnitude of this value and the small difference of the answer choices, it is best to not round at this point of the calculation. This is the amount of mass that has been converted into energy. To obtain energy from mass, we have to multiply by the conversion factor (1amu =932MeV): E=0.003 x 932 = 0.003 x 900 = 2.7MeV At this point we were able to round for an easier calculation that keeps us very near the correct answer choice.

A particle with a net force of 5 N could NOT have which of the combination of forces? A) 2 N and 3 N B) 3 N and 4 N C) 5 N and 11 N D) 14 N and 18 N

C This question asks the examinee to determine a combination of forces that could not result in net force of 5 N. To answer this question, you must consider the range of possibilities given the two forces. The combination of a 2 N and 3 N forces could produce a net force of the smallest magnitude if the forces act in opposite directions. A net force of the greatest magnitude will result if the individual forces act in same direction. Thus, the range for the net force of 2 N and 3 N forces is between 1 N and 5 N. Applying this logic to choice B, the range of the net force is between 1 N and 7 N. For C, the combination of 5 N and 11 N creates a net force in the range of 6 N to 16 N. Lastly, the net force for choice D rages between 4 N and 32 N. Thus, choice C is the only combination that cannot produce a net force of 5 N making C the correct answer.

What is the standard cell potential for the spontaneous reaction below? 2 Ag(s) + Cl2(g) → 2 AgCl(s) Half-reaction Eo (V) Ag+ + e- → Ag 0.80 Cl2 + 2 e- → 2 Cl- 1.36 A. -0.24 V B. -0.56 V C. +0.56 V D. +2.16 V

C This question asks the examinee to determine the standard cell potential for the spontaneous reaction between silver and chlorine gas. Choices A and B can be eliminated because the reaction is spontaneous and should thus have a positive cell potential. The standard cell potential is given by the equation Eocell = Eooxidation + Eoreduction. In the reaction above, silver metal is oxidized and chlorine gas is reduced. According to the table, the standard oxidation potential of Ag is -0.8 V and the standard reduction potential of Cl2 is equal to 1.36 V. Thus, Eocell = -0.8 V+ 1.36 V = 0.56 V, making choice C the correct answer.

A student is analyzing the behavior of a light ray that is passed through a small opening and a lens and allowed to project on a screen a distance away. What happens to the central maximum (the brightest spot on the screen) when the slit becomes narrower? A. The central maximum remains the same. B. The central maximum becomes narrower. C. The central maximum becomes wider. D. The central maximum divides into smaller light fringes.

C This question is testing our understanding of diffraction. When light passes through a narrow opening, the light waves spread out; as the slit narrows, the light waves spread out even more. When a lens is placed between the narrow slit and the screen, a pattern consisting of alternating bright and dark fringes can be observed on the screen. As the slit becomes narrower, the central maximum (the brightest and most central fringe) becomes wider. This can also be seen in the equation for the position of dark fringes in a slit-lens setup (a sintheta=n wavelength). As a, the width of the slit, decrease, sin theta must increase because n(wavelength) is constant for a given fringe. If sin theta increase, theta necessarily increases, implying that the fringes are spreading further apart.

The electronegativity of elements increases going from left to right across a row in the periodic table. Which of the following best accompanies this trend in the periodic table? A. A decrease in ionization energy and an increase in atomic radii B. A decrease in atomic radii and a decrease in ionization energy C. An increase in ionization energy and a decrease in atomic radii D. An increase in ionization energy and an increase in atomic radii

C This question tests the examinees knowledge of trends within the periodic table. In general, as you move from left to right across a row in the periodic table, electronegativity and ionization energy increases while atomic radii decrease.

How many liters of 2M Ba(OH)2 are needed to titrate a 4L solution of 6M H3PO4? A. 1.33L B. 12L C. 18L D. 56L

C Use the equivalence point equation:NaVa=NbVb Ba(OH)2 can dissociate to give two hydroxide ions, so its normality is 2M x 2 = 4N. H3PO4 can dissociate to give three hydronium ions, so its normality is 6M x 3=18N. Plugging into the equation, we get (18N)(4L)=(4N)(Vb). Therefore, Vb is 18L.

One way to test for the presence of iron in solution is by adding potassium thiocyanate to the solution. The product when this reagent reacts with iron is FeSCN2+, which creates a dark red color in solution via the following net ionic equation: Fe3+ + SCN- --> FeSCN2+ How many grams of iron sulfate would be needed to produce 2 moles of FeSCN2+? A. 110g B. 220g C. 400g D. 500g

C When you are shown is a net ionic equation. If two moles of FeSCN are created, two moles of Fe3+ must be used because the mole ratio is 1:1. Iron sulfate has the formula Fe2(SO4)3 because sulfate has a charge of -2 and iron has a charge of +3 (based on the net ionic equation). Therefore, one mole of iron sulfate is needed to make two moles of iron for the reaction. The molar mass of iron sulfate is: 2 x 55.8g/mol + 3 x 32.1g/mol + 12 x 16.0g/mol = 399.9g/mol This most closely matches C. The most common error would be to calculate the amount of iron, which could be A

If the work function of a metal is 6.622E-20J and a ray of electromagnetic radiation with a frequency of 1E14Hz si incident on the metal, what will be the speed of the electrons ejected from the metal? (Note: h=6.626E-34Js and msube-=9.1E-31kg) A. 2.62E-6m/s B. 1.07E-4m/s C. 9.38E3m/s D. 3.81E5m/s

C math on pg 340 physics To determine the speed of the electrons ejected, we must first calculate their kinetic energy; K=hf-W =(6.626E-34Js)(1E14Hz)-6.622E-20J =(6.626-6.622)E-20 =approximately 0 Notice that our initial attempt at estimation via truncation led to a nonviable answer of 0. This sometimes occurs when attempting to estimate. The key is to take a step back and repeat the calculation using the exact numbers. =(6.626-6.622)E-20 = (0.004E-20)=(4E-23J) Now with a usable value for inetic energy, we can calculate the speed of the ejected electrons: K=1/2mv^2 --> v=sqrt(2K/m) =sqrt[(2(4E-23J)/(9.1E-31kg) = sqrt(8E-23)/(9E-31) = sqrt10^8 = 10^4m/s Notice the wide range in the exponents for the answer choices. While the math in this question may seem complex, this allows us to round significantly.

The isotope 12C is not useful for NMR becauase A. it is not abundant in nature B. its resonances are not sensitive to the presence of neighboring atoms C. it has no magnetic moment D. its signal-to-noise ratio in the spectrum is too low

C no magnetic moment and will therefore not exhibit resonance with an applied magnetic field. Nuclei with odd mass numbers or those with an even mass number but an odd atomic number will have a nonzero magnetic moment

What is the binding energy of the argon-40 isotope in MeV? (Note: m proton=1.0073amu, m neutron=1.0087amy, m AR-40 nucleus = 39.9132amu, c^2=932MeV/amu) A. 0.4096 MeV B. 40.3228 MeV C. 381.7 MeV D. 643.8 MeV

C pg 341 physics To determine the binding energy, we must first determine the mass defect. The mass defect is simply the masses of each of the protons and neutrons in the unbound state added together minus the mass of the formed argon-40 nucleus (which contains 18 protons and 40-18=22 neutrons). mass defect=(18)(1.0073amu) + (22)(1.0087amu)-39.9132amu = (18)(1.007)+(22)(1.009)-39.9132=18.126+22.198-39.9132 =18.126+22.198-39.9132 =40.324-39.9132 =0.4(actual=0.4096) This math was difficult without a calculator, but by rounding one value down (proton) and one value up (neutron) by similar amounts we ended up very near the actual value. Calculating 18 x 7=126 and 22 x 9=198 for the decimal values is more manageable than the exact numbers and the spacing of the answer choices allows for our estimation. The binding energy can then be determined from this mass defect: E=mc^2=0.4096amu x 932MeV/amu =0.4 x 900 =360MeV

Which of the following is necessarily true regarding frequency, angular frequency, and period of a given wave? A. The magnitude of the angular frequency is larger than the magnitude of the period B. The product of the frequency and period is equal to the angular frequency C. The magnitude of the angular frequency is larger than the magnitude of the frequency D. The produce of the angular frequency and period is 1

C physics pg 266 The angular frequency is related to the frequency through the equation w=2(pi)f. Therefore, the magnitude of the angular frequency will always be larger than the magnitude of the frequency. The magnitude of the angular frequency may or may not be larger than the magnitude of period; these variables are inversely proportional, eliminating A. The product of the frequency and the period is always 1 because these two are inverses of each other, eliminating B. Finally, the product of the angular frequency and period will always be 2pi because w=2(pi)f=(2pi/T), eliminating D.

A certain sound level is increased by 20dB. By what factor does its intensity increase? A. 2 B. 20 C. 100 D. log 2

C physics pg 267 let Ii be the intensity before the increase and If be the intensity after the increase. Using the equation that relates sound level to intensity, obtain the ratio of Ii to If: Bf=Bi +10log(If/Ii) --> Bf-Bi= 10 log (If/Ii) 20dB=10log(If/Ii) 2=log (If/Ii) 100=If/Ii

The rate of a reaction is calculated as a change in [ ] per time. What are the units of the rate constant, k, in a reaction that is second order overall with respect to one species? (Note: A second-order reaction of this type has a rate law with the form rate = k[A]^2, where [A] is the [ ] of the species.) A. 1/s B. M/s C. L/mol . s D. L^2/mol^2 . s

C rate = k[A]^2 M/s = k[M]^2 --> k = 1/M.s = L/mol . s

Use of a colorimetric assay to determine protein concentration may be subject to all of the following EXCEPT: A. the use of standards B. measurement error C. the Hawthorne effect D. systematic error

C the Hawthorne effect is a change in behavior as a result of the knowledge that one is being observed. its only observed in human subjects. Basic science research generally does not suffer from the Hawthorne effect.

What is the molecular formula of a compound with an empirical formula of B2H5 and a molar mass of 53.2 g/mol? A. B2H5 B. B3H7 C. B4H10 D. B6H15

C the simplest approach is to determine the molar mass of the empirical formula. B2H5 has a molar mass of 26.6 g/mol. A molecular formula is always a multiple of the empirical formula; doubling this quantity will result in the molar mass 53.2 g/mol.

A reaction is found to stop just before all reactants are converted to products. Which of the following could be true about this reaction? A. The reaction is irreversible and the forward rate is greater than the reverse rate B. The reaction is irreversible and the reverse rate is too large for products to from. C. The reaction is reversible, and the forward rate is equal to the reverse rate. D. The reaction is reversible, and the reverse rate is greater than the forward rate

C this scenario likely describes a situation in which a reaction has reached equilibrium very far to the right (with high product [ ] and low reactant [ ]). this reaction must be reversible because the reaction did not proceed all the way to the right. Any reaction in equilibrium has equal forward and reverse rates of reaction

Compound A has a Ka (equilibrium constant of acid dissociation) of approx 10^-4. Which of the following compounds is most likely yo react with a solution of compound A? A. HNO3 B. NO2 C. NH3 D. N2O5

C Ka is equal to the ratio of products to reactants, with each species raised to its stoichiometric coefficient. A compound with a Ka greater than 10^-7 contains more H+ cations than HA- anions at equilibrium, which makes it an acid, This means that the compound in question is likely to react with a compound that is basic. Of the four answer choices, NH3 is the only base

What is the effect of adding KNO3 to a saturated aqueous solution of KCl? A. Ksp increases because more K+ ions are present. B. Ksp decreases because NO3- ions displace Cl- ions. C. The concentration of Cl- ions decreases because KNO3 addition drives KCl dissociation towards the solid, undissociated form. D. More KCl dissolves because more aqueous K+ ions require more Cl- ions

C is correct. Group IA elements, Cl- salts, and nitrate (NO3-) salts are soluble. Ksp for a compound does not change, although the concentration of dissolved species may. When a product concentration is increased in an equilibrium reaction, reactant concentration increases according to Le Chatelier's principle. Here, addition of K+ ions to a saturated aqueous solution of KCl will drive KCl dissociation in the direction of the reactants and cause precipitation of solid KCl. The concentration of Cl- ions in solution will decrease such that the Ksp remains the same for the new product of Ksp=[K+][Cl-]. The only time Ksp changes is when the temperature is changed, which is not the case here. A. This choice states that Ksp increases because more K+ ions are present. Incorrect, Ksp does not change as a result of concentration. B. This answer states that Ksp decreases because NO3- ions displace Cl- ions. Incorrect, Ksp does not change as a result of concentration. C. This option correctly states that the concentration of Cl- ions decreases because KNO3 addition drives KCl dissociation towards the solid, undissociated form. D. In this answer choice, more KCl dissolves because more aqueous K+ ions require more Cl- ions. Incorrect, this contradicts Le Chatelier's principle. In an accurate description of this principle, higher product concentrations should drive the equilibrium towards the reactant, not the product, side. Remember, Le Chatelier's principle states that a reversible process will shift in the direction that relieves stress and re-establishes equilibrium.

A dipole is placed in an electric field and is allowed to come to equilibrium. How would the dipole react if the direction of the electric field is suddenly reversed A. It rotates to align with the new field B. It accelerates linearly along the field lines C. It experiences no rotational or linear movement D. It both rotates to align with the new field and accelerates linearly along the field lines.

C. Torque is a function of both force applied and the angle at which it is applied. A dipole placed in an electric field will experience a torque until it comes to rest oriented within the field, at which point the angle between the plane of the dipole and the electric field is 0 degrees. Once this point is reached, inverting the electric field has no impact on the dipole because it will now have an angle of 180 degrees, the sine of which is still 0. Note, however, that this is an unable setup; any deviation in the dipole from its rest position will immediately result in torque on the dipole and force it to realign with the new field.

The following electron configurations represent elements in their natural form. Which element is the strongest oxidizing agent? A. 1s2 2s2 2p6 3s2 3p6 4s2 B. 1s2 2s2 2p6 3s2 3p6 4s2 3d5 C. 1s2 2s2 2p6 3s2 3p6 4s2 3d10 4p1 D. 1s2 2s2 2p6 3s2 3p6 4s2 3d10 4p5

D A strong oxidizing agent will be easily reduced, meaning that it will have a tendency to gain electrons. Atoms usually gain electrons if they are one or two electrons away from filling up their valence shell. A has a full 4s orbital, meaning that it can only gain an electron if it gains an entire subshell. B has a stable, half-full 3d-orbital, so it is unlikely to pick up electrons unless it can gain five. C has only a single electron in the outer shell, which is more likely lost upon ionization. D would fill up its 4p-orbital by gaining one electron, so it is easily reduced

Which of the following relationships between measurement error and overall error is correct? A. Unreliable data leads to confounding B. Invalid data leads to confounding C. Unreliable data leads to bias D. Invalid data leads to bias

D Data that is off in a systemic way (reads at a value that is not the true value) will cause bias. This type of data error is an example of a lack of validity (or accuracy). Unreliable data suffers from random, not systemic, error. Confounding arises from errors in data analysis, not data collection.

Imagine that a beam of monochromatic light originates in air and is allowed to shine upon the flat surface of a piece of glass at an angle of 60 degrees with the normal. The reflected and refracted beams are perpendicular to each other. What is the index of refraction of the glass? A. (sqrt3)/3 B. 1 C. 2 D. sqrt3

D Drawing a diagram is best here. Use drawing on pg 310 physics or picture in phone

The standard reduction potentials for Zn and Cu are given below: Zn2+(aq) + 2e- → Zn(s) E°= -0.76 V Cu+(aq) + e- → Cu(s) E°= 0.52 V What is the standard state cell potential for the following reaction? Zn(s) + 2Cu+(aq) → Zn2+(aq) + 2Cu(s) A. -1.8V B. -1.28V C. 0.28V D. 1.28V

D Given the standard reduction potentials: Zn2+(aq) + 2e- → Zn(s) E°= -0.76 V Cu+(aq) + e- → Cu(s) E°= 0.52 V The reduction potentials are used as is or multiplied by -1 if the reaction direction is reversed. Stoichiometry does not matter, the reduction potentials are summed for the standard cell potential. Zn(s) + 2Cu+(aq) → Zn2+(aq) + 2Cu(s) 0.76V + 0.52V=1.28V -1.8V, incorrect, This uses Zinc in the wrong direction (it should be +0.76V), and copper is also in the wrong direction and multiplied twice. There should be no multiplying for stoichiometric coefficients. -1.28V, incorrect, This uses Zinc in the wrong direction and copper in the wrong direction. 0.28V, incorrect, This uses Zinc in the right direction and Cu in the wrong direction. 1.28V, correct.

When glucose is in a straight-chain formation, it: A. is an aldoketose B. is a pentose C. has five chiral carbons D. is one of a group of 16 stereoisomers

D Glucose is an aldohexose, meaning that it has one aldehyde group and six carbons. Given this info, A and B can be eliminated. In aldose sugars, each nonterminal carbon is chiral. Therefore, glucose has four chiral centers, not five, as mentioned in C. The number of stereoisomers possible for a chiral molecule is 2^n, where n is the number of chiral carbons. Because glucose has four chiral centers, there are 2^4=16 possible stereoisomers.

Which of the following amino acids does not have an L-enantiomer? A. Cysteine B. Threonine C. Glutamic acid D. Glycine

D Glycine's R group is a hydrogen atom; this amino acid is therefore achiral because the central carbon is not bonded to four different substituents.

Which two polysaccharides share all of their glycosidic linkage types in common? A. Cellulose and Amylopectin B. Amylose and Glycogen C. Amylose and Cellulose D. Glycogen and Amylopectin

D Glycogen and amylopectin are the only polysaccharide forms that demonstrate branching structure, making them most similar in terms of linkage. Both glycogen and amylopectin use a-1,4 and a-1,6 linkages. Cellulose uses B 1,4 linkages and amylose does not contain a-1,6 linkages

Andersen's disease (glycogen storage disease type IV) is a condition characterized by a deficiency in glycogen branching enzyme. Absence of this enzyme would be likely to cause all of the following effects EXCEPT: A. decreased glycogen solubility in human cells B. slower action of glycogen phosphorylase C. less storage of glucose in the body D. glycogen devoid of a-1,4 linkages

D In Andersen's disease, glycogen is less branched than normal, thereby inducing lower solubility of glycogen. Branches reduce the interactions between adjacent chains of glycogen and encourage interactions with the aqueous environment. The smaller number of branches means that glycogen phosphorylase has fewer terminal glucose monomers on which to act, making enzyme activity slower than normal overall. Finally, without branches, the density of glucose monomers cannot be as high; therefore, the total glucose stored is lower than normal. Glycogen synthase is still functioning normally, so we would expect normal a-1,4 linkages in the glycogen of an individual with Andersen's disease but few (if any) a-1,6 linkages

In a certain rigid container, pressure and temperature are directly proportional. If the pressure is changed from 540 torr to 180 torr via a temperature change, by what factor has the temperature changed? A. 360 B. 3 C. 1 D. 1/3

D In a direct relationship, a change in one variable will be associated with a proportional change in the other. Because the pressure was multiplied by 1/3, the temp must also be multiplied by 1/3.

A nuclide undergoes two alpha decays, two positron decays, and two gamma decays. What is the difference between the atomic number of the parent nuclide and the atomic number of the daughter nuclide? A. 0 B. 2 C. 4 D. 6

D In alpha decay, an element loses two protons. In positron decay, a proton is converted into a neutron. Gamma decay has no impact on the atomic number of the nuclide. Therefore, two alpha decays and two positron decays will yield a daughter nuclide with six fewer protons than the parent nuclide.

A child stands between two mirrors with his arms out, perpendicular to the mirrors. One plane mirror is 5m away from his left hand and another plane mirror is 7m away from his right hand. How far apart are the two images produced by the mirrors if the child has an arm span of 0.5m? A. 2m B. 12m C. 12.5m D. 24.5m

D In plane mirrors, the image is as far away from the mirror as the object is. In other words, the image produced by the left mirror is 5m away from the mirror because the child is standing 5m away from the mirror. Similarly, the right mirror produces an image that is 7m away from the center of the mirror. To calculate how far away the two images are, take into consideration not only the image distance but also the distance of the object (the child) from the mirrors and the child's arm span of 0.5m. Therefore, the images are 5+5+0.5+7+7=24.5m apart

If IR spectroscopy were employed to monitor the oxidation of benzyl alcohol to benzaldehyde, which of the following would provide the best evidence that the reaction was proceeding as planned? A. Comparing the fingerprint region of the spectra of starting material and product B. Noting the change in intensity of the peaks corresponding to the benzene ring C. Noting the appearance of a broad absorption peak in the region of 3100-3500 cm^-1 D. Noting the appearance of a strong absorption in the region of 1750 cm^-1

D In this reaction, the functional group is changing from a hydroxyl to an aldehyde. This means that a sharp peak will appear around 1750cm-1., which corresponds to the carbonyl functinoality.

Ignoring attenuation, how does the intensity of a sound change as the distance from the source doubles? A. tis four times as intense B. It is twice as intense C. It is half as intense D. It is one-quarter as intense

D Intensity is equal to power divided by area. In this case, area refers to the surface area of concentric spheres emanating our from the source of the sound. This surface area is given by 4(pi)r^2, so as distance (r) doubles, the intensity will decrease by a factor of four.

Which of the following will not result in the splitting of white light into its component colors? A. Dispersion through a prism B. Diffraction through a grating C. Refraction within a thin film D. Reflection from an ideal convex mirror

D Light can be split into its component colors by dispersion such as that through a prism, eliminating A. Diffraction by a diffraction grating will also separate colors by their wavelengths, eliminating B. The refraction of light within a thin film also leads to light dispersion as the different colors are refracted at slightly different angles in the film, eliminating C. A mirror with significant aberration could lead to a separation of light into its component colors but we are told that this is an ideal mirror.

Carbon dioxide at -20°C is pressurized at 100 atm. If the CO2 undergoes isobaric cooling to -65°C followed by isothermal depressurization to 1 atm, what are the original phase and subsequent phase(s) of the CO2? A. liquid, gas, solid B. gas, solid C. liquid, gas D. liquid, solid, gas

D Phase changes are read from a phase diagram, a plot of compound or element phase as a function of temperature and pressure. Note some commonly used terms when describing phase changes: isobaric indicates constant pressure, isothermal indicates constant temperature. Adiabatic indicates no heat transferred, but it applies to thermodynamic cycles more than phase diagrams. A substance's phase is read by plotting indicated temperatures and pressures on the graph. Here, -20°C and 100 atm indicates a starting phase of liquid. If the CO2 undergoes isobaric cooling to -65°C, it becomes solid (freezes). The isothermal depressurization to 1 atm causes sublimation to a final gas phase. A. liquid, gas, solid, incorrect, Isobaric cooling leads to a solid phase rather than gas from the starting liquid phase. Also, isothermal depressurization causes a vertical decrease on the graph which cannot result in a gas to solid phase change. B. gas, solid, incorrect, The starting phase is liquid, and the isobaric change leads to a solid phase followed by isothermal depressurization and a final gas phase. C. liquid, gas, incorrect, The starting liquid phase undergoes isobaric cooling leading to a solid phase rather than gas. D. liquid, solid, gas, correct.

Pure sodium metal spontaneously combusts upon contact with room temp water. What is true about the equilibrium constant of this combustion reaction at 25C? A. Keq < 0 B. 0 < Keq < 1 C. Keq = 1 D. Keq > 1

D Solve this question using the equation deltaGrxn^not=-RTlnKeq deltaG is negative (spontaneous reaction) R and T are always positive K must also be positive for the sign convention to work out correnctly Since ln(1)=0, the natural log of any number greater than 1 will be positive, and the natural log of any number less than 1 will be negative.

In which of the following media does sound travel the fastest? A. Vacuum B. Air C. Water D. Glass

D Sound is a mechanical disturbance propagated through a deformable medium; it is transmitted by the oscillation of particles parallel to the direction of the sound wave's propagation. As such, sound needs matter to travel through, eliminating A. The speed of propagation is fastest in solid materials, followed by liquids, and slowest in gases.

The coupling constant, J, is A. the value of n+1 when determining splitting in NMR spectra B. measured in parts per million (ppm) C. corrected for by calibration with tetramethylsilane D. a measure of the degree of splitting caused by other atoms in the molecule

D The coupling constant is a measure of the degree fo splitting introduced by other atoms in a molecule, and is the frequency of the distance between subpeaks. It is measured in hertz, eliminating B. The coupling constant is independent of the value of n+1, and is not changed by calibration with tetramethylsilane.

Which of the following reactions would produce the compound below FIGURE 4 (organic chemistry) A. CH3CHO + CH3CH2CH2CHO B. CH3COCH3 + CH3CH2CH2CHO C. CH3CH2COCH3 + CH3CHO D. CH3CH2CHO + CH3CH2CHO

D The reactions listed in the answer choices are examples of aldol condensations. In the presence of a base the alpha hydrogen is abstracted from an aldehyde, forming an enolate ion, [CH3CHCHO]-. This enolate ion then attacks the carbonyl group of the other aldehyde molecule, CH3CH2CHO, forming the pictured aldol.

All of the following are true with respect to carbonyls EXCEPT: A. the carbonyl carbon is electrophilic B. the carbonyl oxygen is electron-withdrawing C. a resonance structure of the functional group places a positive charge on the carbonyl carbon D. the pi electrons are mobile and are pulled toward the carbonyl carbon

D The reactivity of the carbonyl can be attributed to the difference in electronegativity between the carbon and oxygen atoms. The more electronegative oxygen atom attracts the bonding electrons and is therefore electron-withdrawing. Thus, the carbonyl carbon is electrophilic. One resonance structure of the carbonyl pushes the pi electrons onto the oxygen, resulting in a positively charged carbonyl carbon

What would be the product(s) of the reaction in FIGURE 9 ORGANIC CHEMISTRY? A. B. C. D.

D This question acts for the products when ammonia reacts with acetic anhydride. An amide and carboxylic acid will be formed. However, the carboxylic acid-an acid-is in the same environment as ammonia-a base. The two will react, forming the ammonium carboxylate shown.

What is the density of neon gas in g/L at STP? A. 452.3 B. 226 C. 1.802 D. 0.9018

D math on pg 295 gen chem density=mass/volume 20.2g/20.4L=0.092g/L

What is the angular frequency of the third harmonic in a pipe of length 1.5m with one closed end? (Note: The speed of sound is approx. 340m/s). A. 170 radians per second B. 170 radians(pi) per second C. 340 radians per second D. 340 radians(pi) per secon

D physics pg 267 The angular frequency is related to the frequency of a wave through the formula w=2pi(f). thus, our initial task is used to calculate the frequency of the wave. Knowing its speed, we determine the frequency of the wave by first calculating wavelength (v=fλ). For the third harmonic of a standing wave in a pipe with one closed end, the wavelength is λ=4L/n= 4(1.5m)/3= 2m the frequency of the wave is therefore f=v/λ = 340m/s/2m=170Hz Finally, obtain the angular frequency by multiplying the frequency fo the wave by 2pi w=2pi(f)=340pi radians per second

How much heat is required to completely ment 500g gold earrings, given that their initial temp is 25C? (The melting point of gold is 1064C, its heat of fusion is 6.37 x 10^4 J/kg, and its specific heat is 126 J/kg . K) A. 15 kJ B. 32 kJ C. 66 kJ D. 97 kJ

D q=mcdeltaT = 0.5kg (126) (1039K) = 60kJ q=mL = 0.5kg (6.37E4 J) =32kJ 60 + 32 =92

What is the equilibrium expression for the reaction: Cu2SO4(s) <--> 2Cu+(aq) + SO4^2-(aq) See Figure 1 Chemistry A. B. C. D.

D recall that pure solids and liquids do not appear int he equilibrium expression

Vinblastine is a microtubule-disrupting drug that inhibits tubulin polymerization. Which of the following processes would be directly inhibited upon vinblastine treatment?I. Phagosome transport to the lysosomeII. MitosisIII. MeiosisIV. Electron transport A. II only B. I and IV only C. II and III only D. I, II, and III only

D is correct. Microtubules are used in the transport of vesicles and the positioning of organelles within the cell. As part of this role, they form structures that assist in the transport of phagosomes (vesicles that contain particles that have been engulfed via phagocytosis) to the lysosomes of the cell, to which the phagosomes fuse. As such, RN I is correct. RN II and RN III are also correct, since microtubules form the spindle apparatus that is an essential part of both mitosis and meiosis. The general structure of a microtubule is shown below; note that these structures are composed of dimers of the protein tubulin. IV: Microtubules play no direct role in electron transport. A: RN I and RN III are also correct. B: RN II and RN III are also correct, while RN IV is incorrect. C: RN I is also correct. Content Foundations: Structural Proteins The most common function of proteins in the body is not enzymatic, but structural. Structural proteins are fibrous proteins that have an elongated shape and provide structural support for cells and organ tissues. The first type of fibrous proteins are keratins, which form the skin, hair, and nails. Keratins are classified as soft or hard according to their sulfur content (i.e. the relative number of cysteine residues in their polypeptide chains). The low-sulfur keratins of the skin are much more flexible than the high-S, hard keratins. The second type of fibrous proteins you must know are the actin and myosin proteins of muscle tissue. Actin and myosin interact to form cross-linkages that allow the sliding of the filaments over each other in muscle contraction, which takes place through the contraction and relaxation of the sarcomere, the fundamental unit of all muscle fibers. When muscle contracts, the actin and myosin filaments slide over each other and the H-zone (myosin-only region), Z-lines (sarcomere boundaries), and I-band (actin-only region) all shrink, while the A-band (the entire myosin region) remains the same size. The opposite occurs upon muscle relaxation. A third type of structural protein you should know for test day is collagen, which is found in tendons, forms connective ligaments within the body, and gives extra support to the skin. Collagen is a triple helix formed by three proteins that wrap around one another. Many collagen molecules are cross-linked together in the extracellular space to form collagen fibrils to provide structural support for the cell. Elastin polypeptide chains are cross-linked together to form flexible, elastic fibers that give stretched tissues flexibility and the ability to recoil spontaneously as soon as the stretching force is relaxed.

In a protic solvent, which of the following halogens would be the best nucleophile? A. Br- B. Cl- C. F- D. I-

D. In a protic solvent, the protons in solution can attach to the nucleophile, decreasing its nucleophilicity. The larger the nucleophile, and the stronger its conj acid, the stronger the nucleophile will be. Of the options give, I- will be the strongest nucleophile b/c it is least likely to associate with the protons in solution

The behavior of gas will deviate from ideal behaviors LEAST under which of the following sets of conditions: A. A gas at high pressure and temperature B. A gas at low pressure and temperature C. A gas at high pressure and low temperature D. A gas at low pressure and high temperature

D. The ideal gas law, PV=nRT, is based on two assumptions: 1) individual molecular volume is negligible, and 2) intermolecular forces do not exist. A real gas at high temperature and low-pressure conditions, best approximates these two assumptions, and thus the ideal gas law.

Which of the following solvents would be LEAST useful for a nucleophile-electrophile reaction? A. H2O B. CH3CH2OH C. CH3SOCH3 D. CH3CH2CH2CH2CH2CH3

D. To carry out a nucleophile-electrophile reaction, the nucleophile must be able to dissolve in the solvent. Nucleophiles are nearly always polar and often carry a charge. Polar solvents are therefore preferred for these reactions. Hexane is a nonpolar solvent and will not be useful for a nucleophile-electrophile reaction

Explain the respiratory system in terms of pressure

During inspiration, there is a neg pressure gradient that moves air into the lungs. During expiration this gradient reverses. When air reaches the alveoli, it has essentially no speed.

The moon's gravity is 1/6 that of the earth. On earth, the buoyant force on a blueberry in a beer equals the weight of the blueberry, and it bounces up and down in the beer. On the moon, will the blueberry: a) sink and stay down b) float and stay at the surface of the beer c) bounce the same as if on earth d) float away

Gravity contributes equally to both the weight of an object, mg, and the bouyant force of an object, vg. Since the gravity contributes equally, even though it is 1/6 its value on earth, the blueberry bounces in the beer just the same. Note, this does not work with every beer, it must be a slightly darker beer with just the right density. a) sink and stay down, incorrect, The buoyant force is still equal to the weight although both are decreased in magnitude. b) float and stay at the surface of the beer, incorrect, The weight still equals the buoyant force although both are decreased in magnitude. c) bounce the same as if on earth, correct. d) float away, incorrect, This would occur if there were no gravity and the blueberry an external force.

explain the properties of the alkali metals. which group are they?

Group IA or 1 densities Lower than other metals have only 1 loosely bound electron in their outermost shell (Zeff super low). super large atomic radii. low ionization energy. low electron affinity. low electronegativity easily react with nonmetals (esp halogens) ex: NaCl violent reaction with water

explain the properties of the alkaline earth metals. which group are they?

Group IIA or 2. share most of the characteristics of alkali metals except they have slightly higher Zeff and thus slightly smaller atomic radii

what are the properties of the noble gasses? what group are they?

Group VIIIA or 18. inert gases. minimal chemical reactivity due to their filled valence shells high ionization energies, little to no tendency to gain or lose electrons, and no measurable electronegativities (for He, Ne, and Ar) low boiling points, exist as gases as room temp.

Au2S3(s) + H2(g) --> Au(s) + H2S (g) If 2 moles of Au2S3 (s) is reacted with 5 moles of hydrogen gas, what is the limiting reagent?

H2(g) Because Au and H2S are produce, they can't be limiting reagents. The problem says that 2 moles of gold(III) sulfide and 5 moles of hydrogen gas are available. To use up both moles of gold(III) sulfide, 6 moles of hydrogen gas are needed because there is a 1:3 ration between these reactants. Since only 5 moles of hydrogen gas are present, it is the limiting reagent.

What are heat of fusion and heat of vaporization?

Heat of fusion is heat added or taken away during a phase change between solids and liquids. Heat of vaporization is heat added or taken away during a phase change between liquids and gases.

A certain chemical reaction has the following rate law: rate=k[NO2][Br2] Which of the following statements necessarily describe(s) the kinetics of this reaction? I. The reaction is second-order II. The amount of NO2 consumed is equal to the amount of Br2 consumed III. The rate will not be affected by the addition of a compound other than NO2 and Br2

I Only. If the sum of the exponents (orders) of the [ ]s of each species in the rate law is equal to 2, then the reaction is second order. The exponents in the rate law are unrelated to stoichiometric coefficients, so NO2 and Br2 could have any stoichiometric coefficients in the original reaction and still be a second-order reaction. Statement III is incorrect because the rate can be affected by a wide variety of compounds. A catalyst, for example, could increase the rate.

Which of the following types of reactions generally have the same number of reactants and products? I. Double-displacement II. Single-displacement III. Combination

I and II

Consider (E)-2-butene and (Z)-2-butene. This is a pair of what type of isomers? I. Cis-trans isomers II. Diastereomers III. Enantiomers

I and II (E)-2-butene can also be called trans-2-butene; (Z)-2-butene can also be called cis-2-butene. As such, they are cis-trans isomers. Remember that cis-trans isomers are a subtype of diastereomers in which the position of substituents differs about an immovable bond. Diastereomers are molecules that are non-mirror-image stereoisomers.

A hypothesis test was correctly conducted and the experimenter failed to reject the null hypothesis. Which of the following must be true? I. The p-value was greater than alpha II. A type I error did not occur III. The power of the study was too small

I and II A type I error occurs when the null hypothesis is incorrectly rejected. Because we failed to reject the null hypothesis, this could not have occurred. Statement I is correct. If we failed to reject the null hypothesis, the p-value must be greater than the significance level. Statement III is incorrect because we lack info about power in the question stem. In addition, a study could be extremely well-powered and still fail to reject the null hypothesis if no difference truly exists between the two populations.

Why does the equilibrium between keto and enol tautomers lie far to the keto side? I. The keto form is more thermodynamically stable II. The enol form is lower energy III. The enol form is more thermodynamically stable

I only The keto-enol equilibrium lies far to the keto side because the keto form is significantly more thermodynamically stable than the enol form. This thermodynamic stability stems from the fact that the oxygen is more electronegative than the carbon, and the keto tautomer puts more electron density around the oxygen than the enol tautomer. If the enol tautomer is less thermodynamically stable, it is also higher energy than the keto tautomer.

Beta-lactams are: I. cyclic forms of the least reactive type of carboxylic acid derivative II. more reactive than their straight-chain counterparts III. molecules with high levels of ring strain

I, II, and III beta lactams are amides in the form of four-membered rings; amides are generally the least reactive type of carboxylic acid derivative. B-lactams experience significant ring strain from both eclipsing interactions (torsional strain) and angle strain, and are therefore more susceptible to hydrolysis than the linear form of the same molecule.

Which of the following are able to produce a virtual image? I. Convex lens II. Concave lens III. Plane mirror

I, II, and III. All images produced by place mirrors will be virtual, so statement III is true. The same goes for diverging species (convex mirrors and concave lenses), so statement II is true. Converging species (concave mirrors and convex lenses) can produce real or virtual images, depending on how far the object is from the species, so statement I is also true.

An experimenter is attempting to determine the effects of smoking on very low birth weight (VLBW) and of VLBW on IQ. Which of the following statements is correct? I. Smoking is an independent variable II. Smoking is a dependent variable III. VLBW is an independent variable IV. VLBW is a dependent variable

I, III, and IV. Two relationships are being assessed. In the relationship between smoking and very low birth weight (VLBW), smoking is the independent variable and VLBW is the dependent variable. In the second relationship, VLBW is being compared to IQ. VLBW is the independent variable here, while IQ is the dependent variable.

What are the magnitudes and directions of the resultant vectors from the following cross products: C = A x B and D = B x A A: X = -3 N, Y = 0 B: X = 0, Y = +4m

IAI x IBI x sin 90 = 3 N x 4 m x 1 = 12 Nm C is 12 Nm into the page D is 12 Nm out of the page - to determine C, start by pointing your right thumb towards the left (neg x direction). Your fingers will point towards the top of the page (pos y direction). Your palm is pointing out of the page - D is opposite

what are the 3 types of systems? what's the difference?

Isolated systems- incapable of exchanging energy or matter with their surroundings. deltaU=0. very rare. Closed systems- capable of exchanging energy, but not matter, with the surrounding environment. Open systems-capable of exchanging both matter and energy with the environment.

what is the photoelectric effect? What happens to electrons during the photoelectric effect?

It is the release of photoelectrons released by photons with an energy (E=hf) above the work function. When light of a sufficiently high frequency (typically blue to ultraviolet light) is incident on a metal in a vacuum, the metal atoms emit electrons. Electrons liberated from the metal will produce a net charge flow per unit time (current). Provided that the light beam's frequency is above the threshold frequency of the metal, light beams of greater intensity produce larger current in this way. The higher the intensity of the light beam, the greater the number of photons per unit time that fall on an electrode, producing a greater number of electrons per unit time liberated from the metal. When the light's frequency is above the threshold frequency, the magnitude of the resulting current is directly proportional to the intensity (and amplitude) of the light beam.

What are Lewis acids and bases?

Lewis acids accepts electrons while lewis bases donate electrons.

Where are valence electrons located? What kind of interactions do they have with the nucleus and the surrounding environment?

Located farthest from the nucleus. stronger interactions with the surrounding environment and weakest interactions with the nucleus.

What are the 3 types of intermolecular forces? Explain them.

London dispersion forces (a type of van Der Waals force)- weakest of all intermolecular interactions because they are the result of induced dipoles that change and shift moment to moment. Do not extend over long distances, so they are significant only when molecules are in close proximity. Dipole-dipole: present in solid and liquid phase but become negligible in the gas phase b/c of the inc distance between gas particles. Hydrogen bonds- not actual bonds. can be inter or intramolecular. When H atom is bonded to N, O, or F. (Hydrogen picks up the FON (phone))

What is a supercritical fluid?

When temperature exceeds critic temperature and pressure exceeds critical pressure and you can't tell it is a liquid or gas

Explain the retardation factor in TLC.

When thin layer chromatography is performed, compounds are generally identified using the retardation factor (Rf), which is relatively constant for a particular compound in a given solvent. Rf=distance spot moved/distance solvent front moved. Rf can be used to identify unknown compounds.

Explain how parallel plate capacitors work.

When two electrically neutral metal plates are connected to a voltage source, positive charge builds up on the plate connected to the positive (higher potential) terminal, and negative charge builds up on the plate connected to the negative (lower potential) terminal. The two-plate system is a capacitor because it can store a particular amount of charge at a particular voltage. The separation of charges sets up a uniform electric field between the plates with parallel field vectors. The direction of the electric field at any point between the plates is from the positive plate toward the negative plate.

Charges, if allowed, will move spontaneously in whatever direction results in what? Positive charges will move spontaneously in the direction that does what? Negative charges will move spontaneously in the direction that does what?

a decrease in electric potential energy decreases their electric potential (negative voltage) increases their electric potential (positive voltage) in both cases, the electric potential energy is decreasing.

Explain affinity chromatography

a protein of interest is bound by creating a column with high affinity for that protein. This can be accomplished by coating beads with a receptor that binds the protein or a specific antibody to the protein; in either case, the protein is retained in the column. Common stationary phase molecules include nickel, which is used in separation of genetically engineered proteins with histidine tags, antibodies or antigens, and enzyme substrate analogues, which mimic the natural substrate for an enzyme of interest. Once the protein is retained in the column, it can be eluted by washing the column with a free receptor (or target or anitbody), which will compete with the bead-bound receptor and ultimately free the protein form the column. Eluents can also be created with a varying pH or salinity level that disrupts the bonds between the ligand and the protein of interest. The only drawback of the elution step is that the recovered substance can be bound to the eluent. If, for example, the eluent was an inhibitor of the enzyme, it can be difficukt to remove.

What is a skewed distribution? A negatively skewed distribution has a tail where? A positively skewed distribution has a tail where? Which is more susceptible to outliers, the mean or the median? How does it differ for negatively and positively skewed distributions?

a representation of scores that lack symmetry around their average value. it contains a tail on one side or the other of the data set. A negatively skewed distribution has a tail on the left (negative) side. A positively skewed distribution has a tail on the right (positive) side The mean is. The mean of a negatively skewed distribution will be lower than the median while the mean of a positively skewed distribution will be higher than the median.

What is the difference between atomic weight, molecular weight, and formula weight?

atomic weight-a weighted avg of the masses of the naturally occurring isotopes of an element, not their weights molecular weight-sum of atomic weights of all the atoms in a molecule (units are amu) formula weight-adding up atomic weights of the constituent ions according to its empirical formula

the spontaneity of dissolution is dependent on what things?

enthalpy change and the entropy change. At constant temp and pressure, entropy always increases upon dissolution

Materials composed of atoms with unpaired electrons will orient their spins in alignment with a magnetic field, and the material will thus be weakly attracted to the magnetic field. These materials are considered _______.

paramagnetic

What is normality (N))?

the normality of a solution is equal to the number of equivalents of interest per liter of solution. An equivalent is a measure of the reactive capacity of a molecule. Most simply, an equivalent is equal to a mole of the species of interest- protons, hydroxide ions, electrons, or ions

If a charge of +2e and a charge of -3e are separated by a distance of 3 nm, what is the potential energy of the system? (Note: k=8.99E9 Nm^2/C^2)

-4.6E-19J U=kqQ/r U=8.99E9 (2x1.6E-19)(-3x1.6E-19)/3E-9 e is the fundamental unit of charge equal to 1.6E-19C

Calculate the enthalpy change for the following reaction: C(s) +2H2(g) --> CH4(g) deltaH=? Bond dissociation energies of H-H and C-H bonds are 436kJ/mol and 415kJ/mol, respectively. The deltaHf of C9g) is 715kJ/mol.

-73kJ/mol CH4 is formed from free elements in their standard states. The reaction can be written in 3 steps. 1. C(s) -->C(g) deltaH1 2. 2[H2(g) -->2H(g)] 2 x deltaH2 3. C(g) + 4H(g) --> CH4(g) deltaH3 deltaH1=deltaHf of C(g) =715kJ/mol deltaH2 is the energy required to break the H-H bond of one mole of H2 so deltaH2=bond enthalpy of H2=436kJ/mol. deltaH3 is the energy released when 4C-H bonds are formed. Because energy is release when the bonds are formed, deltaH3 is negative. deltaH3= -(4 x bond energy of C-H) = -(4x 415kJ/mol) = -1660kJ/mol Therefore, deltaH(rxn)= 715kJ/mol + (2 x 436kJ/mol) -1660kJ/mol see pg 236 general chemistry for more

5m^3 of a gas are brought from an initial pressure of 1kPa to a pressure of 3kPa through an isochoric process. During this process, the work performed by the gas is:

0 J An isochoric process, by definition, is one in which the gas system undergoes no change in volume. If the gas neither expands nor is compressed, then no work is performed. Remember that work in a thermodynamic system is the area under a P-V curve. if the change in V is 0, the area is 0

How many moles are in 9.53g of MgCl2?

0.1 first find the molar mass of MgCl2 (1 x 24.3g/mol) + (2 x 35.5g/mol) = 95.3 g/mol solve for # of moles 9.53g/95.3 g/mol

If blue light of frequency 6E14Hz is incident on rubidium (W=2.26eV), will there be photoejection of electrons? If so, what is the maximum kinetic energy that an ejected electron will carry away? (Note: h=6.626E-34J(s)=4.14E-15eV(s)

0.22eV If the photons have a frequency of 6E14Hz, each photon has an energy of: E=hf=(4.14E-15eV)(6E14Hz)=2.48eV Clearly then, any given photon has more than enough energy to allow an electron in the metal to overcome the 2.26eV barrier. In fact, the maximum excess kinetic energy carried away by the electrons turns out to be: k-hf-W-2.48-2.26=0.22eV

Sodium nitrate and ammonium sulfate react at 230C to yield sodium sulfate, nitrous oxide, and water vapor. If the total pressure of the products is 1.8 atm, what is the partial pressure of the nitrous oxide? The reaction is as follows: 2 NaNO3 + (NH4)2SO4 --> Na2SO4 (aq) + 2N2o (g) + 4H2O (g) (230-300C)

0.6 atm

FeI(aq) + I2(g) -->FeI3(aq) 1. Which of the following would increase the formation of product? A. Decreasing the volume of the container B. Decreasing the pressure of the container C. Increasing the volume of the container D. Decreasing the volume of the container while maintaining a constant pressure 2. If this reaction were exothermic, what effect would decreasing the temperature have on the equilibrium? A. The forward reaction rate and the reverse reaction rate would both increase B. The forward reaction rate decreases while the reverse reaction rate increases C. The forward reaction rate increases while the reverse reaction rate decreases D. The forward reaction rate and the reverse reaction rate would both decrease

1. A Both increasing the pressure of the container and decreasing the volume would favor the side with fewer moles of gas, which is the product side. D would not disturb the equilibrium- the significance of decreasing the volume fo the container in most equilibria is that there is an inc in pressure 2. C An exothermic reaction produces heat. Decreasing the temp favors product formation, resulting in an inc in the forward reaction rate with a concomitant decrease in the reverse reaction rate

Imagine 2 equal circles with distance 2F between them. The circle on the right is labeled R, on the left is labeled S. There is a vector drawn from S pointing towards R. Its magnitude is half the length between S and R. Figure 1 Physics. 1. The magnitude of the electric force on R due to S is: A. F/2 B. F C. 2F D. 4F 2. If the distance between the centers of the spheres is halved, the magnitude of the force on S due to R will be: A. F/2 B. F/4 C. 2F D. 4F 3. Assume the direction of F is the same direction as the electric field between R and S. If an electron were placed midway between R and S, the resultant electric force on the electron would be: A. toward R B. toward S C. upward in the plane of the page D. downward in the plane of the page

1. B. According to Newton's third law, if R exerts a force on S, then S exerts a force with equal magnitude but opposite direction back on R. Therefore, the magnitude of the force on R due to S is F. 2. D. The force is inversely proportional to r^2. Cutting the distance in half will therefore multiply the force by 2^2., making it four times its original value 3. B. An electric field's direction at a given point is defined as the direction of the force that would be exerted on a positive test charge in that position. Because electrons are negatively charged particles, they will therefore feel a force in the opposite direction of the electric field's vector. In this case, because the force points to the left (toward R), an electron will feel a force pointing to the right (toward S) if E is in the same direction as F

In the reaction, A(aq) +2B <--> C(g) + heat Which will cause a shift to the right and which will cause a shift to the left? 1. A or B is added 2. A or B is removed 3. C is added 4. C is removed 5. the pressure is reduced 6. the pressure is increased 7. the volume is reduced 8. the volume is increased 9. the temperature is reduced 10. the temperature is increased

1. right 2. left 3. left 4. right 5. left 6. right 7. right 8. left 9. right 10. left

What is the density of CO2 gas at 2 atm and 273C?

1.96g/L At STP, a mole of gas occupies 22.4L. Because the increase in pressure to 2 atm decreases volume proportionally, 22.4L must be multiplied by 1atm/2atm=0.5. Because the increase in temp increases volume proportionally ,the temp factor will be 546K/273K=2. V2=(22.4L/mol)(1atm/2atm)(546K/273L)=22.4L/mol density=44g/mol/22.4L/mol= 1.96g/L

Consider two equal resistors wired in parallel. What is the equivalent resistance of the setup?

1/Rp=(1/R1)+(1/R2) next, find the common denominator of the right side: 1/Rp=(R2/R1R2)+(R1/R1R2)=R1+R2/R1R2 Then, take the inverse Rp=R1R2/R1+R2 This is a special case where R1=R2. Substituting in, we get Rp=R^2/2R=R/2

A car's speedometer registers a speed of 33 miles per hour. What was its speed in meters per second?

15m/s math on pg 360 physics and math (33miles/hour)(5280feet/1mile)(12inches/1foot)(2.54cm/1inch)(1m/100cm) =(33 x 5280 x 30)/100 = (1000x5280)/100 = 52800m/mour 52800m/hour(1hour/3600s) = 540/36 = 90/6 = 15m/s

A 1000kg rocket ship, travelling at 100m/s, is acted upon by an avg force of 20kN applied in the direction of its motion for 8s. What is the change in the velocity of the rocket?

160m/s

A car is traveling at 40 km/hr and the driver puts on the brakes, bringing the car to rest in 6s. What is the magnitude of the avg acceleration of the car?

24,000 km/hr^2

Three wires (a, b, and c) meet at a junction point P, as shown in FIGURE 5 PHYSICS. A current of 5A flows into P along wire a, and a current of 3A flows away from P along wire b. What is the magnitude and direction of the current along wire c?

2A flows out of P along wire c the sum of currents entering P must equal the sum of currents leaving P, as per Kirchhoff's Junction Rule. Ia=Ib+Ic 5A=3A+Ic Ic=2A

What is the molar mass of a 22.4 L sample of gas that has a mass of 225 g at a temp of 273C and a pressure of 10 atm?

44.8g/mol Determine how the current conditions compare to STP, and use this to set up a proportionality relationship. P1V1/T1 = P2V2/T2 VSTP=V1(P1/PSTP)(TSTP/T1)=22.4L(10atm/1atm)(273K/546K)=112L 225g/112L=2g/L M=2g/L(22.4L/mol)=44.8g/mol

A firefighter jumps horizontally from a burning building with an initial speed of 1.5 m/s. At what time is the angle between his velocity and acceleration vectors the greatest? A. The instant he jumps B. When he reaches terminal velocity C. Halfway through his fall D. Right before he lands on the ground

A

In a hemicaetal, the central carbon is bonded to : A. -OH, -OR, -H, and -R B. -H, -OR, -OR, and -R C. -OH, -OR, -R, and -R D. -OR, -OR, -R, and -R

A

Reaction of 1-phenylethanone with ethylene glycol, also known as ethane-1,2-diol in aqueous H2SO4 will result in formation of: A. a ketal B. a carboxylic acid C. an aldehyde D. a hemiacetal

A

The fundamental wavelength of a standing wave in a 1 meter long pipe, if the pipe has one open end and one closed end is: A. 4m B. 1m C. 2m D. 4/3m

A

The pressure above a solid compound is slowly reduced. Which of the following is true? A. If the substance is H2O it will transition from solid directly into gas. B. The substance will transition into liquid phase and then gas phase as the pressure is reduced for all substances except H2O C. The substance will transition from solid directly into gas for all substances except H2O D. If the solid transitions from solid directly into gas, it must be H2O

A

Which of the following choices correctly identifies the following three heat transfer processes? I. Heat transferred from the Sun to the Earth II. A metal spoon heating up when placed in a pot of hot soup III. A rising plume of smoke from a fire A. I. Radiation II. Conduction III. Convection B. I. Conduction II. Radiation III. Convection C. I. Radiation II. Convection III. Conduction D. I. Convection II. Conduction III. Radiation

A

Which of the following statements is true regarding a rider in a roller coaster cart moving with a constant speed through a loop? A. The rider is accelerating B. The sum of all the forces acting on the rider is zero C. Gravity is the only force doing work on the rider. D. There are two forces acting on the rider, but neither does any work on the rider.

A

When monochromatic light is refracted as it passes from air to glass, which of the following does NOT remain the same? (Note: assume that the wave is fully transmitted) A. Wavelength B. Frequency C. Amplitude D. Period

A As light rays travel from one medium to another, their wavelengths change. Even if we did not know this immediately, we can determine the answer through process of elimination. Frequency and period are inverses of each other, so if either of those quantities changes, the other would have to change as well. Further, because the wave is fully transmitted, there is no absorption or reflection, and the amplitude (which is related to intensity) should not change. When light is refracted, its speed changes; although the frequency does not change, the wavelength does

In comparison with 1,2-dihydroxyethane, ethanol has weaker intermolecular hydrogen bonds resulting in: A. a lower boiling point for ethanol due to weaker intermolecular forces. B. ethanol being a stronger base. C. a higher boiling point because ethanol is a higher molecular weight compound than 1,2-dihydroxyethane. D. weaker intramolecular bonds in ethanol due to overall reduced bonding energy.

A Ethane-1,2-diol (ethylene glycol) has one more hydroxyl group than ethanol, resulting in greater intermolecular hydrogen bonding. Due to the higher intermolecular forces, ethane-1,2-diol has a boiling point of 200°C in comparison with 78°C for ethanol. A. a lower boiling point for ethanol due to weaker intermolecular forces. Correct. B. ethanol being a stronger base. Incorrect, The two hydroxyl groups on ethylene glycol confer greater basicity than the one hydroxyl group of ethanol. C. a higher boiling point because ethanol is a higher molecular weight compound than ethane-1,2-diol. Incorrect, ethanol is a lower molecular weight compound than ethylene glycol. D. weaker intramolecular bonds in ethanol due to overall reduced bonding energy. Incorrect, Bond energy does not decrease because of intermolecular hydrogen bonding.

In the reaction shown, if 39.05 g of Na2S are reacted with 85.5g of AgNO3, how much of the excess reagent will be left over once the reaction has gone to completion? Na2S + 2AgNO3 --> Ag2S + 2NaNO3 A. 19.5g of Na2S B. 26g of Na2S C. 41.4g of AgNO3 D. 74.3g AgNO3

A The formula weight of Na2S is: (2 x 23g/mol Na) + (1 x 32.1g/mol S) = 78 g/mol Na2S The formula weight of AgNO3 is: (1 x 107.9 g/mol Ag) + (1 x 14g/mol N) + (3 x 16g/mol O) = 169.9g/mol AgNO3 39.05g Na2S/78.1 g/mol = 0.5 mol Na2S 85.5g AgNO3/169.9g/mol = 0.5 mol AgNO3 Because we need 2 moles of AgNO3 for eery mole of Na2S, AgNO3 is the limiting reagent, and the correct answer choice will be in grams of Na2S. If 0.5 mol of AgNO3 are used up, and Na2S will be consumed at half the rate, then 0.25 mol of Na2S will be used 0.25 mol x (78g/mol) =19.5g

Which of the following undergoes a Fischer esterification most rapidly? FIGURE 10 ORGANIC CHEMISTRY A. B. C. D.

A A Fischer esterification involves reacting a carboxylic acid and an alcohol with an acid catalyst. Under these conditions, the carbonyl carbon is open to attack by the nucleophilic alcohol. The rate of this reaction depends on the amount of steric hindrance around the carbonyl carbon because there must be room for the alcohol to approach the carboxylic acid substrate. B, C, and D all have more crowding around the carbonyl carbon, which will decrease reactivity. The additional alkyl groups in the other choices also donate electron density to the carbonyl carbon, making it slightly less electrophilic.

A chemical reaction has a negative enthalpy and a negative entropy. Which of the following terms necessarily describes this reaction? A. Exothermic B. Endothermic C. Exergonic D. Endergonic

A A reaction with a negative enthalpy is, by definition, exothermic. Because both enthalpy and entropy are negative, this is a temperature-dependent process, and the reaction will be both endergonic and exergonic-but only at particular temperatures, eliminating C and D

In a reaction between hydrogen cyanide, butyraldehyde, and ethylmethylketone, which compounds will come together to form the major product? A. Butyraldehyde and hydrogen cyanide B. Ethylmethylketone and butyraldehyde C. Hydrogen cyanide and ethylmethylketone D. No reaction will occur

A Although both the aldehyde and ketone listed will be reactive with the strongly nucleophilic hydrogen cyanide, aldehydes are slightly more reactive toward nucleophiles than ketones for steric reasons, so the aldehyde and HCN will form the major product (which will be a cyanohydrin)

When a bond is created between two nucleotide triphosphates in DNA synthesis, the small molecule released from this reaction is: A. Pyrophosphate B. Inorganic phosphate C. ATP D. Organic phosphate

A As DNA is synthesized, it forms phosphodiester bonds, releasing pyrophosphate, PPi. Pyrophosphate is an inorganic phosphate-containing molecule, but it is not the single phosphate group commonly referred to as inorganic phosphate. The DNA molecule itself is referred to as an organic phosphate.

Assume that blonde hair and blue eyes are independent recessive traits. If one parent is a carrier for each gene while the other parent is homozygous recessive for both genes, what is the probability that the first two offspring will both have blondde hair and blue eyes? A. 6.75% B. 25% C. 43.75% D. 50%

A Because one parent is homozygous for both traits, we are only concerned with the other parent. This parent has a 50% chance of transmitting both (1/2 x 1/2 = 1/4). This probability is the same for both pregnancies because they are independent events; thus, the probability that both children exhibit both traits is 1/4 x 1/4 = 1/16 = 0.0625 = 6.25%.

Butanoic anhydride can be produced by the reaction of butanoic acid with which of the following compounds? A. Butanoic acid B. Ethanoic acid C. Butanol D. Methanal

A Butanoic anhydride is an anhydride with two butane R groups. Anhydrides are produced by the reaction of two carboxylic acids with the loss of a water molecules. therefore, butanoic anhydride would be produced by the reaction of two molecules of butanoic acid

Carbonated beverages are produced by dissolving carbon dioxide in water to produce carbonic acid: CO2(g) + H2O(l)<--> H2CO3(aq) When a bottle containing carbonated water is opened, the taste of the beverage gradually changes as the carbonation is lost. Which of the following statements best explains this phenomenon? A. The change in pressure and volume causes the reaction to shift to the left, thereby decreasing the amount of aqueous carbonic acid B. The change in pressure and volume causes the reaction to shift to the right, thereby decreasing the amount of gaseous carbon dioxide C. Carbonic acid reacts with environmental oxygen and nitrogen D. Carbon dioxide reacts with environmental oxygen and nitrogen

A Carbon dioxide gas evolves and leaves the bottle, which decreases the total pressure of the reactants. Le Chatelier's principle explains that a decrease in pressure shifts the equilibrium to increase the number of moles of gas present. This particular reaction will shift to the left, which in turn will decrease the amount of carbonic acid and increase the amount of carbon dioxide and water. Oxygen and nitrogen are not highly reactive and are unlikely to combine spontaneously with carbon dioxide or carbonic acid.

According to Charles's Law, what happens to a gas as the temperature is decreased? A. Volume decreases B. Pressure decreases C. Volume increases D. Number of moles of gas increases

A Charles's Law states: V1 / T1 = V2 / T2 It represents the direct relationship between volume and temperature for an ideal gas. Thus, as T decreases, so does V, making (A) the right answer. B: This would by true by Gay-Lussac's Law: P1/T1 = P2/T2 C: The relationship between T and V is direct, not inverse. D: The number of moles of gas figures into Avogadro's Law: V1/n1 = V2/n2

The salt KCl is dissolved in a beaker. To an observer holding the beaker, the solution begins to feel colder as the KCl dissolves. From this observation, one could conclude that: A. deltaSsoln is large enough to overcome the unfavorable deltaHsoln B. KCl is mostly insoluble in water C. deltaSsoln must be negative when the KCl dissolves D. boiling point depression will occur in this solution

A Dissolution is governed by enthalpy and entropy, which are related by the equation deltaGsoln=deltaHsoln-TdeltaSsoln. The cooling of the solution indicates that heat is used up in this bond-breaking reaction. In other words, dissolution is endothermic, and deltaH is positive. The reaction is occurring spontaneously, so deltaG must be negative. The only way that a positive deltaH can result in a negative deltaG is if entropy, deltaS, is a large, positive value as in A. Conceptually, that means that the only way the solid can dissolve is if the increase in entropy is great enough to overcome the increase in enthalpy. B is incorrect because it is clearly stated in the question stem that KCl dissolves; urther, all salts of Group I metals are soluble. C is incorrect because deltaSsoln must be positive in order for KCl to dissolve. Finally, D is incorrect because solute dissolution would cause the BP to elevate, not depress. It is also not a piece of evidence that could be found simply by observing the beaker's temp change.

A reaction coordinate for a chemical reaction is displayed in the graph FIGURE 4 (general chemistry). Which of the following terms describes the energy of this reaction? A. Endothermic B. Exothermic C. Endergonic D. Exergonic

A Eliminate C and D, which describe the free energy of reaction and cannot be determined from this graph. While most reaction coordinate graphs we've explored in this book use free energy for the y-axis, this one uses potential energy (enthalpy). If the heat of formation of the products is greater than that of the reactants, the reaction is endothermic. We can determine this information from their relative positions on this graph: because the products are higher than the reactants, this is an endothermic reaction.

Monochromatic red light is allowed to pass between two different media. If the incident angle in medium 1 is 30 degrees and the incident angle in medium 2 is 45 degrees, what is the relationship between the speed of the light in medium 2 compared to that in medium 1? A. v2= v1(sqrt2) B. v2(sqrt2)=v1 C. v2=v1(sqrt3) D. v2(sqrt3)=v1

A First, the color of the light is irrelevant here: the ratio would be the same even if the specific color were not mentioned. Second, recall Snell's Law: n1 sin theta1=n2 sintheta2. Although we don't know the value of n for either medium, you do know the relationship n=c/v. math on pg 310 physics c/v1(sintheta1)=c/v2(sintheta2) sin30/v1 = sin45/v2 1/2v1 = sqrt2/2v2 v2=v1sqrt2

Two wooden balls of equal volume but different density are held beneath the surface of a container of water. Ball A has a density of 0.5g/cm3, and ball B has a density of 0.7g/cm3. When the balls are released, they will accelerate upward to the surface. What is the relationship between the acceleration of ball A and that of ball B? A. Ball A has the greater acceleration B. Ball B has the greater acceleration C. Balls A and B have the same acceleration D. Cannot be determined.

A Fnet=ma Fbouy-mg=ma a=(Fbuoy-mg)/m = (Fbuoy/m) -g Both balls experience the same buoyant forces b/c they are in the same liquid and have the same volume (Fbuoy=rhoVg). Thus the ball with the smaller mass experiences the greater acceleration. Because both balls have the same volume, the ball with the smaller density has the smaller mass, which is ball A.

LiALH4 is often used in laboratories because of its tendency to donate a hydride ion. Which of the following roles would lithium aluminum hydride likely play in a reaction? A. Strong reducing agent only B. Strong oxidizing agent only C. Both a strong reducing agent and strong oxidizing agent D. Neither a strong reducing agent nor a strong oxidizing agent

A Hydride ions are composed of a hydrogen nucleus with two electrons, thereby giving it a negative charge and a considerable tendency to donate electrons. LiAlH2 is therefore a strong reducing agent. Strong reducing agents tend to have metals or hydrides; strong oxidizing agents tend to have oxygen or a similarly electronegative element

If a gas occupies 0.1L at 200atm, what will its volume be at 1atm? A. slightly less than 20L B. 20L C. slightly more than 20L D. 2000L

A Ideal gases undergoing pressure and volume changes are related by the expression P1V1/T1=P2V2/T2. Here, temperature is not involved and P1V1= P2V2. However, since P1 is greater than 10atm, the size of the molecules cannot be ignored and volume V1 is slightly greater than it should be for an ideal gas. Therefore, the calculated volume V2=20L is slightly larger than it should be. Choice A is correct. A. slightly less than 20L, correct. B. 20L, incorrect, The initial volume at a pressure of 200atm includes a small contribution from the molecules themselves, and therefore the initial volume of 0.1L is larger than the volume of an ideal gas. C. slightly more than 20L, incorrect, See explanations for a) and b). D. 2000L, incorrect, This answer choice incorrectly divides the initial pressure by the initial volume.

Explosions are necessarily characterized by: A. deltaG < 0 B. deltaH > 0 C. deltaS < 0 D. T < 0

A In an exploration, a significant amount of heat energy is released, meaning that the reaction is exothermic (delta H < 0), eliminating B. The entropy change associated with an explosion is positive because energy is dispersed over a much larger area, eliminating C. If this is true, the expression deltaH-TdeltaS must be negative, indicating that this is an exergonic process (deltaG < 0). Absolute temperature can never be negative, eliminating D.

Arrange the following molecules in order of increasing pKa: CH3CH2COOH, CH3CHFCOOH, CH3OCH2COOH, CH3CHBrCOOH A) CH3CHFCOOH, CH3OCH2COOH, CH3CHBrCOOH, CH3CH2COOH B) CH3CHBrCOOH, CH3OCH2COOH, CH3CH2COOH, CH3CHFCOOH C) CH3CHFCOOH, CH3CH2COOH, CH3CHBrCOOH, CH3OCH2COOH D) CH3CH2COOH, CH3OCH2COOH, CH3CHFCOOH, CH3CHBrCOOH

A Increasing pKa means decreasing acid strength. A low pKa, just like a low pH, indicates a stronger acid. Here, we need to arrange the molecules from the strongest acid to the weakest. An acid is stronger if the conjugate base is stabilized, and one form of stabilization is inductive stabilization. That is, if an electron withdrawing group pulls electron density off of a carbon, that carbon becomes slightly positive. That slight positive charge can then stabilize the negative charge of the conjugate base. Inductive effects increase with increasing electronegativity. Because fluorine is the most electronegative atom, it has the biggest inductive effect and will most help stabilize the conjugate acid. Thus CH3CHFCOOH is the strongest acid and must be first in our list. That narrows us down to (A) and (C). Propanoic acid with no inductive stabilization would be the weakest acid here, with the highest pKa. So it must be the last item on the list. That tells us choice (A) is the right answer.

Ketose sugars may have the ability to act as reducing sugars. Which process explains this? A. Ketose sugars undergo tautomerization B. The ketone group is oxidized directly C. Ketose sugars undergo anomerization D. The ketone group is reduced directly

A Ketose sugars undergo tautomerization, a rearrangement of bonds, to undergo keto-enol shifts. This forms an aldose, which then allows them to act as reducing sugars. A ketone group alone cannot be oxidized. Anomerization, mentioned in C, refers to ring closure of a monosaccharide, creating an anomeric carbon.

In FIGURE 10 PHYSICS, six currents meet at point P. What is the magnitude and direction of the current between points P and x? A. 2A, toward x B. 2A, toward P C. 10A, toward x D. 10A, toward P

A Kirchhoff's junction rule states that the sum of all currents directed into a point is always equal to the sum of all currents directed out of the point. The currents directed into point P are 8A, 2A, and 3A, so the sum is 13A. The currents directed out of point P are 5A and 6A, so the total is 11A. Because the two numbers must always be equal, an additional current of 2A must be directed away from point P toward point x.

A charge of 2E-6C flows from the positive terminal of a 6V battery, through a 100 omega resistor, and back through the battery to the positive terminal. What is the total potential difference experienced by the charge? A. 0V B. 0.002V C. 0.2V D. 6V

A Kirchhoff's loop rule states that the total potential difference around any closed loop of a circuit is 0V. Another way of saying this is that the voltage gained in the battery (6V) will be used up through the resistors. Because this charge both started and ended at the positive terminal, its total potential difference is therefore 0V. 6V, D, is the voltage gained in the battery as well as the voltage drop in the resistors-creating a net sum of 0V.

Which of the following would be the best method of producing methyl propanoate? A. Reacting propanoic acid and methanol in the presence of a mineral acid. B. Reacting methanoic acid and propanol in the presence of a mineral acid. C. Reacting propanoic anhydride with an aqueous base. D. Reacting propanoic acid with an aqueous base.

A Methyl propanoate is an ester; it can be synthesized by reacting a carboxylic acid with an alcohol in the presence of acid. Here, the parent chain is propanoate, and the esterifying group is a methyl group. B reverses the nomenclature, and would form propyl methanoate. The other reactions listed would not form esters.

Which of the following would react most readily with a carboxylic acid to form an amide? A. Methylamine B. Triethylamine C. Diphenylamine D. Ethylmethylamine

A Methylamine would react readily to form an amide. The less substituted the nucleophile, the easier it will be for the nucleophile to attack the carbonyl carbon and form the amide. In fact, B would not be able to form an amide at all because it does not have a hydrogen to lose while attaching to the carbonyl carbon.

Which of the following combinations of liquids would be expected to have a vapor pressure higher than the vapor pressure that would be predicted by Raoult's law? A. Ethanol and hexane B. Acetone and water C. Isopropanol and methanol D. Nitric acid and water

A Mixtures that have a higher vapor pressure than predicted by Raoult's law have stronger solvent-solvent and solute-solute interactions than solvent-solute interactions. Therefore, particles do not want to stay in solution and more readily evaporate, creating a higher vapor pressure than an ideal solution. Two liquids that have different properties, like hexane (hydrophobic) and ethanol (hydrophilic, small) in A, would not have many interactions with each other and would cause positive deviation, ie higher VP. B and C are composed of liquids that are similar to one another and would not show significant deviation from Raoult's law. D contains 2 liquids that would interact very well with each other, which would cause a negative deviation from Raoult's law-when attracted to one another, solutes and solvents prefer to stay in liquid form and have a lower VP than predicted by Raoult's law

An object is placed at the center of curvature of a concave mirror. Which of the following is true about the image? A. It is real and inverted B. It is virtual and inverted C. It is real and upright D. It is virtual and upright

A Solution pg 308 physics One could solve this question with a ray diagram but don't use these on Test Day. Use the sign convention. If the object is at the center of curvature, its distance is 2f. We can plug into the optics equation 1/f=1/o+1/i --> 1/i=1/f-1/o = 1/f=1/2f = 1/2f i=2f=r Because i is positive, the image is real. For single mirrors or lenses, all real images are inverted.

Consider the following steps in the reaction between oxalic acid and chlorine: I. Cl2 + H2O --> HOCl + Cl- + H+ II. H2C2O4 --> H+ + HC2O4- III. HOCl + H2O4- --> H2O + Cl- + 2CO2 Which of these steps, occurring in aqueous solution, is an example of a disproportionation reaction?

A Step I is a disproportionation reaction because chlorine starts with an oxidation state of 0 in the reactsnts and ends up with an oxidation state of +1 in HOCl and -1 in Cl-. In the other reactions, no element appears with different oxidation states in two different products. Therefore, only step I is a disproportionation reaction

Tertiary alcohols are oxidized with difficulty because A. there is no hydrogen atom attached to the carbon with the hydroxyl group B. there is no hydrogen attaches to the alpha carbon of the carbonyl C. tertiary alcohols contain hydroxyl groups with no polarization D. they are relatively inert

A Tertiary alcohols can be oxidized but only under extreme conditions because their substrate carbons do not have spare hydrogens to give up. Alcohol oxidation involves the removal of such a hydrogen so that carbon can instead make another bond too oxygen. If no hydrogen is present, a carbon carbon bond must be cleaved, which requires a great deal of energy and will, therefore, occur only under extreme conditions. B is incorrect because alcohols are not carbonyl-containing compounds and would more properly describe a carbonyl-containing compound that is unable to form an enolate. C is incorrect because the hydroxyl group of the tertiary carbon is still polarized. D is a false statement; tertiary alcohols are still involved in other reactions, such as SN1 reactions

The kinetic molecular theory states that A. the average KE of a molecules of gas is directly proportional to the temp of the gas in kelvins B. collisions between gas molecules are inelastic C. gas particles occupy discrete amounts of space D. all gas molecules have the same KE at the same temp

A The avg KE is directly proportional to the temp of a gas in kelvins. The kinetic molecular theory states that collisions between molecules are elastic and thus don't result in a loss of energy. Gas particles are assumed to take up negligible space. While the avg KE of any gas as a whole is the same at any given temp, the particles themselves have a distribution of speds (seen in Maxwell-Boltzmann distribution curve)

An experimenter is attempting to determine the internal energy of a well-known compound. He cleans his glassware, completes the synthesis, calibrates a bomb calorimeter, and then uses it to measure the appropriate thermodynamic values. Which of the following errors did he make? A. He did not determine if the compound was novel or if the information has already been determined B. He did not have a specific goal at the beginning of his research. C. He should not be involved in both the synthesis of the compound and later testing. D. He should have calibrated the calorimeter before the synthesis of the compound.

A The experimenter has not completed the initial phases of research. There was no data acquisition or refinement, and there was no indication that the question required an experiment to be answered. Were the experimenter doubtful of the validity of the reported value, an experiment could be appropriate-but there is no info to indicate that this is so. Based on the question stem, it is clear that the experimenter had a clear goal, eliminating B. In human subjects researcher, tasks may be divided to facilitate blinding, but this is generally unnecessary in basic sciences research, eliminating C. As long as the calorimeter was calibrated prior to use, it does not matter when this calibration occurred relative to the synthesis of the compound, eliminating D.

As methanol is converted to methanal, and then methanoic acid, the oxidation number of the carbon A. increases B. decreases C. increases, then decreases D. decreases, then increases

A The formula for methanol is H3COH, for methanal is HCHO, and for methanoic acid is HCOOH. If we assign oxidation numbers to carbon in each molecule, it starts at -2 then becomes 0 them becomes +2. In general, it is often easier to think of oxidation as a gain of bonds to oxygen (or a similarly electronegative element) or loss of bonds to hydrogen for organic compounds. Therefore, because the carbon is oxidized as one converts from an alcohol to an aldehyde to a carboxylic acid, the oxidation number must increase.

All of the following statements about the photoelectric effect are true EXCEPT: A. the intensity of the light beam does not affect the photocurrent B. the kinetic energies of the emitted electrons do not depend on the light intensity C. a weak beam of light of frequency greater than the threshold frequency yields more current than an intense beam of light of frequency lower than the threshold frequency D. for light of a given frequency, the kinetic energy of emitted electrons increases as the value of the work function decreases.

A The greater the intensity, the greater the number of incident photons and , therefore, the greater number of electrons that will be ejected from the mental surface (provided that the frequency of the light remains above the threshold). This means a larger current. Remember that the frequency of the light (assuming it is above the threshold frequency) will determine the kinetic energy of the ejected electrons; the intensity of the light determines the number of electrons ejected per time (the current).

In a molecule containing a carboxylic acid group, what would be expected in a 1H-NMR spectrum? A. A deshielded hydrogen peak for the hydroxyl hydrogen, shifted left B. A deshielded hydrogen peak for the hydroxyl hydrogen, shifted right C. A shielded hydrogen peak for the hydroxyl hydrogen, shifted left D. A shielded hydrogen peak for the hydroxyl hydrogen, shifted right

A The oxygen of the hydroxyl group will deshield the hydroxyl hydrogen, shifting it downfield, or left. Hydrogens in carboxylic acids can have some of the most downfield absorbances, around 10.5 to 12 ppm.

At sea level and 25C, the solubility of oxygen gas in water is 1.25E-3 M. In Denver, a city in the US that lies high above sea level, the atmospheric pressure is 0.800atm. What is the solubility of oxygen in water in Denver? A. 1E-3M B. 1.05E-3M C. 1.50E-3M D. 2.56E-3M

A The solubility of gases in liquids is directly proportional to the atmospheric pressure, as shown by Henry's law. math on pg 296 gen chem [O2]1/P1=[O2]2/P2 1.25E-3M/1atm=[O2]2/0.800atm =1E-3M

FIGURE 17 ORGANIC CHEMISTRY Four compounds, I, II, III, and IV, are separated by chromatographic techniques. Compound III is the most polar, compound II is the least polar, and I and IV have intermediate polarity. The solvent system is 85:15 ethanol:methylene chloride. Which spot on the card below likely belongs to compound III? A. A B. B C. C D. D

A This is an example of reverse-phase chromatogrpahy. The solvent system is polar,which means that the most polar compound will travel the furthest up the card, resulting in the largest Rf. This gives compound III the largest Rf, which corresponds to spot A.

Suppose deltaGrxn^not -=-2000kJ/mol for a chemical reaction. At 300K, what is the change in Gibbs free energy in kJ/mol? A. -2000 + (300K)(8.314)(lnQ) B. -2000 - (300K)(8.314)(lnQ) C. -2000 + (300K)(8.314)(logQ) D. -2000 - (300K)(8.314)(logQ)

A This problem asks for the free energy of a reaction at non-standard conditions, which can be determined with the equation deltaG=deltaG^not + RTlnQ

The electric field a distance rA from a positively charged point charge is equal to EA. What is the ratio EB to EA of the electric field at point B a distance of 3rA from the charge? A. EB = 1/9 EA B. EB = 1/3 EA C. EB = 3EA D. EB = 9EA

A This question asks the examinee to determine the ratio of the electric field due to a point charge as a function of distance. The electric field of a point charge is given by the following equation: E = kq/r2 Therefore, the ratio of the electric field at point B to point A is equal to: EB / EA = (kq/rB2) / (kq/rA2) = rA2 / rB2 = rA2 / 9rA2 = 1/9 Thus, EB = 1/ 9 EA making A the correct answer.

A ray of light (f=5E14 Hz) travels from air into crystal into chromium. If the indices of refraction of air, crystal, and chromium are 1, 2, and 3, respectively, and the incident angle is 30 degrees, then which of the following describes the frequency and the angle of refraction in the chromium? A. 5E14 Hz; 9.6 degrees B. 5E14 Hz, 57 degrees C. 1E10 Hz, 9.6 degrees D. 1E10 Hz, 57 degrees

A This question contains 2 parts- we have to determine the frequency and the angle of refraction of the light ray. The first part is straightforward because the frequency of a light ray traveling from one medium to another does not change. Because the frequency must be 5E14Hz, we can eliminate C and D. For the angle of refraction, we can either calculate it or determine it using logic. The light ray first goes from air into crystal (from a low index of refraction to a higher one). According to Snell's law, the angle of refraction will be smaller than the incident angle (closer to the normal). When the light ray moves from crystal to chromium, it again goes from a lower index of refraction into a higher one, thus making the angle of refraction even smaller.

Thin-layer chromatography (TLC) is a laboratory technique used to separate non-volatile mixtures. Using a silica gel plate and a hexane solvent, which of the following compounds in a mixture containing CH3OH, HCOOH, CH2O, and C6H6 would have the highest Rf value? A. C6H6 B. HCOOH C. CH2O D. CH3OH

A This question is testing your familiarity with thin-layer chromatography (TLC), which is a technique used to separate mixtures. Silica gel is composed of a siloxane matrix (-O-Si-O-). The highly polar substance is capable of forming intermolecular bonds, including hydrogen bonding. As a result, polar compounds adhere to the silica gel plate and travel little from the initial starting position. Meanwhile, nonpolar compounds elute away from the starting position with the nonpolar hexane solvent via capillary action. In the mixture above, HCOOH and CH3OH hydrogen bond with the siloxane matrix. Therefore, B and D will have the lowest Rf values. The oxygen atom in CH2O makes formaldehyde slightly polar allowing for intermolecular interactions between the gel plates. Benzene is the most nonpolar compound of the choices and therefore travels the farthest from the starting position along with the hexane solvent.

A hydraulic lever is used to life a heavy hospital bed, requiring an amount of work. When the same bed with a patient is lifted, the work required is doubled. How can the cross-sectional area of the platform on which the bed if lifted be changed so that the pressure on the hydraulic lever remains constant? A. The cross-sectional area must be doubled B. The cross-sectional area must be halved. C. The cross-sectional area must be divided by four. D. The cross-sectional area must remain the same.

A This question tests our understanding of Pascal's principle which states that a change in pressure applied to an enclosed fluid is transmitted undiminished to every portion of the fluid and to the walls of the containing vessel. We are told that the work required is doubled. In other words, the force required doubles when both the bed and the patient have to be lifted. To maintain the same pressure, we must double the cross-sectional area of the platform of the hydraulic lever on which the patient and bed are lifted.

Amateur chemists make nitrous oxide at home using this (unbalanced) reaction: NH4NO3(s)--> H2O(g) + N2O(g) How many grams of NH4NO3 are required to produce 134.4 L of mixed gas if products are at STP? (Do NOT heat the ammonium nitrate above 240°C or it may detonate) A. 160 B. 240 C. 960 D. 1440

A This solution has 3 steps: balance the equation, determine the required number of moles of product and reactant, and calculate the mass of reactant necessary. The reaction as written is unbalanced. In the balanced equation, 2 moles of H2O and 1 mole of N2O are made for each 1 mole of NH4NO3. 3 moles of product gas result from each mole of reactant. 22.4 L of gas corresponds to 1 mole of gas at STP, and the specified 134.4 L indicates 6 moles of product gases. 6 moles of mixed product gas requires 2 moles of reactant. NH4NO3 weighs 80 g/mole, so 160 g are required. A. 160, correct. 2 moles of NH4NO3 yields 4 moles of H2O and 2 moles of N2O, for 6 moles of mixed product gas occupying 134.4 L at STP. B. 240, incorrect, This answer results from using the unbalanced equation as written. C. 960, incorrect, This answer results from using the unbalanced equation and calculating the mass of 12 moles NH4NO3 for 6 moles N2O product rather than mixed gas. D. 1440, incorrect, This answer results from using the correct balanced equation and calculating the mass of 18 moles of NH4NO3 for 6 moles of N2O product rather than mixed gas.

One mole of an unknown compound is determined to have a molecular weight of 86 grams per mole. The mass percent of the compound is as follows: 55.81% carbon, 6.98% hydrogen, and 37.21% oxygen. What is the empirical formula and molecular formula, respectively, of the unknown compound. A. C2H3O and C4H6O2 B. C2H3O and C6H9O3 C. C5H8O and C5H8O D. C5H8O and C10H16O2

A To answer this question, you must determine the empirical formula of the unknown compound by treating the mass percent as the number of grams of each element out of a 100g sample. The number of moles of each element is determined by the following calculations: Moles of C = (55.81g C) x (1mol C/12g C) = 4.65 mol C Moles of H = (6.98g H) x (1 mol H/1g H) = 6.98 mol H Moles of O = (37.21g O) x (1 mol O/16g O) = 2.33 mol O Dividing each by the largest common denominator of 2.33 gives the empirical formula of the unknown compound: C2H3O (4.65/2.33) mol C = 2 mol C (6.98/2.33) mol H = 3 mol H (2.33/2.33) mol O = 1 mol O Since the molecular weight of the compound is 86 g/mol, the correct answer must be A. Note that if you were in a hurry on Test Day, or if you forgot how to solve this problem, you could instead use the periodic table to calculate the molar mass of the molecular formula given in each answer choice. If only one answer gave a molar mass of 86 g/mol (as is the case here, with choice A), then you could be certain that this option was correct. This is true of many MCAT questions: there may be multiple ways to solve the same problem!

A mobile positive charge Qa is placed a distance "d" from a fixed positive charge Qb of equal magnitude. When released from rest, the positive charge Qa will: A. accelerate away from Qb. B. circle Qb along an equipotential electric field line. C. accelerate towards Qb. D. remain in place while doubling the electrical potential at a point located at d/2 between the charges.

A Two positive point charges will repel each other. The mobile charge will accelerate away from the fixed charge. A. accelerate away from Qb. correct. B. circle Qb along an equipotential electric field line. incorrect, Qa will accelerate away from Qb in a direction defined by the shortest path between the charges. C. accelerate towards Qb. incorrect, Like charges repel each other. D. remain in place while doubling the electrical potential at a point located at d/2 between the charges. incorrect, The mobile charge will accelerate away from the fixed charge.

A transformer is a device that takes an input voltage and produces an output voltage that can be either larger or smaller than the input voltage, depending on the transformer design. Although the voltage is changed by the transformer, energy is not, so the input power equals the output power. A particular transformer produces an output voltage that is 300% of the input voltage. What is the ratio of the output current to the input current? A. 1:3 B. 3:1 C. 1:300 D. 300:1

A We are told that transformers conserve energy so that the output power equals the input power. Thus, p(out)=p(in), or IoutVout=IinVin. There is therefore an inverse proportionality between current and voltage. If the output voltage is 300% of the input voltage (3x its amount), then the output current must be 1/3 of the input voltage. This can be represented as a 1:3 ratio

If two waves with the same frequency are 180 degrees out of phase, what is the amplitude of the resultant wave if the amplitudes of the original waves are 5 cm and 3 cm? A. 2 cm B. 3 cm C. 5 cm D. 8 cm

A When two waves are out of phase by 180degrees, the resultant amplitude is the difference between the two waves' amplitudes. In this case, the resulting wave will have an amplitude of 5cm-3cm=2cm

Which of the following methods would be most appropriate for an initial assessment of hemoglobin saturation during an experiment about breath holding? A. A pulse oximeter, which uses a small light on an adhesive bandage B. An arterial cannula, which permits repeated blood draws with a single puncture C. Repeated venipuncture, because a single puncture would cause data overlap D. A Swan-Ganz catheter inserted through the femoral artery, which can measure saturation nearest the heart

A While it may appear that this question is asking to determine which method is the most accurate or reliable, this is an ethics question. All of these methods (with the exception of C) measure oxygen saturation. According to the principle of beneficence, we must minimize potential harms associated with our investigations; thus, the noninvasive pulse oximeter should be greatly favored over other measurements for an initial assessment.

Which of the following sets of conditions would be LEAST likely to result in ideal gas behavior? A. High pressure and low temperatures B. Low temperature and large volume C. High pressure and large volume D. Low pressure and high temperature

A gases deviate from ideal behavior at higher pressures and lower volumes and temperatures, all of which force molecules closer together. The closer they are, the more they can participate in IMFs, which violates the definition of an ideal gas. At low temps, the KE of the particles is reduced, so collisions with other particles or the walls of the container are more likely to result in significant changes in KE

During uniform circular motion, which of the following relationships is necessarily true? a. No work is done b. The centripetal force does work c. The velocity does work d. Potential energy depends on position of the object around the circle

A in uniform circular motion, the displacement vector and force vector are always perpendicular, therefore no work is done. PE is constant for an obj in uniform circular motion. PE does not change and does not depend on the position of the obj around the circle

How can the value of a natural logarithm be converted to the value of a common logarithm? A. The natural logarithm is divided by a constant. B. A constant is added to or subtracted from the natural logarithm. C. The natural logarithm is raised to an exponent. D. The inverse of the natural logarithm is taken.

A log x = ln x/2.303

Treating 2-methyl-1-propanol with methysulfonyl chloride in base, followed by reaction with pyridinium chlorochromate and a final step in strong acid, will give an end product of A. 2-methyl-1-propanol B. 2-methylpropanol C. 2-methylpropanoic acid D. 2-methyl-1-propane

A methylsplfonyl chloride serves as a protecting group for alcohols, which are converted into mesylates. Reacting with this reagent before continuing with what would normally be an oxidation reaction keeps the alcohol from reacting; when the protecting group is then removed using strong acid, the resultant product is the same as the initial reactant. Neither of the oxidation products in B or C, nor the reduction product in D will be formed

Before absorbing an ultraviolet photon, electrons can be found in: A. The HOMO only B. The LUMO only C. Both the HOMO and LUMO D. neither the HOMO nor the LUMO

A only after absorbing UV light is an electron excited from the HOMO, the highest occupied molecular orbital to the LUMO, the lowest unoccupied molecular orbital.

During the Gabriel synthesis, phthalimide serves as the; A. Nucleophile B. Base C. Leaving group D. Electrophile

A phthalimide attacks a secondary carbon in diethyl bromomalonate. The secondary carbon is the electrophile and bromide is the LG

Which of the following chemical shifts could correspond to an aldehydic proton signal in a 1H-NMR spectrum? A. 9.5 ppm B. 7.0 ppm C. 11.0 ppm D. 1.0 ppm

A this signal lies downfield because the carbonyl oxygen is electron-withdrawing and deshields the protein.

What is a ray diagram? Explain how to use them using Diagrams for Concave (Converging) Mirrors and Ray Diagrams for Convex (Diverging) Mirrors in Physics Notes, and how to draw them.

A ray diagram is useful for getting an approximation of where an image is. On Test Day, ray diagrams can be helpful for a quick determination of the type of image that will be produced by an object some distance from the mirror (real vs virtual, inverted vs upright, and magnified vs reduced). Ray diagrams should be used with caution, however: under the pressure of Test Day, it can be easy to draw them incorrectly. Therefore, it is important to practice drawing them to avoid careless errors on Test Day, and it is important to be familiar with how to solve optics questions mathematically. When drawing a ray diagram, there are 3 important rays to draw. For a concave mirror, a ray that strikes the mirror parallel to the axis (the normal passing through the center of the mirror) is reflected back through the focal point (green lines in Ray Diagrams for Concave (Converging) Mirrors and Ray Diagrams for Convex (Diverging) Mirrors in Physics Notes). A ray that passes through the focal point before reaching the mirror is reflected back parallel to the axis (red lines). A ray that strikes the mirror at the point of intersection with the axis is reflected back with the same angle measured from the normal (blue lines). In figure A, the object is placed beyond F, and the image produced is real, inverted and magnified. In figure B, the object is placed at F and no image is formed because the reflected light rays are parallel to each other. In terms of the mirror equation, we say that the image distance i=infinity here. For the scenario in Figure C, the object is placed between F and the mirror and the image produced is virtual, upright and magnified. A single diverging mirror forms only a virtual, upright, and reduced image, regardless of the position of the object. The further away the object, the smaller the image will be. To quickly remember these rules, recall the convenience store security mirrors. Any time an object is at the focal point of a converging mirror, the reflected rays will be parallel, and thus, the image will be at infinity. Key Concept: To find where the image is (for a mirror), draw the following rays and find a point where any two intersect. This point of intersection marks the tip of the image. If the rays you draw do not appear to intersect, extend them to the other side of the mirror, creating a virtual image. Ray parallel to axis--> reflects back through focal point Ray through focal point--> reflects back parallel to axis Ray to center of mirror--> reflects back at same angle relative to normal

What is the IUPAC name of the following compound: (CH3)2CH-NH-CH-(CH3)2? A. Diisopropylamine B. Diisobutylamine C. Di-tert-propylamine D. 1,1-dimethylethylamine

A. Diisopropylamine, correct. B. Diisobutylamine, incorrect, The two substituent groups have 3 carbons each, isobutyl groups have four carbons, -CH2-CH-(CH3)2. C. Di-tert-propylamine, incorrect, There is no such thing as a tert-propyl group; the tert-butyl group requires 4 carbons. D. N,N-dimethylethylamine, incorrect, The IUPAC name is generated by 1) identifying the longest carbon chain (propane), 2) identifying the highest priority functional group (amine on carbon 2 of the parent propane chain, i.e. 2-amine), identify side chains (N-Isopropyl), and put it together: N-Isopropylpropan-2-amine. The structure of N,N-dimethylethylamine is [(CH3)3]N. The ethane parent carbon chain in choice d) reveals this answer to be incorrect; the parent carbon chain in the question stem is a propane chain with three carbons

At a place where g=9.8m/s^2, an object is thrown vertically downward with a speed of 5m/s while a different object is thrown vertically upward with a speed of 10m/s. Which object undergoes a greater change in speed in a time of 2s? A. the first object has a greater change in speed B. the second object has a greater change in speed C. both objects undergo the same change in speed

A. Each obj experiences an acceleration of 9.8 m/s^2, which means that the obj's velocity changes by that amount each second. Therefore, both objects experience the same change in velocity over the 2 sec period of 19.6m/s. However, this question asks for the change in speed. The first object starts with a velocity of -5m/s and ends with a velocity of -24.g/s. This means change in speed is 19.6m/s. The second obj starts with +10m/s and ends with -9.6m/s. Change in speed is -0.4m/s

A block weighing 100N is pushed up a frictionless incline over a distance of 20m to a height of 10m. Find A. The min force required to push the block B. The work done by the force C. The force required and the work done by the force if the block were simply lifted vertically 10m

A. F=mgsintheta sin theta is the ration of the length of the opposite side to the hypotenuse (10/20) =100N (10/20)=50N B. W=fdcos theta in this case, theta represents the angle between the force and displacement vectors, not the angle of the inclined plane. B/c they are parallel, cos theta = 1. W= 50N (20m) (1)= 1000J C. To raise the block vertically, an upward force equal to the objects weight (100N) would have to be generated. The work done by the lifting force is W= fd cos theta = 100N (10m) (1) = 1000J

A pulley system of 6 pulleys has an efficiency of 80%. A person is lifting a mass of 200kg with the pulley. Find: A. The distance through which the effort must move to raise the load a distance of 4m. B. The effort required to lift the load C. The work done by the person lifting the load through a height of 4m.

A. For the load to move through a vertical distance of 4m, all 6 of the supporting ropes must shorten 4m also. This may only be accomplished by pulling 6x4=24m of the rope through the setup. 24m B. To calc. the effort req, the equation for efficiency should be used. Efficiency= load (load distance)/ effort (effort distance) 0.8=200kg . 9.8m/s^2 (4m)/effort (24m) = 408N C. The work done =Work in Win = effort (effort distance)= 408N x 24 = 9800J

Methanol reacts with acetic acid to form methyl acetate and water Type of Bond Bond Dissociation Energy (kJ/mol) C-C 348 C-H 415 C=O 805 O-H 463 C-O 360 Based on the values in the table above, what is the heat of reaction in kJ/mol? A. 0 B. 464 C. 824 D. 1288

A4At first glance, this may seen math-heavy, but it actually doesn't require any calculations. We just have to keep track of which bonds are broken and which bonds are formed. Breaking bonds requires energy, forming bonds releases energy. Two bonds are broken: a C-O bond between the carbonyl carbon and oxygen of acetic acid, and an O-H bond between the hydroxyl oxygen and hydrogen of methanol. Two bonds are formed: a C-O bond between the carbonyl carbon and the oxygen of methyl acetate, and an O-H bond between a hydroxyl group and a hydrogen to form water. Given that the same two bonds are broken and formed in this reaction, the energy change must be kJ/mol.

What do conductors do?

Allow heat or electricity to easily pass through when a conductor is given a charge, the charges will distribute approximately evenly upon the surface of the conductor conductors are often conceptualized as nuclei surrounded by a sea of free electrons that are able to move rapidly throughout the material and are only loosely associated with the positive charges. conductors are generally metals, although ionic (electrolyte) solutions are also effective conductors.

What is chromatography? What is the general concept? Explain the process.

An analytical method used to separate the substances in a mixture. The more similar a compound is to its surroundings (whether by polarity, charge, or other characteristics), the more it will stick to and move slowly through its surroundings. The process begins by placing the sample onto a solid medium called the stationary phase, or adsorbent. We then run the mobile phase, usually a liquid (or a gas in gas chromatography) through the stationary phase. This will displace (elute) the sample and carry it through the stationary phase. Depending on the characteristics of the substances in the sample and the polarity of the mobile phase, it will adhere to the stationary phase with differing strengths, causing the different substances to migrate at different speeds. This is called partitioning and it represents an equilibrium between the two phases. Different compounds will have different partitioning coefficients and will elute at different rates. This results in separation within the stationary phase, allowing us to isolate each substance individually.

What is Newton's first law?

An object at rest stays at rest and an object in motion stays in motion with the same speed and in the same direction unless acted upon by an unbalanced force. (also called the law of inertia)

Ring strain arises from 3 factors. What are they and define them.

Angle strain, torsional strain, and nonbonded (steric) strain Angle- results when bond angles deviate from their ideal values by being stretched or compressed torsional- results when cyclic molecules must assume conformations that have eclipsed or gauche interactions Nonbonded (steric) strain- (van der Waals repulsion)- results when nonadjacent atoms or groups compete for the same space -nonbonded strain is the dominant source of steric strain in the flagpole interactions of the cyclohexane boat conformation

Suppose an excited parent isotope A/ZX* gamma decays to A'/Z'X, which then undergoes positron emission to form A''/Z''Y, which in turn alpha decays to A'''/Z'''Z. If Z is americium-241, what is A/ZX*?

Answer on pg 328 physics Because the final daughter nucleus is given, it will be necessary to work backwards through the reactions. The last reaction is the following alpha decay: A''/Z'' Y --> 241/95 Am + 4/2 alpha The atomic number fo the parent nucleus must be 97, and the mass number is 245. This is berkelium-245. The preceding reaction is the following positron emission: A'/Z' X--> 245/97 Bk + 0/1B+ The atomic number of the parent nucleus must be 98 and the mass number is 245. This is californium-245. Finally, the preceding reaction is the following gamma decay: A/Z X* --> 245/98Cf + gamma The atomic number of the parent nucleus must be 98, and the mass number is 245. This is a higher-energy form of californium-245: A/Z X* = 245/98 Cf*

Suppose a promethium-146 nucleus beta-decays as follows: 146/61Pd-->A'/Z'Y + B- What are the mass number and atomic number of the daughter isotope Y?

Answer pg 327 physics balance the atomic numbers and mass numbers 146/61Pm --> A/Z+1Y + 0/-1B- 146/61Pm-->146/62Y + 0/-1B- Y must be samarium (Sm) because its atomic number is 62. Therefore, the daughter nucleus is 146/62Sm

Calculate the pH of a 1 x 10^-8M solution of HCl.

At first, one may calculate the pH as -log[H+] = -log10^-8 M=8. However, this answer is not feasible: a pH of 8 cannot describe an acidic solution at 298K because the presence of the acid will increase the hydrogen ion [ ] to above 10^-7M, resulting in an acidic pH below 7. Recognize that the acid [ ] in this question is actually 10 times less than the equilibrium [ ] of hydrogen ions in pure water generated by the autoionization of water. Consequently, the hydrogen ion [ ] from the water itself is significant and cannot be ignored. This can be represented in the equilibrium expression in which x represents the [ ] of H3O+ and OH- resulting from the autoionization of water: Kw=[H3O+][OH-] = [x+10^-8][x] =10^-14 Solving for x (which would require a quadratic equation and is beyond the scope of the MCAT) gives x=9.5 x 10^-8M. The total [ ] of hyrogen ions is [H+]total = (9.5E-8)+(1E-8)=1.05E-7M. Notice that this is extremely close to the [ ] of H+ in pure water. The pH of this acidic solution can now be calculated as pH=-log(1.05E-7) = approx 7 (real: 6.98). This pH is slightly lower than 7, as expected for a very dilute acidic solution.

A test for proteins in urine involves precipitation but is often complicated by precipitation of calcium phosphate. Which procedure prevents precipitation of the salt? A. Addition of buffer to maintain high pH B. Addition of buffer to maintain a neutral pH C. Addition of calcium hydroxide D. Addition of sodium phosphate

B

Bernoulli's equation is the reason for the upward force that permits airplane flight. Which statement best summarizes the equation's relationship to flight? A. The speed of airflow is equal on the top and bottom of a wing, resulting in nonturbulent flight. B. The speed of airflow is greater over the curved top of the wing, resulting in less pressure on the top of the wing and the production of a net upward force on the wing, in turn resulting in flight. C. The speed of airflow on the flat bottom of the wing is greater than over the curved top of the wing, resulting in more rpressure below the wing and the production of a net upward force on the wing, in turn resulting in flight. D. The weight of the wing is directly proportional to the weight of air it displaces.

B

In an adiabatic compression process, the internal energy of the gas: a. increases because the work done on the gas is negative b. increases because the work done on the gas is positive c. decreases because the work done on the gas is negative d. decreases because the work done on the gas is positive

B

The entropy of a system can: A. never decrease B. decrease when the entropy of the surroundings inc by at least as much C. decrease when the system is isolated and the process is irreversible D. decrease during an adiabatic reversible process

B

The function of a buffer is to A. maintain a neutral pH B. resist changes in pH when small amounts of acid or base are added C. slow down reactions between acids and bases D. speed up reactions between acids and bases

B

What is the most specific characterization of the reaction shown? Ca(OH)2 (aq) + H2SO4 (aq) --> CaSO4 (aq) + H2O (l) A. single-displacement B. neutralization C. double-displacement D. redox

B

Which of the following best characterizes the work-energy theorem? A. The work done by any force is proportional to the magnitude of that force B. The total work done on any object is equal to the change in kinetic energy for that object C. The work done on an object by any force is proportional to the change in kinetic energy for that object D. The work done by an applied force on any object is equal to the change in kinetic energy for that object

B

Which of the following best describes an important property of bond energy? A. bond energy increases with inc bond length B. The more shared electron pairs comprising a bond, the higher energy of that bond C. Single bonds are more difficult to break than double bonds D. Bond energy and bond length are unrelated

B

Which of the following quantum number sets is possible? A. n= 2 l=2. ml=1. ms=+1/2 B. n= 2 l=1. ml=-1. ms=+1/2 C. n= 2 l=0. ml=-1. ms=-1/2 D. n= 2. l=0. ml=1. ms=-1/2

B

lithium and sodium have similar chemical properties. For example, both can form ionic bonds with chloride. Which of the following best explains this similarity? A. Both lithium and sodium ions are positively charged. B. lithium and sodium are in the sam group of the periodic table C. lithium and sodium are in the same period of the periodic table D. Both lithium and sodium have low atomic weights

B

CO2 + H2O --> C6H12O6 + O2 What is the theoretical yield of glucose if 30g of water are reaction with excess CO2 and energy? A. 30g B. 50g C. 300.1g D. 1801 g

B 30g H2O x [1 mol H2O/18gH2O][1 mol C6H12O6/6 mol H2O][180g C6H12O6/1mol C6H12O6]

Which of the following devices would be the most appropriate to use to measure the heat capacity of a liquid? A. Thermometer B. Calorimeter C. Barometer D. Volumetric flask

B A calorimeter measures heat transfer. Although calorimeters often incorporate thermometers, the thermometer itself only tracks temperature, not the heat transfer itself, so A is incorrect. C is irrelevant; barometers measure changes in pressure. D si also incorrect because volumetric flasks measure quantities of liquid, not the heat capacity of the liquid.

A researcher wishes to generate a parameter for American women's mean weight. Which of the following is the most significant concern? A. Measuring a person's weight may have psychological consequences which is unethical. B. Gathering all of the necessary study participants would be prohibitive. C. Knowing the average weight of women does not provide any useful info. D. Enough studies have already been conducted on this topic to render it unnecessary

B A parameter is a population measure, so to calculate it, every single member must be measured. Identifying, measuring, and recording data for a population that large-over 160 million-is essentially impossible. Common biometric measures, if not misused, generally neither cause significant psychological harm nor are unethical, eliminating A. Knowing a mean weight could have major ramifications, including public health measures, medical recommendations, and shifting of body image, eliminating C. While a number of studies on weight have been performed, there is not yet a parameter describing the entire population, eliminating D.

Acetic acid dissociates in solution according to the following equation; CHCOOH = CH3COO- + H+ If sodium acetate, is added to a solution of acetic acid in excess water, which of the following effects would be observed in the solution? A. Decreased pH B. Increased pH C. Decreased pKeq (pKa) D. Increased pKeq (pKa)

B Adding sodium acetate increases the number of acetate ions present. According to Le Chatelier's principle, this change will push the reaction to the left, resulting in a dec in the # of free H+ ions. Because pH is determined by the hydrogen ion [ ], a decrease in the number of free protons will increase the pH. An acid's Ka (which is simply the Keq for acid dissociation) will remain constant under a given temp and pressure, eliminating C and D

Alcohols have higher boiling points than their analogous hydrocarbons because; A. The oxygen atoms in alcohols have shorter bond lengths B. Hydrocarbons bonding is present in alcohols C. Alcohols are more acidic than their analogous hydrocarbons D. Alcohols can be oxidized to ketones

B Alcohols have higher BPs than their analogous hydrocarbons as a result of their polarized O-H bonds, in which oxygen is partially negative and hydrogen is partially positive. this enables the oxygen atoms of other alcohol molecules to be attracted to the hydrogen, forming a hydrogen bond. Heat is required to overcome these hydrogen bonds, thereby increasing the BP. the analogous hydrocarbons do not form hydrogen bonds and therefore vaporize at lower temps. A is irrelevant; oxygen's bond length is not a factor in determining a substance's boiling point. C and D are true statements but are also irrelevant to BP determination

When reacted with ammonia (NH3) at 200 degrees celsius., which enolate of a carbonyl containing compound would predominate? A. Kinetic enolate B. Thermodynamic enolate C. Neither enolate; they would be present in roughly equal proportions D. Neither enolate; these reaction conditions would not form either enolate

B At high temperatures and with a weak base like NH3, the thermodynamic enolate will be favored. The reaction proceeds slowly with the weak base, giving the kinetic enolate time to interconvert to the more stable thermodynamic enolate.

Which of the following enzymes cleaves polysaccharide chains and yields maltose exclusively? A. a-Amylase B. B-Amylase C. debranching enzyme D. glycogen phosphorylase

B B-amylase cleaves amylose at the nonreducing end of the polymer to yield maltose exclusively, while a-amylase, A, cleaves amylose anywhere along the chain to yield short polysaccharides, maltose, and glucose. Debranching enzyme, C, removes oligosaccharides from a branch in glycogen or starches, while glycogen phosphorylase, D, yields glucose 1-phosphate

Suppose an extraction with methylene chloride (p=1.33g/mL) is performed, with the desired compound initially in brine (p=1.04g/mL). In a separatory funnel, which layer will be the organic layer? A. The top layer B. The bottom layer C. No layers were observed; methylene chloride and brine are miscible D. More info is needed to answer the question

B Because methylene chloride is denser than brine (salt water), the organic layer will settle at the bottom of the funnel. Methylene chloride is nonpolar, so it cannot mix with brine, eliminating C.

Lactoferrin, a milk protein, is a valuable antimicrobial agent that is extracted from pasteurized, defatted milk utilizing a column containing a charged resin. This is an example fo whic hof the following chromatographic techniques? A. Thin-layer chromatography B. Ion-exchange chromatography C. Affinity chromatography D. Size-exclusion chromatography

B Because the lactoferrin proteins are likely to be charged, as is the resin described in the question, this is an example of ion-exchange chromatography. The charged protein molecules will stick to the column, while the remainder of the milk washes through and can later be washed off the column and collected.

Two organic liquids, pictured in FIGURE 6 CHEMISTRY, are combined to form a solution. Based on their structures, will the solution closely obey Raoult's law? A. Yes; the liquids differ due to the additional methyl group of toluene and, therefore, will not deviate from Raoult's law. B. Yes; the liquids are very similar and, therefore, will not deviate from Raoult's law. C. No; the liquids differ due to the additional methyl group on toulene and, therefore, will deviate from Raoult's law. D. No; the liquids both contain benzene rings, which will interact with each other and cause deviation from Raoult's law.

B Benzene and toluene are both organic liquids and have very similar properties. They are both nonpolar and are almost exactly the same size. Raoult's law states that ideal solutio nbehavior is observed when solute-solute, solvent-solvent, and solute-solvent interactions are all very similar. Therefore, benzene and toluene in solution will be predicted to behave as a nearly ideal solution.

Using the appropriate number of scientific digits, what is the answer to the following math problem? (Note: Assume all numbers are the results of measurements.) 3.060 x 4.10 + 200. = A. 210 B. 213 C. 212.5 D. 212.55

B During multiplication, the answer is maintained to the smallest number of significant digits. During addition, it is maintained to the smallest number of decimal places. By following the order of operations, addition is the last operation; thus, we cannot have a decimal in our answer choice. Because multiplication occurred earlier, the result of that multiplication may be shortened according to the two significant figures in 4.10, but not the entire answer.

If a defibrillator passes 15A of current through a patient's body for 0.1 seconds, how much charge goes through the patient's skin? A. 0.15C B. 1.5C C. 15C D. 150C

B Electrical current is defined as charge flow, or in mathematical terms, charge transferred per time: I=Q/deltat. A 15A current that acts for 0.1s will transfer 15A x 0.1s=1.5C of charge.

Which phenomenon would cause monochromatic light entering the prism along path AB to leave along path CD? FIGURE 12 PHYSICS A. Dispersion B. Refraction C. Diffraction D. Polarization

B Even though the light is traveling through a prism, the change in the light's direction is caused by refraction, not dispersion. Dispersion involves the breaking up of polychromatic light into its component wavelengths because the degree of refraction depends on the wavelength. We are told that the incident light is monochromatic or, in other words, consists of only one wavelength; therefore, light will not be dispersed. Diffraction describes the spreading of light waves as they pass through a small opening. Polarization is the alignment of the electric field component of light waves.

At what temperature do the Fahrenheit and Celsius scales give equal values? A. 0K B. 233K C. 313K D. 273K

B F=(9/5)C +32 T=(9/5)T + 32 --> (-4/5)T = 32 --> T= (-5/4)(32) = -40 -40C +273 = 233K

How many electrons are involved in the following half-reaction after it is balanced? Cr2O7^2- + H+ + e- --> Cr2+ + H2O A. 2 B. 8 C. 12 D. 16

B First, balance the atoms in the equation: Cr2O7^-2 + 14H+ --> 2Cr2+ + 7H2O Now, adjust the number of electrons to balance the charge. Currently, the left side has a charge of +12 (-2 from dichromate and +14 from protons). The right side has a charge of +4 (+2 from each chromium cation). To decrease the charge on the left side from +12 to +4, we should add 8 electrons to the left side.

When ammonia (NH3) is used as a solvent, it can form complex ions. For example, dissolving AgCl in NH3 will result in the complex ion [Ag(NH3)]2+. What effect would the formation of complex ions have on the solubility of a compound like AgCl in NH3? A. The solubility of AgCl will increase because complex ion formation will cause more ions to exist in solution , which interact with AgCl to cause it to dissociate. B. The solubility of AgCl will increase because complex ion formation will consume Ag+ ions and cause the equilibrium to shift away from solid AgCl. C. The solubility of AgCl will decrease because Ag+ ions are in complexes, and the Ag+ ions that are not complexed will associate with Cl- to form solid AgCl. D. The solubility of AgCl will decrease because complex ion formation will consume Ag+ ions and cause the equilibrium to shift toward the solid AgCl.

B Formation of complex ions between silver ions and ammonia will cause more molecules of solid AgCl to dissociate. The equilibrium is driven toward dissociation because the Ag+ ions are essentially being removed from solution when they complex with ammonia. This rationale is based upon Le Chatelier's principle, stating that when a chemical equilibrium experiences a change in [ ], the system will shift to counteract that change.

Fractional distillation under atmospheric pressure would most likely be used to separate which of the following compounds? A. Methylene chloride (boiling point of 40C) and water (BP 10C) B. Ethyl acetate (BP 77C) and ethanol (BP 80C) C. Aniline (BP 184C) and benzyl alcohol (BP 205C) D. Aniline (BP 184C) and water (BP 100C)

B Fractional distillation is the most effective procedure for separating two liquids that boil within a few degrees of each other. Ethyl acetate and ethanol boil well within 35 of each other and thus would be good candidates for fractional distillation. Fractional distillation could also be used for the liquids in C but would require lower pressure because of their high boiling points.

A large cylinder is filled with equal volumes of two immiscible fluids. A balloon is submerged in the first fluid; the gauge pressure in the balloon at the deepest point in the first fluid is 3atm. Next, the balloon is lowered all the way to the bottom of the second fluid, where the hydrostatic pressure in the balloon is 8atm. What is the ratio of the gauge pressure accounted for by the first fluid to the gauge pressure accounted for by the second fluid?

B Gauge pressure is the pressure inside the balloon above and beyond atmospheric pressure. It is the total (absolute or hydrostatic) pressure in the balloon minus the atmospheric pressure. Hydrostatic pressure is the total pressure in the balloon (gauge pressure +atm pressure). Gauge pressure second fluid=8-1-3=4atm. 3:4 is the ratio

Which of the following statements could be true about a Na-Cd cell, based on the info below? Na+ + e- --> Na Ered= -2.71V Cd2+ + 2e- --> Cd Ered= -0.40V A. It is a galvanic cell, and sodium is the cathode B. It is an electrolytic cell, and cadmium is the anode C. It is a galvanic cell, with Ecell= 3.11V D. It is an electrolytic cell, with E cell = -3.11V

B If this were a galvanic cell, the species with the more positive reduction potential (cadmium) would be reduced. The cathode is always reduced in an electrochemical cell, so sodium could not be the cathode in such a galvanic cell, eliminating A. Sodium would be the cathode in an electrolytic cell, however, which woul dmake cadmium the anode. this, the answer is B. Note that we do not have to determine the Ecell because we already know the answer. However, the E cell would be -2.71 - (-0.40) = -2.31V for an electrolytic cell

To form a geminal diol, which of the following could attach a carbonyl carbon? A. Hydrogen peroxide B. Water C. Potassium dichromate D. Ethanol

B In a hydration reaction, water adds to a carbonyl, forming a geminal diol-a compound with two hydroxyl groups on the same carbon. Hydrogen peroxide and potassium dichromate are oxidizing agents that can convert an aldehyde to a carboxylic acid. Ethanol will react with a carbonyl compound to form an acetal or a metal, if excess ethanol is available.

A mixture of sand, benzoic acid, and naphthalene in ether is best separated by: A. filtration, followed by acidic extraction, followed by recrystallization B. filtration, followed by basic extraction, followed by evaporation C. extraction, followed by distillation, followed by gas chromatography D. filtration, followed by size-exclusion column chromatography, followed by extraction

B In this question, three substances must be separated using a combination of techniques. The first step should be the most obvious: remove the sand by filtration. The remaining compounds-benzoic acid and naphthalene-are sill dissolved in ether. If the solution is extracted with aqueous base, the benzoate anion is formed and becomes dissolved in the aqueous layer, while naphthalene, a nonpolar compound, remains in the ether. Finally, evaporation of the ether will yield purified naphthalene.

The formation of a-D-glucopyranose from B-D-glucopyranose is called A. Glycosidation B. Mutarotation C. Enantiomerization D. Racemization

B Mutarotation is the interconversion between anomers of a compound. Enantiomerization and racemization, C and D, are related: enantiomerization is the formation of a mirror-image or optically inverted form of a compound, whereas racemization is moving a solution toward an equal concentration of both enantiomers. Glycosidation, A, is the addition of a sugar to another compound.

Which of the following correctly lists hexane, phenol, and cyclohexanol by increasing acidity of the hydroxyl hydrogen? A.phenol < hexanol < cyclohexanol B. cyclohexanol < hexanol < phenol C. cyclohexanol < phenol < hexanol D. phenol < cyclohexanol < hexanol

B Phenols have significantly more acidic hydroxyl hydrogens than other alcohols because of resonance stabilization of the conjugate base, so this will be the most acidic hydroxyl hydrogen. The acidity of hexanol and cyclohexanol are close, but the hydroxyl hydrogen of hexanol is slightly more acidic because the ring structure of cyclohexanol is slightly electron-donating, which makes its hydroxyl hydrogen slightly less acidic.

The hydrogens of phosphoric acid have pKa values that: A. Allow high buffering capacity over a small pH range B. Allow moderate buffering capacity over a large pH range C. Allow low buffering capacity over a small pH range D. Do not allow buffering

B Phosphoric acid has three hydrogens with pKa values spread across the pH range. This allows some degree fo buggering over almost the entire standard pH range from 0 to 14.

Given an electric dipole, the electric potential is zero A. only at the midpoint of the dipole axis B. anywhere on the perpendicular bisector of the dipole axis and at infinity D. anywhere on the dipole axis D. only for points at infinity

B Potential is a scalar quantity. The total potential is the sum of the potentials of the positive and negative charges: Vsubr=V+ + V- = (+kq/r+)+(-kq/r-) where r+ and r- represent the distances from the positive and negative charge, respectively. The sum of these terms will be zero at any point where r+ = r-. This will be at any point along the perpendicular bisector of the dipole axis, as well as at infinity.

After randomization, it is discovered that one group in a study has almost twice as many women as the other. Which of the following is an appropriate response? A. Move men and women between groups manually so that they have the same gender profile. B. Check the randomization algorithm; if it is fair, continue with the research. C. Eliminate all of these subjects because of potential bias and randomize a new cohort. D. Keep the current cohort and continue randomizing subjects until the gender profiles are equal.

B Randomization is based on the idea that the results will only vary as a result of random chance as long as the assignment is proper. The appropriate response to a fair algorithm that assigns gorups in an unexpected way is to proceed with the research. Participants should never be assigned by the researcher, not should one continue randomizing samples to achieve a desired outcomes-these are likely to introduce more error than leaving unequal groups, eliminating A and D. It is unnecessary to drop this entire cohort, assuming the randomization algorithm was fair, eliminating C.

Which of the following compounds is LEAST likely to be found in the salt bridge of a galvanic cell? A. NaCl B. SO3 C. NgSO3 D. NH4NO3

B Salt bridges contain inert electrolytes. Ionic compounds such as A, C, and D are known to be strong electrolytes because they completely dissociate in solution. B cannot be considered an electrolyte because its atoms are covalently bonded and will not dissociate in aqueous solution . B and C may appear similar, but there is an important distinction to be made. C implies that Mg2+ and SO3^2- are the final, dissociated ionic constituents, while B implies that neutral SO2 would have to be dissolved in solution.

In some forms of otosclerosis, the stapedial foot plate, which transmits vibrations from the bones of the middle ear to the fluid within the cochlea, can become fixed in position. This limits the displacement of the stapedial foot plate during vibration. Based on this mechanism, which of the following symptoms would most likely be seen in an individual with otosclerosis? A. An increase in the perceived volume of sounds B. An decrease in the perceived volume of sounds C. An increase in the perceived pitch of sounds D. An decrease in the perceived pitch of sounds

B Saying that the stapedial footplate has limited displacement during vibration is another way of stating that the amplitude of the vibration has been decreased. Because amplitude is related to intensity, and intensity is related to sound level, the perceived sound level (volume) will be decreased as well. Pitch, described in C and D, is related to the frequency of a sound, not its amplitude.

Shock waves have the greatest impact when the source is traveling: A. just below the speed of sound B. exactly at the speed of sound C. just above the speed of sound D. well above the speed of sound

B Shock waves are the buildup of wave fronts as the distance between those wave fronts decreases. This occurs maximally when an object is traveling at exactly the same speed as the wave is traveling (the speed of sound). Once an object moves faster than the speed of sound, some of the effects of the shock wave are mitigated because all of the wave fronts will trail behind the object, destructively interfering with each other.

For a certain chemical process, deltaG^not=-4.955kJ/mol. What is the equilibrium constant Keq for this reaction? Note R=8.314J/molK A. 1 B. 7.4 C. 8.9 D. 10

B Solve this question using the equation deltaGrxn^not=-RTlnKeq. deltaGrxn^not is -4.955kJ/mol, R is 8.314J/molK, and T=298K because the reaction is occurring under standard conditions. Because deltaGrxn^not uses kilojoules in its units and R uses joules, one will have to be converted -4955J/mol=-(8.314J/molK)(298K)(lnKeq) lnKeq=4955/(8.314 x 298)= 2 In lkKeq=2, then keq=e^2. The value of e is approximately 2.7, so e^2=2.7^2, which will be a number between 2^2=4 and 3^2=9 Both B and C fit these criteria; however, 8.9 is very close to 9, so we can assume that its square root is very, very close to 3. The answer choice should be a bit smaller so B is correct

A medical student attempting to impress her attending physician refers to a recent article that says that there is a statistically significant difference in pregnancy length with a new therapy. Which of the following is most likely to be a valid criticism of the article? A. Medical students usually reference articles in the newest journals, which may be unreliable. B. The effect didn't change patient outcomes, only a secondary measure C. There is a lack of internal validity in the results, despite significance D. Selection bias is inherent in the scientific process

B Statistical significance is not the same as clinical significance. These are medications that increase the length of pregnancy in preterm labor, but some only do so for a few hours. Because this may not impact patient outcomes, it should not inform treatment decisions.

Which of the following carboxylic acids will be the most acidic? A. CH3CHClCH2COOH B. CH3CH2CCL2COOH C. CH3CH2CHClCOOH D. CH3CH2CH2COOH

B The acidity of carboxylic acids is significantly increased by the presence of highly electronegative functional groups. Their electron-withdrawing effect increases the stability of the carboxylate anion, favoring proton dissociation. This effect increases as the number of electronegative groups on the chain increases, and it also increases as the distance between the acid functionality and electronegative group decreases. This answer has two halogens bonded to it at a smaller distance from the carboxyl group compared to the other answers.

A baseball that lands in a lake (water) experiences a buoyant force of approximately 2N while submerged. A baseball's mass is specified as 142-149g, or 5-5¼ oz., according to the official rules of baseball. What is the specific gravity of bromine if the same baseball experiences a buoyant force of 6.2N while submerged after landing in an open vat of liquid bromine at a nearby chemical factory? A. 1.5 B. 3.1 C. 4 D. 40

B The buoyant force exerted on an object by a fluid is given by the equation F=ρVg, where ρ is the density of fluid displaced by an object, V is the volume of fluid displaced, and g is gravitational acceleration. Specific gravity is the ratio of fluid density to density of water, and the student should know the density of water as 1 g/cm3 or 1000kg/m3. The buoyant force in water is Fwater=ρwaterVbaseballg, and the buoyant force in bromine is Fbromine=ρbromineVbaseballg. Since specific gravity is ρbromine/ρwater, and all other variables are constant in the two buoyant force expressions, the specific gravity of bromine for this problem is equal to Fbromine/Fwater. 6.2N/2N equals SGbromine= 3.1. A. 1.5, incorrect, This is the approximate weight of the baseball in newtons. B. 3.1, correct. C. 4, incorrect, This is the buoyant force in bromine divided by the ball's weight in newtons. D. 40, incorrect, This is the buoyant force in bromine divided by the ball's mass in grams.

What is the azimuthal quantum number (l) for the orbital to which a fluorine atom gains an electron to form an F- ion? A. 0 B. 1 C. 2 D. 3

B The electron config for Fl is 1s^2 2s^2 2p^5. When F gains an e- to become F-, the electron adds to the p orbital. P orbitals have l=1.

FIGURE 14 PHYSICS illustrates an electron with initial energy of -10eV moving from point A to point B. What change accompanies the movement of the electron? A. Absorption of a photon B. Emission of a photon C. Decrease in the atom's work function D. Increase in the atom's total energy

B The electron moves from a higher energy level to a lower energy level; this can only occur if the extra energy is dissipated through the emission of a photon. If the electron moved from B to A, it would absorb a photon and increase the atom's total energy; however, the opposite is occurring, so A and D can be eliminated. The work function is the amount of energy required to eject an electron from a material; when moving from A to B, the electrical potential energy of the atom decreases, meaning that more energy will be required to free the electron from the atom, eliminating C.

Which of the following actions does NOT affect the equilibrium position of a reaction? A. Adding or removing heat B. Adding or removing a catalyst C. Increasing or decreasing [ ]s of reactants D> Increasing or decreasing volumes of reactants

B The equilibrium of a reaction can be changed by several factors. Adding or subtracting heat, A, would shift the equilibrium based on the enthalpy change of the reaction. Increasing reactant [ ]s would shift the equilibrium in the direction of the product, and the opposite would occur if reactant [ ]s were decreased, eliminating C. Changing the volume of a reactant would affect any reaction with gaseous reactants or products, eliminating D. While adding or removing a catalyst would change the reaction rates, it would not change where equilibrium lies.

Why is the a-anomer of D-glucose less likely to form than the B-anomer? A. The B-anomer is preferred for metabolism B. The B-anomer undergoes less electron repulsion C. The a-anomer is the more stable anemia D. The a-anomer forms more in L-glucose

B The hydroxyl group on the anomeric carbon of the B-anomer is equatorial, thereby creating less nonbonded strain than the a-anomer, which has the hydroxyl group of the anomeric carbon in axial position

A gaseous mixture contains nitrogen and helium and has a total pressure of 150 torr. The nitrogen particles comprise 80% of the gas, the helium particles make up the other 20% of the gas. What is the pressure exerted by each individual gas? A. 100 torr nitrogen, 50 torr helium B. 120 torr nitrogen, 30 torr helium C. 30 torr nitrogen, 120 torr helium D. 50 torr nitrogen, 100 torr helium

B The partial pressure of each gas is found by multiplying the total pressure by the mole fraction of the gas. Because 80% of the molecules are nitrogen, the mole fraction of nitrogen gas is equal to 0.80. Similarly, for helium, the mole fraction is 0.2. To find the pressure exerted by nitrogen, multiply the total pressure (150torr) by 0.8 to obtain 120 torr of nitrogen. The remainder, 30 torr is attributable to helium.

A 0.040 g piece of magnesium is placed in a beaker of hydrochloric acid. Hydrogen gas is generated according to the following equation: Mg(s) + 2HCl(aq) -->MgCl2(aq) + H2(g) The gas is collected over water at 25C, and the gauge pressure during the experiment reads 784mmHg. The gas displaces a volume of 100mL. The vapor pressure of water at 25C is approximately 24mmHg. Based on this data, how many moles of H are produced in this reaction? Note: R=0.0821Latm/molK or 8.314J/K mol A. 4.04E-5 moles hydrogen B. 4.09E-3 moles hydrogen C. 3.07E-2 moles hydrogen D. 3.11 moles hydrogen

B The pressure of the gas is calculated by subtracting the vapor pressure of water from the measured pressure during the experiment: 784mmHg-24mmHg=760mmHg or 1 atm. This is because the reaction is carried out in an aqueous environment; the water present will contribute to the partial pressures of the gas over the liquid. The ideal gas law can be used to calculate the moles of hydrogen gas. The volume of the gas is 0.100L, the temperature is 298K, and R=0.0821L(atm)/mol(K) math on pg 295 gen chem n=PV/RT=(1atm)(0.1L)/(0.0821)(298K) =0.004

Considering only the 0 to 4.5 ppm region of a 1-NMR spectrum, how could ethanol and isopropanol be distinguished? A. They cannot be distinguished form 1H-NMR alone B. A tripled and quartet are observed for ethanol, whereas a doublet and septet are observed for isopropanol C. A triplet and quartet are observed for isopropanol, whereas a doublet and septet are observed for ethanol D. The alcohol hydrogen in ethanol will appear within that region, whereas the alcohol hydrogen in isopropanol will appear downfield of that region

B The region in question often gives info about the types of alkyl groups present. Specifically, ethanol will give a characteristic triplet for the methyl group (which is coupled to -CH2-) and a quartet for -CH2- (which is coupled to the methyl group). Isopropanol will have a septet for the -CH- group (which is coupled ot both methyl groups combined) and a doublet for the two methyl groups (which are coupled to -CH-). In both cases, the proton in the alcohol does not participate in coupling. The alcohol hydrogen likely lies downfield for both compounds because it is bonded to such an electronegative element.

An anchor made of iron weights 833N on the deck of a ship. If the anchor is now suspended in seawater by a massless chain, what is the tension in the chain? (Note: density of iron =7800kg/m3, density of seawater is 1025kg/m3). A. 100N B. 724N C. 833N D. 957N

B The tension in the chain is the difference between the anchor's weight and the buoyant force because the object is in translational equilibrium: T=Fg-Fbuoy. The object's weight is 833N, and the buoyant force can be found using Archimedes' principle. The magnitude of the buoyant force is equal to the weight of the seawater that the anchor displaces: Fbuoy=rhowater(Vwater)g Because the anchor is submerged entirely, the volume of the water dispaced is equal to the volume of the anchor, which is equal to its mass divided by its density. We are not given the anchor's mass, but its value must be the magnitude of the weight of the anchor divided by g. Putting all of this together, we can obtain the buoyant force. Fbuoy=rhowater(Vwater)g = rhowater(VA)g=rho water (mA/rhoA)g=rhowater(Fg/rhoA x g) =Fg(rho water/rhoA) =833N(1025kg/m3/7800kg/m3)=109N Lastly, we can obtain the tension from T=Fg-Fbuoy T=833N-109N=724N The key to quickly solving this problem on test day is recognizing that the answer choices contain an outlier, A, a value slightly less than the weight of the anchor (B), the weight of the anchor(C), and a value slightly higher than the weight of the anchor(D). Since buoyant force is in the same direction as tension and their sum most equal the weight of the anchor, B is the most likely answer.

Which of the hydrogens in the following molecule Is the most acidic? FIGURE 5 (organic chemistry) A. A B. B C. C D. D

B This hydrogen is on the carbon between wo carbonyls, which means that it is particularly acidic. This is due both to the inductive effects of the two oxygen atoms in the carbonyls and the resonance stabilization of the anion between the carbonyl groups.

3-hydroxybutanal can be formed by the reaction of: A. Methanol in diethyl ether B. Ethanal in base, then in acid C. Butanal in strong acid D. Methanal and ethanal in catalytic base

B This is an example of an aldol condensation, but stopped after aldol formation (before dehydration). After the aldol is formed using strong base, the reaction may be halted by the addition of acid. Butanal in strong acid, described in C, would be likely to deprotonate without gaining the hydroxyl group. Methanal in diethyl ether would not be reactive because diethyl ether is not a strong enough base to abstract the alpha hydrogen, eliminating A. Reaction of the two aldehydes methanol and ethanal in catalytic base would form 3-hydroxypropanal (which would dehydrate to form propenal), not 3-hydroxybutanalE%^WTY'[ \oi7p

It is known that crickets increase their rate of chirping in a direct linear relationship with temp until a maximum chirping rate is reached. Which of the following graphs in FIGURE 19 PHYSICS best represents this relationship? A. B. C. D.

B This question stem indicates that there is a linear relationship, so we know that we are looking for a straight line before a plateau. We also know that linear relationships are represented on linear plots. B satisfies both criteria because the axes show constant intervals. Constant ratios, as shown in C and D, are seen in semilog plots like these, as well as log-log plots.

Which of the following would be formed if methyl bromide were reacted with phthalimide and followed by hydrolysis with an aqueous base? A. C2H5NH2 B. CH3NH2 C. (C2H5)3N D. (CH3)4N+Br-

B This reaction is similar to the Gabriel synthesis. Phthalimide acts as a nucleophile, the methyl carbon acts as an electrophile, and the bromide acts as the LG. Therefore, the reaction between methyl bromide and phthalimide results in the formation of methyl phthalimide. Subseqent hydrolysis then yields methylamine

The reaction in FIGURE 12 ORGANIC CHEMISTRY is important for the breakdown of polypeptides. Under what conditions would the reaction be favored? A. Mild heat B. Acid environment C. Anhydrous environment D. Nonpolar solvent

B This reaction, which is the hydrolysis of an amide, is favored in strong acid. Acid protonates the carbonyl oxygen, which increases the electorphilicity of the carbonyl carbon. This allows water to serve as the nucleophile, attacking the bond and hydrolyzing the molecule.

Given the following standard reduction potentials: Zn2+ + 2e- --> Zn Ered= -0.763V Ag+ + e- --> Ag Ered= +0.337V What is the standard electromotive force of the following reaction? Zn2+ + 2Ag --> 2Ag+ + Zn A. -2.2V B. -1.1V C. +1.1V D. 2.2V

B To determine the standard emf of a cell, simply subtract the standard reduction potentials of the two electrodes. In this case, the cathode is zinc because it is being reduced; the anode is silver because it is being oxidized. Thus, Ecell=Ered,cathode-Ered,anode = -0.763 - 0.337 = 1.10V While we must multiply the silver half-reaction by two to balance electrons, the actual value for the reduction potential does not change. Remember that the standard reduction potential is determined by the identity of the electrode not eh amount of it present.

A water tower operator is interested in increasing the pressure of a column of water that is applied to a piston. She hopes that increasing the pressure will increase the force being applied to the piston. The only way to increase the pressure is to alter the speed of the water as it flows through the pipe to the piston. How should the speed of the water be changed to increase the pressure and force? A. Increase the speed B. Decrease the speed C. Release water intermittently against the pipe D. The speed of water will not change pressure at the piston.

B basic interpretation of Bernoulli's equation that states that at equal heights, speed and pressure of a fluid are inversely related (the Venturi effect). Decreasing the speed o fthe water will therefore inc its pressure. An increase in pressure over a given area will result in inc force being transmitted to the piston

What is the wavelength of a photon that causes an electron to be emitted from a metal with a kinetic energy of 50J? (Note: The work function of the metal is 16J and h=6.262E-34Js)

B math on pg 340 physics To determine the wavelength of the light ray, first calculate its frequency from the photoelectric effect equation: K=hf-W --> f=K+W/h =50J+16J/(6.626E-34Js) = 6.6E1/6.6E-34=10^35Hz In this case, estimation of Planck's constant makes our calculation much simpler without leading us to a nonviable answer. It is worth attempting an estimation first to avoid doing more work than necessary. Now we can determine the wavelength of the incident ray of light by relating the frequency to the speed of light: c=f(wavelength) --> wavelength =c/f =(3E8m/s)/(10^35Hz)=3E-27m

A child is practicing the first overtone on his flute. If his brother covers one end of the flute for a brief second, how will the sound change, assuming that the new pitch represents the first overtone in the new setup? A. The pitch of the sound will go up B. The pitch of the sound will go down C. The pitch of the sound will not change D. The change in the pitch depends on the starting pitch

B physics pg 266 This question is testing our understanding of pipes open at one or both ends. To begin, remember that high-frequency sounds have a high pitch and low-frequency sounds have a low pitch. The pipe in this example begins as one that is open on both ends, and then one end is closed off. Our task, therefore, is to determine how the frequency of the second harmonic differs between a pipe that is open at both ends from one of equal length that is open at only one end. For a pipe of length L open at both ends, the wavelength for the second harmonic (first overtone) is equal to L: λ=(2L/n)=(2L/2)=L In contrast, for a pipe open at one end and closed at the other, the wavelength is equal to 4L/3: λ=(4L/n)=(4L/3) Keep in mind that the first overtone for a closed pipe corresponds to the third harmonic, not the second. Thus, when the brother covers one end of the flute, the wavelength increases. Given that the wavelength and the frequency of a sound are inversely proportional, an increase in wavelength corresponds to a decrease in frequency. Therefore, when the brother covers one end of the flute, the sound produced by the instrument will be slightly lower in pitch than the original sound.

If the speed of a wave is 3m/s and its wavelength is 10 cm, what is its period? A. 0.01s B. 0.03s C. 0.1s D. 0.3s

B physics pg 267 This question is testing our understanding of traveling waves. We know that frequency and wavelength are related through the equation v=f(lambda). Frequency and period are inverses of each other, so this equation could be rearranged to solve for period: v=(1/T)λ-->T=(λ/v)=(0.1m/3m/s)=0.03s

Silica gel is often use in thin-layer chromatography. What property does silica gel probably possess that makes it useful for this purpose? A. Acidity B. Polarity C. Specifically sized pores D. Aqueous solubility

B polarity is used to selectively attract specific solutes within a nonpolar solvent phase. Although silica gels have other properties, this is the most important to TLC.

Which of the following will cause the greatest increase in the boiling point of water when it is dissolves in 1kg H2O? A. 0.4 mol calcium sulfate B. 0.5 mol iron(III) nitrate C. 1 mol acetic acid D. 1 mol sucrose

B solution on pg 340 general chemistry deltaTb=iKbm. m is the molality of the solution, Kb is the boiling point elevation constant. In this case, the solvent is always water, so Kb will be the same for each solution. What is needed is the number of dissociated particles from each of the original species. This is the van't Hoff factor . CaSO4, 0.4 moles, 2 dissolved particles, i x m= 0.8 Fe(NO3)3, 0.5 moles, 4 dissolved particles, i x m= 2 CH3COOH, 1 mole, between 1 and 2 dissolved particles, i x m= between 1 and 2 C12H22O11, 1 mole, 1 dissolved particle, i x m = 1 The choice is between iron (III) ntirate and acetic acid. The fact that acetic acid is a weak acid indicates that only a few particles will dissociate into H+ and acetate. Therefore, the normality of the acetic acid will be much closer to 1 than 2

Lead is a toxic element that can cause many symptoms including mental retardation in children. If a body of water is polluted with lead ions at 200 ppb (parts per billion), what is the [ ] of lead expressed as molarity? (Note: The density of water is 1g/mol and ppb=grams per 10^9 grams of solution). A. 9.7E-10M Pb2+ B. 9.7E-7M Pb2+ C. 6.2E-7M Pb2+ D. 6.2E-6M Pb2+

B solution on pg 341 general chemistry 200 ppb of Pb2+ is equivalent to 200 g of Pb2+ in 10^9 grams of solution; given the extremely low [ ] of lead, the mass of the water can be assumed to be approximately 10^9g as well. To solve, set up a dimensional analysis. The units needed at the end are moles per liter (molarity), so convert from grams of lead to moles of lead and grams of water to liters of water: (200g Pb2+)/(10^9 H2O) x (10^3g H2O)/(1L) x (1 mol Pb2+)/(207.2 g Pb2+) = approx 200E3/200E9 =1E06M = 10E-7 M actual value 9.67E-7M

The catalytic production of dihydroxyacetone and glyceraldehyde 3-phosphate (2-hydroxy-3-oxopropyl dihydrogen phosphate) from fructose-1,6-bisphosphate ({[(2S,3S,4S,5R)-3,4-dihydroxy-5-[(phosphonooxy) methyl] oxolan-2-yl] methyloxy} phosphonic acid) is what type of reaction? A. Aldol condensation B. retro-aldol reaction. C. dehydration D. nucleophilic attack

B the nomenclature in this question is well above MCAT drawing standards. But we can discern that we are forming a ketone and an aldehyde from a single molecule. The hallmark of a reverse aldol reaction is the breakage of a C-C bond, forming two aldehydes, two ketones, or one of each. In an aldol condensation, A, we would expect to form a single product by combining two aldehydes, two ketones, or one of each. A dehydration reaction, C, should release a water molecule, rather than breaking apart a large organic molecule into two smaller molecules. A nucleophilic attack, D, should feature the formation of a bond between a nucleophile and an electrophile; again, we would not expect to break apart a large organic molecule into two smaller molecules. Note that by simply noting how many reactants and products are present in the reaction is sufficient to determine the answer.

What would be the charge of aspartic acid at pH 7? A. Neutral B. Negative C. Positive D. Not enough info

B The amino acid in question is an acidic amino acid. At neutral pH, both of the carboxyl groups are ionized, so there are two negative charges on the molecules. Only one of the charges is neutralized by the positive charge on the amino group, so the molecules has an overall negative charge.

In experiment A, a student mixes ink with water and notices that the two liquids mix evenly. In experiment B, the student mixes oil with water; in this case, the liquids separate into two different layers. The entropy change is: A. positive in A and negative in B B. positive in B and zero in B C. negative in A and positive in B D. zero in A and negative in B

B (physics ch 3)

Increasing temperature can alter the Keq of a reaction. Why might inc temperature indefinitely be unfavorable for changing reaction conditions? A. The equilibrium constant has a definite limit that cannot be surpassed B. The products or reactants can decompose at high temps C. Inc temp would dec pressure, which may or may not alter reaction conditions D. If a reaction is irreversible, its Keq will resist changes in temp

B At extremely high temps, reactants or products may decompose, which will affect the equilibrium and potentially destroy the desired products. A implies that reactions have limits, which is true, however, this does not make increasing temp unfavorable C is false because inc temp would also inc pressure, assuming constant volume D. is false because it refers to properties of irreversible reactions, which would not be involved in an equilibrium between products and reactants

A man drives a go-kart around a circular track with a diameter of 50m, at a rate of 314 m/minute. What is the frequency at which he circles the track in Hertz? A. 0.0027 Hz B. 0.03 Hz C. 2 Hz D. 6.25 Hz

B is correct. Hertz (Hz) is a measure of frequency with units of cycles per second, usually written as 1/sec. The number of cycles around the circle can be found by dividing the velocity by the circumference. The circle circumference is (50 x pi) meters, or approximately 157 meters. The velocity of 314 m/min means that the driver completes 2 laps/min. The frequency in Hz is thus 2 laps/1 min x 1 min/60 sec = 1/30 Hz = 0.03 Hz. 0.0027 Hz, incorrect, This answer results from mistaking the formula for the area of a circle (which is pi times the square of the radius) with the formula for circumference (pi times diameter). 0.03 Hz, correct. 2 Hz, incorrect, This is laps/min rather than laps/second. 6.25 Hz, incorrect, This is the area divided by the velocity, with units of 1/min rather than 1/sec.

Given that R1=20 omega, R2=4 omega, R3=R4=32 omega, R5=15 omega, and R6=5 omega. What is the total resistance in the setup shown in FIGURE 7 PHYSICS? A. 0.15 omega B. 6.67 omega C. 16.7 omega D. 60 omega

B pg 229 physics and math The fastest way to tackle these kinds of questions is to simplify the circuit bit by bit. For example, notice that R3 and R4 are in parallel with each other and are in series with R2; similarly, R5 and R6 are in series. If we determine the total resistance in each branch, we will be left with three branches in parallel. To start, find the total resistance in the middle branch: 1/R(3+4)= (1/R3)+ (1/R4)=(1/32 omega) +(1/32 omega) --> 16 omega R(2+3+4)=R2+R(3+4)= 4omega + 16 omega = 20 omega next, take a look at the total resistance in the bottom branch: R(5+6)= R5+R6=15 omega + 5 omega = 20 omega The circuit can now be viewed as three resistors in parallel, each providing a resistance of 20 omega. The total resistance in the circuit is thus: 1/R(total)=(1/R2)+(1/R2+3+4)+(1/R5+6) = (1/20 omega) + (1/20 omega) + (1/20 omega) = 20/3 omega= 6.67 omega

The production of ATP via cellular respiration is governed by the following balanced reaction: C6H12O6 + 9 O2 > 6 CO2 + 6 H2O + ATP If there are 360 grams of C6H12O6 (MW = 180 g/mole) in the reaction vessel initially and 440 grams of CO2 are produced, what is the limiting reagent? A. C6H12O6 B. O2 C. CO2 D. H2O

B. This question is asking the examinee to determine the limiting reagent of a chemical reaction. To solve this problem, you must calculate the theoretical amount of CO2 that is produced given 360 grams of C6H12O6. If all 360 grams (2 moles) of glucose initially present react, then 528 g (12 moles) of CO2 should theoretically be produced. 360 g C6H12O6 x (1 mol C6H12O6 / 180 g C6H12O6) x (6 mol CO2 / 1 mol C6H12O6) x (44 g CO2 / 1 mol CO2) = 528 g CO2 Since only 440 grams of carbon dioxide are actually produced, not all of the glucose combusts. Thus, the oxygen must be the limiting reagent in this problem, making answer choice B correct. Choices C and D can be eliminated because products are never limiting reagents. Since there will be leftover C6H12O6, it is not the limiting reagent, eliminating answer choice A.

When dissolved in water, which of the following ions is most likely to form a complex ion with H20? A. Na+ B. Fe2+ C. Cl- D. S2-

B. Iron is a transition metal. Transition metals can often form more than one ion. The transition metals, in these various oxidation states, can often form hydration complexes with water. Part of the significance of these complexes is that, when a transition metal can form.a complex, its solubility within the related solvent will increase. The other ions given might dissolve readily in water, but bc none of them are transition metals, they will not likely form complexes

A 20 kg wagon is released from rest from the top of a 15 m long lane, which is angled at 30 degrees with the horizontal. Assuming that there is friction between the ramp and the wagon, how is this frictional force affected if the angle of the incline is increased? A. The frictional force increases B. The frictional force decreases C. The frictional force remains the same

B. The force of friction on an obj sliding down an incline equals the coefficient of friction times the normal force. The normal force is mgcostheta. As theta increase, cos decreases.

A moving negative charge placed in an external magnetic field circulates counterclockwise in the plane of the paper. In which direction is the magnetic field pointing? A. Into the page B. Out of the page C. Toward the center of the circle D. Tangent to the circle

B. This problem is an application of the right-hand rule. The velocity vector v is always tangent to the circle. The magnetic force must always point radially toward the center of the circle. Consider when the negative charge is at the "12 o'clock" position in its circle and apply the right-hand rule. Your thumb points to the left, tangent to the circle at this point. The back of your hand, which represents the force on a negative charge, points down the page, radially toward the center of the circle. Your fingers must point out of the page to get your hand into this position. Therefore, the direction of the magnetic field must be out of the page.

A 10 kg wagon rests on a frictionless inclined plane. The plane makes an angle of 30 degrees with the horizontal. Approx how large is the force required to keep the wagon from sliding down the plane? A. 10N B. 49N C. 85N D. 98N

B. mgcos theta=F friction

Balls A and B of equal mass are fully submerged in a swimming pool. A is much larger than B. Which ball will produce the greater buoyant force?

Ball A Fbuoy=mass fluid displaced (g) = rho fluid (V fluid displaced)g

Suppose a wire of length 2.0m is conducting a current of 5.0A toward the top of the page and through a 30 gauss uniform magnetic field directed into the page. What is the magnitude and direction of the magnetic force on the wire?

Because 1T=10^4 gauss, 1 gauss=E-4T, and 30 gauss =30E-4T=3E-3T. The wire is conducting a current toward the top of the page, and the magnetic field points into the page; therefore, the current is perpendicular to the magnetic field. The angle between them is 90 degrees. fsubB=ILBsintheta =(5A)(2m)(3E-3T)(sin90) =3E-2N To determine the direction, use the right-hand rule. Your thumb should point up the page in the direction of L. Your fingers should point into the page in the direction of B. Current is a flow of positive charge; thus, the force, FsubB is in the direction of your palm, which is to the left.

The H2O molecules has a dipole moment of 1.85D. Calculate the electric potential due to a water molecule at point 89 nm away along the axis of the dipole. (Note: k=8.99E9Nm^2/C^2 and 1D (debeye)=3.34E-30Cm).

Because the questions asks for the potential along the axis of the dipole, the angle is 0. V=(kqd/r^2)cos theta=(kp/r^2)cos theta =8.99E9(1.85)(3.34E-30)/89E-9 =7.01E-6V

A fission reaction occurs when uranium-235 (U-235) absorbs a low-energy neutron, briefly forming an excited state of U=236, which then splits into xenon-140, strontium-94, and more neutrons. In isotopic notion form, the reaction is: PHYSICS FIGURE 13 When balanced, how many neutrons are produced in the last reaction?

By treating each arrow as an equals sign, the problem is simply asking us to balance the last equation. The mass numbers (A) on either side of each arrow must be equal. This is an application of nucleon number conservation, which says that the total number of neutrons plus protons remains the same, even if neutrons are converted to protons and vice-versa, as they are in some decays. Because 235+1=236, the first arrow is balanced. Looking at the atomic numbers, the number of protons are balanced throughout (92+0=92=54+38+0). To find the number of neutrons, determine how many nucleons remain after accounting for xenon-140 and strontium-94: 236-(140+94)=236-234=2 nucleons Because the protons are balanced, these remining nucleons are both neutrons. Therefore, two neutrons are produced in this reaction. These neutrons are free to gone on and be absorbed by more U-235 and cause more fission reactions. Note that it was not necessary to know that the intermediate high-energy state 236/92U* was formed.

Both BF3 and NH3 have 3 atoms bonded to the central atom. Which of the following is the best explanation for why the geometry of these two molecules is different? A. BF3 has 3 bonded atoms and no Lon pairs, which makes its geometry trigonal pyramidal B. NH3 is non polar, while BF3 is polar C. NH3 has 3 bonded atoms and one lone pair, which makes its geometry trigonal pyramidal D. BF3 is non polar, NH3 is polar

C

Both CO3^-2 and ClF3 have 3 atoms bonded to a central atom. What is the best explanation for why CO3^-2 has trigonal planar electronic geometry while ClF3 has trigonal bipyramidal electronic geometry? A. CO3^-2 has multiple resonance structure, while ClF3 does not B. CO3^-2 has a charge of -2 while ClF3 has no charge C. ClF3 has lone pairs on its central atom, while CO3^-2 has none D. CO3^-2 has lone pairs on its central atom, while ClF3 has none

C

The radius of the aorta is about 1 cm and blood passes through it at a velocity of 30 cm/s. A typical capillary has a radius of about 4 x 10^-4cm with blood passing through at a velocity of 5 x 10^-2 cm/s. Using this data, what is the approximate number of capillaries in a human body? A. 1 x 10^4 B. 2 x 10^7 C. 4 x 10^9 D. 7 x 10^12

C

The speed of blood in the aorta is much higher than the speed of blood through a capillary bed. How can this be explained using the continuity equation, assuming that we are interested in average flow and that there is no net fluid loss? A. The aorta is located higher than the capillary bed. B. The pressure in the aorta is the same as the pressure in the capillary bed. C. The cross-sectional area of all the capillaries added together is much greater than the cross-sectional area of the aorta. D. The cross-sectional area of a capillary is much smaller than the cross-sectional area of the aorta.

C

What determines the length of an element's atomic radius? I. The # of valence electrons II. the # of electron shells III. the number of neutrons in the nucleus

C

Which of the following is the best explanation of the phenomenon of hydrogen bonding? A. Hydrogen has a strong affinity for holding onto valence electrons B. Hydrogen can only hold two valence electrons C. Electronegative atoms disproportionately carry shared electron pairs when bonded to hydrogen D. Hydrogen bonds have ionic character

C

Which of the following types of intermolecular forces provides the most accurate explanation for why noble gases can liquify? A. hydrogen bonding B. ion-dipole interactions C. dispersion forces D. dipole-dipole interactions

C

Which phase change has the same magnitude of energy to the heat of vaporization for water? A. sublimation B. freezing C. condensation D. deposition

C

Each of the acyl compounds listed below contain a six-membered ring EXCEPT: A.Delta-lactam B. Cyclohexane carboxylic acid C. Gamma-butyrolactone D. The anhydride formed from intramolecular ring closure of pentanediotic acid.

C A delta-lactam has a bond between the nitrogen and the fourth carbon away from the carbonyl carbon. This ring will have six elements: the nitrogen, the carbonyl carbon, and the four carbons in between. Cyclohexane carboxylic acid has a cyclohexane, a six-membered cycloalkane. The anhydride formed from pentaedioic acid will have the 5 carbons in the parent chain and one oxygen atom closing the ring, meaning there are still 6 elements. Gamma butyrolactone will have 5 elements because it contains a bond between the ester oxygen and the third carbon away from the carbonyl carbon. The five elements will be the oxygen, the carbonyl carbon, and the three carbons in between.

A positive charge of +Q is fixed at point R a distance d away from another positive charge of +2Q fixed at point S. Point A is located midway between the charged, and point B is a distance d/2 from +2Q, as shown in Figure 2 Physics. In which direction will a positive charge move if placed at point A and point B, respectively? A. Toward the +Q charge for both B. toward the +2Q charge for both C. Toward the +Q charge at point A, and toward the right at point B D. Toward the +2Q charge at point A and toward the right at point B

C A positive charge placed at A will experience two forces: a force to the left due to +2Q and a force to the right due to +Q. Because point A is the same distance from +Q and +2Q, the force due to +2Q will be larger than that due to +Q, and there will be a net force to the left (toward +Q). At point B, the forces from both +Q and +2Q will point to the right, so there will be a net force to the right.

Which of the following best describes the final product of an aldol condensation? A. 1,3-dicarbonyl B. 1,2-dicarbonyl C. alpha, Beta-unsaturated carbonyl D. Beta, gamma-unsaturated carbonyl

C Aldol condensations contain two main steps. In the first step, the alpha carbon of an aldehyde or ketone is deprotonated, generating the enolate carbanion. This carbanion can then attack another aldehyde or ketone, generating the aldol. In the second step, the aldol is dehydrated, forming a double bond. This double bond is between the alpha and Beta carbons, so the molecule is an alpha B unsaturated carbonyl.

Aldonic acids are compounds that A. can be oxidized, and therefore act as reducing agents B. can be reduced, and therefore act as reducing agents C. have been oxidized and have acted as reducing agents D. have been oxidized and have acted as oxidizing agents

C Aldonic acids form after the aldehyde group on a reducing sugar reduces another compound, becoming oxidized in the process

An electrolytic cell necessarily has: A. deltaS(not) > 0 B. deltaG(not) < 0 C. Keq < 1 D. E(not)cell > 0`

C An electrolytic cell is nonspontaneous. Therefore, deltaG must be postive and emf must be negative. The change in entropy may be positive or negative depending on the species involved, eliminating A.

Which of the following would best establish a causal link? A. A cross-sectional study using survey data for hand-washing and colds. B. A case-control study of an exposure during childhood and development of a certain disease later in life. C. A randomized clinical controlled trial of a new antipyretic drug. D. An IQ test where the results are later segregated by gender.

C An experiment will always establish a clearer causal link than an observational study. The other choices are all examples of observational data.

Which of the following pure compounds form ionic bonds? I. CaCl2 II. HCl III. NaF IV. Mg(OH)2 A. I only B. I and II only C. I, III, and IV D. I, II, III, and IV

C An ionic bond is formed between two atoms when one atom loses one or more electrons and another atom gains one or more electrons. This results in a positively charged cation and a negatively charged anion. The electrostatic attraction between the oppositely charged ions creates the bond, which can be measured by the finding the difference in electronegativity of the two atoms. Generally, a metal and nonmetal atom form ionic bonds. Metals prefer to give up electrons to form cations whereas nonmetals prefer their reduced anion form. In the case of CaCl2, NaF, and Mg(OH)2, each atom contains a metal that is bonded to a nonmetal. These all represent ionic bonds. While hydrochloric acid tends to dissociate in water to form H+ and Cl-, a pure solution cannot. Pure HCl contains two nonmetals, which forms a polar covalent bond.

The plots of two gases at STP are shown in FIGURE 5 CHEMISTRY. One of the gases is 1.0L helium and the other is 1.0L bromine. Which plot corresponds to each gas and why? A. Curve A is helium and curve B is bromine because helium has a smaller molar mass than bromine. B. Curve A is helium and curve B is bromine because the avg KE of bromine is greater than the avg KE of helium C. Curve A is bromine and curve B is helium because helium has a smaller molar mass than bromine D. Curve A is bromine and curve B is helium because the avg kinetic energy of bromine is greater than the avg KE of helium

C At STP, the difference between the distribution of speeds for helium and bromine gas is due to the difference in molar mass. Helium has a smaller molar mass than bromine. Particles with small masses travel faster than those with large masses, so the helium gas corresponds to curve B, which has a higher avg speed. Because the gases are at the same temp (273K), they have the same avg KE

A 70kg man has a tidal volume of 500mL, a residual volume of 1.2L, an inspiratory reserve volume of 3L, and an anatomic dead space of 150mL. At rest, what breath rate is needed to provide the lungs with 5L of fresh air per minute? A. 2 breaths per minute B. 10 breaths per minute C. 15 breaths per minute D. cannot be determined

C At rest, fresh air delivered to lungs is defined as the tidal volume minus the anatomic dead space. The tidal volume is volume of air taken in with each breath. The anatomic dead space is comprised of of anatomy along the respiratory path lacking alveoli for gas exchange (think mouth, trachea, bronchi). Inspiratory reserve volume is not used at rest, and residual volume does not affect fresh air delivery. The normal human respiratory rate at rest is 12-18 breaths per minute. 500mL-150mL=350mL fresh air per breath. 15 breaths per minute are required to deliver 5L of fresh air in a minute. A) 2 breaths per minute, incorrect, At rest inspiratory reserve volume is not used. B) 10 breaths per minute, incorrect, Dead space must be subtracted from tidal volume to calculate fresh air delivery. C) 15 breaths per minute, correct. D) cannot be determined, incorrect, All necessary measurements are provided in the question stem.

Simple distillation could be used to separate which of the following compounds? A. Toluene (BP 111C) and water (BP 100C) B. Naphthalene (BP 218C) and butyric acid (BP 163C) C. Propionaldehyde (BP 50C) and acetic acid (BP 119C) D. Benzene (BP 80C) and isopropyl alcohol (BP 83C)

C B would require vacuum distillation because the boilign points are over 150C In A and D, the BPs are within 25C of each other and would require fractional distillation to be separated

Oxygen O2 does not exhibit an IR spectrum because: A. It has no molecular motions B. It is not possible to record IR spectra of gaseous molecules C. Molecular vibrations do not result in a change in the dipole moment. D. Molecular oxygen contains four lone paris overall.

C Because molecular oxygen is homonuclear (composed of only one element) and diatomic, there is no net change in its dipole moment during vibration or rotaiton; in other words, the compound does not absorb in a measurable way in the infrared region. IR spectroscopy is based on the principle that, when the molecule vibrates or rotates, there is a change in dipole moment. A is incorrect because oxygen does have molecular motions; they are just not detectable in IR spectroscopy. B is incorrect because it is possible to record the IR of a gaseous molecule as long as it shows a change in its dipole moment when it vibrates. D is incorrect because lone pairs do not have an effect on the ability to generate an IR spectrum of a compound

In which of the following situations is it impossible to predict how the pressure will change for a gas sample? A. The gas is cooled at a constant volume B. The gas is heated at a constant volume C. The gas is heated, and the volume is simultaneously increased D. The gas is cooled and the volume is simultaneously increased

C Both a change in temp and a change in volume can affect a gas's pressure. So if one of those two variables is kept constant, as in A and B, we'll definitely be able to predict which way it will change. At a constant volume, heating the gas will inc the pressure and cooling the gas will decrease it. What about when both temp and volume are changing? If both changes have the same effect on pressure, then we can still predict which way it will change (as in D). Cooling the gas and increasing its volume both decrease pressure. C on the other hand presents too vague a scenario for us to predict definitively the change in pressure. Heating the gas would amplify the pressure, while increase the volume would decrease. Without knowing the magnitude, it's impossible to say whether the pressure would increase, decrease, or stay the same.

What is the structure of methyl propanoate? A. CH3COCH2CH3 B. CH3COOCH3 C. CH3CH2COOCH3 D. CH3CHOCH2CH3

C Break down the compound name: methyl indicates CH3, propane indicates a 3-carbon chain, and "oate" indicates an ester linkage. Only choice C) has the correct groups present. A. CH3COCH2CH3, incorrect, This is a ketone. B. CH3COOCH3, incorrect, This shows an ester but lacks a 3-carbon propane. C. CH3CH2COOCH3, correct. D. CH3CHOCH2CH3, incorrect, This is an ether.

Which of the following compounds would be most effective in extracting benzoic acid from a diethyl ether solution? A. Tetrahydrofuran B. Aqueous hydrocloric acid C. Aqueous sodium hydroxide D. Water

C By extracting with sodium hydroxide, benzoic acid will be converted to its sodium salt, sodium benzoate. Sodium benzoate,unlike its conjugate acid, will dissolve in an aqueous solution. The aqueous layer simply has to be acidified afterward to retrieve benzoic acid. A is incorrect because diethyl ether and tetrahydrofuran are both nonpolar and are miscible. Hydrochloric acid will not transform benzoic acid into a soluble salt, so B is incorrect. Finally, D is incorrect because protonated benzoic acid has limited solubility in water.

Middle-aged men require a base level of 900 Calories per day plus an additional 12 Calories per kilogram of body mass per day. Young adult women require a base level of 500 Calories per day, plus 15 Calories per kilogram of body mass per day. At what mass do middle-aged men and young adult women have the same caloric needs? A. 26kg B. 67kg C. 133kg D. 266kg

C C=900+12m C=500+15m 900+12m=500+15m 400=3m m=133kg

The entropy change when a solution forms can be expressed by the term deltaSsoln. When water molecules become ordered around an ion as it dissolves, the ordering would be expected to make a negative contribution to deltaSsoln. An ion that has more charge density will have a greater hydration effect, or ordering of water molecules. Based on this info, which of the following compound will have the most negative contribution to deltaSsoln? A. KCl B. LiF C. CaS D. NaCl

C CaS will cause the most negative contribution to deltaSsoln through hydration effects because the Ca2+ and S2- ions have the highest charge density compared to other ions. All of the other ions have charges of +1 or -1 whereas Ca2+ and S2- each have charges with a magnitude of 2.

Carboxylic acids can be reacted in one step to form all of the following compounds EXCEPT: A. esters B. amides C. alkenes D. alcohols

C Carboxylic acids cannot be converted into alkenes in one step. Esters, (A), are formed in nucleophilic acyl substitution reactions with alcohols. Amides, (B), are formed by nucleophilic acyl substitution reactions with ammonia. Alcohols, (D), may be formed using a variety of reducing agents. To form alkenes, carboxylic acids may be reduced to alcohols, which can then be transformed into alkenes by elimination in a second step.

A child pushes a 100 kg refrigerator with a force of 50 N, but the refrigerator does not move. Suppose the coefficient of static friction between the floor and the refrigerator is 0.4. What is the force due to friction in this scenario? A. 400N B. 350N C. 50N D. 950N

C Drawing a free-body diagram will indicate that the refrigerator does not have net forces and is in equilibrium. Maximum force of static fraction: F=usN = 400N However, static friction is only as large as necessary for the object to stay static and counteract the opposing force until the force due to static friction reaches its maximum amount. = 50N The force on the refrigerator opposing static force in this scenario is the child's push which amounts to only 50 N, much less than the 400 N threshold.

Intermediates in the Strecker synthesis include all of the following nitrogen-containing functional groups EXCEPT: A. Nitrile B. Imine C. Amide D. Amine

C During the Strecker synthesis, ammonia attacks a carbonyl, forming an imine. This imine is attacked by a cyanide, forming an amine, and a nitrile. Amide bonds are formed between amino acids, but do not appear during the Strecker synthesis

Which of the following best describes why overcharging a Ni-Cd battery is not detrimental? A. The energy density of a Ni-Cd battery is high, so it can store more charge than other batteries per its mass B. the electrodes of a Ni-Cd battery can discharge through the circuit when they are fully charged C. the Ni-Cd battery will stop accepting electrons from an outside source when its electrodes are recharged D. Ni-Cd batteries have a high surge current and can dissipate the overcharge before damage can occur to electrodes

C During the recharge cycle, i-Cd cells will accept current from an outside source until the Cd and NiO(OH) electrodes are pure; at this point, the reaction will stop because Cd(OH)2 runs out and no more electrons are accepted. A and B are both true statements, but they fail to explain why overcharging the battery (continuing to try to run current into the battery even when the electrodes are reverted to their original state) is not a problem with Ni-Cd batteries. Finally, surge current refers to the initial burst of current seen in some batteries; once charged, the surge current will not increase even if the power source continues to be run because no additional charge will be stored on the electrodes, eliminating D.

An assay is performed to determine the gold content in a supply of crushed ore. One method for pulling gold out of ore is to react it in a concentrated cyanide (CN-) solution. The equation is provided below: Au + NaCN + O2 + H2O --> Na[Au(CN)2] + NaOH An indicator is used during this reaction, and approximately 100mL of a 2M NaCN solution is used to reach the endpoint. How many moles of Au are present in the crushed ore? A. 0.01 mol B. 0.02 mol C. 0.10 mol D. 0.20 mol

C First, balance the chemical equation: 4Au + 8NaCN + O2 + 2H2O --> 4Na[Au(CN2)] + 4NaOH Now, determine the number of moles of NaCN used in the reaction: 0.1L x 2mol/L = 0.2mol NaCN If 0.2 mol NaCN are used in the reaction, then 0.2 mol NaCN x 4mol Au/8 mol NaCN = 0.1 mol Au is oxidized

Which of the following statements is true of a process that is spontaneous in the forward direction? A. deltaG > 0 and Keq > 0 B. deltaG > 0 and Keq > 0 C. deltaG > 0 and Keq > 0 D. deltaG > 0 and Keq > 0

C For a process to progress forward spontaneously, Q must be less than Keq and will therefore have a tendency to move in the direction toward equilibrium. A spontaneous reactions free energy is engative by convention

An opera singer has two precisely identical glasses. The singer produces as pure a tone as possible and shatters the first glass at a frequency of 808Hz. She then sings a frequency of 838Hz in the presence of the second glass. The second glass will likely: A. shatter after a longer amount of time because the applied frequency is higher B. shatter after a shorter amount of time because the applied frequency is higher C. not shatter because the applied frequency is not equal to the natural frequency of the glass D. not shatter because higher-frequency sounds are associated with more attenuation

C If these two glasses are perfectly identical, then the fact that the first glass shattered at 808Hz tells us that this is very close (if not identical) to the natural (resonant) frequency of the glass. If she produces a frequency that is not equal (or very close) to the natural frequency, then the applied frequency will not cause the glass to resonate, and there will not be the increase in wave amplitude associated with resonating objects. Attenuation will increase with increased frequency because there is more motion over which nonconservative forces can damp the sound wave; however, even if sound level was matched to that which shattered the first glass would still not shatter for the reasons described above, eliminating D.

What is the oxidation number of chlorine in NaClO? A. -1 B. 0 C. +1 D. +2

C In NaClO, sodium carries its typical +1 charge, and oxygen carries its typical -2 charge. This means that the chlorine atom must carry a +1 charge in order to balance the overall charge of zero

Rusting occurs due to the oxidation-reduction reaction of iron with environmental oxygen: 4Fe(s) + 3O2(g) --> 2Fe2O3(s) Some metals cannot react with oxygen in this fashion. Which of the following best explains why iron can? A. Iron has a more positive reduction potential than those metals, making it more likely to donate electrons to oxygen. B. Iron has a more positive reduction potential than those metals, making it more likely to accept electrons from oxygen C. Iron has a less positive reduction potential than those metals, making it more likely to donate electrons to oxygen D. Iron has a less positive reduction potential than those metals, making it more likely to accept electrons from oxygen

C In the oxidation-reduction reaction of a metal with oxygen, the metal will be oxidized (donate electrons) and oxygen will be reduced (accept electrons). This fact allows us to immediately eliminate B and D. A species with higher reduction potential is more likely to be reduced, and a species with lower reduction potential is more likely to be oxidized. Based on the info in the question, iron is oxidized more readily than those metals; this means that iron has a lower reduction potential.

Experimenters notice that the molar [ ] of dissolved oxygen in an enclosed water tank has decreased to one-half its original value. In an attempt to counter this decrease, they quadruple the partial pressure of oxygen in the container. What is the final [ ] of the gas? A. Half of the original [ ] B. The same as the original [ ] C. Double the original [ ] D. Quadruple the original [ ]

C Initially the [ ] of the gas is decrease to one-half its original value. Recall that the [ ] (solubility) and partial pressure are directly related. If the experiments then quadruple the partial pressure of oxygen in the vessel, the solubility is also increased by a factor of 4. One-half times four gives twice the original [ ] value.

Compared to IR and NMR spectroscopy, UV spectroscopy is preferred for detecting: A. Aldehydes and ketones B. Unconjugated alkenes C. Conjugated alkenes D. Aliphatic acids and amines.

C Most conjugated alkenes have an intense ultraviolet absorption. Akdehydes, ketones, acids, and amines all absorb in the UV range. However, other forms of spectroscopy (mainly IR and MNR) are more useful for precise ID. Isolated alkenes, B, can rarely be identified by UV spectroscopy

What is the approximate pH of a 1.2E-5M aqueous solution of NaOH? A. 4.92 B. 7.50 C. 9.08 D. 12.45

C NaOH is a strong base; as such, there will be 1.2E-5M OH- in solution. Based on this information along, the pOH must be between 4 and 5, and the pH must be between 9 and 10. Using the shortcut, pOH=5-0.12 = 4.88. pH=14-4.88=9.12 (actual 9.08)

Are there any outliers on the box plot? FIGURE 17 PHYSICS A. Yes; 1575 is an outlier B. Yes; 2600 is an outlier C. Yes; both 1575 and 2600 are outliers D. No.

C Outliers can be determined with respect to the interquartile range, Q3-Q1. The IQR for this box plot is 2280-2075, or 205. Values that are 1.5 x IQR below Q1 or above Q3 are considered outliers. 2075 - 1.5x205 = 2075-300-1775 (actual 1767.5). Therefore, 1575 is an outlier. 2280+ 1.5x205 is approx 2580 (actual 2587.5). Therefore 2600 is also an outlier.

Two fluids A and B have densities of x and 2x, respectively. They are tested independently to assess absolute pressure at varying depths. At what depths will the pressure below the surface of these two fluids be equal? a. whenever the depth of fluid A is one half that of fluid B b. Whenever the depth of fluid A equals that of fluid B c. Whenever the depth of fluid A is 2 times that of fluid B d. Whenever the depth of fluid A is 4 times that of fluid B

C Pnot +rhoA(g)(zA) = Pnot = phoB(g)(zB) (x)gzA =(2x)gzB zA=2zB

Potentiometry in an oxidation-reduction titration is analogous to performing an acid-base titration with A. acidic indicator B. basic indicator C. pH meter D. oxidizing agent

C Potentiometry refers to carrying out an oxidation-reduction titration with a voltmeter present to get precise readings of the reaction's electromotive force (EMF) to determine the endpoint. This is analagous to using a pH meter in an acid-base titration because it uses technology to get precise readings for plotting a titration curve. Indicators, as in A and B, can be used in both acid-base and redox titrations, but provide a qualitative (rather than quantitative) analysis of the titration. Oxidizing and reducing agents are used in redox titrations, not acid-base titrations.

The flow of blood through the aorta can be approximated with the volumetric flow rate equation. When the smooth muscle of the aorta contracts, the radius can decrease by as much as 10%. If the blood flow is to remain unchanged, what must happen to the velocity of the blood flow in the aorta? A. Dec by approx 10% B. Inc by approx 10% C. Inc by approx 25% D. Remains unchanged

C Q=a1v1=a2v2 (A= cross sectional area of the pipe, Q= flow rate, V= velocity of fluid) 1 x 1= 0.81 x v2 Because the radius was decreased to 90% of its original value, the cross sectional area will be reduced to 81% of its original value (A is proportional to radius squared and 0.92 = 0.81) r2=0.9r1 A2=0.81A1 v2=1.23, thus the velocity inc by 25%

Consider the following two reactions: 3A + 2B <--> 3C + 4D (Reaction 1) 4D + 3C <--> 3A + 2B (Reaction 2) If Keq for reaction 1 is equal to 0.1, what is Keq for reaction 2? A. 0.1 B. 1 C. 10 D. 100

C Reaction 2 is the reverse of reaction 1. This means that Keq for reactio n2 is the inverse of Keq of reaction 1, so the answer is 1/0.1=10

How does the number of significant digits differ between 14,320,010 and 3.618000? A. 14,320,010 has more significant digits than 3.618000 B. 14,320,010 has fewer significant digits than 3.618000 C. 14,320,010 has the same number of significant digits than 3.618000 D. A comparison cannot be made because the numbers are not both in scientific notation.

C Significant digits include all nonzero digits, all zeroes that are between nonzero digits, and trailing zeroes in any number with a decimal point. In 14,320,010 there is no decimal point; thus, the last zero is insignificant and there are 7 significant digits. In 3.618000 all of the digits are significant thus there are also 7 significant digits.

In the compound KH2PO4, which element has the highest oxidation number? A. K B. H C. P D. O

C Start with the atoms that have oxidation states of which you are certain. Potassium is a Group IA metal, and therefore has an oxidation state of +1. Hydrogen is almost always +1, unless it's paired with a less electronegative element. Oxygen is generally -2. Because there are 4 oxygens, they create a total negative charge of -8 which is partially balanced by two hydrogens (+2) and potassium (+1). Therefore, phosphorus has a +5 charge, making it the highest oxidation state.

A biochemist is synthesizing valine, shown in figure 15 organic chemistry using the Strecker synthesis. Which of the following carbonyl-containing compounds would be an appropriate starting reactant in this synthesis? A. 2-Propanone B. Propanal C. 2-Methylpropanal D>. Butanal

C The Strecker synthesis creates an amino acid from an aldehyde. The carbonyl carbon ultimately becomes the alpha carbon of the amino acid. Any remaining alkyl chain becomes the R group. The starting compound is therefore 2-methylpropanal (isobutyraldehyde). See pg 254 organic chemistry for pic

Objects A and B are submerged at a depth of 1m in a liquid with a specific gravity of 0.877. Given that the density of Object B is 1/3 that of Object A and that the gauge pressure of Object A is 3atm, what is the gauge pressure of Object B? A. 1 atm B. 2 atm C. 3 atm D. 9 atm

C The absolute and gauge pressures depend only on the density of the fluid, not that of the object. When the pressure at the surface is equal to the atmospheric pressure, the gauge pressure is giveb by Pgauge=rho (gz), where rho is the density of the fluid, not the object. These objects are also at the same depth, so they must have the same gauge pressure.

A researcher designing a study has paid for it to be professionally translated into several languages. She discusses the potential risks and benefits with each participant and allows them to bring documentation home for review before committing to the study. This researcher has put special focus on: A. justice by explaining potential risks B. beneficence by describing the potential benefits of the study C. respect for persons by acknowledging the subject's perspective and rights D. selection bias, by making the recruitment documents inclusive

C The behaviors described in the question stem (informing the patient, providing time to make a decision) are consistent with informed consent and autonomy, which are both part of respect for persons.

Given that the gases at the center of the sun have an avg molar mass of 2g/mol, compressed to a density of 1.2g/cm^3 under 1.30E9 atm of pressure, what is the temp at the center of the sun? A. 2.6E4 K B. 2.6E6 K C. 2.6E7 K D.2.6E10 K

C The ideal gas law can be modified to include density. PV=nRT=(m/M)(RT) T=PM/rhoR T=(1.3E9atm)(2g/mol)/(1.2g/cm^3)(1000cm^3/L)(0.0821)

A proton and an alpha particle (a helium nucleus) repel each other with a force of F while they are 20nm apart. If each particle combines with three electrons, what is the magnitude of the new force between them? A. 9F B. 3F C. F D. F/9

C The magnitude of the electrostatic force is given by the equation Fe=kq1q2/2^2. Because the distance does not change during the interaction in the question, the value of r is irrelevant to the answer. Currently, q1 and q2 are equal to +1e and +2e, respectively; the addition of three electrons )each of which carries a charge of -e) will change the charges to -2e and -1e. Therefore, the product q1q2 before the interaction is equal to the product q1q2 after the interaction (+2e^2). Because k and r also remain constant in this system, the magnitude of the force does not change.

An experimenter is attempting to investigate the effect of a new antibiotic on E. coli. He plates cells and administers one milliliter of the antibiotic. Which of the following is an appropriate negative control in this experiment? A. A plate with no cells that was coated with one milliliter of antibiotic B. A plate with E. coli and no additional treatment C. A plate with E. coli and one milliliter of isotonic saline. D. A plate of epithelial cells treated with one milliliter of antibiotic.

C The purpose of a control is to keep the conditions of two experiments as close as possible to establish causality. In this case, the one milliliter volume addition might have impacted the growth of E. coli; thus we must control for this by administering an equal volume of a theoretically inert compound to a plate of E. coli

Which of the following correctly identifies the following process? FIGURE 15 PHYSICS A. B- decay B. B+ decay C. e- capture D. gamma decay

C This process can be described as electron capture. Certain unstable radionuclides are capable of capturing an inner electron that combines with a proton to form a neutron. The atomic number becomes one less than the original, but the mass number remains the same. Electron capture is a relatively rare process and can be thought of as the revers of B- decay. Notice that the equation is similar to that of B+ decay but not identical because a particle is absorbed, not emitted.

Which of the following best accounts for the negative slope of the liquid-solid equilibrium line in the phase diagram for water? A. H2O(s) has a greater density than H2O(l), which causes the solid to form liquid under high pressure conditions. B. H2O(s) has a greater density than H2O(l), which results from the hydrogen bonds formed between water molecules. C. H2O(s) has a lower density than H2O(l) which results from the crystalline framework that forms due to hydrogen bonds. D. H2O(s) has a lower density than H2O(l) which causes the solid to form liquid under low pressure conditions.

C This question asks the examinee to identify the correct explanation for the negatively sloped phase diagram for water. During the crystallization of water, each molecule forms four hydrogen bonds with adjacent water molecules resulting in three-dimensional hexagonal lattice structure. Thus, the density of solid water is less than the density of liquid water eliminating choices A and B. The negatively sloped liquid-solid equilibrium line in the phase diagram means that an increase in pressure at a constant temperature can cause water to change phases from solid to liquid. The increase in pressure causes water to convert to the more compact liquid phase in order to relieve the increased pressure.

Which of the following statements accurately orders the speed of sound through the following mediums: air (ρair = 1.275 kg/m3), water (ρwater = 1000 kg/m3), and copper (ρcopper = 8960 kg/m3)? A. Air > water > copper B. Air > copper > water C. Copper > water > air D. Copper > air > water

C This question asks the examinee to identify the correct order for the velocity of sound waves through various media. The velocity of sound is given by the equation, v = (B/ρ)1/2, where B is the bulk modulus and ρ is the density of the material. The densities of solids are the greatest because of the strong intermolecular forces, which create lattice structures with closely bonded molecules. While solids tend to be densest, they also tend to be the least compressible, which corresponds to a high bulk's modulus. As a result, the B/ρ quotient is the highest for solids, followed by liquids, and then gases. Therefore, the velocity of sound through copper is greater than water, which is greater than air. Thus, C is the correct answer.

Which of the following best characterizes ideal voltmeters and ammeters? A. ideal voltmeters and ammeters have infinite resistance B. ideal voltmeters and ammeters have no resistance C. ideal voltmeters have infinite resistance, and ideal ammeters have no resistance D. ideal voltmeters have no resistance, and ideal ammeters have infinite resistance

C while this is primarily a recall question, it should also be intuitive. Voltmeters are attempting to determine a change in potential from one point to another. To do this, they should not provide an alternate route for charge flow and should therefore have infinite resistance. Ammeters attempt to determine the flow of charge at a single point and should not contribute to the resistance of a series circuit; therefore, they should have no resistance.

If 1-Hexanol was treated with Pyridinium Chlorochromate (PCC) what would be the end result? 2-Hexanol 2-Hexanone Hexanal Hexanoic acid

C (hexanal) pyridinium chlorochromate is a weak oxidizing agent, and will oxidize an alcohol to an aldehyde. Stronger oxidizing agents are required to convert a primary alcohol to a carboxylic acid

Which of the following processes is LEAST likely to be accompanied by a change in temp? A. the kinetic energy of a gas is increased though a chemical reaction B. energy is transferred to a solid via electromagnetic waves C. a boiling liquid is heated on a hot plate D. a warm gas is mixed with a cold gas

C (physics ch 3)

Nitrogen primarily exists in the atmosphere as a diatomic gas. Which of the following is true about this form of nitrogen? A. The presence of a lone pair of electrons on each nitrogen atom in the molecule allows it to act as a strong Lewis base. B. The triple bond of electrons creates a region of high electron density that allows N2 to be very reactive as a nucleophile. C. Diatomic nitrogen gas is relatively inert and can be used as the atmosphere in laboratory conditions to prevent unwanted side reactions. D. Atmospheric nitrogen reacts spontaneously with carbon dioxide, which keeps atmospheric CO2 levels at a relatively low 0.04% (on a molar basis) of the atmosphere.

C is correct. The MCAT will expect you to be familiar with N2 as a very inert gas. It makes up approximately 80% of the air you breathe, yet has no significant chemical reactions with your lungs - or with anything other than nitrogen-fixing plants. This information implies that nitrogen is very inert (unreactive). As such, it would serve as a good artificial atmosphere when working with reagents that might react with oxygen or other gases. The Lewis structure of N2 is shown below. A, B: Although N2 does have relatively high electron density, the strength and stability of the triple bond means it doesn't react strongly as many other compounds. D: Atmospheric CO2 levels are primarily determined by the seasons (when the northern hemisphere enters spring and summer, many plants grow leaves and pull CO2 out of the atmosphere) and by humans burning fossil fuels, which puts CO2 into the atmosphere. Nitrogen and carbon dioxide do not react readily with each other. Content Foundations: Gases A gas is a phase of matter in which the material does not have a fixed shape or volume of its own, but instead adopts the form and size of its container. Once inside a container, gas molecules are distributed uniformly with no spatial organization, and they undergo continuous, random motion. Both liquids and gases are considered fluids because of their tendency to flow. A gas is a low-density fluid because the molecules are much farther apart than in a liquid, where the molecules are in close contact with each other. There are five basic, measurable properties of a gas: pressure (P), temperature (T), volume (V), mass (m), and number of moles (n). Pressure is defined as force divided by area, and is commonly given in pascals (Pa), atmospheres (atm), mmHg, or torr, where 1 atm = 760 mmHg = 760 Torr = 105 Pa (N/m2). Unless otherwise indicated, we can assume that gases are under 1 atm of pressure, which is the magnitude of the pressure exerted by Earth's atmosphere at sea level. Temperature is an approximation of the average kinetic energy of the molecules in a gaseous sample (T ~ KEavg = ½mv2). On the exam, T may be given in degrees Celsius or Kelvin, which are related by the equation TK = TC + 273. For the sake of the MCAT, absolute zero (0 K) is the lowest temperature at which any substance can exist, and at this temperature, the molecules in a gaseous sample will have no energy and will be motionless. Volume is the size of the space a gas occupies. Volume can be related to pressure using Boyle's law, which states that for an ideal gas at a constant temperature, PV = constant. Thus, we can see there is an inverse relationship between P and V. Volume and temperature are related in Charles's law: T/V = constant if P is constant, indicating a direct relationship between T and V. Volume can be expressed as any unit of distance cubed or as any standard unit of volume, but the most common units are mL, L, or cm3, where 1 L = 103 mL = 103 cm3. The number of gas particles is measured through moles. One mole of any substance is defined as 6 × 1023 atoms/molecules of that substance. Each substance will have an atomic or molecular weight defined as grams per mole; this variability means that moles (n) are more convenient than mass as a way of expressing the quantity of a gas present. The ideal gas law summarizes relates all the above properties: PV = nRT, where R is the universal gas constant (do not memorize its value) and T is temperature in Kelvin. Unless told otherwise, on the MCAT we can assume that we are dealing with ideal gases. Gases are more likely to behave non-ideally under extremely high pressures or at very low temperatures.

The energy stored in a fully charged capacitor is given by U=1/2CV^2. In a typical cardiac defibrillator, a capacitor charged to 7500V has a stored energy of 400J. Based on this information, what is the charge on the capacitor in this cardiac defibrillator? A. 1.1E-5C B. 5E-2C C. 1.1E-1C D. 3.1E6C

C pg 230 physics and math Because the question is asking us to calculate the charge on the capacitor, using the formula Q=CV. We are given V=75000V and can calculate C from the formula for energy, U=1/2CV^2: U=1/2CV^2=1/2(Q/V)V^2=QV/2 Q=2U/V=2(400J)/7500V= 800/8000=0.1C

In the circuit in PHYSICS FIGURE 8, what is the voltage drop across the 2/3 omega resistor? A. 1/2V B. 2/3V C. 5V D. 7.5V

C pg 230 physics and math to determine the voltage drop across the 2/3omega resistor, start by calculating the total resistance in the circuit. For the resistors in parallel, the equivalent resistance is: 1/Rp=(1/2omega)+(3/2omega) --> Rp=1/2omega The total resistance in the circuit is the sum of the remaining resistor and the equivalent resistance of the other two: Rs=1/2omega + 1/2omega = 1 omega Now that we know the equivalent resistance, we can calaculate the total current using Ohm's law: I=V/R=10V/1omega=10A Finally, we can determine the voltage drop across the parallel resistors. The voltage drop across the 1/2omega resistor must be (10A)(1/2omega)=5V. Therefore, there must be a 5V drop across both the 2/3 omega resistor and 2omega resistor, according to Kirchhoff's loop rule. Each of these resistors forms a complete loop in combination with the 1/2omega resistor and 10V voltage source, and the net potential difference around any closed loop must be 0V.

A 10 omega resistor carries a current that varies as a function of time as shown in FIGURE 9 PHYSICS. How much energy has been dissipated by the resistor after 5s? A. 40J B. 50J C. 120J D. 160J

C pg 230 physics and math. Power is energy dissipated per unit time; therefore, the energy dissipated is E=Pdeltat. In the five second interval during which the resistor is active, it has a 2A current for three of those seconds. The power dissipated by a resistor R carrying a current I is P=I^2R. Therefore, the energy dissipated is: E=I^2Rdeltat=(2A)^2(10 omega)(3s)= 4 x 10 x 3= 120J

If an object with an initial temperature of 300 K increases its temperature by 1 degree Celsius every minute, by how many degrees Fahrenheit will its temp have increased in 10 minutes? A. 6F B. 10F C. 18F D. 30C

C.

If the GPE of an object has doubled in the absence of nonconservative forces, which of the following must be true, assuming the total mechanical energy of the object is constant? A. The object has been lifted to twice its initial height B. The kinetic energy of the object has been halved C. The kinetic energy has decreased by the same quantity as the potential energy has increased D. The mass of the object has doubled

C. In the absence of nonconservative forces, all changes in PE must be met by an equal change in KE. Both A and D could be true statements but do not necessarily have to be

In 2-butanol was treated with dichromate, what reaction would occur? A. Reduction, resulting in the formation of butene B. Oxidation, resulting in the formation of butanoic acid C. Oxidation, resulting in the formation of butanone D. no reaction would occur

C. Because 2-butanol is a secondary alcohol, oxidation by a strong oxidizing agent like dichromate will result in a ketone, butanone

Centrifugal force is an apparent outward force during circular motion. It has been described as a reaction force to Newton's third law. Which of the following statements is most likely to be correct regarding centrifugal force? A. Centrifugal force exists only for uniform circular motion, not nonuniform circular motion. B. Centrifugal force exists only when tension or a normal force provides centripetal acceleration C. Centrifugal force always acts antiparallel to the centripetal force vector. D. Centrifugal force is a result of repulsive electrostatic interactions.

C. Because the question stem indicates that centrifugal force is reactionary and gives its direction, we can draw the conclusion that it is a reaction to centripetal force. According to Newton's third law, these forces must have an equal magnitude and opposite directions (antiparallel).

Two parallel conducting plates are separated by a distance, d. One plate carries a charge +Q and the other carries a charge -Q. The voltage between the plates is 12V. If a +2uC (u=mu) charge is released from rest at the positive plate, how much kinetic energy does it have when it reaches the negative plate? A. 2.4E-6J B. 4.8E-6J C. 2.4E-5J D. 4.8E-5J

C. Recall that the change in potential energy, deltaU, and the change in potential, deltaV, are releated by E=delta U=q delta V. Therefore, delta U=(2E-6C)(-12V)=-2.4E-5J. The positive charge is moving from the positive to the negative plate , and is therefore decreasing in potential energy; this is reflected by the fact that the voltage is -12v rather than +12V. The potential energy that is lost is converted into kinetic energy, so the charge must gain 2.4E-5J of kinetic energy.

The negative charge in the figure below (-1,uC) u=mu moves from y=-5 to y=+5 and is made to follow the dashed line. What is the work required to move the negative charge along this dashed line? A. -10J B. -5J C. 0J D. 10J

C. There will be work done in moving the negative charge from its initial potition to y=0. However, in moving the negative charge from y=0 to the final position, the same amount of work is done but with the opposite sign, This is because the force changes direction as the electron crosses y=0. Therefore, the two quantities of work cancel each other out. This argument depends crucially on the symmetry of the initial and final positions.

A certain 9V battery is used as a power source to move a 2C charge. How much work is done by the batter? A. 4.5J B. 9J C. 18J D. 36J

C. Voltage (deltaV) is equal to the quotient of the amount of work done (W) divided by the charge of the particle on which the work is done (q), according to the equaltion deltaV=[deltaU/q]=W/q Because the voltage equals 9V and the charge equals 2C, the work done must equal 9Vx 2C=18J

A rock (m=2kg) is shot up vertically at the same time that a ball (m=0.5kg) is projected horizontally. If both start from the same height A. the rock and the ball will reach the ground at the same time B. the rock will reach the ground first C. the ball will reach the ground first D. the rock and ball will collide in the air before reaching the ground

C. We only need to analyze the motion in the vertical dimension to answer this question. If both the rock and ball began with no vertical velocity, they would reach the ground at the same time. However, because the rock begins with an upward component of velocity, it will take time to reach a max height before falling back towards the ground. Functionally, the rock's free fall thus starts higher and later than the ball's. the rock will hit the ground after the bll.

The following data shown in the table were collected for the combustion of the theoretical compound XH4: XH4+2O2-->XO2+2H2O Trial. [XH4] [O2]. [Rate] 1. 0.6 0.6 12.4 2 0.6 2.4 49.9 3 1.2 2.4 198.3 A. rate=k[XH4][O2] B. rate=k[XH4][O2]^2 C. rate=k[XH4]^2 [O2] D. rate=k[XH4]^2 [O2]^2

C. math given on pg 182 of Gen Chemistry

Rank the following in order of decreasing electrophilicity: CR3+, CH3OH, CH3OCH3, CH3Cl

CR3+ > CH3Cl > CH3OH > CH3OCH3 Good electrophiles are pos charged or polarized. CH3+ is a tertiary carbocation and it has a pos charge so it's super electrophilic. CH3Cl and CH3OH are both polarized, but Cl- is a weaker base than OH-. As such, Cl- will be more stable in solution than OH-, which inc the electrophilic reactivity of CH3Cl above CH3OH. CH3OCH3 has a much less stable LG, CH3O-, and is therefore significantly less electrophilic

REFER TO FIGURE 8 GENERAL CHEMISTRY 1. What is the approximate value of pKa1? A. 1.9 B. 2.9 C. 3.8 D. 4.1 2. At what pH is the second equivalence point? A. pH=3.0 B. pH=4.1 C. pH=5.9 D. pH=7.2 3. What is the approximate value of pKa2? A. 3.6 B. 4.1 C. 5.5 D. 7.2

Can use pg 382 general chemistry if graph I drew is wrong. 1. A The first pKa in this curve can be estimated by eye. It is located halfway between the starting point (when no base had yet been added) and the first equivalence point (the first steep portion of the graph, around 15mL). This point is approx 7-8mL on the x-axis, which corresponds to a pH of approx 1.9. Notice that this region experiences very little change in pH, which is the defining characteristic of a buffer region. 2. C The second equivalence point is the midpoint of the second steep increase in slope. 3. B The value of the second pKa is found at the midpoint between the first and second equivalence points. In this curve, that corresponds to pH=4.1. Just like the first pKa, it is in the center of a flat buffering region.

What are catalysts? How do they work? What affect do they have on deltaG and Keq? What's the difference between homogenous catalysis and heterogenous catalysis?

Chemical agents that selectively speed up chemical reactions without being consumed by the reaction. Catalysts interact with the reactants, either by adsorption or through the formation of intermediates, and stabilize them so as to reduce the activation energy necessary for the reaction to proceed. While many catalysts chemically interact with the reactants they return to their OG chemical state upon forming the products. They may inc the frequency of collisions between the reactants, change the relative orientation of the reactants (making a higher % of the collisions effective), donate electron density to the reactants, or reduce intramolecular bonding within reactant molecules. -The presence of the catalyst has no impact on the free energies of the reactants or the products of the difference between them. It only can change the rate of reactions, and change the forward rate and reverse rate by the same factor. Consequently, they have no impact on the equilibrium position or Keq. In homogenous catalysis, the catalyst is in the same phase as the reactants. In heterogenous catalysis, the catalyst is in a distinct phase.

Explain cohesion and adhesion. Explain this in terms of meniscus vs a convex meniscus.

Cohesion is the attractive force that a molecule of liquid feels toward other molecules of the same liquid. Think: cohort Ex: when oil and water mix, cohesion happens between oil molecules and other oil molecules, as well as between water and other water molecules Adhesion is the attractive force that a molecule of the liquid feels toward the molecules of some other substance. Ex: adhesive forces cause water molecules to form droplets on the windshield of a car even though gravity is pulling them downward. When liquids are placed in containers, a meniscus, or curved surface in which the liquid "crawls" up the side of the container a small amount, will form when the adhesive forces are greater than the cohesive forces. A backward (convex) meniscus (with the liquid level higher in the middle than at the edges) occurs when the cohesive forces are greater than the adhesive forces.

What are structural (constitutional) isomers?

Compounds with the same molecular formula but different structural formula`

Despite the fact that both C2H2 and HCN contain triple bonds, the lengths of these triple bonds are not equal. Which of the following is the best explanation for this finding? A. In C2H2, the bond is shorter because it is between atoms of the same element. B. The two molecules have different resonance structures C. Caron is more electronegative than hydrogen D. Nitrogen is more electronegative than carbon

D

A woman stands 1m in front of a plane mirror. What is the magnification and distance from the woman to her mirror-image? A. 1/2 and 2m B. 1 and 1m C. 2 and 1m D. 1 and 2m

D A flat plane mirror has a radius of curvature that is infinite (it doesn't curve!) and thus a focal point distance that is also infinite. Thus the equation 1/f = 1/object + 1/image can be simplified to 1/object = -1/image So a flat plane mirror will always have an image distance that is equal to the object distance. The negative sign tells us that the image is virtual. Using the equation m = -i/o we can also see that a flat plane mirror will always produce an object with a magnification of 1. That is, when you look at yourself in the bathroom mirror, your image is not magnified nor reduced. If a person is standing 1 m in front of their mirror, then the image will be 1 m away from the mirror as well ("inside" the mirror). So the total distance between the person and the image will be 2 m. 1/2 and 2m, incorrect, Uses image distance of 2m rather than 1m and inverts the magnification equation. 1 and 1m, incorrect, Does not sum object and image distance for distance between woman and image. 2 and 1m, incorrect, Uses image distance of 2m in magnification equation and distance from woman to image also incorrect. 1 and 2m, correct.

Which of the following is the correct net ionic reaction for the reaction of copper with silver(I) nitrate? A. Cu + AgNO3 --> Cu(NO3)2 + Ag B. Cu + 2Ag+ + 2NO3- --> Cu2+ + 2NO3- + 2Ag C. 2Ag+ + 2NO3- --> 2NO3- + 2 Ag+ D. Cu + 2Ag+ --> Cu2+ + 2Ag

D A net ionic equation represents each of the aqueous ions comprising the reactants and products as individual ions, instead of combining them as formula units. Thus, A is not a net ionic reaction. The term net means that the correct answer does not include any spectator ions (ions that do not participate in the reaction. In this reaction, nitrate (NO3-) remains unchanged. Therefore B and C are eliminated.

Which of the following choices is indicative of a spontaneous reaction, assuming standard conditions? A. E(not)cell is negative B. Q=Keq C. The cell is a concentration cell D. Keq > 1

D A spontaneous electrochemical reaction has a negative deltaG. Using the equation deltaG(not)=-RTlnKeq, Keq>1 would result in lnKeq>0, which means deltaG(not)<0. A negative emf, A, or equilibrium state, B, would not correspond to a spontaneous reaction. [ ] cells can be spontaneous; however, if the [ ] cell had reached equilibrium, it would cease to be a spontaneous reaction, eliminating C.

The voltage of a galvanic cell, E, depends on the concentration of reactants and products. The Nernst equation, E = Eo - (0.0592/n)log(Q) can be used to calculate the cell potential. Eo is the standard cell potential, n is the number of moles of electrons exchanged in the reaction, and Q is the reaction quotient. Which of the following is true regarding the value of E? A. E is always less than Eo B. E is always greater than Eo C. E is always equal to Eo D. E can be less than, greater than, or equal to Eo

D According to the Nernst equation, the cell potential E will vary based on the logarithm of the reactant quotient Q. The reactant quotient follows the same equation as the equilibrium constant K, but measures the ratio of the actual concentration of products to concentration of reactants at any time during the reaction. Therefore, when Q > 1, log(Q) is positive and E < Eo. If Q < 1, log(Q) is negative, then E > Eo. Lastly, E is equal to Eo when Q = 1 because the log of one is equal to zero. Thus, E can be less than, greater than, or equal to Eo

Which of the following is NOT a characteristic of an amphoteric species? A. Amphoteric species can act as an acid or a base, depending on its environment B. Amphoteric species can act as an oxidizing or reducing agent, depending on its environment C. Amphoteric species are sometimes amphiprotic D. Amphoteric species are always nonpolar

D An amphoteric species is one that can act either as an acid or a base, depending on its environment. Proton transfers are classic oxidation-reduction reactions, so A and B are true. C is true because may amphoteric species, such as water and bicarbonate, can either donate or accept a proton. D is false, and thus the correct answer because amphoteric species can be either polar or nonpolar in nature.

An electron is known to be in the n=4 shell and the l=2 subshell. How many possible combinations of quantum numbers could this electron have? A. 1 B. 2 C. 5 D. 10

D An electron in the n=4 shell and the l=2 subs jell can have 5 different values for msubl: -2, -1, 0, 1, 2. In each of these orbitals, electrons can have a pos or neg spin.

The reaction of ammonia with caprylic acid, found in coconuts would produce; A. ester B. anhydride C. alcohol D. water molecule

D Based on its name, caprylic acid must be a carboxylic acid. The reaction between a carboxylic acid and ammonia (NH3) would produce an amide-which is not one of the options listed. Instead, we should take a look at the type of reaction occurring. The production of an amide from a carboxylic acid and ammonia occurs through a condensation reaction in which a molecule of water is removed as a leaving group.

When benzaldehyde is reacted with acetone, which will act as the nucleophile? A. Benzaldehdye, after addition of strong acid B. Benzaldehyde after reaction with strong base C. Acetone, after addition of strong acid D. Acetone, after reaction with strong base

D Because benzaldhyde lacks an alpha proton, it cannot be reacted with base to form the nucleophilic enolate carbanion. Therefore, acetone will act as our nucleophile, and both A and B can be eliminated. In order to perform this reaction, which is an aldol condensation, acetone will be reacted with a strong base-bot a strong acid- in order to extract the alpha hydrogen and form the enolate anion, which will act as a nucleophile.

A cross-sectional study in which current smoking status and cancer history are assessed simultaneously cannot satisfy which of Hill's criteria? A. Stength B. Coherence C. Plausibility D. Temporality

D Because both the exposure and outcome are measured at the same time, we cannot make any conclusions about the temporality. The cancer patient may have only begun smoking after diagnosis, and this type of study doesn't examine that possibility.

Which of the following methods of amino acid synthesis results in an optically active solution? A. The Strecker synthesis only B. The Gabriel synthesis only C. Both D. Neither

D Both the Strecker and Gabriel syntheses contain planar intermediates, which can be attacked from either side by a nucleophile. This results in a racemic mixture of enantiomers, and the solution will therefore be optically inactive.

Which of the following characterizes butane? A. high solubility in water B. strong dipole moment C. high molecular weight D. low solubility in water

D Butane is a four carbon alkane. As such, it is non-polar and low in molecular weight. Solubility in water is primarily governed by polarity and ability to hydrogen-bond, both of which butane lacks. Choice d is the best choice, low solubility in water. A. high solubility in water, incorrect, Nonpolar compounds do not dissolve well in water. B. strong dipole moment, incorrect, Alkanes lack dipole moments. C. high molecular weight, incorrect, A four carbon alkane has low molecular weight. High molecular weight on the MCAT is often used to describe long-chain polymers and very large molecules, and butane does not qualify for either designation. D.low solubility in water, correct.

Which of the following processes has the most exothermic standard heat of combustion? A. Combustion of ethane B. Combustion of propane C. Combustion of n-butane D. Combustion of n-pentane

D Combustion often involves the reaction of a hydrocarbon with oxygen to produce carbon dioxide and water. Longer hydrocarbon chains yield greater amounts of combustion products and release more heat In the process--that is, the reaction is more exothermic. Of the hydrocarbons listed here, n-pentane is the longest chain.

Which of the following is the best explanation for the fact that a larger mass of electrodes are required for lead-acid batteries, as compared to other batteries, to produce a certain current? A. The lead-acid electrolyte, sulfuric acid, is diprotic and incompletely dissociates in solution B. The energy density of lead-acid electrodes is higher than that of other batteries C. The electrolytes in other batteries less readily dissociate than those of lead-acid batteries D. the energy density of lead-acid electrodes is lower than that of other batteries

D Compared to other cell types, lead-acid batteries have a characteristically low energy-density. While A is true, the incomplete dissociation of sulfuric acid does not fully explain the low energy density of lead-acid batteries. C is likely to be an opposite; the more easily the electrodes dissociate, the easier it is to carry out oxidation-reduction reaction with them.

A leak of helium gas through a small hole occurs at a rate of 3.22E-5 mol/s. How will the leakage rates of neon and oxygen gases compare to helium at the same temperature and pressure? A. Neon will leak faster than helium; oxygen will leak faster than helium B. Neon will leak faster than helium; oxygen will leak slower than helium C. Neon will leak slower than helium; oxygen will leak faster than helium D. Neon will leak slower than helium; oxygen will leak slower than helium

D Graham's law of effusion states that the relative rates of effusion of two gases at the same temp and pressure are given by the inverse ratio of the square roots o the masses of the gas particles. In other words, a gas with a higher molar mass will leak more slowly than a gas with a lower molar mass. Both neon and oxygen gases will leak at slower rates than helium b/c they both have more mass than helium

A new medication for heart failure is being developed and has had a statistically significant effect on contractility in clinical trials. Which of the following would NOT likely cause the drug to be held back from common use? A. The value of a used was 0.5 B. Similar compounds display toxicity C. The effect size is clinically insignificant D. The study had low power to detect a difference

D If a study has low power, it is more difficult to get results that are statistically significant. Therefore, if the results are still statistically significant even with low power, then there is likely a large effect size that makes the effect clinically significant. If the value of alpha used in the study was 0.5, then statistically significant results do not mean much- traditionally, alpha = 0.05 or a smaller probability is used, eliminating A.

Which of the following would be the best procedure for extracting acetaldehyde from an aqueous solution? A. A single extraction with 100mL of ether B. Two successive extractions with 50mL portions of ether C. Three successive extractions with 33.3mL portions of ether D. Four successive extractions with 25mL portions of ether

D It is more effective to perform four successive extractions with small amounts of ether than to perform one extraction with a large amount of ether.

The reduction of a carboxylic acid using lithium aluminum hydride will yield what final product? A. An aldehyde B. An ester C. A ketone D. An alcohol

D Lithium aluminum hydride (LiAlH2 or LAH) is a strong reducing agent. LAH can completely reduce carboxylic acids to primary alcohols. Aldehydes are intermediate products of this reaction; therefore, (A) is incorrect. The other compounds are not created through the reduction of a carboxylic acid.

Which of the following is true with respect to a micelle in a hydrophilic environment? A. The interior is hydrophilic B. The structure, as a whole, is hydrophobic C. It is composed of short-chain fatty acids with polar heads D. It can dissolve nonpolar molecules deep in its core

D Micelles are self-assembled aggregates of soap in which the interior is composed of long hydrocarbon (fatty) tails, which can dissolve nonpolar molecules. The outer surgace is covered with carboxylate groups, which makes the overall structure water-soluble. Soaps, in general, are salts of long-chain hydrocarbons with carboxylate head groups.

A saturated solution of cobalt(III) hydroxide (Ksp=1.6E-44) is added to a saturated solution of thallium(III) hydroxide (Ksp=6.3E-46). What is likely to occur? A. Both cobalt(III) hydroxide and thallium(III) hydroxide remain stable in solution. B. Cobalt(III) hydroxide precipitates and thallium(III) hydroxide remains stable in solution C. Thallium(III) hydroxide precipitates and cobalt(III) hydroxide remains stable in solution D. Both thallium(III) hydroxide and cobalt(III) hydroxide precipitate.

D Since both salts have a formula MX3, it is possible to directly compare the molar solubilities of each. When the solutions are mixed, [OH-] is above saturation levels for both the cobalt and the thallium in the solution. Since thallium(III) hydroxide has a smaller Ksp than that of cobalt (III) hydroxide, it will react first. The ion product of the mixed solution is higher than the Ksp for thallium(III) hydroxide precipitates, a small excess of OH- will remain, which gives an ion product slightly above the Ksp of cobalt (III) hydroxide. This will cause a small amount (1%-3%) of cobalt (III) hydroxide to also precipitate.

Which of the following will most likely increase the electric field between the plates of a parallel plate capacitor? A. adding a resistor that is connected to the capacitor in series B. adding a resistor that is connected to the capacitor in parallel C. increasing the distance between the plates D. adding an extra battery to the system

D The electric field between two plates of a parallel plate capacitor is related to the potential difference between the plates of the capacitor and the distance between the plates, as shown in the formula E=V/d. The addition of another battery will increase the total voltage applied ot the circuit, which, consequently, will increase the electric field. The addition of a resistor in series will increase the resistance and decrease the voltage applied to the capacitor, eliminating A. Adding a resistor in parallel will not change the voltage drop across the capacitor and should not change the electric field, eliminating B. Increasing the distance between the plates, C, would decrease the electric field, not increase it

The process of formation of a salt solution can be better understood by breaking the process into three steps: 1. Breaking the solute into its individual components 2. Making room for the solute in the solvent by overcoming intermolecular forces in the solvent 3. Allowing solute-solvent interactions to occur to form the solution. Which of the following correctly lists the enthalpy changes for the three steps, respectively? A. Endothermic, exothermic, endothermic B. Exo, endo, endo C. Exo, exo, endo D. Endo, endo, exo

D The first step will most likely be endothermic because energy is required to break molecules apart. The second step is also endothermic because the intermolecular forces in the solvent must be overcome to allow incorporation of solute particles. The third step will most likely be exothermic because polar water molecules will interact with the dissolved ions, creating a stable solution and releasing energy.

An 8.00 g sample of NH4NO3(s) is placed into an evacuated 10L flask and heated to 227C. After the NH4NO3 completely decomposes, what is the approximate pressure in the flask? NH4NO3(s) --> N2O(g) + H2O (g) A. 0.410 atm B. 0.600 atm C. 0.821 atm D. 1.23 atm

D The first thing to do is balance the given chemical equation: NH4NO3(s)-->N2O(g) + 2H2O(g). The mass given is 8g, which represents 0.1 mol NH4NO3 (molar mass=80g/mol). When 0.1 mol of the solid decomposes, it will form 0.1 mol N2O and 0.2 mol water. This gives approx. 0.3 moles of gas product. The ideal gas equation can be used: math pg 296 gen chem P=nRT/V=(0.3mol)(0.0821)(500K)/10L=1.2 atm C is the result if you don't balance the equation first

An elevator with a passenger accelerates upwards from rest at a value of 3 m/s^22squared. What is the normal force of the elevator on the passenger if the passenger has a mass of 50 kg? A. 500N B. 350N C. 150N D. 650N

D The net force of the passenger should equal the sum of individual forces on the passenger. Individual forces that are directed upwards towards the direction of the net force are positive in sum of forces equation: Fnet = Fnorm - Fg Fn= 50 x 10 + 0 x 3

A scientist looks through a microscope with two thin lenses with m1=10 and m2=40. What is the overall magnification of this microscope? A. 0.25 B. 30 B. 50 D. 400

D The overall magnification of a system of multiple lenses is simply the product of each lens's magnification. In this case, that is 10 x 40 = 400

Ultraviolet light is more likely to induce a current in a metal than visible light. This is because photons of ultraviolet light: A. have a longer wavelength B. have a higher velocity C. are not visible D. have a higher energy

D The photoelectric effect occurs when a photon of sufficiently high energy strikes an atom with a sufficiently low work function. This means that a photon with higher energy is more likely to produce the effect. Because ultraviolet light has a higher frequency and lower wavelength than visible light, it also carries more energy according to the equation E=hf. All light travels at the speed of light, eliminating B. As mentioned earlier, ultraviolet light has a shorter wavelength than visible light, eliminating A> The visibility of a wave plays no role in its ability to cause the photoelectric effect, eliminating C.

Why should esterification reactions NOT be carried out in water? A. Carboxylic acids, from which esters are made, are generally insoluble in water B. The polar nature of water overshadows the polar nature of the leaving group C. The extensive hydrogen bonding of water interferes with the nucleophilic addition mechanism D. Water molecule would hydrolyze the desired products back into the parent carboxylic acid.

D The presence of water in an esterification reaction would likely revert some of the desired esters back into carboxylic acids. Small carboxylic acids, like formic or acetic acid, are easily dissolved in water, eliminating A. The polarity of water plays little role in affecting the leaving group; if anything, water can be used to increase the electrophilicity of the carbonyl carbon by protonating the carbonyl oxygen- eliminating B. Finally, this is a nucleophilic substitution mechanism, not a nucleophilic addition mechanism, as mentioned in C. Further, hydrogen bonding would likely augment the reaction.

In the presence of an acid catalyst, the major product of butanoic acid and 1-pentanol is A. 1-butoxy-1-pentanol B. butyl pentanoate C. 1-pentoxy-1-butanol D. pentyl butanoate

D The reaction described is esterification, in which the nucleophilic oxygen atom of 1-pentanol attacks the electrophilic carbonyl carbon of butanoic acid, ultimately displacing water to form pentyl butanoate. The acid catalyst is regenerated from 1-pentabol's released proton. (A) reserves the carbon chains, considering the butyl tail to be the esterifying group. Esters do not form under these conditions, so (B) and (C) are also incorrect.

Which of the following expressions correctly describes the relationship between standard electromotive force and standard change in free energy? A. deltaG(not) = -nF(Ered,anode-Ered,cathode) B. E(not)cell= (nF/RT)ln Keq C. E(not)cell=-(RT/nF) ln Keq D. deltaG(not) = nF(Ered, anode-Ered,cathode)

D There are only two equations involving standard change in free energy in electrochemical cells: deltaG(not)=-nFE(not)cell and deltaG(not)=-RTlnKeq. Substituting E(not)cell=Ered,cathode-Ered,anode into the first equation and distributing the negative signs gives D.

Which conversion between carboxylic acid derivates is NOT possible by a nucleophilic reaction? A. carboxylic acid to ester B. ester to carboxylic acid C. anhydride to amide D. ester to anhydrive

D There is a hierarchy to the reactivity of carboxylic acid derivatives that dictates how reactive the are towards nucleophilic attack. In order from highest to lowest, this hierarchy is anhydrides > carboxylic acids and esters > amides. In practical terms, this means the derivates of higher reactivity can form derivatives of lower reactivity but not vice versa. Nucleophilic attack of an ester cannot result in the corresponding anhydride because anhydride are more reactive than esters.

Consider the chemical reaction in the vessel depicted in FIGURE 3 (general chemistry) A. The reaction is spontaneous B. The reaction is nonspontaneous C. The reaction is at equilibrium D. Not enough info

D There is not enough info available to determine the free energy of this reaction. While the entropy is clearly increasing (there are more particles in the system), it is unclear what the enthalpy change is. Because bonds are breaking, the reaction should be endothermic, meaning that both deltaS and deltaH are positive. In this case, it is a temperature-dependent process, and-without a temperature given-we cannot determine the sign on deltaG

A reaction has a positive entropy and enthalpy. What can be inferred about the progress of this reaction from this information? A. The reaction is spontaneous B. The reaction is nonspontaneous C. The reaction is at equilibrium D. There is not enough info

D There is not enough info in this problem to determine whether or not the reaction is spontaneous. If the signs of enthalpy and entropy are the same, the reaction is temp dependent according to deltaG=deltaH-TdeltaS. W/o temp, we can't determine if this reaction is spontaneous, nonspontaneous, or at equilibrium.

Which of these compounds would be expected to decarboxylate when heated? FIGURE 7 ORGANIC CHEMISTRY A B C D

D This compound is a B-keto acid: a carboyl functional group at the B-position from a carboxyl group. Decarboxylation occurs with B-keto acids and B-dicarboxylic acids because they can form a cyclic transition state that permits simultaneous hydrogen transfer and loss of carbon dioxide. (B) is a diketone and does not have a single carboxyl group. (A) and (C) and gamma and delta dicarboxylic acids, respectively, and can decarboxylate but with more difficulty

A positive charge on the molecule shown in FIGURE 13 ORGANIC CHEMISTRY would have greater stability than a positive charge on a straight-chain alkane version of the same molecules. What property explains this effect? A. Steric hindrance B. Nitrogen electronegativity C. Induction D. Conjugation

D This molecule is more stable with a positive charge than a straight-chain alkane due to the conjugation of the benzene ring. This permits delocalization of the charge through resonance. Although induction does have an effect on the stabilization of the molecule, this effect is much less significant than the impact of having a conjugated system. The electronegativity of N, which primarily affects induction, is also not a vital component of the stabilization by this molecule of a positive charge because oxygen is more electronegative. Steric hindrance would affect the reactivity of a molecule, but not its ability to stabilize charge.

A submarine is inspecting the surface of the water with a laser that points from the submarine to the surface of the water and through the air. At what angle will the laser not penetrate the surface of the water but rather reflect entirely back into the water? (Assume n water=1.33 and n air=1). A. 19 degrees B. 29 degrees C. 39 degrees D. 49 degrees

D This question is testing our understanding of total internal reflection. As the laser beam travels from water to air (from a higher to a lower index of refraction), the angle of refraction increases. At the critical angle, the angle of refraction is 90 degrees; at this point, the refracted ray is parallel to the surface of the water. When the incident angle is greater than the critical angle, all the light is reflected back into the water. The question is asking for the critical angle. Math on pg 309 physics theta c= sin-1 (n2/n1)= sin-1(1/1.33)= sin-1(0.75) The inverse sine of 0.75 must be slightly higher than 45 (sin 45=sqrt2/2=0.707). 48.59 is the exact answer.

A student is measuring sound frequencies from the side of a road while walking east. For which of the following situations could the student determine that the difference between the perceived frequency and the actual emitted frequency is zero? A. A plane flying directly above him from east to west B. A police car passing the student with its siren on C. A person playing piano in a house on the street D. A dog barking in a car that moves east

D This question is testing us on our understanding of the Doppler effect. A difference of zero between the perceived and the emitted frequency implies that the source of the sound is not moving relative to the student. if the car in D is moving at the same speed as the student, then the relative motion between them could be 0. In all of the other cases, the student and the sound source are necessarily moving relative to each other.

Given the reduction potentials for the following half-reactions, Br2 (l) + 2e- → 2 Br-(aq) Ered= + 1.07 V Au3+ (aq) + 3 e- → Au (s) Ered= + 1.55 V What is the electrochemical potential for the following chemical reaction? 2 Au3+ (aq) + 6 Br-(aq) → 3 Br2 + 2 Au (s) Ecell = ? A. Eo = - 0.48 V B. Eo = - 0.11 V C. Eo = + 0.11 V D. Eo = + 0.48 V

D This question is testing your understanding of electrochemical cells and using half-reactions to determine the cell potential. To answer this question, you must first know that the electrochemical potential for a cell is equal to the reduction potential plus the oxidation potential: Eocell = Eored + Eoox In this chemical reaction, Au3+ is reduced to Au making this the reduction half-reaction (Eored= + 1.55 V). Oppositely, Br- is oxidized to Br2 so this would be the oxidation half-reaction. Therefore, you must reverse the reduction potential given for Br2 so that it matches the chemical reaction given, as follows: 2 Br-(aq) → Br2 (l) + 2e- Eoox= - Eored= -(+1.07 V) = -1.07 V Therefore, the Eocell = Eored + Eoox = (+1.55 V) + (-1.07 V) = + 0.48 V making D the correct answer. Note, when calculating the cell potentials, do not multiply the reduction potential or oxidation potential by the coefficients.

What is the [H3O+] of a 2M aqueous solution of a weak acid HXO2 with Ka=3.2E-5? A. 6.4E-5M B. 1.3E-4M C. 4.0E-3M D. 8.0E-3M

D This question requires the application of the acid dissociation constant. Weak acids do not dissociate completely; therefore, all three species that appear in the balanced equation will be present in solution. Hydrogen ions and conjugate base anions dissociate in equal amounts, so [H+] = [XO2-]. If the initial [ ] of HXO2 was 2M and some amount x dissociates, we will have x amount of H3O+ and XO2- at equilibrium, with 2M-x amount of HXO2 at equilibrium: Ka=([H3O+][XO2-]/[HXO2])=(x)(x)/2M-x = x^2/2 Note that x was considered negligible when added or subtracted, per usual. Solving for x, we get: x^2/2 = 3.2E-5 --> x^2 = 6.4E-5 = 64E-6 --> x=8E-3M

A voltaic cell provides a current of 0.5A when in a circuit with a 3 omega resistor. If the internal resistance of the cell is 0.1 omega, what is the voltage across the terminals of the battery when there is no current flowing? A. 0.05V B. 1.5V C. 1.505V D. 1.55V

D This question tests our understanding of batteries in a circuit. The voltage across the terminals of the battery when there is no current flowing is referred to as the electromotive force (emf or E of the battery). However, when a current is flowing through the circuit,the voltage across the terminals of the battery is decreased by an amount equal to the current multiplied by the internal resistance of the battery. Mathematically, this is given by the equation V=E-ir(int) To determine the emf of the battery, first calculate the voltage across the battery when the current is flowing. For this, we can use Ohm's law: V=IR =0.5A(e omega)=1.5V Because we know the internal resistance of the battery, the current, and the voltage, we can calculate the emf: E=V+ir(int) =1.5V + (0.5A)(0.1 omega) =1.55V The answer makes sense in the context of a real bettery because tis internal resistance is supposed to be very small so that the voltage provided to the circuit is as closed as possible to the emf of the cell when there is no current running.

The combination of the cornea and the crystalline lens shown below serves as a converging lens. In a perfectly functioning eye, an image is projected at a fixed distance on the retina, which is approximately 2cm from the lens. What is the power of the converging lens when a person without any visual impairment stares at a distant object? image A. 0.5 diopters B. 10 diopters C. 25 diopters D. 50 diopters

D To answer this question, you must use the thin lens equation 1/f = 1/do + 1/di, where f is the focal length, do is the object distance, and di is the image distance. Since the power (P) of a lens is equal to 1/f, you can set P = 1/do + 1/di. The question stem states that image distance is fixed and equal to a length of 2 cm. Since the question states that the observer stares at a distant object, we can assume that 1/do is negligible and equal to zero. Therefore, the equation simplifies to P = 1/di. Since power is measured in diopters, which has the units of m-1, you must convert 2 cm to 0.02 m and solve. Thus, the power of the lens is equal to 50 diopters, or answer choice D.

Mg(OH)2 is slowly dissolved in 500 mL of 25oC water until the solution becomes fully saturated. Which of the following occurs when 10.0 mL of 0.1 M HCl is added? A. MgCl2 precipitates B. Mg(OH)2 precipitates C. Ksp for Mg(OH)2 increases D. [H2O] increases

D When HCl is added to the solution, the pH of the solution decreases. The additional H+ ions react with the dissolved OH- ions, forming water. Magnesium chloride is soluble in water, eliminating A. When HCl is added, the hydroxide [ ] decreases shifting the equilibrium to the right, making B false. The solubility product, Ksp, is a constant that varies only with temp, making C incorrect.

What is the pH of a solution with an ammonium [ ] of 70mM and an ammonia [ ] of 712mM? (Note: The pKb of ammonia is 3.45) A. 2.45 B. 4.45 C. 9.55 D. 11.55

D pOH=pKb + log([conjugate acid]/[conjugate base]) = 3.45 + (log70mM/712mM) =approx 3.45 + log1/10 = 3.45-1=2.45 If the pOH =2.45, the pH = 14-2.45=11.55

In 1H-NMR, splitting of spectral lines is due to: A. coupling between a carbon atom and protons attached to that carbon atom B. coupling between a carbon atom and protons attached to adjacent carbon atoms C. coupling between adjacent carbon atoms D. coupling between protons on adjacent carbon atoms

D spin-spin coupling (splitting) is due to influence on the magnetic environment of one proton by protons on the adjacent atom. These protons are three bonds away from each other.

If the area of a capacitor's plates is doubled while the distance between them is halved, how will the final capacitance (Cf) compare to the original capacitance (Ci)? A. Cf=Ci B. Cf=1/2Ci C. Cf=2Ci D. Cf=4Ci

D pg 230 physics and math This question should bring to mind the equation C=Esub0(A/d), where Esub0 is the permittivity of free space, A is the area of the plates, and d is the distance between the plates. From this equation, we can infer that doubling the area will double the capacitance, and halving the distance will also double the capacitance. Therefore, the new capacitance is four times larger than the original capacitance.

Each of the resistors shown in FIGURE 11 PHYSICS carries an individual resistance of 4 omega. Assuming negligible resistance in the wire, what is the overall resistance of the circuit? A. 16 omega B. 8 omega C. 4 omega D. 3 omega

D pg 231 physics and math The resistance of the three resistors wired in series is equal to the sum of the individual resistances (12 omega). This means that the circuit functionally contains a 12 omega resistor and a 4 omega resistor in parallel. To determine the overall resistance of this system, use the formula 1/Rp=(1/R1)+(1/R2)= (1/12 omega)+(1/4 omega)=(1+3/12 omega)-->Rp=3 omega

A block is released from the top of an inclined plane and allowed to slide to the bottom. The speed of the block at the bottom is maximized if the angle of incline is closest to: (assume a frictionless track and all inclines start from the same height) A. 90º B. 45º C. 0º D. The angle of incline is irrelevant

D. This is a conservation of energy question. The block begins with only gravitational potential energy. When reaching the ground, all of that energy has been converted to kinetic energy. Because the block is starting with the same mass and the same height, it will therefore start and end with the same total energy. Thus it will have the same speed upon reaching the ground, regardless of the angle of inclination. Note that the block is never released from an incline with exactly 0º (flat ground), since the incline has some height and the question only says "closest to", not "equals".

A reactant in a second order reaction at a certain temperature is increased by a factor of 4. By how much is the rate of the reaction altered? A. It is unchanged B. It is increased by a factor of 4 C. It is increased by a factor of 16 D. It cannot be determined from the information given

D. A second-order reaction can be second-order with respect to one reactant or first-order with respect to both reactants. Need this info to determine the rate of the reaction change.

In a third order reaction involving two reactants and two products, doubling the concentration of the first reactant causes the rate to increase by a factor of 2. What will happen to the rate of this reaction if the concentration of the second reactant is cut in half? A. It will increase by a factor of 2 B. It will increase by a factor of 4 C. It will decrease by a factor of 2 D. It will decrease by a factor of 4

D. Based on the info given, the rate is first-order with respect to the [ ] of the 1st reactant; when the [ ] of the reactant doubles, the rate also doubles. Because the reaction is third order, the sum of the exponents in the rate law must be equal to 3. Therefore, the reaction order with respect to the other reactant must be 3-1-2. If the concentration of this second reactant is multiplied by 1/2, the rate will be multiplied by (1/2)^2=1/4

If the magnetic field a distance r away from a current-carrying wire is 10T, what will be the net magnetic field at r if another wire is placed a distance 2r from the original wire (with r in the middle) and has a current twice as strong flowing in the opposite direction? A. 0T B. 15T C. 20T D. 30T

D. The safest way to answer this question is to quickly draw a diagram: (View on pg 192 of physics and math). Notice that between the two wires, the direction of the magnetic field is the same: into the page. Therefore, because the vector direction is the same, we can just focus on the magnitudes of the two magnetic fields. We know that B1=10T at distance r. Consider the relationships int he equation B=mu sub0 (I)/2 pi r. Magnetic field and current are directly proportional, whereas magnetic field and distance are inversely proportional. Therefore, doubling the current will result in double the magnetic field of the first wire, or 20T. The overall magnitude of the magnetic field is 10T+20T=30T into the page

What happens when the product itself is a liquid that is soluble in the solvent? How can you separate them?

Distillation.

An elevator's motion is through the action of a single cable. The elevator has a gravitational force of 2000 N. When the elevator is moving downwards at a constant velocity, how much tension is in this cable? A. slightly less than 2000N B. equal to 2000N C. greater than 2000N D. close to 0N

Drawing a free-body diagram of a elevator in motion will elucidate the forces on the elevator The forces on the elevator when it is moving downwards at a constant velocity are the gravitational force and the force due to the tension on the cable. If the elevator is moving at constant velocity, then there is no net acceleration and no net force. The tension in the cable must equal the gravitational force of the elevator.

A 0.1 mol sample of CuS is added to 1L of 1M NH3. What is the final [ ] of the complex ion, tetraammine copper(II)? CuS(s) <--> Cu2+(aq) + S2-(aq) Ksp=8E-37 Cu2+(aq) + 4NH3 (aq)<--> [Cu(NH3)4]2+(aq) Kf-1.1E13

First determine the amount of copper ion produced from CuS in solution. Comparison of the two equilibrium constants shows that the CuS dissociation has a Ksp of 8E-37, indicating that this reaction is unlikely to proceed as the forward reaction is not favorable. However if the formation of the complex ion occurs simultaneously, the large Kf of 1.1E13 for this process will drive the dissociation of CuS forward as the Cu2+ ions are consumed in the second reaction due to Le Chatlier's principle effects. Note that, even without quantitative analysis, the large value of Kf is a sign that the formation of the product of the second reaction will be highly favorable. In fact, if these two reactions are simultaneous, the CuS will ultimately be completely consumed due to this effect. Thus, the amount of Cu2+ available to react is given by [Cu2+] = 0/1mol/1L - 0/1M. The reactions given show a 1:1 relationship between the Cu2+ available and the [Cu(NH3)4]2+ that is generated. In other words, almost all the Cu2+ ions will be used up to form the complex, Therefore the [ ] of [Cu(NH3)4]2+ complex ion is 0.1M.

If enough water is added to 11 g of CaCl2 to make 100mL of solution, what is the molarity of the solution?

First, calculate the number of moles of CaCl2: 11gCaCl2 [1mol/111.1g]=0.1 mol CaCl2 Then determine the molarity: M=moles of solute/liters of solution M=0.1mol/(100mL)(1L/1000mL) = 0.1/0.1=1M

If 10 g NaOH are dissolved in 500g of water what is the molality of the solution?

First, calculate the number of moles of NAOH: 10g NaOH [1mol/40g] =0.25 mol NaOH Then determine the molality m=moles of solute/kilograms of solvent m=0.25 mol/ (500g)(1kg/1000g) = 0.35/0.5 = 0.5m

If 184g glycerol (C3H8O3) is mixed with 180g water, what will be the mole fractions of the two components? Note the molar mass of H2) = 18g/mol and molar mass of C3H8O3=92 g/mol.

First, determine the number of moles of each compound: 180 g water x [1mol/18g] = 10 mol water 184 g glycerol x [1mol/92g] = 2 moles glycerol Total number of moles = 10+2= 12 moles The determine the mole fraction" Xwater = 10 mol water/ 12 mol total = 5/6 = 0.83 Xglycerol = 2 mol glycerol/12 mol total = 1/6 = 0.17

Give that the standard reduction potentials for Sm3+ and [RhCl6]3- are -2.41V and +0.44V, respectively, calcuate the electromotive force of the following reaction: Sm3+ + Rh + 6Cl- --> [RhCl6]3- + Sm

First, determine the oxidation and reduction half-reactions. As written, the Rh is oxidized and the Sm3- is reduced: Sm3+ + 3e- --> Sm Rh + 6Cl- --> [RhCl6]3- + 3e- Now we take the difference between the Sm3+ reduction potential and the [RhCl6]3- reduction potential. We need not change the sign on the [RhCl6]3 reduction potential because we are subtracting it from that of Sm3+. Using the equation: Ecell=Ered,cathode = Ered,anode, the emf can be calculated as -2.41V - (+0.44V) = -2.85V The cell is thus electrolytic.

Find the emf of a galvanic cell at 25C based on the following standard reduction potentials: Fe2+ + 2e- --> Fe Ered=-0.44V Cl2 + 2e- --> 2Cl- Ered=+1.36V In this cell, [Fe2+]=0.01M and [Cl-]=0.1M

First, determine the standard cell potential. Because the chlorine half-reaction has a higher reduction potential, it will be the cathode. Iron will act as the anode. The standard cell potential is: Ecell=Ered,cathode-Ered,anode=1.36V-(-0.44V) =+1.8V Now determine the net ionic equation. remember that iron is being oxidized so its reduction half reaction in the question stem will have to be reversed. The net ionic equation is: Fe + Cl2 --> Fe2+ + 2Cl- From this equation we can determine the value of the reaction quotient: Q=[Fe2+][Cl-]^2 = (0.01M)(0.1M)^2 = 10^-4 Now plug into the Nernst equation, keeping in mind that two electrons are transferred (N=2) Ecell=E(not)cell -(0.0592/n)logQ = 1.8-(0.0592/2)log10^-4 =1.8+[(4 x 0.0592)/2] = 1.8 + (2 x 0.6) = 1.92V In this case, the cell actually has a higher voltage than it normally would due to the [ ] of ions present

What are the empirical and molecular formulas of a carbohydrate that contains 40.9% carbon, 4.58% hydrogen, and 54.52% oxygen and has a molar mass of 264g/mol?

First, determined the # of moles of each element in the compound by assuming a 100g sample; this converts the percentage of each element present directly into grams of that element. Then convert grams to moles: moles C = 40.9g/ 12 g/mol = 3.4 moles moles H = 4.58g/ 1 g/mol = 4.6 moles moles O = 54.52g/ 16g/mol= 3.4 moles Next, find the simplest whole number ratio of the elements by dividing the number of moles for each compound by the smallest number out of all obtained in the previous step. C: 3.4/3.4= 1 H: 4.6/3.4 = 1.33 O: 3.4/3.4= 1 The empirical formula is obtained by converting the numbers obtained into whole numbers Empirical formula =C1H1.22O1 x 3 = C3H4O3 To determine the molecular formula, divide the molar mass (264g/mol) by the empirical formula weight. The resulting value gives the number of empirical formula units in the molecular formula (3 x 12 g/mol C) + (4 x 1 g/mol H) + (3 x 12 g/mol O) = 88g/mol total 263/88 =3 Molecular formula =C3H4O3 x 3 = C9H12O9

Determine the standard change in free energy of a cell with the following net reaction. Note the standard reduction potential of iron(III) is +0.77V; the standard reduction potential of molecular chlorine is +1.36V 2Fe3+(aq) + 2Cl-(aq) --> 2Fe2+(aq) + Cl2(g)

First, separate the reaction into the half-reactions: 2Fe3+ + 2e- --> 2Fe2+ 2Cl- --> Cl2 + 2e- In this reaction, iron(III) is reduced and is the cathode, whereas Cl- is oxidized and is the anode. The reduction potential of chlorine is actually higher than iron(III); this means that the electrodes are serving the opposite role from their natural tendency, and the reaction is nonspontaneous. this is an electrolytic cell, and should have a negative emf value. Now, determine the emf: Ecell = Ered,cathode - Ered,anode = 0.77V-1.36V= -0.59V Use the emf to determine the free energy change (note that 2 electrons are transferred, n=2): deltaG = -nFEcell = -(2 mol e-)(96485C/mol e-)(-0.59V) = approx 2E5 (0.6) = 1.2E5J (actual 1.14E5J) The free energy change is about +120kJ, which represents a non-spontaneous reaction

Consider two resistors wired in parallel with R1=5omega and R2=10omega. If the voltage across them is 10V, what is the current through each of the two resistors?

First, the current flowing through the whole circuit must be found. To do this, the equivalent resistance must be calculated: 1/Rp=1/R1=1/R2 1/10omega+1/5ohm=3/10omega Using Ohm's law to calculate the current flowing through the circuit give Ip=Vp/Rp=10V/(10/3omega)=3A Three amps flow through the combination of R1 and R2. Because the resistors are in parallel, Vp=V1=V2=10V. Apply Ohm's law to each resistor individually I1=Vp/R1=10V/5omega=2A I2=Vp/R2=10V/10omega-1A As a check, note that Ip=3A=I1+I2. More current flows through the smaller resistor. In particular, note that R1, with half the resistance of R2, has twice the current. Once Ip was found to be 3A, the problem could have been solved by noting the ratio of the resistances of the two branches.

In a closed system with liquid flowing, how do flow rate and linear speed change when a fluid flows in a pipe with a small cross sectional area to a pipe with a large cross sectional area?

Flow rate (volume per unit time) is CONSTANT and independent of changes in cross-sectional area Linear speed (a measure of linear displacement of fluid particles in a given amount of time) changes.

An electrolytic cell consists of two half-reactions: Ag+(aq) + e- → Ag(s) E° = 0.80 Cu2+(aq) + 2e- → Cu(s) E° = 0.34 What is the standard-state cell potential for the electrolytic cell? a) -1.14 V b) -0.46 V c) 0.46 V d) 1.14 V

For the MCAT, an electrolytic cell will always have a negative standard-state cell potential. A power supply must be used to drive the non-spontaneous reaction. Here, we have two half-reactions: Ag+(aq) + e- → Ag(s) E° = 0.80 Cu2+(aq) + 2e- → Cu(s) E° = 0.34 One of the reactions must run in reverse in order to pair a reduction with an oxidation (which is necessary for a redox reaction). If the copper reaction runs in reverse, the standard reduction potential will be -0.34 + 0.8 = 0.46 V. This cannot be correct as the voltage is positive. Try running the Ag reaction in reverse and the copper forward: the standard-state cell potential is -0.80 + 0.34 = -0.46 V. (Note that we should not multiply the Ag potential by two, even though it only contains one electron while the other reaction contains two. This is important to remember for electrochemistry problems!) This could work for an electrolytic cell being driven by a power source. Both half-reactions cannot run in reverse, as we must always pair an oxidation with a reduction, and both half-reactions cannot run forward for the same reason. a) -1.14 V, incorrect, This answer has both half-reactions running in reverse. b) -0.46 V, correct. This answer has a negative standard-state cell potential, and one reaction runs forward (or reduces) while the other runs in reverse (or oxidizes). c) 0.46 V, incorrect, An electrolytic cell on the MCAT will have a negative standard-state cell potential. d) 1.14 V, incorrect, Both reactions cannot run forward, and the standard-state cell potential must be negative for an electrolytic cell.

The gaseous state of matter is characterized by which of the following properties? I. Gases are compressible II. Gases assume the volume of their containers III. Gas particles exist as diatomic molecules

I and II Gases are easily compressible because they travel freely with large amounts of space between molecules. Because gas particles are far apart from each other and in rapid motion, they tend to take up the volume of their container. Many gases exist as diatomic molecules, but this is not a property that characterizes all gases.

Nucleophilic acyl substitution is favored by: I. basic solution II. acidic solution III. leaving groups that are strong bases

I and II Nucleophilic acyl substitutions are favored in basic solution, which makes the nucleophile more nucleophilic; in acidic solution, which makes the electrophile more electrophilic; and by good leaving groups. However, strong bases do not make good leaving groups; weak bases do.

Which of the following are common names for carboxylic acid derivatives? I. Acetic anhydride II. Formic acid III. methyl formate

I and III

When succinaldehyde is treated with lithium diisopropylamide (LDA), it: I. becomes more nucleophilic II. becomes less nucleophilic III. generates a carbanion

I and III When succinaldehyde (or any aldehyde or ketone with alpha hydrogens) is treated with a strong base like lithium diisopropylamide (LDA), it forms the more nucleophilic enolate carbanion.

A parachutist jumps from a plane. Beginning at the point when she reaches terminal velocity, which of the following is/are true? I. The jumper is in translational equilibrium II. The jumper is not being acted upon by any forces III. There is an equal amount of work being done by gravity and air resistance.

I and III at terminal velocity, the force of gravity and force of air resistance are equal in magnitude, leading to translational equilibrium. If these forces have the same magnitude and act over the same displacement, then the work performed is the same as well, making statement III true.

An electrolytic cell is filled with water. Which of the following will move toward the cathode of such a cell? I. H+ ions II. O2- ions III. Electrons

I and III only In an electrolytic cell, ionic compounds are broken up into their constituents: the cations migrate toward the cathode and anions migrate toward the anode. In this case, the cations are H+ ions so option I is correct. Electrons flow from anode to cathode in all types of cells, meaning that option III is also correct. Option II is incorrect for two reasons. First, it is unlikely that the anions in any cell would be O2- rather than OH-. Second, and more significantly, these anions flow to the anode not the cathode.

Aluminum metal can be used to remove tarnish from silver when the two solid metals are placed in water, according to the following reaction: 3AgO +2Al --> 3Ag +Al2O3 The reaction is a : I. double-displacement reaction II. single-displacement reaction III. oxidation-reduction reaction IV. combustion reaction

II and III

The alpha hydrogen of a carboxylic acid is I. more acidic than the hydroxyl hydrogen II. less acidic than the hydroxyl hydrogen III. relatively acidic, as organic compounds go

II and III The alpha hydrogen of a carboxylic acid is relatively acidic as far as organic compounds go, due to resonance stabilization. However, the hydroxyl hydrogen is significantly more acidic because it is able to share the negative charge resulting from deprotonation between both electronegative oxygen atoms in the functional groups

Which of the following is true of equilibrium reactions? I. An inc in k1 results in a decrease in k-1 II. As the [ ] of products increases, the [ ] of reactants decreases III. The equilibrium constant is altered by changes in temp

II and III only Statement I is false because the addition of a catalyst could inc the rate constants of both the forward and reverse reactions. Statement II is true because for products to come into existence, reactants must be used up Statement III is also true, all K values are temp-dependent

Why is a single bond stronger than a pi bond? I. Pi bonds have greater orbital overlap II. s-orbitals have more overlap than p-orbitals III. sp3 hybridization is always unstable

II only

Which of the following will convert CH3CH2CH2OH to CH3CH2CHO? I. CrO3 II. PCC III. K2Cr2O7

II only CH3CH2CH2OH is 1-propanol, a primary alcohol. The desired end product is propanal, an aldehyde. Of the available options, the only reactant capable of oxidizing primary alcohols to aldehydes is PCC. Chronic trioxide and dichromate salts will both oxidize primary alcohols to carboxylic acids

What are the exceptions to the octet rule?

Incomplete octet: these elements are stable with fewer than 8 electrons in their valance shell and include Hydrogen (2), Helium (2), Lithium (2), Beryllium (4), and Boron (6) Expanded octet: any element in period 3 and greater can hold more than 8 electrons, include Phosphorus (10), Sulfur (12), Chlorine (14), and many others Odd # of Electrons: any molecule with an odd # of valence electrons cannot distribute those electrons to give eight to each atom (EX: NO (nitric oxide) has 11 valence electrons) Octet rule abiders: Oxygen, Carbon, Nitrogen, Fluorine, Sodium, and Magnesium

What is an isolated system? What is a closed system? What is an open system?

Isolated: The system cannot exchange energy (heat and work) or matter with the surroundings; for example, an insulated bomb calorimeter Closed: The system can exchange energy but not matter with the surroundings; ex: a steam radiator Open: The system can exchange both energy and matter with the surroundings; ex: a pot of boiling water

When we perform extractions, what is important? Explain the process of an extraction.

It is important to make sure that the two solvents are immiscible, meaning that they form two layers that do not mix, like water and oil. The two layers are temporarily mixed by shaking so that solute can pass from one solvent to the other. Ex: In a solution of isobutyric acid and diethyl ether we can extract the isobutyric acid with water. Isobutyric acid, with its polar carboxyl group, is more soluble in a polar solvent like water than in a nonpolar solvent like ether. When the two solvents are mixed together, isobutyric acid will transfer to the water layer, which is called the aqueous phase (layer). The nonpolar ether layer is called the organic phase (layer). The water (aqueous) and ether (organic) phases will separate on their own, given time. In order to isolate these two phases, we use a separatory funnel. Gravitational forces cause the densey layer to sink to the bottom of the funnel, where it can then be removed by turning the stopcock at the bottom. In this example we'll assume that the aqueous layer is more dense and settles to he bottom of the separatory funnel. Once we drain the aqueous layer from the separatory funnel, we repeat the extraction several times. Additional water is added to the separatory funnel; it is shaken and allowed to settle, and the aqueous layer is once again drained off. This is done in order to extract as much of the isobutyric acid from the ether layer as possible because it does not completely transfer with the first extraction. Multiple extractions with fresh water are more effective for obtaining the most product, rather than a single extraction with a larger volume of water. Once the desired product has been isolated in the solvent, we can obtain the product alone by evaporating the solvent, usually by using a rotary evaporator (rotovap).

What is the reaction quotient, Q? Q<Keq means what? Q=Keq means what? Q>Keq means what?

It is the "timer" to indicate how far the reaction has proceeded. Q<Keq, then the forward reaction has not yet reached equilibrium - deltaG<0 - There is a greater [ ] of reactants and a smaller [ ] of products than at equilibrium - The forward rate of the reaction is increased to restore equilibrium Q=Keq, then the reaction is in dynamic equilibrium - deltaG=0 - The reactants and products are present in equilibrium proportions -The forward and reverse rates of reaction are equal Q>Keq, then the forward reaction has exceeded equilibrium -deltaG>0 - There is a greater [ ] of products and smaller [ ] of reactants than at equilibrium - The reverse rate of reaction is increased to restore equilibrium

Explain reverse-phase chromatography.

It is the exact opposite of thin layer chromatography. The stationary phase used is nonpolar , so polar molecules move up the plate quickly, while nonpolar molecules stick more tightly to the stationary phase. The spots of individual compounds are usually white, which makes them difficult or impossible to see on the white paper or TLC plate. To get around this problem, the developed plate can be placed under UV light, which will show any compounds that are UV-sensitive. Alternatively, iodine, phosphomolybdic acid, or vanillin can be used to stain the spots, although this will destroy the compounds such that they cannot be recovered.

Keq=?/? A reaction that strongly favors _____(products or reactants) will have a large, positive exponent, and the larger the exponent, the less ______ that will be present at equilibrium. A large, negative exponent indicates that a reaction strongly favors ______ (products or reactants) at equilibrium.

Keq=[products]/[reactants] A reaction that strongly favors products will have a large, positive exponent, and the larger the exponent, the less reactant that will be present at equilibrium. A large, negative exponent indicates that a reaction strongly favors reactants at equilibrium.

For a plane mirror, how can you calculate the image distance?

Normal equation: 1/f=1/o=1/i=2/r Plane mirrors can be thought of as spherical mirrors with infinitely large focal distances. As such, for a plane mirror r=f=infinity, and the equation becomes 1/o+1/i=0 or i=-o. This can be interpreted as saying the virtual image is at a distance behind the mirror equal to the distance the object is in front of the mirror.

A detector with a surface area of 1 square meter is placed 1 meter from a blender. It measures the average power of the blender's sound as being 10^-3W. Find the intensity and sound level of the blender, and the ratio of the intensities of the blender and a jet engine.

PHYSICS pg 250 I=P/A= (10^-3W)/(1m^2)= 10^-3W/m^2 Sound level B=10log(I/Isub0)= 10log (10^-3W/m^2)/(10^-12W/m^2)=10log10^9=90dB Finally, the ratio of two sound intensities can be found from the difference of their sound levels: Bjet=Bblender + 10log(Ijet/Iblender) 150dB=90dB + 10 log (Ijet/Iblender) 6=log (Ijet/Iblender) 10^6= (Ijet/Iblender) Thus, the jet engine's sound is 1,000,000 times more intense than the blender's sound

Rank the following in order of decreasing nucleophilicity in an aprotic solvent: RO-, RCOOH, ROH, HO-

RO- > HO- > ROH > RCOOH Remember, good nucleophiles tend to have lone pairs or pi bonds and are negatively charged or polarized. ROH and RCOOH are weak nucleophiles The alkyl group of an alkoxide anion donates additional electron density, making it more reactive then the hydroxide ion. The carboxylic acid contains more electron-withdrawing oxygen atoms than the alcohol, making it less nucleophilic

Omeprazole is a proton pump inhibitor commonly used in gastroesophageal reflux disease. When omeprazole, a racemic mixture, went off patent, pharmaceutical companies began to manufacture esomeprazole, the (S)- enantiomer of omeprazole, by itself. Given 1M solution of omeprazole and esomeprazole, which solution would likely exhibit optical activity? A. Omeprazole only B. Esomeprazole only C. Both D. Neither

Racemic mixtures like omeprazole contain equimolar amounts of two enantiomers and thus have no observed optical activity. Each of the two enantiomers causes rotation in opposite directions, so their effects cancel out. Esomeprazole only contains one of the two enantiomers and thus should cause rotation of plane-polarized light

A circuit is wired with one cell supplying 5V in series with three resistors of 3omega, 5omega, and 7omega, also wired in series as shown in FIGURE 6 PHYSICS. What is the resulting voltage across and current through each resistor of this circuit, as well as the entire circuit?

Rs=R1+R2+R3=3+5+7=15omega Is=Vs/Rs=(5V/15omega)=0.33A Because everything is in series, this is also the current through each circuit element. Now, use Ohm's law for each of the resistors in turn. From a to b, the voltage drop across R1 is IR1=0.33A(3omega)=1V From b to c the voltage drop across R2 is IR2=0.33A(5omega)=1.67V From c to d the voltage drop across R3 is IR3=0.33A(7ohm)=2.33V

Sulfide

S2-

The coefficient of kinetic friction for a car against a road is 0.05. If a car moving at 10 m/s suddenly stops its wheels from moving, how far will the car go before it comes to a complete stop? A. 10m B. 1000m C. 100m D. 0.01m

Since the normal and gravitational forces cancel out, the force due to kinetic friction is the only force relevant to our determination of the net acceleration. This force is represented by Fnet =Ffriction = ukmg. ma(net) = ukmg vf2 - Vi^2 =2ad

what is ksp?

Solubility product is the equilibrium constant for the reaction in which the solid salt dissolves to give its constituent ions in solution.

If at time t=0 there is a 2 mole sample of radioactive isotopes, how many nuclei remain after 45 minutes, assuming a decay constant of 2hr^-1? (Hint: e^-3.2 =0.22)

Solution on pg 330 physics This question is asking for an application of the exponential decay equation: n=nsubo(e)^gamma(t) = nsubo(e)^(2 hr^-1)(3/4hr) = nsuboe^-3/2 Raising Euler's number (e) to an exponent- especially a fractional exponent, is beyond the scope of the math on the MCAT, but the value of e^-3/2 is 0.22. Thus, 22% of the original 2 mole sample remains. This constitutes 0.44 mol, which, if multiplied by avagadros number, gives us the number of nuclei remaining 0.44 mol x 6.02E23nuclei/mol = 2.64E23 nuclei

After balancing the following oxidation-reduction reaction, what is the sum of the stoichiometric coefficients of all of the reactants and products? S8 (s) + NO3^- (aq) -> SO3^2- (aq) + NO (g) A.4 B. 50 C. 91 D. 115

The balanced half reactions are: S8 + 24H2O --> 8SO3^2- + 48H+ + 32e- NO3- + 4H+ + 3e- --> NO + 2H2O To get equal numbers of electrons in each half reaction, the oxidation half reaction will have to be multiplied by 3 and the reduction half reaction will have to be multiplied by 32. This makes the overall reaction: 3S9 + 32NO3- + 8H2O --> 24SO3^2- + 32NO + 16H+ The sum of the stoichiometric oefficients is therefore 3 + 32 + 8 + 24 + 32 + 16 = 115

What is the change in vapor pressure when 180 g of glyceraldehyde (C3H6O3) are added to 0.18L of water at 100C?

The density of water at 100C is close to 1g/mL and the vapor pressure of water at the same temp is 1 atm because this is the BP of water. In order to find the mole fraction of the solvent, first find the molar mass of the solute (glyceraldehyde) and solvent (water). 180g glyceraldehyde represents 2 moles of glycrealdehyde. 0.18L of water has a mass around 180 g, which represents 10 mole water. The mole fraction of water is therefore 10mol water/12 mol total =5/6=0.83 Pa=XaPa(pure) = (0.83)(1atm)=0.83atm The change in vapor pressure is therefore 1atm-0.83atm=0.17 atm

Measurements of the atomic mass of a neutron and a proton yield these results: proton=1.00728 amu neutron=1.00867 amu 4/2He contains two protons and two neutrons, which should theoretically give a helium nucleus a mass of 2x1.00728+2x1.00867=4.03190amu. However, the true mass of the helium nucleus is 4.00260amu. What is the mass defect and binding energy of this nucleus? (c^2=932MeV/amu)

The difference 4.03190-4.00260=0.02930 is the mass defect fo the helium nucleus. This is the mass that contributed to the binding energy of the nucleus: E=mc^2 =0.02930amu(932MeV/amu) =27.3MeV

What is the interquartile range? What are quartiles? How do you calculate quartiles.

The difference between the upper quartile and the lower quartile. IQR=Q3-Q1 Quartiles, including the median (Q2), divide data (when placed in ascending order) into groups that comprise one-fourth of the entire set. 1. To calculate the position of the first quartile (Q1) in a set of data sorted in ascending order, multiply n by 1/4 2. If this is a whole number, the quartile is the mean of the value at this position and the next highest position. 3. If this is a decimal, round UP to the next whole number, and take that as the quartile position. 4. To calculate the quartile position of the third quartile (Q3), multiply the value of n by 3/4. Again, if this is a whole number, take the mean of this position and the next. If it is a decimal, round up to the next whole number, and take that quartile position.

The voltage across the terminals of an isolated 3E-6F capacitor is 4V. If a piece of ceramic having dielectric constant k=2 is placed between the plates, find the new charge, capacitance, and voltage of the capacitor.

The introduction of the dielectric by itself has no effect on the charge stored on the isolated capacitor. There is no new charge, so the charge is the same as before. The charge stored is therefore give by Q'=Q=CV=(3E-6F)(4V)= 12E-6C By introducing a dielectric with a dielectric constant of 2, the capacitance of the capacitor is multiplied by 2 (C'=kC). Hence the new capacitance is 6E-6F. Now, the new voltage across the capacitor can be determined V'=Q'/C'=(12E-6C)/(6E-6F)=2V

What are the null and alternative hypotheses?

The null hypothesis says that two populations are equal or that a single population can be described by a parameter equal to a given value. The alternative hypothesis may be nondirectional (the populations are not equal) or directional (Ex: that the mean of population A is greater than the mean of population B)

When a packet of light called a photon strikes the electron cloud, the electron may jump to a higher energy shell. Although the electron might stay in the excited state without returning to the ground state, we observe electrons returning to ground rather than staying in excited states. If an additional photon can strike the excited electron to stimulate a return to ground state, how many photons are given off by an electron returning to ground state from a shell at one higher energy level after excitation by a second photon? A. 0 B. 1 C. 2 D. 3

The photoelectric effect describes the phenomenon of electrons jumping to discrete energy levels after excitation by packets of light called photons. Photons can excite electrons to higher energy states in a process called absorption, and photons are given off as electrons return to ground states in a process called emission. We do not observe electrons staying at an excited state and spontaneously returning to ground. In a process called stimulated emission, described by Einstein, excitation from a second photon from incident light again excites the electron, which then returns to ground state. As stated in the question stem, a second photon excites the electron before the electron returns to ground state. This phenomenon of one photon striking an excited electron in order to release two photons in a return to ground state is the theory behind lasers. One in yields two out, but the electron must be in an excited state in order to undergo stimulated emission. A) 0, incorrect, When electrons return to ground state there is an emission of photons. B) 1, incorrect, Although the initial excitation of the electron will yield one photon out in a return to ground state, the question stem states that an additional photon has struck the electron in order to stimulate the emission. C) 2, correct, Both the initial excitatory photon and the second excitatory photon are released. D) 3, incorrect, There is no third photon mentioned in the question.

Explain column chromatography.

The principles behind column chromatography are the same as for thin-layer chromatography, although there are some differences. First, column chromatography uses an entire column filled with silica or aluminum beads as an adsorbent, allowing for much greater separation. In addition, TLC uses capillary action to move the solvent and compounds down the column. To speed up this process, one can force the solvent through the column using gas pressure, a technique called flash column chromatography. In column chromatography, the solvent polarity an also be changed to help elute the desired compounds. Eventually, the solvent drips out of the end of the column, and the different fractions that leave the column can be collected over time. Each fraction will contain different compounds. After collection,t he solvent can be evaporated, leaving behind the compounds of interest. Column chromatography is particularly useful in biochemistry because it can be used to separate and collect macromolecules such as proteins or nucleic acids.

Explain the process of thin layer chromatography (also the same process as paper chromatography).

The sample that we want to separate is placed directly onto the adsorbent itself; this is called spotting because we apply a small, well-defined spot of the sample directly onto the silica (thin layer) or paper (paper) plate. The plate is then developed, which involves placing the adsorbent upright in a developing chamber, usually a beaker with a lid or a wide-mouthed jar. At the bottom of this jar is a shallow pool of solvent, called the eluent. The spots of sample must be above the level of the solvent, or else they will dissolve into the pool of solvent rather than running up the plate. When set up correctly, the solvent will creep up the plate by capillary action, carrying the various compounds in the sample with it at varying rates. When the solvent front nears the top of the plate, the plate is removed from the chamber and allowed to dry. TLC is often done with silica gel, which is polar and hydrophilic. The mobile phase, on the other hand is usually an organic solvent of weak to moderate polarity so it doesnt bind well to the gel. Because of this, nonpolar compounds dissolve in the organic solvent and move quickly as the solvent moves up the plate, whereas the more polar molecules stick to the gel. Thus, the more nonpolar the sample is, the further up the plate it will move.

what is heat?

The thermal energy transferred between objects that are at different temperatures

One cup containing 100 grams of water at 300K is mixed into another cup containing 200g of water at 450K. What is the equilibrium temperature of the system? (Note: Assume that the pressure is sufficiently high to avoid boiling).

The two liquids undergo thermal exchange; thus, the heat given off by one liquid will be equal to the heat absorbed by the other. q(cold)=-q(hot) m(cold)c(H2O)deltaT(cold)=m(hot)c(H2O)[-deltaT(hot)] (100g)(1cal/gK)(Tf-300K)=(200g)(1cal/gK)(450K-Tf) Tf=400K

what is the electromotive force (EMF)? what does it mean when its positive? when its negative?

The voltage or electrical potential difference of the cell. If the emf is positive, the cell is able to release energy (deltaG < 0), which means it is spontaneous. If the emf is negative, the cell must absorb energy (deltaG > 0), which means it is nonspontentous. Always opposite from deltaG.

What is the electron configuration of calcium in CaCl2? A) 1s22s22p63s23p5 B) 1s22s22p63s23p6 C) 1s22s22p63s23p64s1 D) 1s22s22p63s23p64s2

This question asks the examinee to determine the electron configuration of the calcium ion, as it exists in calcium chloride. Calcium in its elemental state has a total of 20 electrons. Therefore Ca2+ has a total of 18 electrons. In the answer choices provided, choice B is the only electron configuration that contains 18 electrons making B the correct answer.

What is the atmospheric pressure at the top of Mount Everest, which stands approximately 9000 meters above sea level? (Assume an average air density = 0.8 kg/m3 and g = 10 m/s2) A. 7200 Pa B. 29000 Pa C. 72,000 Pa D. 101,000 Pa

This question asks the examinee to determine the pressure due to the atmosphere at the top of Mount Everest. To answer this question, you must determine the difference in air pressure between sea level (101,000 Pa) and the top of Everest. The best method to calculate the pressure differential is to apply the gauge pressure formula, Pgauge = ρgh, where Pgauge is the pressure due to the atmospheric column of height h, ρ is the density of the fluid, and g is the gravitational acceleration constant. The difference between the atmospheric pressure at sea level and the gauge pressure applying the equation above yields the atmospheric pressure at the top of Mt. Everest, as follows: PEverest = PSea Level - Pgauge PEverest = 101,000 Pa- ρgh PEverest = 101,000 Pa- (0.8 kg/m3)(10 m/s2)(9000 m) PEverest =29,000 Pa Thus, the correct answer is 29,000 Pa, which is answer choice B.

Which of the following accurately compares the final velocity (vf) and acceleration (af) of an object as it strikes the ground to the velocity (v) and acceleration (a) immediately after being launched from a platform 10 meters above than the striking surface? A. |af| > |a| and |vf| > |v| B. |af| < |a| and |vf| < |v| C. |af| = |a| and |vf| > |v| D. |af| = |a| and |vf| < |v|

This question requires the examinee to understand projectile motion characteristics throughout flight. At the time of launch, the projectile has kinetic energy and potential energy that is converted entirely into kinetic energy as the projectile strikes the surface. Thus the magnitude of the projectile's final velocity (vf) is greater than the initial velocity (v). Additionally, the acceleration of the object is equal to gravity and does not change during flight making choice C the correct answer.

The sound level can be determined using the equation, dB = 10log10(I/Io), where dB is the sound level measured in decibels, I is the intensity of the sound measured in W/m2, and Io (W/m2) is the threshold of hearing in a healthy human. How many orders of magnitude greater is the intensity of sound measured at 120 decibels compared to a sound of 40 decibels? A. 2 B. 8 C. 12 D. 80

This question requires the examinee to understand the logarithmic relationship between the sound level and intensity. As a general rule, an increase in 10 dB corresponds to an increase in intensity by a factor of 10. Therefore, the 80-decibel difference in sound levels in this problem corresponds to a difference in intensity by a factor of 108. dB = 10log10(I/Io) 120 = 10log10(I120/Io) 12 = log10(I120/Io) I120 = Io1012 dB = 10log10(I40/Io) 40 = 10log10(I40/Io) 4 = log10(I40/Io) I40 = Io104 I120/I40 = Io1012/Io104 I120/I40 = 108 Thus, the intensity of the sound at 120 decibels is eight orders of magnitude greater than sound at 40 decibels making B the correct answer.

Using the interquartile range, determine whether the 36 year old from Ray's party is a outlier. The ages are provided in numerical order: 20, 22, 22, 22, 23, 24, 25, 36

To determine if this point is an outliner, we must first determine the interquartile range. To do so, we must determine the first and third quartiles. This data set contains eight values. Multiplying 8 by 1/4 gives us 2, so the first quartile is the mean of the second and third values in the ordered data set: Q1=22+22/2 = 22 Multiplying 8 by 3/4 gives us 6, so the third quartile is the mean o fthe sixth and seventh values in the ordered data set: Q3=24+25/2=24.5 IQR=Q3-Q1=24.5-22=2.5 Outliers are data values more than 1.5 interquartile ranges below Q1 or above Q3. Thus, any value above 24.5 + 1.5 x 2.5 + 3.75 = 28.25 or below 22- 1.5 x 2.5 = 22-3.75=18.25 will be an outlier. So 36 is an outlier.

Explain real vs virtual lenses compared to mirrors.

To identify the real side (R), the real side is where light actually goes after interaction with the lens or mirror. For mirrors, light is reflected and therefore stays in front of the mirror. Hence, for a mirror, the real side is in front of the mirror, and the virtual side is behind the mirror. For lenses, the convention is different: because light travels through the lens and comes out on the other side, the real side is on the opposite side of the lens from the original light source, and the virtual side is on the same side of the lens as the original light source. Although the object of a single lens is on the virtual side, this does not make the object virtual. Objects are real, with a positive object distance, unless they are placed in certain multiple lens systems in which the image of one lens becomes the object for another (RARE).

What is fractional distillation and when is it used?

To separate two liquids with similar boiling points (less than 25C apart), we use fractional distillation. In this technique, a fractionalation column connects the distillation flask to the condenser. A fractionation column is a column in which the surface area is increased by the inclusion of inert objects like glass beads or steel wool. As the vapor rises up the column, it condenses on these surfaces and refluxes back down until rising heat causes it to evaporate again, only to condense again higher in the column. Each time the condensate evaporates, the vapor consists of a high er proportion of the compound with the lower P. By the time the top of the column is reached, only the desired product drips down to the receiving flask.

Triacylglycerides consist of I. a ribose backbone II. a glycerol backbone III. three phosphodiester linkages IV. three ester linkages a) I and III b) II only c) II and III d) II and IV

Triacylglycerides consist of a glycerol backbone bonded to three fatty acids. The bond between each acid and the glycerol is an ester group. Phosphodiester linkages are found in DNA. a) I and III, incorrect, Glycerol, not ribose, forms the backbone of triacylglycerides. b) II only, incorrect, There are three ester linkages in addition to the glycerol backbone. c) II and III, incorrect, Phosphodiester linkages are found in DNA not triacylglycerides. d) II and IV, correct. Phospholipids contain two ester linkages between the glycerol backbone and the two fatty acid tails, and one ester linkage between glycerol and the phosphate group.

Explain the SN1 reaction

Two steps. Unimolecular. 1st step is rate limiting step in which the LG leaves, generating a positively charged carbocation. The nucleophile then attacks the carbocation, resulting in the substitution product The more substituted the carbocation, the more stable it is b/c the alkyl groups act as e- donors, stabilizing the positive charge. B/c the formation of the carbocation is the rate limiting step, the rate of the reaction depends only on the [ ] of the substrate. This is a first-order reaction; anything that accelerates the formation of the carbocation will inc the rate. B/c SN1 reactions pass thorough a planar intermediate before the nucleophile attacks, the product will usually be a racemic mixture.

What mass of copper will be deposited in a Daniell cell if a current of 2A flows through the cell for 3 hours?

Use the equation mol M = It/nF Moles of Metal It is Not Fun (mnemonic to help remember) A Daniell cell uses a copper electrode in copper sulfate (CuSO4) solution. Because the oxidation state of copper in solution is +2, n=2. Now we can plug into the equation. molM =It/nF = (2A)(3hr)(3600s/hr)/(s mol e-)(96485C?mole-) = approx 3 x 3600/10^5 =0.1 mol Cu (actual 0.11 mol( Then, we must determine the actual mass of copper being deposited. 0.1 mol Cu should have a mass of 6.35 g because the molar mass of copper is 63.5 g/mol (actual 7.11g)

Balance this redox reaction MnO4- + I- --> I2 + Mn2+

Use the half-reaction method Step 1: Separate the two half reactions: I- --> I2 MnO4- --> Mn2+ Step 2: Balance the atoms of each half-reaction. First, balance all atoms except H and O. Next, in an acidic solution, add H2O to balance the O atoms and then add H+ to balance the H atoms. In a basic solution, use OH- and H2O to balance the O and H atoms. 2I- --> I2 MnO4- + 8H+ --> Mn2+ + 4H2O Step 3: Balance the charges of each half-reaction. Add electrons as necessary to one side of the reaction so that the charges are equal on both sides. 2I- --> I2 + 2e- MnO4- + 8H+ + 5e- ---> Mn2+ + 4H2O Step 4: Both half-reactions must have the same number of electrons so that they cancel each other out in the next step. In this example, you need to multiple the oxidation half reaction by 5 and the reduction half reaction by 2. 10I- --> 5I2 + 10e- 2MnO4- + 16H+ + 10e- --> 2Mn2+ + 8H2O Steph 5: Add the half-reactions, canceling out terms that appear on both sides of the reaction arrow. 2MnO4- + 16H+ + 10I- --> 2Mn2+ + 5I2 + 8H20 Step 6: Confirm that mass and charge are balanced. There is a +4 net charge on each side of the reaction equation, and the atoms are stoichiometrically balanced.

How does a hydraulic life allow an auto mechanic to raise a heavy car with far less force than the weight of the car?

Using Pascal's principle, P=F1/A1 =F2/A2 F2=F1(A2/A1) **Use pic drawn in physics section** We have a closed container that is filled with an incompressible liquid. On the left side of the life, there is piston of cross sectional area A1, when this piston is pushed down the column, it exerts a force with a magnitude equal to F1 and generates a pressure equal to P1. The piston displaces a volume of liquid equal to A1d1 (the cross sectional area times the distance gives a volume). Because the liquid inside is incompressible, the same volume of fluid must be displaced on the right side of the hydraulic life, where we find a second piston with a much larger surface area, A2. The pressure generated by piston 1 is transmitted undiminished to all points within the system, including A2. As A2 is larger than A1 by some factor, the magnitude of the force, F2, exerted against A2 must be greater than F1 by the same factor so that P1=P2, according to Pascal's principle.

What is the difference between thin layer chromatography and paper chromatography?

Very similar techniques, varying only in the medium used for the stationary phase. For thin-layer chromatography, a thin layer of silica gel or alumina adherent to an inert carrier sheet is used. For paper chromatography, as the name suggests, the medium used is paper, which is composed of cellulose.

A lead ball of mass 0.125 kg is thrown straight up in the air with an initial velocity of 30 m/s. Assuming no air resistance, find the work done by the force of gravity by the time the ball is at its max height.

W=Kf-Ki 0-1/2mv^2 =-56.25J the answer could be found using W=Fdcostheta but the other way is easier

What are transverse waves? Give examples.

Waves that move at right angles to the direction of the wave (perpendicular) the direction of particle oscillation is perpendicular to the propagation (movement) of the wave. Ex: electromagnetic waves (visible light), microwaves, x-rays

Explain what happens when an object sinks vs floats in a fluid using Archimedes' principle.

When an object is placed in a fluid, it will sink into the fluid only to the point at which the volume of displaced fluid exerts a force that is equal to the weight of the object. If the object is denser than the fluid it's in, it will sink to the bottom. If an object is less dense than the fluid it's in, it will float. These objects will submerge enough of their volume to displace a volume of water equal to the objects weight.

When light enters a medium with a higher index of refraction, what happens? What about when it enters a medium with a lower index of refraction?

When light enters a medium with a higher index of refraction, it bends toward the normal. When it enters a medium with a lower index of refraction, it bends away from the normal.

Explain the critical angle and the total internal reflection.

When light travels from a medium with a higher index of refraction (such as water) to a medium with a lower index of refraction (such as air), the refracted angle is larger than the incident angle (theta2>theta1); that is, the refracted light ray bends away from the normal. As the incident angle is increased, the refracted angle also increases, and eventually, a special incident angle called the critical angle (theta subc) is reached, for which the refracted angle (theat2)=90 degrees. At the critical angle, the refracted light ray passes along the interface between the two media. Total internal reflection, a phenomenon in which all the light incident on a boundary is reflected back into the original material, results with any angle of incidence greater than the critical angle. Total internal reflection occurs as the light moves from a medium with a higher refractive index to a medium with a lower one.

What are confounding variables?

a data analysis error. The data may or may not be flawed, but an incorrect relationship is characterized. For example, consider the statement: Having natural red hair leads to a decreased pain tolerance and higher opiate tolerance. There are two flaws with this statement. First, the statement implies a causal relationship as a result of what would almost certainly be an observational study. Second, this is not realistic. However, a third variable, such as a gene mutation, could potentially cause both parts of this statement. These third party variables are called confounding variables or confounders.

what is a dielectric material? give examples? what happens when one is introduced between the plates of a capacitor? Explain this and the dielectric constant.

a dielectric material is another way of saying insulation. When a dielectric material, such as air, glass, plastic, ceramic, or certain metal oxides, is introduced between the plates of a capacitor, it increases the capacitance by a factor called the dielectric constant (k). The dielectric constant of a material is a measure of its insulating ability, and a vacuum has a dielectric constant of 1, by definition. For reference, the dielectric constant of air is slightly above 1, glass is 4.7, and rubber is 7.

What is a blackbody?

a hypothetical object capable of absorbing all the electromagnetic radiation falling on it. can absorb all wavelengths of light, which would appear completely black if it were at a lower temp than its surroundings.

What is a cell diagram? What are the rules for cell diagrams?

a shorthand notation representing the reactions in an electrochemical cell. Ex: Zn(s) I Zn2+ + (1M) II Cu2+ (1M) I Cu (s) 1. The reactants and products are always listed from left to right in this form: anode I anode solution ([ ]) II cathode solution ([ ]) I cathode 2. A single vertical line indicates a phase boundary 3. A double vertical line indicates the presence of a salt bridge or some other type of barrier

Explain a concentration cell.

a special type of galvanic cell. like all galvanic cells, it contains two half-cells connected by a conductive material, allowing a spontaneous redox reaction to proceed. The distinguishing characteristic of a [ ] cell is its design: the electrodes are chemically identical. Ex, if both electrodes are copper metal, they have the same reduction potential. Therefore, current is generated as a function of a [ ] gradient established between the two solutions surrounding the electrodes. The [ ] gradient results in a potential difference between the two compartments and drives the movement of electrons in the direction that results in equilibrium of the ion gradient. The current will stop when the [ ]s of ionic species in the half-cells are equal. This implies that the voltage (V) or electromotive force of a [ ] cell is 0 when the [ ]s are equal; the voltage, as a function of [ ], can be calculated using the Nernst equation.

Explain lead-acid batterys (aka lead storage battery).

a specific type of rechargeable battery. As a voltaic cell, when fully charged, it consists of two half cells- a Pb anode and a porous PbO2 cathode, connected by a conductive material (concentrated 4M H2SO4). When fully discharged, it consists of two PbSO4 electroplated lead electrodes with a dilute [ ] of H2SO4. Both half reactions cause the electrodes to plate with lead sulfate (PbSO4) and dilute the acid electrolyte when discharging. The lead anode is negatively charged and attracts the anionic bisulfate. The lead (IV) oxide cathode is a bit more complicated. This electrode is porous, which allows the electrolyte (sulfuric acid) to solvate the cathode into lead and oxide ions. The, the hydrogen ions in solution react with the oxide ions to produce water, and the remaining sulfate ions react with the lead to produce the electroplated lead sulfate. When charging, the lead-acid cell is part of an electrolytic circuit. These equations and electrode charge designations are the opposite because an external source reverses the electroplating process and concentrates the acid solution-this external source is very evident when one uses jumper cables to restart a car. Lead-acid batteries, as compared to other cells, have some of the lowest energy-to-weight ratios (otherwise known as energy density). Energy density is a measure of a battery's ability to produce power as a function of its weight. Therefore, lead-acid batteries require a heavier amount of battery material to produce a certain output as compared to other batteries.

What is disproportionation (dismutation)?

a specific type of redox reaction in which an element undergoes both oxidation and reduction in producing its products. Many biological enzymes utilize a disproportionation mechanism. 2H2O2(aq) ---> 2H2O(l) + O2(g)

Explain a system vs surroundings.

a system is the matter that is being observed- the total amount of reactants and products in a chemical reaction. It could be the amount of solute and solvent used to create a solution. It could be the gas inside a balloon. the surroundings, or environment, are everything outside of that system. However, the boundary between system and surroundings is not permanently fixed and can be moved. Ex: one might consider the mass of coffee in a coffee cup to be the system and the cup containing it to be part of the environment. Alternatively, one might define the system as the hot coffee and the cup together, and the environment as the air surrounding the coffee cup. The boundary can be extended our farther and farther.

What is isoelectric focusing?

a technique used to separate amino acids or polypeptides based on their isoelectric points (pI). The positively charged amino acids (protonated at the solution's pH) will migrate toward the cathode; negatively charged amino acids (deprotonated at the solution's pH) will migrate toward the anode.

what is the hawthorne effect?

aka observation bias. posits that the behavior of study participants is altered because they recognize that they are being studied.

what is the Doppler effect? give a real life example

ambulances! it is quickly approaching from the other land and as it passes one can hear a distinct drop in the pitch of the siren. Doppler effect describes the difference between the actual frequency of a sound and its perceived frequency when the source of the sound and the sound's detector are moving relative to one another. If the source and detector are moving toward each other, the perceived frequency, f', is greater than the actual frequency,f. If the source and the detector are moving away from each other, the perceived frequency is less than the actual frequency. This can be seen from the Doppler effect equation: f'=f(v+/- VsubD)/(V-/+ VsubS)

Rank anhydrides, amides, carboxylic acids, and esters in terms of reactivity. Explain.

anhydrides are the most reactive, followed by esters (essentially tied with carboxylic acids), followed by amides. This can be explained by the structure of these molecules. Anhydriges, with their resonance stabilization and three electron withdrawing oxygen atoms, are the most electrophilic. Esters, by comparison, lack one electron-withdrawing carbonyl oxygen and are slightly less reactive, Finally, amides, with an electron-donating amino group,a re the least reactive toward nucleophiles.

Explain gas chromatography

another method that can be used for qualitative separation. GC, also known as vapor-phase chromatography (VPC) is similar to the other types of chromatography. The main conceptual difference is that the eluent is a gas (usually helium or nitrogen) instead of a liquid. The adsorbent is a crushed metal or polymer inside a 30-foot column. This oclumn is coiled and kept inside an oven to control its temperature. The mixture is then injected into the column and vaporized. The gaseous compounds travel through the column at different rates because they adhere to the adsorbent in the column to different degrees and will separate in space by the time they reach the end of the volumn. The injected compounds must be volatile: low melting point, sublimable solids or vaporizable liquids. The compounds are registered by a detector, which records them as a peak on a chart. It is common to separate molecules using GC and then to inject the pure molecules into a mass spectrometer for molecular weight determination

What is Kirchhoff's Loop Rule?

around any closed circuit loop, the sum of voltage sources will always be equal to the sum of voltage (potential) drops. This is a consequence of the conservation of energy. All the electrical energy supplied by a source gets fully used up by the other elements within that loop. No excess energy appears, and no energy disappears that cannot be accounted for. Of course, energy can be changed from one form to another, so the kinetic energy of the electrons can be converted to thermal energy, light, or sound by the particular apparatus that is connected to the circuit. This law is in terms of voltage (J/C) Vsource=Vdrop

explain binary vs continuous or categorical variables.

binary (yes vs no, better vs worse), continuous (amount of weight lost, % improvement in cardiac output), categorical variables (state of residence, socioeconomic status)

What are the 3 important characteristics of a covalent bond. Explain them

bond length- the avg distance between the 2 nuclei of atoms in a bond *as the # of shard electron pairs increases, the two atoms are pulled closer together, resulting in a decrease in bond length. * single bond > double bond > triple bond bond energy- the energy required to break a bond by separating its components into their isolated, gaseous atomic states * the greater the # of pairs of electrons shared between the atomic nuclei, the more energy is required to break the bonds holding the atom together * triple bond > double bond > single bond polarity- occurs when two atoms have a relative difference in electronegativities. *when these atoms come together in covalent bonds, they must negotiate the degree to which the electron pairs will be shared. The atom with the higher electronegativity gets the larger share of the electron density. A polar bond creates a dipole, with the positive end of the dipole at the less electroneg atom and vice versa. TWO TYPES: *Nonpolar covalent bond- when atoms that have identical or close electronegativities share electron pairs, they do so with equal electron distribution *Polar covalent bond- atoms differ in their electronegativities. difference in electronegs is not enough to form an ionic bond, so it causes a separation of charge across the bond. The more electroneg element has a greater portion of the electron density and has a partial negative charge. vice versa.

How is sound produced?

by the mechanical disturbance of particles in a material along the sound wave's direction of propagation. Although the particles themselves do not travel along with the wave, they do vibrate or oscillate about an equilibrium position, which causes small regions of compression to alternate with small regions of rarefaction (decompression). These alternating regions of increased and decreased particle density travel through the material, allowing the sound wave to propagate sound involves vibration of material particles, so the source of any sound is ultimately a mechanical vibration of some frequency.

Rank the following in order of decreasing oxidation state: amine, carboxylic acid, aldehyde, alkane

carboxylic acid, aldehyde, amine, alkane Carboxylic acids are the second most oxidized form of carbon (only CO2 is more oxidized). In carb acids, the carbon atom has 3 bonds to O. In aldehydes, the C atom has 2 bonds to O. In amines, the C atom has 1 bond to N. In an alkane, the C only has bonds to other Cs and Hs

What is the law of conservation of charge?

charge can be neither created nor destroyed.

torques that generate clockwise rotation are considered? counterclockwise?

clockwise- negative counterclockwise-positive

The circulatory system is a ________ loop that has a _______ flow rate. Explain. This flow rate can be felt and measured how?

closed loop with a nonconstant flow rate. The nonconstant flow is a result of valves, gravity, the physical properties of our vessels (elasticity, in particular), and the mechanics of the heart. Can be felt and measured as a pulse!

explain constant pressure calorimeters vs constant volume calorimeters

constant pressure calorimeter (coffee cup calorimeter) coffee cup covered with a lid filled with a solution in which a reaction or some physical process is occurring. The pressure, which is atm pressure, is constant and the temp can be measured as the reaction progresses. constant volume calorimeter (bomb calorimeter) decomposition vessel. a sample of matter, typically a hydrocarbon is placed in the steel decomposition vessel, which is then filled with almost pure oxygen gas. The vessel is then placed in an insulated container holding a known mass of water. The contents of the composition vessel are ignited by an electric ignition mechanism. The material combusts (burns) in the presence of oxygen, and the heat that evolves is the heat of the combustion reaction. Because W=PdeltaV, no work is done in an isovolumetric proves so Wcalorimeter=0. B/c of the insulation, the whole calorimeter can be isolated from the rest of the universe, so the system is the sample plus the oxygen and the steel vessel, the surroundings are the water.

What amount of energy is required to change a 90 gram ice cube at -10C to vapor at 110C? Note: C(H2O, l)=4.18J/gK C(H2O, s)=2.18J/gK C(H2O, g)=2.00J/gK deltaH(fus)=6.02kJ/mol deltaH(vap)=40.67kJ/mol

convert mass to moles [90g H2O/(18g/mol)]=10/2 = 5 mol H2O because we are beginning in the ice phase, we must heat the ice cube to the solid-liquid phase transition, which occurs at OC. this first step involves a change in temp, so we must use the heat formula that contains deltaT and all the pertinent variables for ice. q1=m(ice)c(ice)deltaT(1) q1=(90g)(2.18J/gK)(10K)=1.8kJ in step 2, we must convert the ice into liquid form. During this phase change, there will be no temp change. q2=mL=n(H2O)deltaH(fus) q2=(5 mol)(6.02kJ/mol)=30kJ In step 3, we heat the water to its liquid-gas phase transition tempaerature at 100C q3=m(water)c(water)deltaT(3) q3=(90g)(4.18J/gK)(100K)=36kJ In step 4, we vaporize the water. q4=mL=n(H2O)deltaH(vap) q4=(5 mol)(40.67kJ/mol)=200kJ In step 5, we must heat the water to the target temp of 110C. q5=m(steam)c(steam)deltaT(s) q5=(90g)(2.00J/gK)(10K)=1.8kJ Add them all qtot=274.8kJ

400g of AlCl3 is dissolved in 1.5L of water at room temp (Kf=1.86K.kg/mol). What is the new freezing point of this solution?

deltaTf=iKfm=4(1.86)(2m)=15 The normal freezing point of water is 273K. The freezing point is then going to be depressed by 15K. The new freezing point is therefore 273-15=258K=-15C.

What is Hill's criteria? What are his 9 criteria?

describe the components of an observed relationship that increase the likelihood of causality in the relationship. While only the first criterion is necessary for the relationship to be casual, it is not sufficient. The more criteria that are satisfied by a relationship, the likelier it is that the relationship is causal. Hill's criteria do not provide an absolute guideline on whether a relationship is causal; thus, for any observational study, the relationship could be described as a correlation. Temporality-the exposure (independent variable) must occur before the outcome (dependent variable) Strength- as more variability in the outcome variable is explained by variability in the study variable, the relationship is more likely to be causal Dose-response relationship- As the study or independent variable increases, there is a proportional increase in the response. The more consistent this relationship, the more likely it is to be causal. Consistency-The relationship is found to be similar in multiple settings Plausibility- There is a reasonable mechanism for the independent variable to impact the dependent variable supported by existing literature Consideration of alternative explanations- If all other plausible explanations have been eliminated, the remaining explanation is more likely Experiment- If an experiment can be performed, a causal relationship can be determined conclusively Specificity- The change in the outcome variable is only produced by an associated change in the independent variable Coherence- The new data and hypothesis are consistent with the current state of scientific knowledge.

what are the two patterns of current flow? explain the difference. give examples of both.

direct current (DC)- charge flows in one direction only -Ex: household batteries alternating current (AC)-the flow changes direction periodically. -Ex: current supplied over long distances to homes and other buildings

what does the length of a resistor do to it's resistance?

directly proportional. a longer resistor means that electrons will have to travel a greater distance through a resistant material. This factor scales linearly: if a resistor doubles its length, it will also double its resistance.

some materials allow free flow of electric charge within them; these materials are called what? why are metals good electrical and thermal conductors? how is the metallic bond visualized and described?

electrical conductors metal atoms can easily lose one or more of their outer electrons, which are then free to move around in the larger collection of metal atoms. the metallic bond has often been visualized as a sea of electrons flowing over and past a rigid lattice of metal cations. Metallic bonding is more accurately described as an equal distribution of the charge density of free electrons across all of the neutral atoms within the metallic mass.

according the the Aufbau principle...

electrons fill from lower to higher energy subshells and each sub shell will fill completely before electrons begin to enter the next one

The energy of an electron ______ (becomes _____ negative), the farther out from the nucleus it is located (_________ n)

energy of an e- inc (becomes less negative) the farther out from the nucleus it is (increasing n)

What is Hund's rule?

every orbital in a subshell is singly occupied with one electron before any one orbital is doubly occupied, and all electrons in singly occupied orbitals have the same spin. What's the big takeaway that results in lots of testing on the MCAT? half and fully filled orbitals are more stable than other Staes. Chromium(and other elements in its group) and copper(and other elements in its group) will have one electron moved from the highest s sub shell to the highest d subshell to create a Half or fully filled orbital creates extra stability. Note: this is never observed for the p shell

what is current?

flow of positive charge. this is due to tradition, negative charges are the only charges that are actually moving.

Explain turbulent flow. what does is form? how does it arise?

flow which is rough and disorderly. causes the formation of eddies, which are swirls of fluid of varying sizes occurring typically on the downstream side of an obstacle arises when the speed of the fluid exceeds a certain critical speed, which depends on the physical properties of the fluid. when critical speed is exceeded, the fluid demonstrates complex flow patterns, and laminar flow occurs only in the thin layer of fluid adjacent to the wall, called the boundary layer. The flow speed immediately at the wall is zero and inc uniformly throughout the layer. Beyond the boundary layer, however, the motion is highly irregular and turbulent. A significant amount of energy is dissipated from the system as a result of the increased frictional forces.

What is Pascal's Law?

fluid mechanics principle. A pressure change occurring anywhere in a confined incompressible fluid is transmitted throughout the fluid such that the same change occurs everywhere Ex: an unopened carton of milk could be considered an incompressible fluid in a closed container. If one were to squeeze the container, exerting an inc pressure on the sides of the milk carton, the applied pressure would be transmitted through the entire volume of milk. If the cap were to suddenly pop off, the resulting geyser of milk would be evidence of this increased pressure

How does temperature affect the rate of a reaction?

for nearly all reactions, the reaction rate will increase and the temperature increases. b/c the temp of a substance is a measure of the particles' avg KE, inc the temp increases the avg KE of the molecules.

For galvanic cells, the electron with the most _________ reduction potential is the cathode and the most ___________ reduction potential is the anode. Why? How about for electrolytic cells? Why?

galvanic: more positive Ered=cathode. more negative Ered=anode. Because the species with a stronger tendency to gain electrons (that wants to gain electrons more) is actually doing so, the reaction is spontaneous and delta G is negative. Electrolytic cells: the electrode with the more positive reduction potential is forced by the external voltage source to be oxidized and is, therefore, the anode. The electrode with the less positive reduction potential is forced to be reduced and is the cathode. Because the movement of electrons is in the direction against the tendency or desires of the respective electrochemical species, the reaction is nonspontaneous and deltaG is positive

What are the four fundamental forces of nature?

gravity, electromagnetic force, strong nuclear force, weak nuclear force

When do you expect the most deviations from the ideal gas law?

high pressure, low volume, or low temp high volume, low pressure, and high temp deviations are usually small and good approximations can still be made from the ideal gas law

What is the difference between a homopolysaccharide and a heteropolysaccharide?

homopolysaccharide is a polysaccharide composed entirely of glucose. hetero is when a polymer is made up of more than one type of monosaccharide

What is isotopic notation?

how elements are written, preceded by their atomic number as a subscript and mass number as a superscript (a/zX). The atomic number (z) correspons to the number of protons in the nucleus. The mass number (a) corresponds to the number of protons plus neutrons.

what is the range of human hearing? what are infrasonic and ultrasonic waves?

human hearing from 20Hz-20000Hz infrasonic waves are sound waves with frequencies below 20 Hz ultrasonic waves have frequencies above 20000 Hz

Explain image distance

if an image has a positive distance ( i>0), it is a real image, which implies that the image is in front of the mirror. If the image has a negative distance (i<0), it is virtual and thus located behind the mirror.

Extraction requires two solvents that are _________ in order to separate the products.

immiscible

Explain the electrode charge designations in a galvanic cell vs an electrolytic cell? Explain. In all cells, anions are attracted to the __________ and cations are attracted to the ______.

in a galvanic cell the anode is negative and the cathode is positive. In an electrolytic cell, the anode is positive and the cathode is negative. This is because an external source is used to reverse the charge of an electrolytic cell. However, in both types of cells, reduction occurs at the cathode and oxidation occurs at the anode. Anions attracted to the Anode. Cations attracted to the Cathode.

What are shock waves and how are they produced? What happens when one passes thorough another object?

in a special case of the Doppler effect, an object that is producing sound while traveling at or above the speed of sound allows wave fronts to build upon one another at the front of the object. This creates a much larger amplitude at that point. Because the amplitude for sound waves is related to the degree of compression of the medium, this creates a large pressure differential or pressure gradient. This highly condensed wave front is called a shock wave, and it can cause physical disturbances as it passes through other objects. The passing of a shock wave creates very high pressure, followed by very low pressure, which is responsible for the phenomenon known as a sonic boom. Unlike its depiction in movies and television, a sonic boom can be heart any time that an object traveling at or faster than the speed of sound passes a detector, not just at the point that the speed of sound is exceeded (Mach 1). Once an object moves faster than the speed of sound, some of the effects of the shock wave are mitigated because all of the wave fronts will trail behind the object, destructively interfering with each other.

Protons are found where? The have what charge? What is their mass?

in the nucleus of an atom. +1. 1 atomic mass unit

As one moves down the elements of a given group, what happens to the principal quantum number? what does this mean?

increases, by one each time means that the valence electrons are increasingly separated from the nucleus by a greater number of filled principal energy levels, which can also be called inner shells. The result of this inc separation is a reduction in the electrostatic attraction between the valence electrons and the positively charged nucleus. These outermost electrons are held less tightly as the principal quantum number increases. As one goes down a group, the inc shielding created by the inner shell electrons cancels the increased positivity of the nucleus. Thus, the Zeff is more or less constant among the elements within a given group.

how are intensity and the distance for the source of the sound wave related?

intensity is inversely proportional to the square of the distance from the source. Ex: sound waves that have traveled 2 meters from the source have spread their energy out of are a surface area that is four times larger than that for identical sound waves that have traveled 1 meter from their source.

What is the intensity of sound? What about the loudness or volume?

intensity is the average rate of energy transfer per area across a surface that is perpendicular to the wave loudness or volume of a sound is the way in which we perceive its intensity. It is subjective, and depends not only on brain function, but also physical factors such as obstruction of the ear canal, etc.

how does the cross sectional area of the resistor affect it's resistance?

inversely. if a resistor's cross-sectional area is doubled, tis resistance will be cut in half. This is because the inc int he cross-sectional area inc the number of pathways through the resistor, called conduction pathways. The wider the resistor, the more current that can flow, analogous to a river.

What are enantiomers?

isomers that are nonsuperimopsable mirror images of each other. They have the same connectivity but opposite configurations at every chiral center in the molecule - enantiomers have identical physical properties with only 2 exceptions: optical activity and reactions in chiral environments

When capacitors are connected in series, what happens to the total capacitance?

it decreases in similar fashion to the decreases in resistance seen in parallel resistors. This is because the capacitors must share the voltage drop int he loop and therefore cannot store as much charge. Functionally, a group of capacitors in series acts like one equivalent capacitor with a much larger distance between its plates. This inc in distance means a smaller capacitance.

What is an equipotential line? What does it look like? How much work is done in moving a test charge from one point on an equipotential line to another point on the same line?

it is a line on which the potential at every point is the same. On paper, equipotential lines may look like concentric circles surrounding the source charge. In 3-D space, these lines would be spheres surrounding the charge. No work is done

What is the Heisenberg Uncertainty Principle?

it is impossible to simultaneously determine, with perfect accuracy, the momentum and the position of an electron

What is important about the perpendicular bisector of the dipole. What is it?

it is the equipotential line that lies halfway between the two source charges, +q and -q. Because the angle between this plane along the dipole axis is 90 (cos90=0), the electric potential at any point along this plane is 0.

Why would a spontaneous reaction not take place?

it may have a very high activation energy ex: when was the last time you saw a match ignite itself? j

Explain the Arrhenius equation, including the relationships between the variables and the exponent rules that govern the reaction. How can the frequency factor be increased?

k=Ae^(-Ea/RT) k-rate constant A-frequency factor(attempt frequency; a measure of how often molecules in a certain reaction collide, unit s^-1) Ea-activation energy R-ideal gas constant T-temp in kelvin low activation energy and high temps make the negative exponent of the Arrhenius equation smaller in magnitude and thus increase the rate constant, k. as A increases, k also increases the frequency factor can be increased by increasing the number of molecules in a vessel. More molecules = more opportunities for collision.

What is the difference between kinetic and potential energy?

kinetic energy is the energy of motion potential energy is associated with an object's given position in space or other intrinsic qualities of the system

Explain the differences between kinetic and thermodynamic products.

kinetic products are higher in free energy than thermodynamic products and can form at lower temps. These are sometimes termed "fast" products because they can form more quickly under such conditions thermodynamic products are lower in free energy than kinetic products and are therefore more stable. Despite proceeding more slowly than the kinetic pathway, the thermodynamic pathway is more spontaneous (more negative delta G)

What is the azimuthal (angular momentum) quantum number? Why is it so important ? How does the principal quantum number limit this number?

l. refers to the shape and number of sub shells within a given principal energy level (shell) it's v important because it has implications for chemical bonding and bond angles for any given value of n, the range of possible values for l is 0 to (n-1). Ex: n=1, l=0. n=2, l=0, 1 (the n value tells you the number of sub shells) n=1, 1 subshell (l=0). n=2, 2 sub shells (l=0, 1)

What are cyclic amides called? How are they named?

lactams replace "-oic acid" with "-lactam" may also be named by which carbon (beta, gamma, sigma, etc) is used to form bond with N to complete the ring . Just count how many carbons away the bond is to N from other side of carbonyl

what is plane polarized (linear polarized) light? Explain it vs unpolarized light? How will this show up on the MCAT? Explain polarizers.

light in which the electric fields of all the waves are oriented in the same direction (that is, their electric field vectors are parallel). If follows that their magnetic fields vectors are also parallel, but convention dictates that the plane of the electric field identifies the plane of polarization. Unpolarized light has a random orientation of its electric field vectors; sunlight and light emitted from a light bulb are prime examples. One of the most common applications on the MCAT is the classification of stereoisomers. The optical activity of a compound, due to the presence of chiral centers, causes plane-polarized light to rotate clockwise or counterclockwise by a given number of degrees relative to its [ ] (its specific rotation). Remember that enantiomers, as nonsuperimposable images, will have opposite specific rotations. There are filters called polarizers, often used in cameras and sunglasses, which allow only light with an electric field pointing in a particular direction to pass through. If one passes a beam of light through a polarizer, it will only let through that portion of the light parallel to the axis of the polarizer. If a second polarizer is then held up to the first, the angle between the polarizers' axes will determine how much light passes through. When the polarizers are aligned, all the light that passes through the first polarizer passes through the second. When the second polarizer is turned so that its axis is perpendicular, no light gets through at all.

what are ohmmeters and how do they work

measures current in a circuit. Unlike ammeters and voltmeters, they do not require a circuit to be active. ohmmeters will often have their own battery of known voltage and then function as ammeters through another point in the circuit. Because only one circuit element is being analyzed, Ohm's law can be used to calculate resistance by knowing the ohmmeter's voltage and the current created through another point in the circuit.

what are voltmeters and how do they work?

measures current in a circuit. like ammeters, they require a circuit to be active. Voltmeters also use magnetic properties of current-carrying wires. However, voltmeters are used to measure the voltage drop across two points in a circuit. They are wired in parallel to these two points. Because the goal with any meter is to minimize its impact on the rest of the circuit, and voltmeters are wired in parallel, an ideal voltmeter has infinite resistance

In general, metals _____ electrons and become ____, while nonmetals ____ electrons and become ____. Metalloids do that?

metals lose e-s, become +, nonmetals gain e-s, become - metalloids can go in either direction, but tend to follow the trend based on which side of the metalloids line they fall on (Si behaves like a nonmetal, Ge behaves like a metal)

how does temperature effect resistance?

most conductors have greater resistance at higher temperatures. this is due to increase thermal oscillation of the atoms in the conductive material, which produces a greater resistance to electron flow.

What are diffraction gratings?

multiple slits arranged in patters. they can create colorful patterns similar to a prism as the different wavelengths interfere in characteristic patters. Ex: organization of the grooves on a DV act like a diffraction grating, crating an iridescent rainbow pattern on the surface of the disc.

If deltaG is negative, the reaction is positive? zero?

negative-spontaneous positive-nonspontaneous zero-equilibrium

What is the Pauli Exclusion Principle?

no two electrons in the same atom can have the same set of four quantum numbers

is sound transmitted undiminished, even after the decrease in intensity associated with distance? Why?

no. this is a result of damping, or attenuation. Oscillations are a form of repeated linear motion, so sound is subject to the same nonconservative forces as any other system, including friction, air resistance, and viscous drag. The presence of a nonconservative force causes the system to decrease in amplitude during each oscillation. Because amplitude, intensity, and sound level (loudness) are related, there is a corresponding gradual loss of sound. Note that damping does not have an effet on the frequency of the wave, so the pitch will not change.

what is the second law of thermodynamics?

objects in thermal contact and not in thermal equilibrium will exchange heat energy such that the object with a higher temp will give off heat energy to the object with the lower temp until both objects have the same temp at thermal equilibrium energy spontaneously disperses from being localized to become spread out if it is not hindered from doing so

Explain cross-sectional studies.

one of 3 types of observational studies in medicine. attempt to categorize patients into different groups at a single point in time. For example, a study to determine the prevalence of lung cancer in smokers and nonsmokers at a given point in time

Explain case-control studies

one of 3 types of observational studies in medicine. start by identifying the number of subjects with or without a particular outcome and then look backwards to assess how many subjects in each group had exposure to a particular risk factor. Ex: a study in which 100 patients with lung cancer and 100 patients without lung cancer are assessed for their smoking history

explain cohort studies.

one of 3 types of observational studies in medicine. those which the subjects ar sorted into groups based on differences in risk factors (exposures), and then assessed at various intervals to determine how many subjects in each group had a certain outcome. For example, a study in which 100 smokers and 100 nonsmokers are followed for 20 years while counting the number of subjects who develop lung cancer in each group.

Opposite charges exert ______ forces. Like charges exert ______ forces.

opposite charges exert attractive forces. Like charges exert repulsive forces

Explain electrolytic cells. Compare to galvanic cells.

opposite of galvanic cells. (Except for the commonality in that all electrolytic cells have reduction at the cathode and oxidation at the anode). Whereas galvanic cells house spontaneous redox reactions that generate electrical energy, electrolytic cells house nonspontaneous reactions that require the input of energy to proceed. Therefore, the change in free energy for an electrolytic cell is positive. This type of redox reaction driven by an external voltage source is called electrolysis, in which chemical compounds are decomposed.

what is pitch? what sounds have lower pitch and vice versa?

our perception of the frequency of sound. lower frequency sounds have lower pitch and vice versa.

For all electrochemical cells, the electrode where oxidation occurs is called the _________ and the electrode where reduction occurs is called the _______.

oxidation- anode reduction- cathode AN OX RED CAT

What are oxidation and reduction?

oxidation- refers to an increase in oxidation state (loss of electrons) (view as inc number of bonds to atoms besidesC and H) reduction- decrease in oxidation state. (view as inc the # of bonds to H)

explain oxidizing and reducing agents

oxidizing agents cause another atom in a redox reaction to undergo oxidation and is itself is reduced. A reducing agent causes the other atoms to be reduce and is itself oxidized.

If [H+]=0.001 or 10^-3. What are pH and pOH?

pH=3 pOH=11

If Kb=1.0 x 10^-12, what is pKb?

pKb=12

Suppose a proton is moving with a velocity of 15 m/s toward the top of the page through a uniform magnetic field of 3.0T directed into the page. What is the magnitude and direction of the magnetic force on the proton? Describe the motion that will result from this setup. (Note: the charge of a proton is 1.6E-19C and its mass is 1.67E-27kg).

pg 178 physics and Math Start by determining the magnitude of the force: FsubB=qvBsintheta (-1.6E-19C)(15m/s)(3.0T)(sin90) =7.2E-18N To determine the direction, use the right-hand rule. Your thumb should point up the page in the direction of v. Your fingers should point into the page in the direction of B. Protons are positively charged; thus the force, FsubB, is in the direction of your palm, which is to the left. Note that v and FsubB will always be perpendicular to each other; this implies that uniform circular motion will occur in this field, with FsubB pointing radially inward toward the center of the circle. If the centripetal force is the magnetic force, then we can set these two equations equal to each other Fsubc=FsubB [(mv^2)/r]=qvBsintheta mv=qBrsin90 =5.2E-8m

A train traveling south at 216km/hr is sounding its whistle while passing by a stationary observer. The whistle emits sound at a frequency of 1400Hz. What is the frequency heard by the stationary observer when the train is moving toward the observer, and when the train has passed the observer? (Note: the speed of sound in air is approx. 340m/s).

pg 246 PHYSICS To solve this problem, convert the speed of the train (VsubS) to m/s: 216km/hr=60m/s when the train is moving toward the stationary observer, the top sign sould be used in the denominator. The numerator is simply v because VsubD=0. This gives: f'=f(v)/(v-VsubS) =(1400Hz)(340m/s /340-60)=1700Hz When the train is moving away from the observer, the sign in the denominator changes. The numerator remains unchanged because the observer is still stationary: f'=v(v+vsubS) = (1400Hz)(340+60)= 1190Hz

A positively charged atom is called a _______ a negatively charged atom is called a _______.

pos- cation neg- anion

A clinical trial is devised to quantify the effectiveness of a retinal scanning system for the detection of Alzheimer's disease. The trial will include a positive control group, a negative control group, and an experimental group. The experimental group is a large sample of 70 year olds showing symptoms of Alzheimer's disease. Assuming the retinal scanning system is effective at detecting Alzheimer's disease, rank the groups in decreasing order of percentage of expected disease detection.

positive control group and negative control group establish the upper and lower bounds of percent detection, respectively. The experimental group is likely to fall between those two extremes. Positive, experimental, negative.

What is the difference between positive and negative controls in an experiment? What do negative controls used to assess for?

positive controls ensure a change in the dependent variable when it is expected. In the development of a new assay for detection of HIV, for example, administering the test to a group of blood samples known to contain HIV could constitute a positive control. negative controls ensure no change in the dependent variable when no change is expected. With the same assay, administering the test to a group samples known not to contain the HIV virus could constitute a negative control. In drug trials, a negative control group is often used to assess for the placebo effect- an observed or reported change when an individual is given a sugar pill or sham intervention.

a positive deltaH corresponds to a _________ process a negative deltaH corresponds to a __________ process

positive=endothermic negative=exothermic

Explain circular polarization.

rarely seen natural phenomenon that results form the interaction of light with certain pigments or highly specialized filters. Circularly polarized light has a uniform amplitude but a continuously changing direction, which causes a helical orientation in the propagating wave. The helix has average electric field vectors and magnetic field vectors that lie perpendicular to one another, like other waves, with maxima that fall on the outer border of the helix.

Explain nickel-cadmium batteries?

rechargeable cells. they consist of two half cells made of solid cadmium (the anode) and nickel(II) oxide-hydroxide (the cathode) connected by a conductive material, typically potassium hydroxide (KOH). Both half reactions cause the electrodes to plate with their respective products. Charging reverses the electrolytic cell potentials. Some Ni-Cd designs are vented for this reason to allow for the release of built up hydrogen and oxygen gas during electrolysis. Ni-Cd batteries have a higher energy density than lead-acid batteries. The electrochemistry of the Ni-Cd half reactions tends to provide higher surge current. This is preferable in appliances such as remote controls that demand rapid responses. It is important to note that modern Ni-Cd batteries have largely been replaced by more efficient nickel-metal hydride (MiMH)batteries. These newer batteries have more energy density, are more cost effective, and are significantly less toxic. In lieu of a pure metal anode a metal hydride is used instead.

what is optical activity? when is a solution racemic

refers to the rotation of this plane-polarized light by a chiral molecule. At the molecular level, one enantiomer will rotate the plane-polarized light to the same magnitude but in the opposite direction of its mirror image - a compound that rotates the plane of polarized light to the right, clockwise, is dextrorotatory (d-) and is labeled (+) - a compound that rotates light to the left, counterclockwise, is levorotatory (l-) and is labeled (-) - The direction of rotation cannot be determined from the structure of the molecule and must be determined experimentally -when both + and - enantiomers are present in equal concentrations, they form a racemic mixture. in these solutions, the rotations cancel each other out, and no optical activity is observed.

What is detection bias?

results from educated professionals using their knowledge in an inconsistent way. Because prior studies have indicated that there is a correlation between two variables, finding one of them increases the likelihood that the researcher will search for the second.

explain metalloids

separating the metals and nonmetals are a stair-step group of elements called the metalloids. The metalloids are also called semimetals b/c they share some characteristics with metals and nonmetals - physical properties (densities, melting points, etc) vary wildly

What are sigma and pi bonds? The more bonds that are formed between atoms, the ____ the overall bond length. Therefore, a double bond is ______ than a single bond, and a triple bond is _____ than a double bond.

sigma bonds happen when a molecular orbital is formed by head-to-head or tail-to-tail overlap. -All single bonds are sigma bonds pi bonds happen when 2 p-orbitals line up in a parallel (side-by-side) fashion, their electron clouds overlap -one pi bond on top of a sigma bond is a double bond -two pi bonds and 1 sigma bond form a triple bond -a pi bond cannot exist independently of a sigma bond. Only after the formation of a sigma bond will the p-orbitals of adjacent carbons be parallel and in position to form the pi bond. shorter the overall bond length. double bond shorter than single bond. triple bond shorter than a double bond

The molar solubility of Fe(OH)3 in an aqueous solution was determined to be 4E-10mol/L. What is the value of the Ksp for Fe(OH)3 at the same temp and pressure?

solution on pg 318 general chemistry Fe(OH)3 (s) <--> Fe3+ (aq) + 3 OH- (aq) Ksp= [Fe3+][OH-]^3 Ksp=[x][3x]^3 Ksp= (4E-10M)(3x4x10E-10M)^3 = [4E-10][3^3 x (4E-10)^3] Ksp=3^3 (4E-10)^4 = 27(4E-10)^4= 27 x 256 x 10^-40 = 30 x 250 x 10^-40 =75000x10^-40 = 7.5E-37 (actual 6.9E-37)

What are the [ ]s of each of the ions in a saturated solution of CuBr, given that the Ksp of CuBr is 6.27E-9 at 25C? If 3 g CuBr are dissolved in water to make 1L of solution at 25C, would the solution be saturated, unsaturated, or supersaturated?

solution on pg 318 general chemistry The first step is to write out the dissociation reaction" CuBr (s) --> Cu+ (aq) + Br- (aq) Ksp=[Cu+][Br-] Let x equal the molar solubility of CuBr, which is the amount of CuBr, which is the amount of CuBr that dissolves at equilibrium. The [ ] of Cu+ and Br- will each equal x. Ksp= x^2 6.27E-9=x^2 6.3E-9=x^2 63E-10 = x^2 = 64E-10 x=8E-5 Therefore, [Cu+] is about 8E-5 M and [Br-] is also about 8E-5M. Note that 8E-5 M also represents the molar solubility of copper(I) bromide. Next, convert 3 g of CuBr into moles: 3g (1mol CuBr/143.5g) = 3/150 = 3/1.5E-2 = 2E-2 mol 2E-2 mol CuBr in 1L of solution represents a molarity of 2E-2M, which is more than 100 times higher than the molar solubility. Therefore, this is a supersaturated solution.

400g AlCl3 is dissolved in 1.5L of water at room temperature (Kb-=0.512K(kg)/mol) How much does the boiling point increase after adding the aluminum chloride?

solution on pg 326 general chemistry Water at room temp has a density of 1g/mL. Therefore, 1.5L is the same as 1.5kg. The van't Hoff facrot for aluminum chloride is 4 because it breaks down to form 1 aluminum cation and 3 chloride anions. To determine the molality, we will also need to know how many moles 400g AlCl3 represents. 400gAlCl2 x 1mol/133.5g = 3 mol AlCl2 The molality is therefore 3 mol AlCl2/1.5kg = 2m Then plug into the BP elevation equation deltaTsubb = iKbm=(4)(0.512)(2m)=4K

An aqueous solution was prepared by mixing 70 g of an unknown nondissociating solute into 100g of water. The solution has a boiling point of 101C. What is the molar mass of the solute? (Kb=0.512 K , kg/mol). A. 358.4g/mol B. 32.3g/mol C. 123.2g/mol D. 233.6g/mol

solution on pg 339 general chemistry A The equation deltaTb=iKsubb(m) can be used. The change in BP is 101=100=1C. m=deltaTb/iKb=1K/(1)(0.512K.kg/mol) = 2m The van't Hoff factor is 1 because the molecules does not dissociate into smaller components. Then, convert to grams of solute using the definition of molality: molality=moles of solute/kg of solvent --> moles of solute = (2m)(0.1kg)=0.2mol The mass used in this equation is 0.1kg because 100mL of water has a mass of 0.1kg. Then, determine the molar mass: molar mass=70g/0.2mol=350g/mol

The Ksp of AgI in aqueous sollution is 8.5E-17. If a 1E-5M solution of AgNO3 is saturated with AgI, what will be the final [ ] of the iodide ion?

solution pg 321 general chemistry The [ ] of Ag+ in the original AgNO3 solution will be 1E-5 M because the AgNO3 will fully dissociate. Some small amount of AgI will dissociate into the solution, which is the molar solubility x of AgI under these conditions. The net silver [ ] from both AgNO3 and AgI will become 1E-5M + X. Because no iodide was present in solution until the AgI began dissociating, the [ ] of iodide will be x. Thus the Ksp expression for the dissociation of AgI is : Ksp = [Ag+][I-] 8.5E-17 = [1E-5M+x][x] 8.5E-12=x

What is the vapor pressure at room temp of a mixture containing 58g butane (C4H10) amd 172 hexane (C6H14)? (Note: The vapor pressure of pure butane and pure hexane are 172kPa and 17.6kPa respectively at 25C).

solution pg 325 general chemistry first determine the number of moles of each substance. 58 g butane represents 1 mol butane. 172 g hexane is 2 moles of hexane. Then, determine the mole fractions of each component mixture Xbutane=1/3 Xhexane=2/3 Then calculate the VP of each component: Pbutane (pure)=XbutatePbutane= 1/3(172kPa)=1/3(180kPa)=60kPa Phexane(pure)=XhexanePhexane= 2/3(17.6kPa)=2/3(18kPa)=12kPa The total VP is the sum of the two so its around 72kPa(actual 69.1kPa)

What are solutions? What are mixtures? What are solutes and solvents?

solutions are homogeneous mixtures of two or more substances that combine to form a single phase, usually the liquid phase. Mixtures are gases "dissolved" into gases. a solution consists of a solute dissolved in a solvent. The solvent is the component of the solution that remains in the same phase after mixing. If the two substances are already in the same phase the solvent is the component present in greater quantity. If the two same-phase components are in equal proportions in the solution, then the component that is more commonly used as a solvent in other contexts is considered the solvent.

What is solvation? What is another name for it? What is it called when water is the solvent?

solvation is the electrostatic interaction between solute and solvent molecules. This is also known as dissolution and when water is the solvent is it known as hydration. solvation involves breaking intermolecular interactions between solute molecules and between solvent molecules and forming new intermolecular interactions between solute and solvent molecules.

What is resistivity?

some materials are intrinsically better conductors of electricity than others. ex: copper conducts electricity better than plastic. the number that characterizes the intrinsic resistance to current flow in a material is called the resistivity (rho), for which the SI unit is the ohm-meter

How does the medium affect the rate of a reaction?

some molecules are more likely to react with each other in aqueous environments, while others are more likely to react in nonaqueous solvents. The physical state of the medium (solid, liquid, gas) can also have a significant effect.

What is spherical aberration?

spherical mirrors and lenses are imperfect. They are therefore subject to specific types of errors or aberrations. Spherical aberration is a blurring of the periphery of an image as a result of inadequate reflection of parallel beams at the edge of a mirror or inadequate refraction of parallel beans at the edge of a lens. This creates an area of multiple images with very slightly different image distances at the edge of the image, which appears blurry.

What is diffraction? What happens when there is interference between diffracted light rays?

spreading out of light as it passes through a narrow opening or around an obstacle. Interference between diffracted light rays can lead to characteristic fringes in slit-lens and double-slit systems.

What is starch and what is it's function in the body?

starches are polysaccharides that are more digestable by humans because they are linked a-D-glucose monomers. Plants predominantly store starch as amylose, a linear glucose polymer linked via a-1,4 glycosidic bonds. Another type of starch is amylopectin, which starts off with the same type of linkage that amylose exhibits, but also contains branches via a-1,6 glycosidic bonds. Starches like amylose and amylpectin are broken down by enzymes in the body and are used as a source of energy.

Explain stage vs process functions. Label each as a state or a process function Work (W) Pressure (P) Density (p) Temperature (T) Heat (Q) Volume (V) Enthalpy (H) Internal Energy (U) Gibbs Free Energy (G) Entropy (S)

state functions describe the system in an equilibrium state. Process function describe the process of a system, how a system got to its current equilibrium Work and Heat are process functions, everything else is a state function

What is the collision theory of chemical kinetics? Do all collisions result in a chemical reaction? What is the activation barrier?

states that the rate of a reaction is proportional to the number of collisions per second between the reacting molecules The theory suggests that not all collisions result in a chemical reaction. An effective collision (one that leads to the formation of products) occurs only if the molecules collide with each other in the correct orientation and with sufficient energy to break their existing bonds and form new ones. The minimum energy of collision necessary for a reaction to take place.

what is the difference between statistical significance and clinical significance?

statistical significance is not the result of random chance. clinical significance is a notable or worthwhile change in health status as a result of our intervention

what is the Belmont report?

summarizes ethical principles and guidelines for research involving human subjects. delineates the three necessary pillars of research ethics: respect for persons, justice, and a slightly more inclusive version of benicence

What is superheating?

superheating occurs when a liquid is heated to a temperature above its boiling point without vaporization. Superheating situations occur when gas bubbles within a liquid are unable to overcome the combination of atmospheric pressure and surface tension.

Explain simple distillation. When should it be used? What is the apparatus for this technique? How does it work?

takes advantage of differences in BP to separate two liquids by evaporation and condensation. The liquid with the lower BP will vaporize first, and the vapors will rise up the distillation column to condense in a water-cooled condenser. This condensate then drips down into a vessel. The end product is called the distillate. The heating temp is kept low so that the liquid with the higher BP will not be able to boil and therefore will remain liquid in the initial container. Should be used only to separate liquids that boil below 150C and have at least a 25C difference in BPs. The apparatus for this technique consists of a distilling flask containing the combined liquid solution, a distillation column consisting of a thermometer and a condenser, and a receiving flask to collect the distillate.

In uniform circular motion, the instantaneous velocity vector is always ______ to the circular path. -what does this mean?

tangent -the object moving in the circular path has a tendency (inertia) to break out of its circular path and move in a linear direction along the tangent

What is echolocation?

the Doppler effect used by animals. The animal emitting the sound serves as both the source and the detector of the sound. The sound bounces off a surface and is reflected back to the animal. How long it takes for the sound to return, and the change in frequency of the sound, can be used to determine the position of objects in the environment and the speed at which they are moving.

The work done on or by a system undergoing a thermodynamic process can be determined by finding what? When work is done by a system (the gas ______), the work is _______. When work is done on a system (the gas ______), the work is _____.

the area enclosed by the pressure volume curve work done by a system (gas expands), work = positive work done on a system (gas contracts), work= negative

What is Ohm's law?

the basic law of electricity because it states that for a magnitude of resistance, the voltage drop across the resistor will be proportional to the magnitude of the current. likewise, for a given resistance, the magnitude of the current will be proportional to the magnitude of the emf (voltage) impressed upon the current. V=IR the equation applies to a single resistor within a circuit, to any part of a circuit, or to an entire circuit as current moves through a set of resistors in a circuit, the voltage drops some amount in each resistor; the current (or sumof currents for a divided circuit) is constant. No charge is gained or lost through a resistor; thus, if resistors are connected in series, all of the current must pass through each resistor.

Explain ion-exchange chromatography

the beads in the column are coated with charged substances so that they attract or bind compounds that have an opposite charge. For instance, a positively charged compound will attract and hold a negatively charged backbone of DNA or protein as it passes through the column, either increasing its retention time or retaining it completely. After all other compounds have moved through the column, a salt gradient is used to elute the charged molecules that have stuck to the column.

Explain size exclusion chromatography

the beads used in the column contain tiny pores of varying sizes. These tiny pores allow small compounds to enter the beads, thus slowing them down. Large compounds can't fit into the pores, so they will move around them and travel through the column faster. It si important to remember that in this type of chromatography, the small compounds are slowed down and retained longer-which may feel counter intuitive. The size of the pores may be varied so that molecules with different molecular weights can be fractionated. A common approach in protein purification is to use an ion-exchange column followed by a size-exclusion column.

What is refraction?

the bending of light as it passes from one medium to another and changes speed. The speed of light through any medium is always less than its speed through a vacuum.

How does the bonded system of protons and neutrons compare to the unbonded constituents? What results?

the bonded system is at a lower energy level than the unbonded constituents, and this difference in energy must be radiated away in the form of heat, light, or other electromagnetic radiation before the mass defect becomes apparent. This energy, called binding energy, allows the nucleons to bind together in the nucleus.

what happens when a dielectric material is placed in a charged capacitor within a circuit?

the charge on the capacitor increases. The voltage must remain constant because it must be equal to that of the voltage source. By increasing the amount of charge stored on the capacitor, the dielectric has increased the capacitance of the capacitor by a factor of the dielectric constant. Thus, when a dielectric material is introduced into a circuit capacitor, the increase in capacitance arises from an increase in stored charge.

Explain how the eye is a refractive instrument.

the cornea acts as the primary source of refractive power because the change in refractive index from air is so significant. Then, light is passed through an adaptive lens that can change its focal length before reaching the vitreous humor. It is further diffused through layers of retinal tissue to reach the rods and cones. Here, the image has been focused and minimized significantly, but it is still relatively blurry. Our nervous system processes the remaining errors to provide a crisp view of the world.

What is the free energy change (delta G) of a reaction? endergonic or exergonic reaction? +deltaG= - deltaG= how does changing deltaG affect the rate of a reaction?

the difference between the free energy of the products and the free energy of the reactants +deltaG=endergonic=energy absorbed =deltaG=exergonic=energy given off it doesn't! kinetics and thermodynamics are separate.

what is the mass defect?

the difference between the mass of an isotope and its mass number. while one would assume that the mass of the nucleus is simply the sum of the masses of all the protons and neutrons within it, the actual mass of every nucleus (other than hydrogen) is smaller than that. This difference is the mass defect. The mass defect is a result of matter that has been converted to energy.

What is induction? Explain.

the distribution of charge across sigma bonds. Electrons are attracted to atoms that are more electronegative, generating a dipole across the sigma bond. the less electronegative atom acquires a slightly positive charge, and the more electronegative atom acquires a slightly negative charge. This effect is relatively weak and gets increasingly weaker as one moves further away within the molecules from the more electronegative atom. This effect is responsible for the dipole character of the carbonyl group, as well as the increased dipole character (and therefore susceptibility to nucleophilic attack) or carboxylic acids- which contain an additional oxygen atom in their leaving group.

what is high-performance liquid chromatography?

the eluent is a liquid and it travels through a column of a defined composition. There are a variety of stationary phases that can be chosed depending on the target molecule and the quantity of material that needs to be purified. This is fairly similar to column chromatogrpahy because the various compounds in solution will react differently with the adsorbent material. In HPLC, a small sample is injected into the column, and separation occurs as it flows throught. The compounds pass through a detector and are collected as the solvent flows out of the end of the apparatus. The interface is similar to that used for GC because the entire process is computeried, but uses liquid under pressure instead of gas. Because the whole process is under computer control, sophisticated solvent gradients as well as temp can be applied to the column to help resolve the various compounds in the sample- hence, the higher performance of HPLC over regular column chromatography.

What is Beta decay? What are the two types?

the emission of a beta particle, which is an electron and is given the symbol e- or B-. Electrons do not reside in the nucleus, but they are emitted by the nucleus when a neutron decays into a proton, a B-particle, and an antineutrino. In some cases of induced decay (positron emission), a positron is released, which has the mass of an electron but carries a positive charge. The positron is given the symbol e+ or B+. A neutrino (v) is emitted in positron decay as well. During B- decay, a neutron is converted into a proton and a B- particle (Z=-1, A=0) is emitted. Hence, the atomic number of the daughter nucleus will be one higher than that of the parent nucleus, and the mass number will not change. During B+ decay, a proton is converted into a neutron and a B+ particle (Z=+1, A=0) is emitted. Hence, the atomic number of the daughter nucleus will be one lower than that of the parent nucleus, and the mass number will not change.

What is electron affinity? endothermic or exothermic process? what is its trend across a periodic table?

the energy dissipated by a gaseous species when it gains an electron (essentially the opposite concept of ionization energy). Exothermic. Because this is an exothermic process, deltaHrxn is negative, but the electron affinity is positive. The stronger the electrostatic pull (higher the Zeff), between the nucleus and the valence shell electrons, the greater the energy release will be when the atom gains the electron. Electron affinity inc across a period from left to right. decreases in a group from top to bottom

What is ionization energy (ionization potential)? Endothermic or exothermic process? What is it's trend across a periodic table?

the energy required to remove an electron from a gaseous species. Endothermic. the greater the atom's Zeff (or closer the valence electrons are to the nucleus), the more tightly bound they are. This makes it more difficult to remove one or more electrons, increasing the ionization energy. Thus, ionization energy inc from left to right across a period and from bottom to top in a group.

What is the transition state? How is it different from reaction intermediates?

the intermediate complex formed as covalent bonds in the reactants are being broken and re-formed during a reaction the transition state, also called the activated complex, has greater energy than both the reactants and the products. transition states are distinguished from reaction intermediates in that transition states are theoretical constructs that exist at the point of maximum energy, rather than distinct identities with finite lifetimes

What is the law of reflection and the normal?

the law of reflection means theta 1 (incident angle) = theta2 (reflected angle) the normal is a line drawn perpendicular to the boundary of a medium; all angles in optics are measured from the normal, not the surface of the medium.

Explain randomization in an experiment.

the method used to control for differences between subject groups in biomedical research. Randomization uses an algorithm to determine the placement of each subject into either a control group that receives no treatment or a sham treatment, or one or more treatment groups. A proper randomization algorithm will be equivalent to a coin toss or die roll. Once each individual is assigned to a group, the intervention is performed and the results are measured. Ideally, each group is perfectly matched on conditions such as age and gender; however, as long as there is an appropriate randomization algorithm, the collected data may be analyzed without concern.

What are z or t tests? How are they done? Explain type 1 and type 2 errors.

the most common hypotheses tests. rely on the standard distribution or the closely related t-distribution. From the data collected, a test statistic is calculated and compared to a table to determine the likelihood that the statistic was obtained by random chance (under the assumption that our null hypothesis is true). This is our p value. We then compare our p-value to a significance level (alpha); 0.05 is commonly used. If the p-value is greater than alpha, then we fail to reject the null hypothesis, which means that there is not a statistically significant difference between the two populations. If the p-value is less than alpha, we reject the null hypothesis and state that there is a statistically significant difference between the two groups. Reject null hypothesis=our results our statistically significant. The value of alpha is the level of risk that we are willing to accept for incorrectly rejecting the null hypothesis. This is called a type 1 error. In other words, a type 1 error is the likelihood that we report a difference between two populations when one does not actually exist. A type II error occurs when we incorrectly fail to reject the null hypothesis. In other words, a type II error is the likelihood that we report no difference between two populations when one actually exists. The probability of a type II error is sometimes symbolized by Beta. The probability of correctly rejecting a false null hypothesis is referred to as power and equals 1-Beta. Finally, the probability of correctly failing to reject a true null hypothesis is referred to as confidence.

What is resistance? materials that offer almost no resistance are called what? what about those materials that offer very high resistance? what are resistors?

the opposition within any material to the movement and flow of charge. Electrical resistance can be thought of like friction, air resistance, or viscous drag: motion is being opposed. materials that offer almost no resistance are called conductors. materials that offer very high resistance are called insulators. resistors are conductive materials that offer amounts of resistance between these two extremes

what is the capacitance of a capacitor?

the ratio of the magnitude of the charge stored on one plate to the potential difference (voltage) across the capacitor. Therefore if voltage V is applied across the plates of a capacitor and charge Q collects on it (with +Q on the positive plate and -Q on the negative plate) then the capacitance is given by: C=Q/V SI unit: farad, 1F=1C/V capacitances are usually given in microfarads (1muF=1E-6F) or picofarads (1pF=1E-12F)

what is conductance?

the reciprocal of resistance SI unit- Siemens (S)

What is viscosity? What about viscous drag? Explain the two in terms of thin fluids vs thick fluids.

the resistance of a liquid to flow drag- a nonconservative force that is analagous to air resistance. Thin fluids, like gases, water, and dilute aqueous solutions, have low viscosity and so they flow easily. Objects can move through these fluids with low viscous drag. Whole blood, vegetable oil, honey, cream, and molasses are thick fluids and flow more slowly. Objects can move through these fluids, but with significantly more viscous drag.

What is the electric dipole? It can be either ______ or _______.

the result of two equal and opposite charges being separated a small distance, d, from each other transient (as in the case of the moment-to-moment changes in electron distribution that create London dispersion forces) or permanent (as in the case of the molecular dipole of water or the carbonyl functional group)

What is rotational equilibrium (also known as what?)

the second condition of equilibrium- exists only when the vector sum of all the torques acting on an object is zero clockwise torques - negative counterclockwise torques -positive so all the pos torques must exactly cancel out the neg torques

Straight wires create magnetic fields in what shape? How can you determine the direction of field vectors of the magnetic field in straight wires and looped wires?

the shape of concentric rings use the right hand rule (one of 2 right hand rules). point your thumb in the direction of the current and wrap your fingers around the current-varying wire. Your fingers mimic the circular field lines, curling the wire

What is the rate-determining step in a reaction? Why is it important?

the slowest step in a reaction the rate of the whole reaction in only as fast as the rate-determining step

Explain splitting in NMR

the splitting of the peak represents the number of adjacent hydrogens. A peak will be split into n+1 subpeaks, where n is the number of adjacent hydrogens. doublet: two peaks of identical intensity, equally spaced around the true chemical shift of Ha. Ha and Hb will both appear as doublets because each one is coupled with one other hydrogen.

what does electrolytic conductivity depend mostly on? how is conductivity in an electrolyte solution measured?

the strength of the solution. distilled deionized water has such a low ion [ ] that it may be considered an insulator, while sea water and orange juice and excellent conductors. measured by placing the solution as a resistor in a circuit and measuring changes in voltage across the solution. Because [ ] and conductivity are directly related, this method is often used to determine ionic [ ]s in solutions, such as blood. One caveat is that conductivity in nonionic solutions is always lower than in ionic solutions. While the [ ] of total dissolved solids does relate to conductivity, the contribution of nonionic solids is much, much less important than ion [ ].

what is selection bias?

the subjects used for the study are not representative of the target population. People who volunteer for a study in a particular area may be significantly different from people who do not volunteer.

what are the triple point and the critical point on a phase diagram?

the triple point is where the three phase boundaries meet. this is the temperature and pressure at which the three phases exist in equilibrium. The phase boundary that separates the solid and the liquid phases extends indefinitely from the triple point. the phase boundary between the liquid and gas phases, however, terminates at a point called the critical point. This is the temp and pressure above which there is no distinction between the phases.

What happens when a dielectric material is placed in an isolated, charged capacitor (a charged capacitor disconnected from any circuit)?

the voltage across the capacitor decreases. This is the result of the dielectric material shielding the opposite charges from each other. By lowering the voltage across a charged capacitor, the dielectric has increased the capacitance of the capacitor by a factor of the dielectric constant. Thus, when a dielectric material is introduced into an isolated capacitor, the increase in capacitance arises from a decrease in voltage.

What does a system in translational equilibrium look like?

the weight (the load) is balanced by the total tension in the ropes holding it delta T (tension) = mg also, the tension in the ropes must be equal to each other, otherwise the pulleys would turn until the tensions were equal on both sides. Therefore, if there are 2 ropes holding an object up, each rope supports only 1/2 of the crate's total weight. By extension, only half the force (effort) is required to life the crate.

Explain galvanic (voltaic) cells. what are they in your house? what is the delta G and emf? Explain the inner workings of a galvanic (voltaic) cell.

they are commonly non-rechargeable batteries. the reactions in these cells must be spontaneous. delta G < 0, emf>0. Two electrodes of distinct chemical identity are placed in separate compartments, which are called half-cells. The two electrodes are connected to each other by a conductive material, such as a copper wire. Along the wire, there may be other components of a circuit, such as resistors. Surrounding each of the electrodes is an aqueous electrolyte solution composed of cations and anions. Connecting the two solutions is a structure called a salt bridge, which consists of an inert salt which contains ions that will not react with the electrodes or with the ions in the solution. When the electrons are connected to each other by a conductive material, charge will begin to flow as the result of a redox reaction that is taking place between the two half-cells. This reaction is spontaneous. As this reaction proceeds towards equilibrium, the movement of electrons results in a conversion of electrical potential energy into kinetic energy. By separating the reduction from the oxidation half-reactions, we are able to harness this energy and use it to do work by connecting various electrical devices into the circuit between the two electrodes.

what are ammeters how do they work

they are used to measure the current as some point within a circuit. Using an ammeter requires the circuit to be on, or the current will be 0A. Ammeters are inserted in series where the current is being measured and use the magnetic properties of a current-carrying wire to cause a visible needle movement or calibrated displaty of the current.

how do ultrasounds work?

they use high frequency sound waves outside the range of human hearing to compare the relative densities of tissues in the body. an ultrasound machine consists of a transmitter than generates a pressure gradient, which also functions as a receiver that processes the reflected sound. Because the speed of the wave and travel time is known, the machine can generate a graphical representation of borders and edges within the body by calculating the traversed distance.

How do you perform a titration?

titrationa are performed by adding small volumes of a solution of known [ ](the titrant) to a known volume of a solution of unknown [ ](the titrand) until completion of the reaction is achieved at the equivalence point. In acid-base titrations, the equivalence point is reached when the number of acid equivalents present in the original solution equals the number of base equivalents added, or vice versa. The equivalence point is determined either evaluated by using a graphical method, plotting the pH of the unknown solution as a function of added titrant by using a pH meter, or estimated by watching for a color change of an added indicator.

How do you determine the direction of the magnetic force on a moving charge?

use the right hand rule (one of two right hand rules). first, position you right thumb in the direction of the velocity vector. then, put your fingers in the direction of the magnetic field lines. Your palm will point in the direction of the force vector for a positive charge, whereas the back of your hand will point in the direction of the force vector for a negative charge Thumb-velocity (indicates direction of movement, like a hitchhikers thumb) Fingers-field lines (fingers are parallel like the uniform magnetic field lines) Palm-force on a positive charge (you must give a high five to a positive person) Back of hand-force on a negative charge (you must give a backhand to a negative person)

How does distillation work?

used when the product itself is a liquid that is soluble in the solvent. takes advantage of differences in BP to separate two liquids by evaporation and condensation. The liquid with the lower BP will vaporize first, and the vapors will rise up the distillation column to condense in a water-cooled condenser. This condensate then drips down into a vessel. The end product is called the distillate. The heating temp is kept low so that the liquid with the higher BP will not be able to boil and therefore will remain liquid in the initial container. This is the process that is used to make liquor at a distillery.

What is a first-order reaction? What is the rate law for this type of reaction? What are then units for k?

using the generic aA + bB --> cC+ dD - has a rate that is directly proportional to only one reactant, such that doubling the [ ] of that reactant results in a doubling of the rate of formation of the product Rate Law: rate=k[A]^1 or rate = k[B]^1 k units (s^-1)

What is a zero-order reaction? What is the rate law for this type of reaction? What are the units of k?

using the generic aA + bB --> cC+ dD - the rate of formation of product C is independent of changes in [ ]s of any of the reactants, A and B. - These reactions have a constant reaction rate equal to the rate constant, k. The rate law for a zero-order reaction is: rate =k[A]^0 [B]^0 = k k units (M/s)

What is a second-order reaction? What is the rate law for this type of reaction? What are then units for k?

using the generic aA + bB --> cC+ dD -has a rate that is proportional to either the [ ] of two reactants or to the square of the [ ] of a single reactant -It's important to recognize that a second-order rate law often suggests a physical collision between two reactant molecules Rate Law: rate =k[A]^1 [B]^1 or rate=k[A]^2 or rate=k[B]^2 k units (M^-1 s^-1)

Electrons move around the nucleus at varying ______, which correspond to varying what?

varying distances which correspond to varying levels of electrical potential energy.

how can a potential difference (_______) be produced? what is electromotive force (emf)?

voltage- can be produced by an electrical generator, a galvanic (voltaic) cell, a group of cells wired into a battery or even a potato. emf is when no charge is moving between the two terminals of a cell that are at different potential values. -ps, this is not actually a force, it is a potential difference (voltage)and it's units are J/C not N

What are longitudinal waves

waves in which the particles of the wave oscillate parallel to the direction of propagation; that is, the wave particles are oscillating in the direction of energy transfer

What is vacuum distillation and when is it used?

we use vacuum distillation whenever we want to distill a liquid with a BP over 150C. By using a vacuum, we lower the ambient pressure, thereby decreasing the temp that the liquid must reach in order to have sufficient vapor pressure to boil. This allows us to distill compounds with higher boiling points at lower temps so that we do not have to worry about degrading the product.

What is steric hindrance?

when a reaction does not proceed due to the size of the substituents. A good example is in SN2 reactions, which won't occur at tertiary carbons. This can be used to control where a reaction occurs in a molecule. Protecting groups may make it too hard for a nucleophile, oxidizing agent, or reducing agent to access or react with a part of the molecule.

What is the strong nuclear force?

when protons and neutrons (nucleons) come together to form the nucleus, they are attracted to each other by the strong nuclear force, which is strong enough to more than compensate for the repulsive electromagnetic force between the protons. Although the strong nuclear force is the strongest of the four fundamental forces, it only acts over extremely short distances, less than a few times the diameter of a proton or neutrons. The nucleons have to get very close together in order for the strong nuclear force to hold them together.

How do resistors in parallel work?

when resistors are connected in parallel, they are wired with a common high-potential terminal and a common low-potential terminal. This configuration allows charge to follow different parallel paths between the high-potential terminal and the low-potential terminal. Electrons have a "choice" regarding which path they will take. No matter which path is taken, the voltage drop experienced by each division of current is the same because all pathways originate form a common point and end at a common point within the circuit. This is analogous to a river that splits into mult streams before plunging over different waterfalls, which then come back together to reform the river at a lower height. If all the water starts at some common height and ends at a lower common height, then it doesn't matter how many steps the water fell over to get to the bottom of the falls: the change in height is the same for each stream. While the voltage is the same for all parallel pathways, the resistance of each pathway may differ. In this case, electrons prefer the path of least resistance, the current will be largest hrough the pathways with the lowest resistance.

When is solvation exothermic vs endothermic? When is it favored at low temps vs high temps? Which is more common? Give exs of each.

when the new interactions are stronger than the original ones, solvation is exothermic and the process is favored at low temps. when the new interactions are weaker than the original ones. Ex: gases into liquids solvation is endothermic and the process is favored at high temps. Most dissolutions are of this type. Ex: ammonium nitrate or sugar into water

What is an isobaric process? How does it appear on a pressure volume graph.

when the pressure of the system is constant. Isothermal and isobaric processes are common because it is usually easy to control temperature and pressure. Isobaric processes do not alter the first law, but note that an isobaric process appears as a flat line on a P-V graph

Explain dispersion. What is the most common example? Explain how it works.

when various wavelengths of light separate from each other, this is called dispersion. The most common example of dispersion is the splitting of white light into its component colors using a prism. If a source of white light is incident on one of the faces of a prism, the light emerging from the prism is spread out into a fan-shaped beam. This occurs because violet light has a smaller wavelength than red light and so is bent to a greater extent. Because red experiences the least amount of refraction, it is always on top of the spectrum; violet, having experienced the greatest amount of refraction, is always on the bottom of the spectrum. Note that as light enters a medium with a different index of refraction, the wavelength changes but the frequency of the light does not.

What is the principle of superposition? What is the difference between constructive and destructive interference in waves?

when waves interact with each other, the displacement of the resultant wave at any point is the sum of the displacements of the two interacting waves. Constructive Interference- When the waves are perfectly in phase, the displacements always add together and the amplitude of the resultant is equal to the sum of the amplitudes of the two waves. Destructive Interference- When waves are perfectly out of phase, the displacements always counteract each other and the amplitude of the resultant wave is the difference between the amplitudes of the interacting waves.


Ensembles d'études connexes

Senderos 1 NN En La Clase Part 3

View Set

Ch 13: Normal and Altered Immune Responses

View Set

Drivers Ed Segment 2: Study Guide

View Set

Chemistry- chapter 2 Chemical equations and air quality

View Set

Chapter 3 - Organic compounds, DNA,

View Set

EDS500 Chapter 5: Learners with Intellectual and Developmental Disabilities

View Set